Download as docx, pdf, or txt
Download as docx, pdf, or txt
You are on page 1of 353

1.

    Which Republic Act provides government assistance to students and teachers in private education?

a. RA 7836                  c. RA 6728
b. RA 7784                 d. RA 6675

2.    In the preamble of the CODE of ethics for Professional Teachers, which of the following teacher descriptions is
included in the Code of Ethics for Professional Teachers?

a. A. With satisfactory teaching performance


b. Passed the licensure exams for teachers
c. Duly licensed professional
d. Persons of dignity and reputation

3.    Which stage according to Erickson is the stage of trust and mistrust?

a. Young adulthood                c. Adolescence    


b. Early childhood                  d. Infancy

4.    In the history of education system, including  that of the Philippines, which system was first and last remained a
partner of other systems of education?
a. Formal  b. Non-formal c. Informal  d. Pre-school
5.    Who among the following subscribe to the theory that the good, the true and the beautiful are universally valid in
all places at all times?
a.    Realists and idealists
b.    Realist and existentialists    
c.    Pragmatists and idealists
d.    Pragmatists and realists
6.    The 1987 Constitution mandates the state to encourage among others self-learning, independent and out-of-school
study programs particularly those that respond to community needs. Which is an off shoot of this mandate?

a. Institutionalization of early childhood care and development


b. Principal empowerment program
c. Multi-grade teaching
d. Practices of community-based learning

7.    Authors claim that education is given greater in the 1987 Philippine Constitution than in the past Constitution.
Which article provision on education is found only in the 1987 Constitution?

a. Support to teacher education


b. Provision on assigning the highest budget priority to education
c. The establishment of the integrated system of education
d. The creation of CHED

8.    According to Hurlock, studies of children’s play revealed that toy play reaches its peak during the

a. Middle childhood years                   


b. Early childhood years            
c. Babyhood
d. Late childhood

9.    In a DECS memorandum issued last 1998, the new name for multi-grade class is:

a. A.Friendly school      c. Paaralan ng bahay


b. School-of-the-future d. Escuela nueva

10.The student-centered curriculum belongs to which group of educators?

a. Reconstructionist-Essentialist
b. Progressivist, Perennialist   
c. Perennialist, Essentialist      
d. Progressivist, Reconstructionist

11.Daniel Goleman talks about emotional intelligence. Which of the following characterize a student with a high degree
of emotional intelligence?

a. Sensitive to points of view and feeling of others


b. Prodded by others
c. Obsessed with achieving at any cost
d. Unable to delay gratification

12.The history of curriculum development includes the hidden curriculum approach. Which is the content emphasis of
the hidden curriculum?

a. Students experiences and activities


b. Implicit processes and social norms
c. Student needs and interest
d. Introspection and choice

13.Teacher T is concerned with conceptual matters since reality is mental teacher T’s thinking is quite-

a. Realistic                  c. Existential
b. Pragmatic               d. Idealistic

14.The authoritarian setting in the Filipino home is reinforced by a classroom teacher who-

a. Asks open –ended questions


b. Encourages pupils to ask questions
c. Prescribes what pupils should do
d. Is open to suggestions

15.A number of researches found that the effects of maternal employment on children’s achievement are-

a. Hardly established         


b. Positive and negative         
c. Fully established    
d. Negative

16.Which one correctly describes the phenomenon of latchkey children?

a. They turn to empty homes after school either to sit by the television or to roam the streets
b. They are truant school children
c. They are engaged in child labor
d. They are engaged in child labor

17.Teacher Q regards the student as a spiritual entity and as part of the larger spiritual universe. To what philosophy
does teacher Q subscribe?

a. Idealism                 c. Realism


b. Existentialism           d. Pragmatism

18.The 1987 Constitution mandates the state to assign the highest budgetary priority to-

a. Health                    c. Social work


b. Education                d. Defense

19.Moral development in early childhood is characterized by-


a. Questioning rules
b. Willful disobedience of rules
c. Acting out of conviction
d. Acting without knowing why they do so

20.According to Piaget, by adolescence boys and girls must have reached what stage?

a. Concrete operations stage


b. Sensorimotor stage
c. Formal operations stage
d. Pre-operational stage

21.Ruth dances well. She can figure out how something works or how to fix something that is broken, without asking
for help. Based on Garden’s theory of multiple intelligence under what intelligence is she strong?

a. Musical                  
b. Logical-mathematical
c. Spatial intelligence
d. Kinesthetic intelligence

22.Who among the following believes that learning requires disciplined attention regular homework, and respect for
legitimate authority?

a. Reconstructionist        c. Essetialist
b. Perennialist               d.Progressivist

23.Which developmental task is expected of the adolescent according to havughurst?

a. Achieving new and more mature relations with age-mates of both sexes
b. Skilled games
c. Getting started in an occupation
d. Learning to get along with age-mates

24.Mr. Roy wants his pupils to be creative. Which of the following will he REFRAIN from doing?

a. Heighten the student’s sense of unusual


b. Develops students’ ability to recognize and analyze problems and relationships
c. Encourage sustained attention in an area of interest
d. Develop solely skill mastery

25.The creation of Student’s Loan Fund to give equal opportunity to all persons who desire to pursue higher education  
is the essence of:

a. RA 6014                 c. RA 6655   


b. RA  7836                 d. RA 7784

26.In his/her teaching, Teacher D moves from particular instances to tentative generalizations that are Subjected to
further verification . Teacher D  engages  himself/herself in:

a.  Intuition                  
b. Inductive logic
c. Deductive logic  
d. Philosophical analysis

27.Which of the correct statement of emotional intelligence is based on Daniel Goleman’s theory?

a. Emotional intelligence is feeling approximately and effectively


b. Emotional intelligence id being nice to people
c. Emotional intelligence is giving free rein to feeling
d. Emotional intelligence changes less considerably than IQ through life.

28.Which of the following is not a subject-centered curriculum?

a. Perennialist                c. Values-centered    


b. Back-to-basics     d. Subject area

29.Which program is assisted financially by the World Bank and OECF and is meant for the improvement of elementary
education in the SRA provinces?

a. Program for Decentralized Education   


b. First Elementary Education Project   
c. Kinesthetic intelligence
d. Second Elementary Education Project

30.Mr. Z, A fifty-five year old American citizen, even though very educationally qualified to be president an educational
institution in third country cannot be appointed as President because: He_____

a. Does not belong to a religious group 


b. Is nearing retirement         
c. Is not Filipino Citizen    
d. Is not an official of a corporation

31.Which of the following is not a hazard to the mastery of developmental tasks?

a. Unfavorable social judgment


b. Bypassing of stage of development as a result of failure to master the tasks for that stage of development
c. Crisis when individuals pass from on stage to another
d. Inappropriate or impossible expectations

32.Teacher U emphasizes to her students the importance of deep personal reflection on one’s commitments and
choices. Teacher U subscribes to which Philosophy?

a. Realism                   c. Existentialism       


b. Idealism                  d. Pragmatism

33.The constitutional provision on language has the following aims, Except:


a.    To make Filipino the national language and medium of instruction and official communication
b.    To make  the regional  dialect  as auxiliary media of instructions in regional schools
c.    To make Filipino the sole  medium of instruction
d.    To maintain  English as a second language
34.Which periods in the life span of an individual are characterized by growth spurt?

a. Early childhood and adolescence  


b. Prenatal period and early childhood
c. Early childhood 
d. Pre-natal and puberty

35.The introduction of non-formal education in line with the Constitutional Provision on:

a. Protection of teachers
b. Promoting the rights of all citizens to make quality education accessible to all
c. Optional religious instructions
d. Promoting the rights of all citizens to quality education at all levels

36.Teacher X is often times frustrated. The students in her class hardly volunteer to recite and to do other learning-
related task. This is a proof of the Filipino’s:

a. Sense of humor       
b. Lack of resourcefulness
c. Lack of reflection
d. Passivity and lack of initiative

37.The tendency to emphasize so much on school beautification to the detriment of pupil’s performance illustrated the:

a. Filipino’s sense of humor


b. Filipino’s lack of reflection
c. Filipino’s love for “ Porma” at the expense of substance
d. Filipino’s lack of seriousness

38.A president  body  to study Philippines education created by virtue of Executive Order No. 46 during the incumbency
of DECS secretary Andrew Gonzales  with an aim to study Philippine Education is the:

a. Presidential Commission on Educational Reform


b. Philippine Commission to Survey Philippine Education
c. Survey to outcomes of elementary Education
d. Commission on Philippine Education

39.Which is a danger signal of adolescent mal-adjustment?

a. Inappropriate body-build
b. Irresponsibility as shown in neglect  of studies in favor of having a good time
c. Attraction to the opposite sex
d. Prolonged treatment  as children

40.To democratize access to secondary education, public secondary education was made free. In whose presidency was
this implemented?

a. Ferdinand Marcos     c. Corazon Aquino


b. Joseph Estrada          d.Fidel Ramos

41.Which is not a characteristic of democratic discipline?

a. Child obeys blindly


b. Child understands the meaning of rules
c. Child is given punishment  us related to the misdeed
d. Child has opportunity to express his/her opinion

42.The three A’s of happiness according to Hurlock are:

a. Adjustments, affection, altruism 


b. Attitude, ability, adjustment
c. Acceptance, affection, and achievement
d. Affection, ability, attitude

43.Babyhood is often referred to as a “ Critical period” in the development of personality because:

a. At this time the foundations are laid upon which the adult personality structure will be built
b. Changes in the personality pattern take place
c. The brain grows and develops at such an accelerated rate during babyhood
d. At this time the baby is exposed to many hazards both physical and psychological

44.Who among the following stressed the processes of experiences and problem solving?

a. Plato                      c. Dewey 


b. Hegel                     d. Aristotle
45.Teacher L believes that creation of knowledge in by way of the learners’ interaction with their environment. Teacher
L is more of:

a. An idealist               c. An existentialis
b. A pragmatist            d. A realist

46.To reach out clientele who cannot be in the classroom for one reason or another, which of the following was
established

a. Non-formal education
b. Informal education    
c. Pre-school education
d. formal education

47.The encouragement of self-learning, independent, and out-of-school study programs as stated in the 1987
constitution has given rise to:

a. The implementation of open universities and distance learning programs


b. Institutionalization of early childhood care and development
c. The conduct of NEAT and NSAT
d. The clamor for a Grade VII

48.Which of the following statement on developmental tasks is wrong?

a. Failure to master developmental tasks at a certain developmental stage has far reaching consequences in
a person’s development
b. The mastery of development tasks is a result of physical maturation, societal pressure and individual’s
aspiration
c. There are essential skills expected to be acquired  and mastered in each developmental stage
d. Retirees are not expected to work on mastering certain develop mental tasks

49.The 1987 Philippines Constitution states the following, Except:

a. The official languages of the Philippines are Filipino and English


b. The national language of the Philippines is Filipino
c. Filipino is the Tagalog of the Tagalog –speaking provinces
d. The government shall initiate and sustain the use of Filipino as a medium of official communication and as
a language of instruction in the education system

50.Which among the following has been said to play an irreplaceable role in the education of the young?
     

a. Community c. Church
b. School d. Home

Answer Key:
1.     C
2.     D
3.     D
4.     A
5.     D
6.     D
7.     B
8.     D
9.     B
10.  C
11.  A
12.  B
13.  D
14.  C
15.  A
16.  A
17.  B
18.  B
19.  D
20.  C
21.  D
22.  B
23.  A
24.  D
25.  A
26.  B
27.  A
28.  C
29.  A
30.  C
31.  C
32.  C
33.  C
34.  A
35.  B
36.  D
37.  C
38.  A
39.  B
40.  C
41.  A
42.  C
43.  A
44.  C
45.  D
46.  A
47.  A
48.  D
49.  A
50.  D

51.For the drill method to be effective there should be:

a. Much practice on a few skills


b. Little practices on few skills
c. Much practice on many skills
d. Little practice on many skills

52.Which of the following classroom management practices runs counter to the reality therapy approach of William
Glasser?

a. The teacher asks a misbehaving student what he or she is doing


b. Teacher requires the students to prepare a plan in writing and sign it as means of increasing personal
motivation to maintain and fulfill the plan
c. The teacher evaluates pupil’s behavior as good or bad
d. Teacher assists the students in making realistic plans to change behavior

53.Which of the following does not apply to mastery learning?

a. It makes use of varied instructional time for different groups of students


b. It makes use of norm-reference tests
c. It is effective in teaching basic skills
d. It requires the use of carefully crafted instructional objective

54.The teacher’s role in problem solving methods is:

a. To test the conclusions


b. To set up the problem
c. To propose ways of obtaining the needed data
d. To help the learners what is it that is being solved

55.To nurture creativity in student, teacher should:

a. Vary the length and difficulty of question


b. Allow for one-minute wait-time
c. Ask convergent questions
d. Emphasize the necessity of giving right answers

56.The content of a good lesson plan is self-sufficient. This means that:

a. The content should match with student’s aptitude


b. The content should help students learn how to learn
c. The content should be broad and treated thoroughly
d. The content should be verifiable

57.Which one can NOT help individuals reduce their own fears about change?

a. Reflect on the extent of change in one’s life so far and how it has coped with
b. Identify shortfalls in skills or knowledge and take action to remedy them
c. Identify skills which will be useful in a new situation
d. Keep feeling of anxiety to themselves

58.The following are generating thinking skills, Except:

a. Connecting new ideas c. Classifying


b. Predicting d. Inferring

59.Mrs. Valdez wants to generates as many ideas as she can as the class is about to embark on a community outreach
program. Which of the following will she employ?

a. role playing c. brainwashing


b. brainstorming d.simulation

60.Which teaching practice will most likely decrease learner’s attention?

a. Questioning
b. Assessing learning as an integral part of instruction
c. Assessing learning as an integral part of instruction
d. Teaching by telling

61.For efficiency, if in the process of teaching a teacher realizes that too little has been planned, which of the following
may she NOT do?

a. Discuss possible problems of the new assignment.


b. Pose additional questions to explore various facets of the content
c. Drill the students on the major points of the lesson.
d. Give a quiz

62.Correct practice makes perfect. This maxim is based on Thorndike’s law of exercise and the finding that
reinforcement of a response increases the likelihood its occurrence. Whose research finding is the underlined
statement?

a. Skinner c. Palo
b. Bruner d. Lewin
63.Here is a lesson objective: Given a microscope and a slide, the students must be able to focus the prepared slide.
Applying Robert F. Mager’s principle, which does this lesson objective lack?

a. Criterion measure c. Performance


b. Condition d. time element

64.Which is a focusing thinking skill?

a. Identifying key concepts


b. Summarizing
c. Obtaining information
d. Clarifying through inquiry

65.In the use of television in the classroom, which of the following should be avoided?

a. The TV program lasts the whole class period.


b. There is a pre-viewing orientation
c. Select programs that match the learners’ level of interest and maturity
d. Life should be left on if students are to take notes

66.Based on Victor Lowenfield’s classification, which sequence in the developmental stages of children in art is
followed?

a. Scribbling stage, pre-schematic stage, schematic stage, dawning realism stage, pseudo-realistic stage
b. Scribbling stage, pre-schematic stage, schematic stage, pseudorealistic stage, dawning realism stage
c. Pre-schematic stage, schematic stage, scribbling stage, pseudorealistic stage, dawning realism stage
d. Pre-schematic, schematic stage, scribbling dawning realism stage, pseudo-realistic stage

67.In relation to teacher’s ratings research suggests the following, Except one:

a. Teachers favor self-evaluation over all other forms of evaluation


b. Peer and supervisory evaluation are not reliable
c. Student raters of teachers are more reliable and valid than other raters
d. Student work and test outcomes are supplementary sources for evaluating teachers

68.Suppose defined 3 levels of computer-assisted instruction: dialogue, tutoring, practice-drill. Which ones are two
higher levels in the correct order?

a. Tutoring and dialogue


b. Practices-drill and tutoring
c. Dialogue and tutoring
d. Practice-drill and dialogue

69.In what condition is the use of the lecture method appropriate?

a. Higher cognitive learning is sought


b. The information is not available
c. The subject matter is quite easy
d. Long term learning is desired

70.Which refers to the time when students learn at a maximum level?

a. Wait time c. Allocated time


b. Engaged time d. Academic learning time

71.What is a sign of the underachiever in the classroom?

a. Resist authority and carry on a power struggle with a teacher.


b. Holds back from class participant unless sure of self
c. Frustrated about quality of work
d. Minimum work output.

72.If after calling on a number of students, a teacher is unable to obtain the desired response, what should teacher do

a. Ask leading questions


b. Re-teach parts of the lesson that need re-teaching
c. Probe student’s answers.
d. Rephrase the questions

73.For global competitiveness, a school must embark on a proactive change. Which one is a characteristic of proactive
change?

a. radical and inventive c. late in the game


b. imitative of others d. problem-driven

74.Which guideline on the use of the chalkboard should a teacher AVOID?

a. Establish routine uses for the chalkboard


b. While writing, proceed from right to left
c. Don’t talk to the chalkboard while writing on it
d. Limit your board writing to major ideas

75.To teach the democratic process to the pupils, Mabuhay Elementary School decided that the election of class and
school officers shall be patterned after local election of the class and school officers shall be patterned after local
election process. There are qualifications set for candidates, limited period for campaign and rules for
postingcampaign materials, etc. Which of the following did the school use?

a. Simulation c. Role playing


b. Philips 66 d. Symposium

76.Which is NOT a characteristic of preventive discipline.

a. Proactive c. Inventive
b. Anticipatory d. Reactive.

77.One criterion that has been proposed in the selection and organization of content in the development section of a
lesson plan is balance. When does lesson content possess balance.

a. The content should have practical application for the learners


b. The content is not cluttered by masses of more trivial content
c. The content should enable the learners to experience the broad sweep of content and give them the
opportunity to go deeper
d. The content should help student learns how to learn

78.The burnout malady gets worse if a teacher doesn’t intervene to change whatever areas he or she can control.
Which one can renew a teachers enthusiasm.

a. Engage in self-pity
b. Stick to the job
c. Judge someone else as wrong
d. Initiate changes in job.

79.Although learning can take place anywhere and anytime, the more systematic the teacher, the greater the
probability for success. This points out the need for teachers to:

a. Specify their objectives


b. Plan their lessons
c. Set their teaching priorities
d. Select their instructional materials

80.Below are teachers effective classroom management practices, Except One.

a. Teachers make sure students understand and follow rules and procedures
b. Disruptive behavior is handled every weekend
c. Teachers clearly establish consequences for not following rules
d. Teachers spend more time in the beginning of the year explaining and reminding students of rules.

81.A teacher’s summary of a lesson serves the following functions except;

a. It links the parts of the less


b. It makes provisions for full participation of the students
c. It clinches the basic ideas or concepts of the lesson
d. It brings together the information that has been discussed

82.Which ones are projected visuals?

a. Graphs on a book c. Realias


b. Models d. Slides

83.The lesson is on the pros and cons of capital punishment. Mr. Milan wants to do high level thinking and to develop a
view of capital punishment from a different perspective. Which technique will be most appropriate?

a. Lecture c. Role playing


b. Simulation d. Panel discussion

84.Which is used to emphasize individualized instruction?

a. Tutorial group c. Inquiry group


b. Philips 66 d. Task group

85.Here a question: FROM THE DATA PRESENTED FORM GENERALIZATIONS THAT ARE SUPPORTED BY THE DATA.
Under what type of question does this test item fall?

a. Evaluate c. Divergent
b. Convergent d. Application

86.Ms. Estira cannot bring her pupils to the sea for a lesson on marine community. Which of the following will be
closest to an actual experience of marine community?

a. Motion picture on marine community


b. Description of marine community
c. Pictures of marine community
d. Taped lesson of marine community

87.Which can NoT help organizations reduces fears about change?

a. By training people in new skills


b. By keeping people informed about plans
c. By cladestine moves
d. By consultation

88.After a lesson in air pollution, teacher C gives each pupil mimeographed sheets, which contain statements to which
each is expected to react. Examples of these statements are. Do you in any way contribute to air pollution? What
solutions do you propose to minimize, if not eliminate, air pollution. Then the students are grouped for sharing.
In this case what does the teacher make use of
a.         Value sheet c. Rank ordering
b.         Conflict story d. Contrived incident
89.Which orders consist of the goal-oriented instructional model?

a. Pre-assessment, specification of objectives, pre-assessment, instruction, evaluation


b. Specification of objectives, pre-assessment, instruction evaluation
c. Pre-assessment, specification of objectives, instruction, evaluation
d. Specification of objectives, evaluation, pre-assessment, instruction

90.Teacher B’s lesson was on abortion. She wanted to get her student’s maximum participation and reaction. She
showed slides on abortion accompanied by an audiotape that presented the reasons for and against abortion.
Teacher took a stand in favor of abortion and came up with all her justifications. At the end, however, she made
clear her stand on abortion. Which did the teacher use?

a. She made use of value clarifying discussion


b. She made use of contrived incident
c. She made use of simulation
d. She played the role of the devil’s advocate

91.In on of the pages of her reference material, Ms. Estrada finds the life cycle of a frog. Assuming that the following
are available to her, the quickest way to effectively present the life cycle of a frog to the class is by way of

a. An overhead projector
b. An opaque projector
c. Describing the life cycle
d. A drawing on the chalkboard

92.Below are questions that must be considered in developing appropriate learning activities/ experience, Except one

a. Do the experiences encourage pupils to inquire further?


b. Do the experiences save the pupils from learning difficulties?
c. Can be experiences profit the pupils?
d. Are the experiences in accordance with an increasing amount of learning?

93.Which order of the senses goes with an increasing amount of learning?

a. Sight, hearing, touch, taste and smell


b. Taste and smell, sight , touch, hearing
c. Taste and smell, touch, sight, hearing
d. Taste and smell, touch, hearing, sight

94.Which of the following statements on peer tutoring is true?

a. Teacher has time with more severe learning problems


b. The atmosphere is threatening
c. The social skills of pear tutors are not enhanced
d. the peer tutor’s achievement goes down

95.A lesson in Philippines history was presented by Mrs. Palaroan by making her class view a videotape on EDSA
revolution. The videotape which Mrs. Palaroan used in class points to what property of instructional media?

a. Manipulative c. Fixative
b. Multi-sensory d. Distributive

96.Read the following teacher-student- situation. Teacher: Why is the process called photosynthesis? Student: I don’t
know. Which questioning technique should the teacher use?

a. Concept c. Clarification
b. Prompting d. Multiple response
97.Miss Lee’s objective is to focus student’s attention on and quickly create interest in a problem or concept. She
makes use of:

a. Task group c. Tutorial group


b. Panel discussion d. Philips 66

98.The content of the lesson must be feasible. This means that:

a. The lesson should be interesting to the student


b. The lesson must be within the capacity of the students
c. The teacher must considered the time needed and resources available
d. The lesson must be useful

99.Which questioning behavior is appropriate:

a. Not allowing a student to complete a long response


b. Repeating all student’s response
c. Allowing choral responses
d. Asking varied questions

100.      Research on teacher effective practices has shown the following except:

a. Planning has title impact on student learning


b. Questioning strategies are ineffective monitoring technique
c. Teaching procedures on classroom routines early in the school year are essential
d. Directives should be few and best delivered in a casual manner

Answer Key:

51.     A
52.     A
53.     B
54.     D
55.     A
56.     B
57.     D
58.     C
59.     B
60.  D
61.  D
62.  A
63.  A
64.  B
65.  B
66.  A
67.  B
68.  B
69.  B
70.  A
71.  B
72.  D
73.  A
74.  C
75.  A
76.  D
77.  C
78.  D
79.  B
80.  B
81.  B
82.  D
83.  D
84.  A
85.  A
86.  A
87.  C
88.  A
89.  C
90.  D
91.  D
92.  B
93.  D
94.  A
95.  B
96.  C
97.  B
98.  B
99.  D
100.  C

101.Teacher Y wants to measure student’s ability to organize thoughts and ideas. Which type of test is most
appropriate?

a. Objectives test
b. Limited response essay
c. Extended response essay
d. Short answer type of test

102.Which of the following refers to organized services specially designed to improve the adjustment of students?

a. Group growth sessions c. Cumulative records


b. Test profiles d. Guidance program

103.Which tests survey existing problems of academic progress?

a. Achievement tests c. Problem checklist


b. Interest inventories d. Aptitude tests

104.Which of the following statements about guidance is NOT true?

a. Guidance is a continuous process


b. Guidance is concerned with “ Whole” student, not only with his or her intellectual and academic aspect
c. Guidance is telling a student what is best for him, and advising him about what he should do
d. Guidance is primary concerned with prevention rather than cure

105.Which of the following is NOT a guidance service?

a. Psychological testing
b. Counseling
c. Observation
d. Education placement

106.Which of the following tests is given at the end of instruction?

a. Summative c. formative test


b. Placement test d. diagnostic test

107.Special or exceptional students are such in certain aspects, Except for one:

a. Sensory abilities
b. Gender orientation
c. Mental characteristics
d. Communication abilities

108.Information from achievement tests can be used in the following, Except in:

a. Grading a student
b. Labeling pupils
c. Establishing proficiency
d. Giving feedback regarding effectiveness of learning

109.Which of the following occurs when the teachers have a general tendency to rate all individuals’ performance at
approximately the same position on the rating scale?

a. Personal bias error c. Logical error


b. Halo effect d. Severity error

110.The environment must be interactive to facilitate learning, which of the following situations is an example of this?

a. The class copies a list of facts concerning the habitat of insects


b. The teacher lectures on the habitat of insects
c. The class goes out and discovers the habitat of insects
d. The teacher shows posters of the habitat of insects

111.When teachers, administrators and counselors come together to synthesize or coordinate in interpreting data
about a student for the purpose of a more intensive study, this is called

a. Longitudinal study c. Case conference


b. Case study d. Case analysis

112.Which type of test can help teachers predict the probability of success of students in certain areas or endeavors?

a. Achievement c. Aptitude
b. Intelligence d. Personality

113.Which of the following correctly describes dyslexia?

a. A. It is a developmental expressive writing disorder


b. It is a developmental arithmetic disorder
c. It is a developmental reading disorder
d. It is a developmental articulation disorder

114.Free and compulsory education as mandated in the 1987 constitution holds true for_____________

a. Elementary and secondary education


b. Education in the primary grades
c. All children of school age
d. Grade VI pupils

115.Readiness pretests is give at the beginning of an instructional unit is a:

a. Summative tests c. Placement test


b. Formative test d. Diagnostic test

116.The distribution of the scores is negatively skewed. This means that:

a. Most of the score are low


b. Most of the scores are high
c. The score are concentrated on the left of the distribution curve and most of the score are low
d. The scores are concentrated on the left of the distribution curve
117.Aside from professional competence, what other important qualification must a teacher have for guidance work?

a. Similar cultural background with the students


b. Personal qualities, such as warmth, open-mindedness, interest in people
c. Affiliation with a professional group
d. Several year of experience as teacher

118.Which of the following is meant to determine student performance at the beginning of instruction?

a. Diagnostic assessment
b. Placement assessment
c. Summative assessment
d. Summative assessment

119.Mean is to central tendency as standard deviation is to:

a. Discrimination c. Correlation
b. Level of difficulty d. Variability

120.      The following characterize school guidance programs, Except one:

a. A part of every school activity


b. Separate from the general life of the school
c. A function sheared by all
d. Located in every part of the school

121.      Which factor is most likely to make a school’s guidance program succeed?

a. Administrative support
b. The availability of the specialist
c. The favorable attitude of teachers toward guidance
d. Located in every part of the school

122.      The results if this type of test serve as basis for remedial instruction. What is the type of test referred to?

a. Prognotic test c. Diagnostic test


b. Speed test d. Achievement test

123.      The difficulty index of a test item is 1. This implies that:

a. The test item must be moderate in difficulty because 50% got the item correctly and 50% got it wrongly
b. The test item must be very difficult because nobody got the item correctly
c. The test item must be neither difficult because nor easy because this depends on the ability of the
students
d. The test item must be very easy because everybody got the item correctly

124.      In his conduct of item analysis Mr. Milanes discovered that a significant greater number from the lower group of
the class that from the upper group got item number 10 correctly. This implies that:

a. The test item is reliable


b. The test item has a negative discriminating power
c. The item has a positive discrimination power
d. The tests item is valid

125.      Guidance is concerned with

a. All students, even if some seem not to have problems


b. Students who manifests undesirable behavior only
c. Underachieving students only
d. Students with emotional problems only

126.      Which statement explains the primary focus of the non-directive approach in counseling?

a. It focuses in the person’s responsibility and capacity to discover more appropriate behavior
b. It places confidence in a person’s ability to deal with his or her needs through a realistic process
c. it helps confidence to develop a more rational to deal with his or her needs through a realistic process
d. it modifies behavior by providing appropriate learning conditions and experiences

127.      An algebra test was designed to measure what the students learned at the end of the course. It was also to
predict success in future mathematics subjects. This algebra test functioned as-

a. Diagnostic and placement test


b. An aptitude test
c. An achievement test
d. An aptitude and an achievement test

128.      Giftedness is a form of exceptionality. Students who fall under this category demonstrate high performance in
special areas, such as those below-

a. Athletic prowess c. General intellectual ability


b. Creative thinking d. Visual and performing arts

129.      Which of the following is the least stable measure of central tendency?

a. Median and mode c. Median


b. Mode d. Mean

130.      Which assessment is concerned with identifying learning difficulties during instruction

a. Summative assessment
b. Placement assessment
c. Formative assessment
d. Visual and performing arts

131.      These are significant information about a student, gathered through the use of various techniques, assembled,
summarized and organized in such a way that they may be used effectively:

a. Cumulative records
b. Test profile
c. Personal inventories
d. Case studies

132.      Research findings show that autism is

a. Either more prevalent among girls or among boys depending on their nationality
b. Equally prevalent among boys and girls
c. More prevalent among boys than among girls
d. More prevalent among girls than among boys

133.      In interpreting test results which statements are true?


I.       A raw score itself is meaningless but becomes meaningful once it is interpreted
II.     An analysis of test results is useless without interpretation
III.   Test result interpretation is possible without analysis
IV.    The use of statistical techniques gives meaning to student’s scores
a.    I, II, IV c. I, III, IV
b.    I, II, III d. II, III, IV
134.      This guidance service to help students carry out their plans and act on their choices so that they become
adjusted to their chosen field or career is called:
a. Counseling c.placement
b. information d. follow-up

135.      The tendency for the rather to use only the lower end of rating scale in rating performance is referred to as

a. Personal bias error c. Generosity error


b. Severity error d. logical error

136.      What must be employed to continuously strengthen a school guidance program

a. Assignment of responsibilities
b. research
c. Evaluation
d. Program Planning

137.      Which tests are given before instruction?

a. Placement c. Summative
b. Formative d. Achievement

138.      The operation “Return to the basics” saw its embodiment in the _________
             a.      National Elementary Achievement Test
             b.      New Secondary Education Curriculum
              c.      National Secondary Achievement Test
             d.      New Elementary School Curriculum
139.      The following are computed difficulty indices: 1, . 80, .50, .30 which one indicates the most difficulty item?
a.    30 b. 50 c. 1 d. 80
140.      Which of the following explains why the plateau phenomenon of ten found in human being does NOT exist in
animal learning experiments?
a.    Animals are unable to profit from errors
b.    The incentive in animal learning is constant
c.    Animals have weak incentive in learning
d.    Animals are unable to spurt in learning
141.      All are components of remedial guidance in the classroom, Except:
a.    Dealing with discipline problems
b.    Dealing with poor study habits and skills
c.    Dealing with underachievement
d.    Dealing with career choices or vocational plans
142.      Which method provides a more or less objective assessment of different aspects of an individual?
a.    Home visits
b.    Counseling interview
c.    Observations
d.    Standardized testing
143.      Which of the following behavior indicates that a child has developed conventional maturity? The behavior is
based on _________
a.    The desire to avoid severe physical punishment by a superior authority
b.    Personal decisions based on his satisfaction
c.    The expectations of the group or society in general to gain approval
d.    Internalized ideals to avoid self condemnation rather than social censure
144.      A school guidance program should arise mainly out of:
a.    The curriculum and other programs of the school
b.    The school administrator’s directives from his/her perceptions
c.    The students’ needs and the problems
d.    The availability of guidance resources in the schools
145.      A test in which the options are dependent upon a foundation of some sort such as graphical representation,
paragraph, and pictures is:
a.    Contained-options test
b.    Setting-and-options test
c.    Structured-response
d.    Stem-and-options test
146.      In the scoring of essay tests, experts advise teachers to score all answer to an easy question before going to the
next question for which reason?
a.    It reduce the chances that ratings will be influenced by a halo effect
b.    It is more convenient
c.    It is easier to apply the criteria more uniformly when considering only a single task at a time when going from task
to task for each student
d.    Checking is faster this way
147.      Whose performance does stanine of 5 represent?
a.    Performance of the upper 40%
b.    Performance of the upper 20%
c.    Performance of the lower 20%
d.    Performance of the middle 20%
148.      In making and reporting observations of students’ behavior, which ones should be avoided?
a.    Descriptions of student’s gestures and expressions
b.    The names of other students with whom the one being observed interact
c.    Opinions and judgment about students
d.    Direct quotations of student’s statements
149.      This is the stage when the learner become confused and starts to experience identity crisis. This is the
____________
a.    Late childhood stage
b.    Early childhood stage
c.    Early adulthood stage
d.    Adolescent stage
150.      All are basic functions of psychological tests EXCEPT ONE.
a.    For selection or admission
b.    Identification of students who need special attention
c.    For promotion or acceleration
d.    Integration of life experiences and future directions of students

Answer Key:
101.     C
102.     D
103.     C
104.     C
105.     C
106.     A
107.     B
108.     C
109.     C
110.  C
111.  C
112.  C
113.  C
114.  C
115.  D
116.  C
117.  B
118.  B
119.  D
120.  B
121.  C
122.  A
123.  D
124.  D
125.  A
126.  C
127.  D
128.  C
129.  B
130.  A
131.  C
132.  C
133.  A
134.  C
135.  B
136.  A
137.  A
138.  D
139.  D
140.  B
141.  D
142.  C
143.  C
144.  C
145.  B
146.  A
147.  B
148.  C
149.  D
150.  D
151.      Upon what shall a teacher base the evaluation of the learner’s scholarship
a.    Merit, learner’s scholarship
b.    Merit, quality of performance
c.    Merit, attendance
d. Quality of performance, learner’s scholarship
152.      The following are some of what a professional teachers may NOT do EXCEPT:
a.   Accept remuneration from tutorials other than what is authorized for such service
b.   Make deductions from student’s scholastic ratings as a punishment
c.   Base the evaluation of the learner’s work on merit and quality of academic performance
d.   Inflict corporal punishment an offending learners
153.      The value of education, deference to authority, and filial piety in the Philippines in an influenced of
a.    Taoist tradition c. Buddhist tradition
b.    Confucianist tradition d. Hindu tradition
154.      Which statement regarding teacher’s ethical behavior is wrong?
a.    A teacher may not fall in love with his/her students
b.    Teachers shall support one another at all times when the best interest of the learners, the school, or the profession
at stake
c.    A teachers may submit to the proper authorities any justifiable criticisms against an associate
d.    It is every teacher’s responsibility to seek correctives for any unprofessional and unethical conduct of any associate
155.      Which is the first and foremost concern of a teacher? The interest and welfare of:
a.    Learners
b.    Colleagues
c.    Parents
d.    The teaching profession
156.      The ideal society in the context of Plato is one where:
a.    There is hope c. Justice reigns
b.    Love prevails d.Faith matters
157.      The survivors of a shipwreck are now packed in a lifeboat which may capsize unless partly deloaded. Which of
the following acts will be morally justifiable?
a.  Pushing the young and old ones and leaving them behind to drown
b.  Trying to save everyone as much as possible
c.  Shooting some passengers and leaving them behind
d.  Doing nothing and waiting for the boat to capsize
158.      For the advocates of value clarification, values are
a.    Universal c. Objective
b.    Personal d.Unchanging
159.      With regard to business, which does the Code of Ethics NOT say about teachers?
a.   A teacher shall maintain a good reputation with respect to debts loans and other financial matters
b.   No teachers shall be financially interested in any commercial venture involving textbooks and other school
commodities where he/she can exercise official influence
c.   no teacher shall act as agent of textbooks and other school commodities where he/she can exercise official
influence
d.   a teacher may not engage in any kind of business
160.      The more consumeristics a person is, the more he/she gets attached to this material world, the farther he is
from the universal self, the more miserable he/she becomes. This is a teaching
a.    Confucianism c. Hinduism
b.    Taoism d. Buddhism
161.      Who believed that there should be less reliance on the school, and the world of work?
a.    Paolo Freire c. Paul Goodman
b.    Ivan Illich d. Jean Paul Sartre
162.      Human dignity is inherent in every person. This means that
a.   The inner worth of a person depends on what he has
b.   The inner worth of a person depends on how he/she looks
c.   The inner worth of a person depends on what he can do
d.   The inner worth of a person is in him/her she looks
163.      Which of the following runs counter to the development of honesty and accountability?
a.    Being vigilant about weights and measures used in the markets
b.    Equal payment of government employee’s productivity pay
c.    Payment of just wages to workers and employees
d.    Transparent in operations of the barangay and other organizations through public reporting
164.      Which are said to be our wellspring of excellence?
a.    Third largest English-speaking nation, tiger economy, inventor of people’s power revolution
b.    Tiger economy, superior human resources
c.    Tiger largest English-speaking nation, superior human resources, inventor of people’s power revolution
d.    Tiger economy, the inventor of people’s power revolution, superior, human resources
165.      Which among these goals for change was proposed by the moral Recovery Program?
a.    A sense of justice and outrage over its violation, a sense of national pride, a sense of seriousness
b.    A sense of national guide, a sense of seriousness, sense of common good
c.    A sense of national guide, a sense of seriousness, sense of national justice and outrage over its violation
d.    A sense of national guide, a sense of the common good, a sense of justice and outrage over its violation
166.      The code of ethics for teachers states that school officials shall encourage and attend to the professional growth
of all teachers. According to the Code school officials can do this by:
a.  Giving them due recognition for meritorious performance by allowing them to participate in conferences and
training programs ecommending teachers for promotion
b.  Recommending teachers for promotion, organizing teachers in a professional organization
c.  Allowing them to participate in conferences and training programs
d.  Organizing teachers into a professional organization, giving them due recognition for meritorious performance and
recommending teachers for promotion
167.      Which thrust on value formation is intended to help the students identify and be aware of their values?
a.    Analysis c. Value inculcation
b.    Value clarification d.Moral development
168.      The idea that there should be less reliance on the school and greater use of education potential of the
community and the world of work is the essence of:
a.    Community-based education
b.    Formal education
c.    Multi-grade teaching
d.    Mobile teaching
169.      It is said that the big drama development is played out in small scenes. Which one does not serve as a further
explanation of this statement?
a.   Ordinary things done by ordinary people are the solid building blocks on which a nation develops
b.   Details can make a significant difference
c.   Much of our personal development and progress of the nation depends on how extraordinary well we do the
ordinary things
d.   Let us give attention to major things because they are the ones that matter
170.      Which of the following characteristics does not apply to Plato’s just society?
a.    The artists are creative
b.    The leaders are wise
c.    The soldiers are courageous
d.    The working class are temperant
171.      Martin Buber’s “ dialogic principle” requires as restructuring of society which can be achieved by:
a.   Communism which cannot tolerate multiplicity and freedom
b.   Fascism which cannot allow dialogue
c.   Democracy which requires centralistic political set up
d.   Democratic socialism which advocates a genuine dialogue life
172.      Which of the following statements about society is TRUE?
a.    Revolution produce the opposite of their goals
b.    Ethnic cleansing leads to social unanimity
c.    “ Social education “ leads to a mature society
d.    Cooperative education is a means to Social education?
173.      Which of the following statements defines ethics?
a.    The science of correct thinking
b.    The study of being a general
c.    The study of the nature of human knowledge
d.    The study of rightness or wrongness of human knowledge
174.      Which of the following dose not characterize fascism?
a.    A government of laws and not of men
b.    Rigid hierarchy and authority of single leader
c.    Militarist, nationalist and dictatorial regime
d.    It justifies any of its means by its end
175.      Which of the following is not Anarchism?
a.  Freedom and responsibility
b.  Social organization without hierarchy
c.  Extreme view of individual freedom
d.  Destruction of the state and its supporting institution
176.      In which of the following acts would you attribute responsibility?
a.  A barkeeper sells liquor to a minor not knowing he is an adult
b.  A barkeeper sells liquor to a minor not knowing it is illegal to do so
c.  A barkeeper sells liquor to a client not knowing it is contaminated with poison
d.  A barkeeper sells liquor not knowing it would cause the death of a diabetic person
177.      The values of the people are revealed in the following approaches, Except One
a.   When they are asked to choose freely among alternative
b.   When they act out what they choose but for a short while and not repeatedly
c.   When they act out what they choose
d.   When they are asked to choose among alternatives
178.      To attain harmony in society people must rule their subjects with benevolence . whose teaching was this?
a.    Mohammad c. Buddha
b.    Kung-fu-zu d.Lao Tzu
179.      The concept that school failed to encourage positive learning and were opposed to the production of
independently-minded adults was popularized by:
a.    Paolo Freire c. Paul Goodman
b.    John Dewey d. Ivan Illich
180.      Which type of conscience is possessed by one who is extremely rigorous and is constantly afraid of committing
evil?
a.    Scrupulous c. Certain conscience
b.    Lax conscience d.Certain conscience
181.      The following EXCEPT ONE are the strength of Filipino character
a.    Pakikipagkapwa-tao c. Frankness
b.    Family orientation d.Joy of humor
182.      Which of the following is the Aristotlelian definition of happiness?
a.  One’s ultimate achievements in life
b.  A state made perfectly the aggregation of good things
c.  The satisfaction of all desires and the absence of pain
d.  The experience of the fulfillment of possibilities
183.      One of the following statements about passion is NOT TRUE?
a.   Passion does not affect voluntariness
b.   Passion may completely destroy freedom
c.   passion tends to blind the judgment of the intellect
d.   passion cannot affect freedom
184.      Which does not characterize a non-violent society?
a.    Solves problems in non-adversarial way
b.    Prefers indigenous ways of solving problems
c.    A passive society
d.    Solves problems in a non-confrontational way
185.      Which statement holds true of values clarification?
a.  Values are objective; no person has the right set of values to pass on to others
b.  It is meant to help students get at their own feelings, ideas, and belief’s; no person has the right set of values to
pass on to others
c.  No person has the right set of values to pass on to others; values are independent of time
d.  Values are independent of time, place, and persons; meant to help students get at their own feelings and ideas
186.      Based on the Code of Ethics for teachers, which of the following may teachers Not do?
a.   Welcomes the opportunity to lead in barangay
b.   Studies and understands local customs and traditions
c.   Deducts points from students scholastic ratings as a form of punishment
d.   Attends church and worship of his choice joy and humor
187.      Which type of justice implies the duty of one individual to give another what he is due?
a.    International justice
b.    Distributive justice
c.    Social justice
d.    Commutative justice
188.      It is said that our level of achievement is always lower than our level of aspiration. This suggests that:
a.   We should aim high
b.   We should not be over ambitious
c.   We should think and act local
d.   We should set over aspirations because it is damaging if we cannot realize them
189.      John Dewey advocates which of the following?
a.   Something is true if it works
b.   Morality is for persons
c.   Everything in this world is tentative, a series of means and ends
d.   What is right or wrong depends on the situation
190.      Which is NOT the goal of true authority?
a.  To bring out the best in others
b.  To set the examples for others
c.  To coerce others to do what people in authority desire them to do
d.  To help, from and guide others
191.      In moral life, responsibility depends on one’s knowingly and freely doing an act. In which of the following
situations would you attribute responsibility?
a.  A nurse administers a medicine despite strong doubts about it
b.  A four year old fires a loaded gun killing his own father
c.  A person unknowingly passes counterfeit money
d.  An insane person strikes someone’s head with a hammer
192.      “ Life is what you make it”. What you become is up to you. This thought comes from the:
a.    Existentialist c. Idealist
b.    Realist d. Philosophical analyst
193.      According to Plato, the good form of government is
a.    Oligarchy c. emocracy
b.    Aristocracy d. Monarchy
194.      Which of the following ways of life refer to Taoism?
a.    Practice an all-embracing love
b.    Practice the golden rule
c.    Love one another
d.    Be natural
195.      “ Without a vision, a people perish.” Says the Book of Proverbs. This shows how important a vision is. What
does a vision Not do to a person’s life?
a.   It guides him/her to follow the most efficient route
b.   It dilutes his/her attention
c.    It keeps him/her focused on one direction
d.    It keeps him/her inspired
196.      Which lie is the worst of all lies?
a.    An officious lie c. A malicious lie
b.    A material lie d.A jocose lie
197.      Which of the following does not refer to existentialism?
a.    Each man shapes his own life and destiny
b.    Existence means self-choice and self-determination
c.    Existence means creativity and uniqueness
d.    Social existence means anarchy and violence
198.      Which is ethical for a teacher to do in a situation where she/he falls in love with a student or when a student falls
in love with his/her teacher?
a.  The teacher avoids that learner
b.  The teacher exercise discretion to avoid scandal and gossip and preferential treatment of that learned
c.  The teacher resigns from his/her teaching job
d.  The teacher advises the student to discontinue with his/her studies
199.      Which of the following statements is NOT TRUE of an ideal social principle?
a.  Restructure society to crates a union of communities
b.  Advocate direct life-relations between men and men
c.  De-emphasize centralization of power
d.  Advocate an ethics of manipulation
200.      If a ruler wants to maintain himself in power he must govern by moral power. Upon whose teaching is this
based?
a.    Kung-fu-tzu c. Buddha
b.    Lao Tzu d. Mohammad

Answer Key:

151.     B
152.     C
153.     B
154.     A
155.     A
156.     C
157.     B
158.     B
159.     D
160.  C
161.  C
162.  A
163.  B
164.  A
165.  C
166.  A
167.  B
168.  A
169.  D
170.  D
171.  D
172.  C
173.  D
174.  A
175.  A
176.  C
177.  B
178.  D
179.  A
180.  A
181.  C
182.  C
183.  A
184.  C
185.  B
186.  C
187.  C
188.  A
189.  C
190.  C
191.  A
192.  A
193.  B
194.  D
195.  B
196.  D
197.  D
198.  B
199.  C
200.  C

Professional Education - Drill 10 - Part 1

- December 09, 2018


1. You are very much interested in a quality professional development program for teachers. What characteristic should you look for?
a. Prescribe by top educational teachers
b. Dependent on the availability of funds
c. Required for renewal of professional license
d. Responsive to identified teacher’s needs.

2. To ensure high standards of teachers’ personal and professional development, which of the following measures must be implemented?
I. A school head plans the professional development of his/her teachers.
II. Every teacher formulates his/her own professional development plan
III. The implementation of what is leaned in a training must be monitored.
a. I only       c. II and III
b. I and III               d. II only

3. As a community leader, which of the following should a teacher NOT do?


a. Support effort of the community to improve their status in life.
b. Make herself aloof to ensure that her decisions will not be influenced bu the community politics.
c. Solicit donation from philanthropists in the community.
d. Play an active part in the activities of the community.

4. In a highly pluralistic society, what type of learning environment is the responsibility of the teacher?
I. Safe
II. Gender-biased
III. Secure
a. I and II         c. II only
b. I, II and III d. I and III

5. A teacher is said to be “trustee of the cultural and educational heritage of the nation and is under obligation to transmit to learners such
heritage”. Which practice makes the teacher fulfill such obligation?
a. Use interactive teaching strategies.
b. Use the latest educational technology.
c. Observe continuing professional education
d. As a class, study the life of Filipino heroes.

6. Which actions show that political factors affect schools as agents of change?
I. The strengthening of the teaching of English in Philippines school.
II. The introduction of mandated subjects such as Rizal in the curriculum
III. The practice of mainstreaming
IV. The turnover of day care centers for DSWD to DepEd for supervision.

a. I and III c. II and III


b. I and II d. II and IV

7. For more efficient and effective management of school as agents of change, one proposal is for the DepEd to cluster remote stand-alone
schools under one lead school head. Which factor has the strongest influence on this proposal?
a. Psychological c. Geographical
b. Historical d. Social

8. What does the acronym EFA imply for schools?


a. The acceptance of exclusive schools for boys and for girls.
b. The stress on the superiority of formal education over that of alternative learning system.
c. Practice of inclusive education
d. The concentration on formal education system

9. The wide acceptance of “bottom up” management style has influenced schools to practice which management practice?
a. Exclusion of politicians from the pool of guest speakers during graduation exercises.
b. Prescription of what ought to be done from the Center Office.
c. Involvement of students, parents, teachers, and community in school planning
d. Allowing schools to do what they think is best

10. Large class size in congested cities is a common problem in our public schools. Which measure/s have schools taken to offset the effects of
large class?
I. The deployment of more teachers
II. The implementation of 1:1 pupil textbook ratio
III. The conduct of morning and afternoon sessions

a. I, II and III c. III only


b. I and II         d. II only

11. The failure of independent study with most Filipino students may be attributed to students’
a. unpreparedness fro schooling
b. ambivalence
c. high degree of independence
d. high degree of independence on authority

12. The schooling incidents in school campuses abroad have made school to rethink the curriculum. Which is believed to counteract such
incidents and so is being introduced in schools?
I. Inclusion of socio-emotional teaching
II. The emphasis on the concept of competition against self and not against others
III. Focus on academic achievement and productivity

a. I and III c. I and II


b. II and III d. I, II and III

13. Widespread abuse of Mother Earth prompted schools to teach sustainable development. which one does this prove about schools?
a. The curricula of schools are centered on Mother Earth.
b. Schools can easily integrate sustained development in their curriculum.
c. Sustained development cannot be effectively taught in the classroom.
d. Environment factors influence the school as an agent of change.

14. A father tells his daughter “You are a woman. You are meant for the home and so for you, going to school is not necessary.” Is the father
CORRECT?
a. It depends on the place when the daughter and the father live.
b. Yes, women are meant to be a mother only.
c. No. today women can take on the jobs of men.
d. No, there is gender equality in education.

15. Is there a legal basis for increasing the teacher’s starting salary to P18,000 an months?
a. No, it is a gift to teachers from Congress
b. Yes, R.A 7836
c. No, it is simply an act of benevolence from President GMA
d. Yes, the Phil. Constitution

16. As provided for the Educational Act of 1982, how are the institutions of learning encourage to set higher standards of equality over the
minimum standards required for state recognition?
a. Granting of Special Permit
b. Academic freedom
c. Continuing Professional Education
d. Voluntary accreditation

17. Despite of opposition from some school official, DepEd has continuously enforced the “no collection of fees” policy during enrolment period
in public schools. In this policy in accordance with EFA goals?
a. No, it violates the mandate of equality education
b. Yes, it somewhat eliminates gender disparities
c. Yes, it supports equitable access to basic education
d. No. it does not support parent  of adult education

18. “Specialization is knowing more and more about less and less”. Hence, it is better to be a generalist, claims Teacher F. Which Philosophy
does Teacher F. subscribe to?
a. Existentialism c. Essentialism
b. Perennialism d. Progressivism

19. Mencius believed that all people are born good. This thought on the innate goodness of people makes it easier to our pupils.
a. teach c. like
b. Respect         d. motivate

20. The specialization requires of every professional teacher for him/her to be competent is in line with which pillar of learning?
a. Learning to know
b. Learning to be
c. Learning to live together
d. Learning to do

21. Which pillar of learning is aimed at the wholistic development of man and his complete fulfillment?
a. Learning to be
b. Learning to know
c. Learning to live together
d. Learning to do

22. Material development at the expense of human development points to the need to do more in school.
a. “Learning to do”
b. “Learning to know”
c. “Learning to live together”
d. “Learning to be”

23. A student complains to you about his failing grade. When you recomputed you found out that you committed an error in this grade
computation. Your decision is not accept the erroneous computation before the student and so leave the failing grade as is for fear that you may
lose credibility. Is this morally right?
a. No, the reason for not accepting the error before the students in flimsy.
b. No, the end does not justify the means
c. Yes, the end justifies the means
d. Yes, as a teacher you must maintain your credibility

24. Which violate(s) the principle of respect?


I. Teacher A tells her students that what Teacher B taught is wrong.
II. To retaliate, Teacher B advises students not to enroll in Teacher A’s class.
III. Teacher C secretly gives way to a special favor (e.g. add 2 points to grade) requested by student A who is vying for honors.
a. II and III         c. I and II
b. I, II and III d. I and III

25. Which is/are in accordance with the principle of pedagogical competence?


I. Communication of objectives of the course to students
II. Awareness of alternative instruction strategies
III. Selection of appropriate methods of instruction
a. I and III            c. III only
b. I, II and III d. II and III

26. With a death threat over his head, Teacher D is directed to pass an undeserving student. Which will a utilitarianist do?
a. Pass the student, why suffer the threat?
b. Don’t pass him; live by your principle of justice. You will get reward, if not in this life, in the next.
c. Pass the student. That will be of use to the student, his parents and you.
d. Don’t pass him. You surely will not like someone to give you a death threat in other to pass.

27. Teacher A knows of the illegal activities of a neighbor but keeps quiet in order not to be involved in any investigation. Which foundational
principle of morality does Teacher A fail to apply?
a. The end does not justify the means
b. The end justifies the means
c. Always do what is right
d. Between two evils, do the lesser evil

28. To earn units for promotion, a teacher pays her fee but does not attend class at all. Does this constitute professional growth?
a. Not immediately but yes after promotion
b. It depends on the school she is enrolled in
c. No, it is simply earning MA units for promotion
d. Yes, just enrolling in an MA program is already professional growth

29. If a teacher asks more higher-order questions, he has to ask more questions.
a. fact         c. convergent
b. close d. concept

30. Misdemeanor has a “ripple effect”. This implies that as a classroom manager, a teacher
a. reinforces positive behavior
b. responds to misbehavior promptly
c. is consistent in her classroom management practice
d. count 1 to 10 before she deals with a misbehaving student

31. Based on Edgar Dale’s “Cone of Experience”, which activity is farthest from the real thing?
a. Watching demo c. Video disc
b. Attending exhibit d. Viewing images

32. The students of Teacher Y scan an electronic encyclopedia, view a film on subject, or look at related topics at the touch of a button right there
in the classroom. Which device/s des teacher Y’s class have?
a. Teaching machines
b. CD
c. Video disc
d. Videotaped lesson

33. Which is an INAPPROPIATE way to manage off-task behavior?


a. Redirect a child’s attention to task and check his progress to make sure he is continuing work
b. Make eye contact to communicate what you wish to communicate
c. Move closer to the child to make him feel your presence
d. Stop your class activity to correct a child who is no longer on task

34. To be an effective classroom manager, a teacher must be friendly but must at the same time be .
a. confident c. analytical
b. businesslike d. buddy-buddy

35. Which software is needed when one wants to perform automatic calculations on numerical data?
a. Database
b. Spreadsheet Program
c. Microsoft Word
d. Microsoft Powerpoint

36. Which of the following questions must be considered in evaluating teacher-made materials?
a. In the material new?
b. Does the material simulate individualism?
c. Is the material expensive?

37. Kounin claims that “with-it-ness” is one of the characteristics of an effective classroom manager. What is one sign of “with-it-ness”?
a. Giving attention to students who are having difficulty with school work
b. Seeing only a portion of the class but intensively
c. Knowing where instructional materials are kept
d. Aware of what’s happening in all parts of the classroom
38. Which of these is one of the ways by which the internet enables people to browse documents connected by hypertext links?
a. URL
b. Browser
c. Welcome page
d. World Wide Web

39. Which characteristics must be primary considered as a choice of instructional aids?


a. Stimulate and maintain students interests
b. Suited to the lesson objectives
c. Updated and relevant to Filipino setting
d. New and skillfully made

40. You can exhibit referent power on the first day of school by
a. telling them the importance of good grades
b. giving your students a sense of belongingness and acceptance
c. making them feel you know what you are taking about
d. reminding your students your authority over them again and again

41. I would like to use a model to emphasize particular part. Which of these would be MOST appropriate?
a. Regalia         c. Stimulation
b. Audio recording d. Mock up

42. What must a teacher do to ensure orderly transitions between activities?


a. Allow time for the students to socialize in between activities
b. Have the materials ready at the start of the activity
c. Assign fewer exercise to fill the allotted time
d. Wait for students who lag behind

43. The task of setting up routine activities for effective classroom management is a task that a teacher should undertake
a. as soon as the students have adjusted to their schedules
b. on the very first day of school
c. every day at the start of the session
d. every homeroom day

44. Teacher S uses the low-profile classroom control technique most of the time. what does this imply?
a. She is reactive in her disciplinary orientation
b. She manages pupils personalities
c. She reacts severely to a misbehaving student
d. She stops misbehaving without disrupting lesson flow

45. When teacher tries to elicit clarification on a student response or solicits additional information, which of these should be use?
a. Directing c. Structuring
b. Probing         d. Cross examining

46. Which priority criterion should guide a teacher in the choice of instructional devices?
a. Novelty c. Attractiveness
b. Cost d. Appropriateness

47. Which learning activity is MOST appropriate if a teacher’s focus is attitudinal change?
a. Fieldtrip         c. Role play
b. Exhibit d. Game

48. Teacher H strives to draw participation of every student into her classroom discussion. Which of these student needs is she trying to address?
The need to .
a. shoe one’s oral abilities to the rest of the class
b. feel significant and be part of a group
c. get everything and be part of a group
d. be creative

49. Instead of teacher giving this comment a student response. “You’re on the wrong track!”, what should be teacher do?
a. Change the question to an easier one
b. Redirect the question by calling another student to recite
c. Probe to redirect the response into a more productive area
d. Pause, ask the question, lecture, then ask the question again

50. If curriculum is designed following the traditional approach, which feature(s) apply(ies)?
I. The aims of the curriculum are set by professionals and experts
II. Interested groups (teachers, students, communities) are assumed to agree with the aims of the curriculum
III. Consensus building in not necessary
a. III only c. I, II
b. I, II, III d. I, III

Answer Key:
1C
2B
3C
4D
5D
6D
7C
8C
9C
10 A
11 D
12 D
13 B
14 D
15 D
16 C
17 C
18 C
19 A
20 A
21 A
22 A
23 B
24 B
25 B
26 C
27 C
28 C
29 D
30 B
31 D
32 A
33 D
34 B
35 B
36 C
37 D
38 A
39 B
40 C
41 D
42 B
43 C
44 A
45 B
46 D
47 C
48 B
49 B
50 B

Professional Education - Drill 10 - Part 2

- December 09, 2018


51. I want my student to develop the ability to look at a problem from various perspectives. Which approach will be MOST fit?
a. Behaviorist approach
b. Computer-based Education
c. Modular approach
d. Cognitive approach

52. One’s approach to teaching is influenced by Howard Gardner’s MI Theory. What is he/she challenged to do?
I. To come up with 9 different ways of approaching  lesson to cater to the 9 multiple intelligence
II. To develop all student’s skill in all nine intelligences
III. To provide worthwhile activities that acknowledge individual difference in children
a. I, II and III c. II only
b. II, III d. III only

53. If my approach to my lesson is behaviorist, what features will dominate my lesson?


I. Copying notes III. Lecturing
II. Reasoning IV. Demonstration

a. III, IV c. I, II, III, IV


b. I, III, IV         d. II, III, IV

54. You practice inclusive education. Which of these applies to you?


I. You accept every student as full and valued member of the class and school community
II. Your special attention is on learners with specific learning or social needs
III. Your address the needs of the class as a whole within the context of the learners with specific learning or social needs

a. II only c. I only
b. I and II d. I and III

55. School curriculum reflects the world’s economic and political integration and industrialization. What does this point in curriculum
development?
a. The trend towards the classical approach to curriculum development
b. The trend toward the globalization and localization
c. The trend toward participatory curriculum development
d. The shift in the paradigm of curriculum development from a process-oriented to a product-oriented one

56. You choose cooperative learning as a teaching approach. What thought is impressed on your students?
a. Interaction is a must, but not necessarily face to-face interaction
b. Student’s success depends on the success of the group
c. Student’s individuality evaluate how effectively their group worked.
d. The accountability for learning is on the group not on the individual

57. What principle is violated by overusing the chalkboard, as though it is the only education technology available?
a. Isolated use c. Variety
b. Flexibility d. Uniformity

58. Which statement applies a CORRECTLY to Edgar Dale’s ”Cone of Experience”


a. The farther you are from the base, the more direct the learning experience becomes.
b. The farther you are from the bottom, the more direct the learning experience becomes.
c. The closer you are from the base, the more indirect the learning experience becomes
d. The closer you are from the base, the more direct the learning experience becomes

59. “When more senses are stimulated, teaching and learning become more effective.” What is an application of this principle?
a. Appeal to student’s sense of imagination
b. Use multisensory aids
c. Make your students touch the instructional material
d. Use audiovisual aids because the eyes and the eras are the most important senses in learning.

60. Which is a classroom application of the theory of “operant conditioning”?


a. Help student see the connectedness of facts, concepts, and principles
b. Create a classroom atmosphere that elicits relaxation
c. Reinforce a good behavior to increase the likelihood that the learner will repeat the response
d. Make students learn by operating manipulatives

Read the following teacher-student situation


61. TEACHER: Why is the process called photosynthesis?
STUDENT: I don’t know
Which questioning technique should be the teacher use?

a. Clarification c. Prompting
b. Multiple response d. Concept review

62. Here is the test item.


“From the data presented in the table, from generalizations that are supported by the data”.
Under what type of question does this item fall?

a. Convergent c. Application
b. Evaluative d. Divergent

63. I want to teach concepts, patterns and abstractions. Which method will be MOST appropriate?
a. Discovery c. Direct instruction
b. Indirect instruction d. Problem solving

64. Teacher A teaches English as a Second Language. She uses vocabulary cards, fill-in-the-blanks sentences, dialogues, dictation and writing
exercises in teaching a lesson about grocery shopping. Based on this information, which of the following is a valid conclusion?
a. The teacher wants to make her teaching easier by having les talk
b. The teacher emphasizing reading and writing skills
c. The teacher is teaching in a variety of ways because not all students learn in the same manner
d. The teacher is applying Bloom’s hierarchy of cognitive learning

65. Teacher A an experienced teacher, does daily review of past lessons in order to    
a. introduce a new lesson
b. reflect on how he presented the previous lessons
c. provide his pupils with a sense of continuity
d. determine who among his pupils are studying

66. I combined several subject areas in order to focus on a single concept for interdisciplinary teaching. Which strategy did I use?
a. Reading-writing activity
b. Thematic introduction
c. Unit method
d. Problem-centered learning

67. To teach the democratic process to the pupils, Biag Elementary School decided that the election of class officers shall be patterned after local
elections. There are qualifications set for candidates, limited period for campaign and rules for posting campaign materials, etc. Which of the
following did the school use?
a. Symposium c. Pole playing
b. Simulation d. Philips 66

68. Which are effective methods in teaching student critical reading skills?
I. Interpret editorial
II. Read and interpret three different movie reviews
III. Read a position paper and deduce underlying assumptions of the position papers

a. II and III c. I and II


b. I and III d. I, II and III

69. Here is a test item


“The improvement of basic education should be the top priority of the Philippine government. Defend or refute this position”.
Under what type of question does this test item fall?

a. Low-level c. Analysis
b. Evaluative d. Convergent

70. When I teach, I often engage in brainstorming. Which do I avoid?


a. Break down barriers
b. Increase creativity
c. Generate many ideas
d. Selectively involves pupils

71. Teacher S teaches a lesson in which students must recognize that ¼ is the same 0.25. They use this relationship to determine that 0.15 and
0.20 are slightly less than ¼. Which of the following concept/s is/are being taught?
a. Numeration skills
b. Place value of decimals
c. Numeration skills of decimals and relationships between fractions and decimals
d. Relationship between fraction and decimals

72. What is the best way to develop math concept?


a. Solving problems using multiple approaches
b. Solving problems by looking for correct answer
c. Learning math as applied to situations, such as being a tool of science
d. solving problems by applying learned formulas

73. After the reading of a selection in the class, which of these activities can enhance students creativity.
I. Reader’s theater
II. Reading aloud
III. Silent reading

a. I and II         c. I only
b. II only d. III only

74. Teacher C, a Reading teacher, advised he class to “read between the lines”. What does she want her pupils to do?
a. Make an educated guess
b. Determine what is meant by what is stated
c. Apply the information read
d. Describe the characters in the story

75. To nurture students’ creativity, which activity should a teacher AVOID?


a. Ask “hat if…” questions
b. Ask divergent thinking questions
c. Emphasize  the need to give right answers
d. Be open to “out-of-this-world” ideas

76. Teacher R wants to develop his student’s creativity. Which type of questions will be MOST appropriate?
a. Synthesis questions
b. Fact questions
c. “What if….” questions
d. Analysis questions

77. In my attempt to develop creative thinking skills, I want to test fluency of ideas. Which activity for my students will be MOST appropriate?
a. Solve this math problem
b. List animals  covered with hair in 1 minute
c. Solve this puzzle
d. Compare pictures 1 and 2. Where are the differences?

78. You want your students to answer the questions at the end of a reading lesson. “What did I learn did?”,”What still puzzle me?”, “What did I
enjoy, hate accomplish in the class today”?,”How did I learn from the lesson?”.Which of the following are you asking them to do?
a. Work on an assignment
b. Make journal entry
c. Work on a drill
d. Apply what they learned

79. After reading an essay. Teacher B wants to help sharpen her students’ ability to interpret. Which of these activities will be most appropriate?
a. Drawing conclusions
b. Making inferences
c. Getting the main idea
d. Listing facts separately from opinion

Read the following then answer the question


80. A man and his son are driving in a car. The car crashes into a tree, killing the father and seriously injury his son. At the hospital, the boy
needs to have surgery. Looking at the boy, the doctor says (telling the truth),”I cannot operate on him. He is my son. How can this be?
ASWER: The doctor is the boy’s mother.
The above brain twister helps develop critical reading skills. Which activity was used?

a. Comparing
b. Classifying
c. Inferring meaning
d. Looking for cause and effect

81. Research says that mastery experiences increase confidence and willingness to try similar or more challenging tasks as reading. What does
this imply for children’s reading performance?
a. Children who have not mastered the basic skills are more likely to be motivated to read in order to gain mastery over basic skills.
b. Children who have mastered basic skills are more likely to be less motivated to read because they get fed up with too much reading.
c. Children who have a high sense of self-confidence are not necessarily those who can read
d. Children who have gained mastery over basic skills are more motivated to read

82. The value that students put on reading is critical to their success. In what way/s can teachers inculcate his value for reading?
I. Sharing the excitement of read-aloud
II. Showing their passion for reading
III. Being rewarded to demonstrate the value of reading

a. II and III c. I, II and III


b. I and II d. II only

83. Bruner’s theory on intellectual development moves from enactive to iconic and symbolic stages. Applying Bruner’s theory. How would you
teach?
a. Be interactive in approach
b. Begin with the abstract
c. Begin with the concrete
d. do direct instruction

84. A person who has painful experiences at the dentist’s office may become fearful at the mere sight of the dentist’s office building. Which
theory can explain this?
a. Generalization
b. Operant Conditioning
c. Attribution theory
d. Classical conditioning

85. Which is/are the basic assumption/s of behaviorists?


I. The mind of newborn child is a blank state.
II. all behaviors are determined by environmental events
III. The child has a certain degree of freedom not to allow himself to be shaped by his environment.

a. III only c. II only


b. I and II d. I and II

86. If a student is encourage to develop himself to the fullest and must satisfy his hierarchy of needs, the highest needs to satisfy according to
Maslow is         .
a. psychological need c. belongingness
b. self-actualization         d. safety needs

87. In a Social studies class. Teacher I presents a morally ambiguous situation and asks student what they would do. On whose theory is Teacher
I’s technique based?
a. Bandura         c. Kohberg
b. Piaget d. Bruner

88. Teacher F is convinced that whatever a student performs a desired behavior, provide reinforcement and soon the student learns to perform the
behavior on his own. On which principle is Teacher F’s conviction based?
a. Environmentalism c. Cognitivism
b. Behaviorism d. Constructivism

89. Bandura’s social learning theory, states that children often imitate those who
I. have substantial influence over their lives
II. belong their peer group
III. belong to other races
IV. are successful and seem admired

a. IV only c. I and II
b. I and IV d. II and IV

90. According to Erikson, what years are critical for the development of self-confidence?
a. High school years
b. College years
c. Preschool years
d. Elementary school years

91. Which of the following does NOT describe the development of children aged 11 to 13?
a. They exhibit increased objectivity in thinking
b. They shift from impulsivity to adaptive ability
c. Sex difference in IQ become more evident
d. They show abstract thinking and judgment

92. Teacher H begins a lesson on tumbling, demonstrating front and back somersaults in slow motion and physically guiding his students through
the correct movements. As his students become more skillful, he stands back from the man and gives verbal feedback about how to improve.
With Vygotsky’s theory in mind, what did Teacher H do?
a. Apprenticeship
b. Guided participation
c. Peer interaction
d. Scaffolding

93. What does Gagne’s hierarchy theory propose for effective instruction?
a. Be concerned with the socio-emotional climate in the classroom
b. Teach beginning with the concrete
c. Sequence instruction
d. Reward good behavior

94. William Glasser’s control theory states that behavior in inspired by what satisfies a person’s want at any given time. What then must a teacher
do to motivate students to learn?
a. Make teaching-leaning interactive
b. Avoid giving assignments
c. Organize a curriculum in a spiral manner
d. Make schoolwork relevant to students’ basic human needs.
95. Soc exhibit fear response to freely roaming dogs but does not show fear when a dog is on a leash or confined to a pen. Which conditioning
process is illustrated?
a. extinction c. acquisition
b. generalization d. discrimination

96. Based on Freud’s theory, which operate/s when a student strikes a classmates at the height of anger?
a. Ego c. Id and Ego interact
b. Id d. Superego

97. Bernadette enjoyed the roller coaster when he and her family went to Enchanted Kingdom. The mere sight of a roller coaster gets her excited.
Which theory explains Bernadette’s behavior?
a. Operant conditioning
b. Social learning theory
c. Attribution theory
d. Pavlovian conditioning

98. According to Frued, with which should one be concerned if he/she has to develop in the students a correct sense of right and wrong?
I. Super-ego    II. Ego         III. Id

a. I and II c. I
b. II d. III

99. When small children call animals “dog”, what process is illustrated on Piaget’s cognitive development theory?
a. reversion c. accommodation
b. assimilation d. conservation

100. Researchers found that when a child is engaged in a learning experience a number of areas of the brain are simultaneously activated. Which
of the following is/are implication/s of this research finding?
I. Make use of field trips, guest speakers
II. Do multicultural units of study
III. Stick to the “left brain and right brain” approach

a. I and III c. I and II


b. I only d. II only

Answer Key:
51 D
52 B
53 B
54 D
55 D
56 C
57 C
58 D
59 B
60 C
61 A
62 B
63 A
64 D
65 C
66 B
67 B
68 D
69 C
70 D
71 D
72 C
73 C
74 B
75 C
76 C
77 B
78 C
79 B
80 D
81 D
82 C
83 A
84 D
85 C
86 B
87 C
88 D
89 C
90 D
91 C
92 B
93 C
94 D
95 D
96 B
97 A
98 C
99 D
100 C

Professional Education - Drill 10 - Part 3

- December 09, 2018

101. Which appropriate teaching practice flows from this research finding on the brain: “The brain’s emotional center is tied into its ability to
learn”.
a. Establish the discipline of being judgmental in attitude
b. Come up with highly competitive games where winners will feel happy
c. Tell the students to participate in class activities or else won’t receive plus points in class recitation
d. Create a learning environment that encourages students to explore their feeling and ideas freely

102. Research on Piagetian tasks indicates that thinking becomes more logical and abstract as children reach the formal operations stage. What
is an educational implication of this finding?
a. Engage children in analogical reasoning as early as preschool to train them for higher order thinking skills (HOTS)
b. Learners who are not capable of logical reasoning from ages 8 to 11 lag behind in their cognitive development
c. Let children be children
d. Expect hypothetical reasoning for learners between 12 to 15 years of age

103. Research says: “People tend to attribute their successes to internal causes and their failures to external causes.”Based on this finding, what
should be taught to students for them to be genuinely motivated to succeed?
a. Tell them the research finding when applied will make them genuinely motivated
b. Convince them that genuine motivation is the only factor that matters for a person to succeed
c. Make them realize that failure is a part of life
d. Make them realize that both success and failure are more a function of internal causes.

104. Which characterize/s a learning environment that promotes fairness among learners of various cultures, family background and gender?
I. Inclusive
II. Exclusive
III. Gender-sensitive

a. I only c. I and III


b. III only         d. II and III

105. Which of the following steps should be completed first in planning an achievement test?
a. Define the instructional objective
b. Set up a table of specialization
c. Select the types of test items to use
d. Decide on the length of the test

106. The computed r fro scores in Math and Science in 0.92. What does this mean?
a. Math score is positive related to Science score
b. The higher the Math score, the lower the Science score
c. Math score is not in any way related to Science score
d. Science score is slightly related to math score

107. Which types of test is most appropriate if Teacher Y wants to measure student’s ability to organize thoughts and ideas?
a. Short answer type of test
b. Extended response essay
c. Modified alternative response
d. Limited response essay

108. With assessment of affective learning in mind, which does NOT belong to the group?
a. Cloze test c. Reflective writing
b. Moral dilemma d. Diary entry

109. I want to test student’s synthesizing skills. Which has the highest diagnostic value?
a. multiple choice test c. Essay test
b. Performance test d. Completion test

110.  Why is this test item poor?

 _______ is an example of a leafy vegetable.

I. The test item does not pose a problem to the examinee


II. There are variety of possible correct answer to this item
III. the language used in the question is not precise
IV. The blank is near the beginning of a sentence

A. I and III
B. II and IV
C. I and IV
D. I and II

111. What makes the multiple choice type of test poor?

What follows is a multiple choice type of test.


Some test items                   .

a.    are too difficult

b.    are objective

c.    are poorly constructed

d.    have multiple defensible answers

a. The options are not grammatically connected to the stem


b. The stem fails to present a problem
c. There are grammatical clues
d. The options are not parallel

112. If a teacher wants to measure her students’ ability to discriminate, which of these is an appropriate type of test item as implied by the
direction?
a. “Outline the Chapter on The Cell”.
b. “Summarize the lesson yesterday”.
c. “Group the following items according to shape”.
d. “State a set of principle that can explain the following events”.
113. A test item has a difficult index of 0.89 and a discrimination index of 0.44. What should the teacher do?
a. Reject the item
b. Retain the item
c. Make it a bonus item
d. Make it a bonus item and reject it

114. Which form of assessment is consistent with the saying “The proof of the pudding is in the eating”.
a. Contrived c. Traditional
b. Authentic d. Indirect

115. What is WRONG with this item?

Who is best admired for outstanding contribution to world peace?

a. Kissinger       c. Kennedy


b. Clinton          d. Mother Teresa

a. Item is overly specific


b. Content is trivial
c. Test item is option-based
d. There is a cue to the right answer

116. Student’s score were as follows: 82, 83, 84, 86, 88, 84, 83, 85. The score 86 is the.
a. mode c. median
b. average         d. mean

117. Which text form would you choose if you want to have a valid and reliable test based on the table below?
Test Form Validity Index Reliability Index

A .47 .68

B .87 .57

C .20 .86

D .40 .41

E .63 .07

Test Form Validity Index Reliability Index


A .47 .68
B .87 .57
C .20 .86
D .40 .41
E .63 .07

a. A only c. A and D
b. B only d. B and E

118. A mathematicians test was given to all Grade V pupils to determine the contestants for the Math Quiz Bee. Which statistical measure
should be used to identify the top 15?
a. Mean percentage score
b. Quartile Deviation
c. Percentile Rank
d. Percentage Score

119. Nellie’s score is within x±1 SD. To which of the following groups does she belong?
a. Below average
b. Average
c. Needs Improvement
d. Above average

120.

a. The median is a score of 80 and the range is 60.


b. The median is a score of 70 and the range is 60.
c. The median is a score of 80 and the range is 20.
d. The median is a score of 70 and the range is 20.

121. Which can be said of Arielle who obtained a score of 75 out of 100 items in a Grammar objective test?
a. She performed better than 25% of her classmates
b. She answered 75 items in the test correctly
c. Her rating is 75
d. She answered 75% of the test items correctly

122. The criterion of success in Teacher D’s objective is that “the pupils must be able to spell 90% of the words correctly”. Ana and 24 others in
the class spelled only 40 out of 50 words correctly while the rest scored 45 and above. This means that Teacher D
a. attained her lesson objective
b. did not attain her lesson objective because of the pupils’ lack of attention
c. failed to attain her lesson objective as far as the 25 pupils are concerned
d. attained her lesson objective because of her effective spelling drill

123. If the scores of your test follow a negatively skewed score distribution, what should you do?
Find out ?
a. why your items were easy
b. why most of the scores are high
c. why most of the scores are low
d. why some pupils scored high

124. Principal A is talking about “grading on the curve” in a faculty meeting. What does this expression refer to?
a. A student mark compares his achievement to his effort.
b. A student’s grade or mark depends on how his achievement compares with the achievement of other students in a class.
c. A student’s grade determines whether or not a student attains a defined standard of achievement
d. A student mark tells how closely he is achieving to his potential.

125. Which tests determine whether students accept responsibility for their own behavior or pass on responsibility for their own behavior to
other people?
a. Thematic tests
b. Sentence-completion tests
c. Stylistic test
d. Locus-of-control tests

126. Which of the one weakness of self-supporting personality checklists?


a. Many personality measures have built-in lie scales
b. They lack stability
c. They may not get true information because individuals can hide or disguise feelings
d. They have poor internal consistency

127. Which of these can measure awareness of values?


a. Sociogram
b. Moral dilemmas
c. Projective techniques
d. Rating scales

128. Marking on a normative basis means that


a. the normal distribution curve should be followed
b. some should fall
c. some get high marks
d. the grading is based on a present criteria

129. Which process enhances the comparability of grades?


a. Using a table specifications
b. Determining the level of difficulty of the tests
c. Giving more HOTS (higher order thinking skills)
d. Constructing departmentalized examinations for each subject area.

SITUATIONAL

Situation 1- In a faculty meeting, the principle told his teacher: “We need to improve our school performancein
the National Achievement Test. What should we do?

The teacher gave varied answers as follows:

1. Let’s give incentives and rewards to students who get a rating of 85%
2. Let’s teach them to accept complete responsibility for their performance
3. Let’s make the schoolenvironment conducive for learning
4. Let’s make use of the experimental methods of teaching

130. Which response/s come/s from a behaviorist?


a. #2 and #4 c. #3 and #4
b. #1 and #2 d. #1 and #3

131. On which educational philosophy is response #1 anchored?


a. Existentialism c. Progressivism
b. Essentialism d. Bahaviorism

132. If you learned toward a progressivist philosophy, with which response would you agree?
a. #2       b. #3       c. #4       d. #1

Situation 2- One principle in the utilization of technology of the classroom is appropriateness of material or activity.

133. Teacher C wants his students to master the concept of social justice. Which series of activities will be most effective?
a. Pretest-teaching-posttest
b. Pretest-teaching-posttest-re-teaching for unlearned concepts-posttest
c. Review-pretest-teaching-posttest
d. Teaching-posttest

134. Teacher A likes to show how the launching of spaceships takes place. Which of the following materials available is most fit?
a. Model c. Replica
b. Mock-up         d. Realia

135. Teacher B likes to concretize the abstract concepts of an atom. She came up with a concrete presentation of the atom by using wire and
plastic balls. How would you classify Teacher B’s visual aids?
a. Chart c. Model
b. Replica d. Realia

Situation 3- After reading and paraphrasing Robert frost’s “Stopping by the Wood on a snowy Evening”. Mr. Sales
asked the class to share any insight derived from the poem.

136. The class was asked to share their insights about the poem. The ability to come up with a n insight stems from the ability to
a. analyze the parts of a whole
b. evaluate the worthiness of a thing
c. relate and organize things and ideas
d. comprehend the subject that is being studied

137. To ask the class any insight derived from the poem is based on the theory of
a. realism         c. conditioning
b. behaviorism d. constructivism

138. On which assumption about the learner is Mr. Marquez’s act of asking the class to share their insight based?
a. Learners are like empty receptacles waiting to be filled up
b. Learners are meant to interact with one another
c. Learners have multiple intelligence and varied learning styles
d. Learners are producers of knowledge not only passive recipients of information

Situation 4- Principal E wants her teachers to apply constructivism in teaching


139. On which assumption/s is the principal’s action anchored?

I. Students learn by personally constructing meaning of what is taught.


II. Students are construct and reconstruct meaning based on experiences
III. Students derive meaning from the meaning that the teacher gives
a. II only c. I, II, and III
b. I and II d. I only

140. Which materials will her teachers LEAST prefers?


a. Controversial issues
b. Open-ended topics
c. Unquestionable laws
d. Problem or cases

141. Which concept/s of the learner will Principal E NOT accept?


I. “Empty vesse!”
II. “Tabula rasa”
III. Candle to be lighted

a. III only         c. II only


b. I only d. I and II

Situation 5- Study the matching type of test then answer the 3 questions that follow:
          Column A                         Column B

1.    equilateral triangle         A. With 3 equal sides

2.    right triangle                 B. With 5 equal sides

3.    octagon                        C. Has 90- degree   

                                       angle       

4.    pentagon                      D. Means many

5.    heptagon                      E. with 7 sides

6.    poly                             F. with 8 sides


142. How can you make the items homogeneous?
a. Increase the number of items in Column B
b. All items should be on polygons
c. Remove the word triangle in items #1 and #2 in column A
d. The word “gon” must be included in column B

143. What is the main defect of this matching test?


a. the matching type is an imperfect type
b. the items are NOT homogeneous
c. the items quite easy
d. an obvious pattern is followed in the answering

144. Which should be done to improve the matching type of test?


a. Capitalize the items in Column A
b. Items in Column A and B should be exchanged
c. Drop #6 item in Column A
d. The item in Column A should be increased

Situation 6- Below the template for Scoring Rubric.

5-Demonstrate complete understanding of the problem. All requirements of task are included in response

4-Demonstrate considerable understanding of the problem. All requirements of task are included

3- Demonstrate partial understanding of the problem. Most requirements of task are included

2- Demonstrate little understanding of the problem. Many requirements of task are missing

1- Demonstrate no understanding of the problem

0-No response/task not attempted

145. Which of these is/are essential in constructing a scoring rubric?


I. Description of criteria to serve as standard
II. Clear descriptions of performance at each level
III. Levels of achievement (mastery)
IV. Rating scheme

a. I, II, III c. I, II, III, IV


b. I, II d. I only

146. Which statement is TRUE of the rubric?


a. It is developmental
b. It is analytical
c. It is both holistic and developmental
d. It is holistic

147. Which is TRUE of the scoring rubric?


I. It describes criteria of levels of achievement
II. It has a rating scheme
III. It limit itself to 4 levels of achievement
a. I and II
b. I and III
c. II and III
d. I, II and III
Situation 7- Study the table on item analysis for non-attractiveness and non-plausibility of distracters based on the
results of a try-out test in Science. The letter marked with a asterisk is the correct answer.

Item No. 1 A B C D E

Upper 27% 10 4 1 1 0

Lower 27% 6 5 2 2 0

148. The table shows that the test item analyzed .


a. has a positive discrimination index
b. has a negative discrimination index
c. is extremely easy
d. is extremely difficult

149. Based on the table, which is the most effective distracter?


a. Option D
b. Option A
c. Option C
d. Option B

150. Based on the table, which group got more correct answer?
a. Upper group
b. It cannot be determined
c. Lower group
d. Data are not sufficient to give an answer

Answer Key:

101 D

102 D

103 C

104 C

105 A

106 A

107 B

108 A

109 B

110 D

111 B

112 C
113 B

114 B

115 D

116 D

117 A

118 C

119 D

120 A

121 B

122 C

123 B

124 B

125 D

126 C

127 B

128 D

129 D

130 B

131 D

132 B

133 B

134 D

135 B

136 A

137 D

138 B

139 B

140 C

141 A

142 B

143 B
144 C

145 C

146 C

147 A

148 D

149 D

150 C

Professional Education Drill 9 - Part 1

- December 09, 2018


1.     Principal B tells her teachers that training in the humanities is most important. To which educational philosophy does he
adhere?
a.     Existentialism c. Progressivism
b.     Perennialism d. Essentialism

2.     Principal C shares this thought with his teachers: "Subject matter should help students understand and appreciate themselves as
unique individuals who accept complete responsibility for their thoughts, feelings, and actions." From which philosophy is this
thought based?
a.     Perennialism c. Existentialism
b.     Essentialism d. Progressivism

3.     To come closer to the truth we need to "go back to the things themselves." This is the advice of the
a.     behaviorists c. idealists
b.     phenomenologists d. pragmatists

4.     Student B claims: "I cannot see perfection but I long for it. So it must be real." Under which group can he be classified?
a.     Idealist c. Realist
b.     Empiridst d. Pragmatist.

5.     Which of the following prepositions is attributed to Plato?


a.     Truth is relative to a particular time and place
b.     Human beings create their own truths.
c.     Learning is the discovery of truth as Iatent ideas are brought to consciousness.
d.     Sense perception is the most accurate guide to knowledge.

6.     On whose philosophy was A. S. Neil's Summerhill, one of the most experimental schools, based?
a.     Rousseau c. Montessori
b.     Pestalozzi d. John Locke

7.     As a teacher, you are a rationalist. Which among these will be your guiding principle?
a.     I must teach the child that we can never have real knowledge of anything.
b.     I must teach the child to develop his mental powers to the full.
c.     I must teach the child so he is assured of heaven.
d.     I must teach the child every knowledge, skill, and value that he needs for a better future.

8.     Teacher U teaches to his pupils that pleasure is not the highest good. Teacher's teaching is against what philosophy?
a.     Realism c. Epicureanism
b.     Hedonism d. Empiricism

9.     Who among the following puts more emphasis on core requirements, longer school day, longer academic year and more challenging
textbooks?
a.     Perennialist c. Progressivist
b.     Essentialist d. Existentialist

10.  Which group of philosophers maintain that "truth exists in an objective order that is independent of the knower"?
a.     Idealists c. Existentialists
b.     Pragmatists d. Realists
11.  You arrive at knowledge by re-thinking of latent ideas. From whom does this thought come?
a.     Experimentalist c. Idealist
b.     Realist d. Existentialist

12.  As a teacher, you are a reconstructionist. Which among these will be your guiding principle?
a.     I must teach the child every knowledge, skill, and value that he needs for a better future.
b.     I must teach the child to develop his mental powers to the full.
c.     I must teach the child so he is assured of heaven.
d.     I must teach the child that we can never have real knowledge of anything.

13.  Teacher B engages her students with information for thorough understanding for meaning and for competent application. Which
principle governs Teacher B's practice?
a.     Contructivist c. Behaviorist
b.     Gestalt d. Cognitivist

14.  Which is/are the sources of man's intellectual drives, according to Freud?
a.     Id c. Id and ego
b.     Superego d. Ego

15.  Soc exhibits fear response to freely roaming dogs but does not show fear when a dog is on a leash or confined to a pen. Which
conditioning process is illustrated
a.     Generalization c. Acquisition
b.     Extinction d. Discrimination

16.  The concepts of trust vs. maturity, autonomy vs. self-doubt, and initiative vs. guilt are most closely related with the works of
__________.
a.     Erikson b. Piaget c. Freud d. Jung
17.  Teacher F is convinced that whenever a student performs a desired behavior, provided reinforcement and soon the student will learn
to perform the behavior on his own. On which principle is Teacher F's conviction based?
a.     Cognitivism c. Behaviorism
b.     Environmentalism d. Constructivism

18.  In a social studies class, Teacher I presents a morally ambiguous situation and asks his students what they would do. On whose
theory is Teacher I's technique based?
a.     Kohlberg c. Piaget
b.     Bandura d. Bruner

19.  Based on Freud's psychoanalytic theory which component(s) of personality is (are) concerned with a sense of right and wrong?
a.     Super-ego c. ld
b.     Super-ego and Ego d. Ego

20.  Which does Naom Chomsky, assert about language learning for children?
I. Young children learn and apply grammatical rules and vocabulary as they are exposed to them.
II. Begin formal teaching of grammatical rules to children as early as possible.
III. Do not require initial formal language teaching for children.
a.     I and III c. I only
b.     II only d. I and II

21.  Which teaching activity is founded on Bandura's Social Learning Theory?


a.     Lecturing c. Questioning
b.     Modeling d. lnductive Reasoning

22.  Behavior followed by pleasant consequences will be strengthened and will be more likely to occur in the future. Behavior followed by
unpleasant consequences will be weakened and will be less likely to be repeated in the future. Which one is explained?
a.     Freud's Psychoanalytic Theory
b.     Thorndike's Law of Effect
c.     B. F. Skinner's Operant Conditioning Theory
d.     Bandura's Social Learning Theory

23.  Bruner's theory on intellectual development moves from enactive to iconic and symbolic stages. In which stage(s) are diagrams
helpful to accompany verbal information?
a.     Enactive and iconic             c. Symbolic and enactive
b.     Symbolic d. Iconic

24.  In a treatment for alcoholism, Ramil was made to drink an alcoholic beverage and then made to ingest a drug that produces nausea.
Eventually, he was nauseated at the sight and smell of alcohol and stopped drinking alcohoL Which theory explains this?
a.     Operant conditioning
b.     Social Learning Theory
c.     Associative Learning
d.     Attribution Theory
25.  A mother gives his boy his favorite snack every time the boy cleans up his room. Afterwards, the boy cleaned his room everyday in
anticipation of the snack. Which theory is illustrated?
a.     Associative Learning
b.     Classical Conditioning
c.     Operant Conditioning
d.     Pavlonian Conditioning

26.  Researches conducted show that teacher's expectations of students become. Do not require initial formal language teaching for
children self-fulfilling prophecies. What is this phenomenon called?
a.     Halo effect c. Ripple effect
b.     Pygmalion effect d. Hawthorne effect

27.  What does extreme authoritarianism in the home reinforce in learners?


a.     Doing things on their own initiative
b.     Ability to direct themselves.
c.     Dependence on others for direction.
d.     Creativity in work.

28.  Theft of school equipment like tv, computer, etc. by teenagers in the community itself is becoming a common phenomenon. What
does this incident signify?
a.     Prevalence of poverty in the community.
b.     Inability of school to hire security guards.
c.     Deprivation of Filipino schools.
d.     Community's lack of sense of co-ownership.

29.  A student passes a research report poorly written but ornately presented in a folder to make up for the poor quality of the book report
content. Which Filipino trait does this practice prove? Emphasis on __________.
a.     art over academics
b.     substance over "porma"
c.     art over science
d.     "porma" over substance

30.  Student Z does not study at all but when the Licensure Examination for Teachers (LET) comes, before he takes the LET, he spends
one hour or more praying for a miracle, i.e. to pass the exam. Which attitude towards religion or God is displayed?
a.     Religion as fake
b.     Religion as magic
c.     Religion as authentic
d.     Religion as real

31.  During the Spanish period, what was/were the medium/media of instruction in schools?
a.     The Vernacular
b.     English
c.     Spanish
d.     Spanish and the Vernacular

32.  All subjects in Philippine elementary and secondary schools are expected to be taught using the integrated approach. This came
about as a result of the implementation of _________.
a.     Program for Decentralized Education
b.     School-Based Management
c.     Basic Education Curriculum
d.     Schools First Initiative

33.  Under which program were students who were not accommodated in public elementary and secondary schools because of lack of
classroom, teachers, and instructional materials, were enrolled in private schools in their respective communities at the
government's expense?
a.     Government Assistance Program
b.     Study Now-Pay Later
c.     Educational Service Contract System
d.     National Scholarship Program

34.  What was the most prominent educational issue of the mid 1980s?
a.     Bilingual Education c. Accountability
b.     Values Education d. Mainstreaming

35.  Availment of the Philippine Education Placement Test (PEPT) for adults and out-of-school youths is in support of the government’s
educational program towards __________
a.     equitable access           c. quality and relevance
b.     quality d. relevance
36.  The main purpose of compulsory study of the Constitution is to __________
a.     develop students into responsible, thinking citizens
b.     acquaint students with the historical development of the Philippine Constitution
c.     make constitutional experts of the students
d.     prepare students for law-making

37.  Which one may support equitable access but may sacrifice quality?
a.     Open admission
b.     School accreditation
c.     Deregulated tuition fee hike
d.     Selective retention

38.  With which goals of educational institutions as provided for by the Constitution is the development of work skills aligned?
a.     To develop moral character
b.     To teach the duties of citizenship
c.     To inculcate love of country
d.     To develop vocational efficiency

39.  Studies in the areas of neurosciences disclosed that the human brain has limitless capacity. What does this imply?
a.     Some pupils are admittedly not capable of learning.
b.     Every pupil has his own native ability and his learning is limited to this nativeabilty.
c.     Every child is a potential genius.
d.     Pupils can possibly reach a point where they have learned everything.

40.  Based on Piaget's theory, what should a teacher provide for children in the concrete operational stage?
a.     Activities for hypothesis formulation.
b.     Learning activities that involve problems of classification and ordering.
c.     Games and other physical activities to develop motor skills.
d.     Stimulating environment with ample objects to play with.

41.  Based on Piaget's theory, what should a teacher provide for children in the sensimotor stage?
a.     Games and other physical activities to develop motor skill.
b.     Learning activities that involve problems of classification and ordering.
c.     Activities for hypothesis formulation.
d.     Stimulating environment with ample objects to play with.

42.  Which behavior is exhibited by a student who is strong in interpersonal intelligence?


a.     Works on his/her own.
b.     Keeps interest to himself/herself.
c.     Seeks out a classmate for help when problem occurs.
d.     Spends time meditating.

43.  A sixth grade twelve-year old boy comes from a dysfunctional family and has been abused and neglected. He has been to two
orphanages and three different elementary schools. The student can decode on the second grade level, but he can comprehend
orally material at the fourth or fifth grade level. The most probable cause/s of this student's reading problem is/are __________.
a.     emotional factors c. neurological factors
b.     poor teaching d. immaturity

44.  A child who gets punished for stealing candy may not steal again immediately. But this does not mean that the child may not steal
again. Based on Thorndike's theory on punishment and learning, this shows that __________
a.     punishment strengthens a response
b.     punishment removes a response
c.     punishment does not remove a response
d.     punishment weakens a response

45.  It is not wise to laugh at a two-year old child when he utters bad word because in his stage he is learning to __________.
a.     consider other's views
b.     distinguish sex differences
c.     socialize
d.     distinguish right from wrong

46.  John Watson said: "Men are built not born." What does this statement point to?
a.     The ineffectiveness of training on a person's development.
b.     The effect of environmental stimulation on a person's development.
c.     The absence of genetic influence on a person's development.
d.     The effect of heredity.

47.  Which types of play is most characteristic of a four to six-year old child?
a.     Solitary and onlooker plays
b.     Associative and cooperative plays
c.     Associative and onlooker plays
d.     Cooperative and solitary plays

48.  All of the following describe the development of children aged eleven to thirteen EXCEPT __________.
a.     they shift from impulsivity to adaptive ability
b.     sex differences in IQ becomes more evident
c.     they exhibit increase objectivity in thinking
d.     they show abstract thinking and judgment

49.  Rodel is very aloof and cold in his relationships with his classmates. Which basic goal must have not been attained by Rodel during
his developmental years, according to Erikson's theory on psychological development?
a.     Autonomy c. Initiative
b.     Trust d. Generativity

50.  Ruben is very attached to his mother and Ruth to her father. In what developmental stage are they according to Freudian
psychological theory?
a.     Oedipal stage c. Annal stage
b.     Latent stage d. Pre-genital stage

Answer Key:

1. a
2. c
3. a
4. d
5. c
6. a
7. b
8. b
9. b
10. d
11. a
12. a
13. d
14. c
15. d
16. b
17. c
18. b
19. b
20. d
21. d
22. c
23. b
24. c
25. d
26. b
27. b
28. d
29. d
30. b
31. d
32. c
33. d
34. a
35. a
36. a
37. a
38. d
39. c
40. b
41. a
42. c
43. c
44. c
45. a
46. b
47. b
48. a
49. b
50. a

Professional Education Drill 9 - Part 2

- December 09, 2018

51. Which assumption underlies the teacher's use of performance objectives?

a. Not every form of learning is observable.


b. Performance objectives assure the learner of learning.

c. Learning is defined as a change in the learner's observable performance.

d. The success of learner is based on teacher performance.

52. The principle of individual differences requires teachers to __________.

a. give greater attention to gifted learners

b. provide for a variety of learning activities

c. treat all learners alike while in the classroom

d. prepare modules for slow learners in class

53. In instructional planning it is necessary that the parts of the plan from the first to the last have __________.

a. clarity                  c. coherence

b. symmetry                  d. conciseness

54. A goal-oriented instruction culminates in __________.

a. planning of activities         c. identification of topics

b. evaluation                  d. formulation of objectives

55. A teacher's summary of a lesson serves the following functions, EXCEPT

a. it links the parts of the lesson

b. lt brings together the information that has been discussed

c. it makes provisions for full participation of students.

d. it clinches the basic ideas or concepts of the lesson.

56. In Krathwohl's affective domain of objectives, which of the following is the lowest level of affective behavior?

a. Valuing                        c. Responding


b. Characterization        d. Organization

57. The following are used in writing performance objectives, EXCEPT

a. delineate        c. integrate

b. diagram                d. comprehend

58. If a teacher plans a constructivist lesson, what will he most likely do? Plan how he can

a. do evaluate his students' work

b. do reciprocal teaching

c. lecture to his students

d. engage his students in convergent thinking

59. In mastery learning, the definition of an acceptable standard of performance is called a

a. SMART                    c. behavior

b. criterion measure            d. condition

60. The primary objective of my lesson is: "To add similar fractions correctly." Before I can do this I must first aim at this specific
objective: "To distinguish a numerator from a nominator." What kind of objective is the latter?

a. Major c. Enabling

b. Terminal         d. Primary

61. Which behavioral term describes a lesson outcome in the highest level of Bloom's cognitive domain?

a. Create            c. Analyze

b. Evaluate                    d. Design

62. As a teacher, what do you do when you engage yourself in major task analysis?

a. Test if learning reached higher level thinking skills.

b. Breakdown a complex task into sub-skills.

c. Determine the level of thinking involved.

d. Revise lesson objectives.

63. Teacher G's lesson objective has something to do with the skill of synthesizing? Which behavioral term is most appropriate?
a. Test                  c. Appraise

b. Assess                  d. Theorize

64. In Krathwohl's taxonomy of objectives in the affective, which is most authentic?

a. Characterization           c. Responding

b. Organization           d. Valuing

65. "A stitch on time saves nine", so goes the adage.. Applied to classroom management, this means that we __________

a. may not occupy ourselves with disruptions which are worth ignoring because  they are minor

b. must be reactive in our approach to discipline

c. have to resolve minor disruptions before they are out of control

d. may apply 9 rules out of 10 consistently

66. How can you exhibit referent power on the first day of school?

a. By making them feel you know what you are talking about.

b. By telling them the importance of good grades.

c. By reminding your students your authority over them again and again.

d. By giving your students a sense of belonging and acceptance.

67. Teacher B clears his throat to communicate disapproval of a student's behavior. Which specific influence technique is this?

a. Signal interference c. Interest boosting

b. Direct appeal d. Proximity control

68. How can you exhibit expert power on the first day of school?

a. By making them feel you know what you are talking about.

b. By making them realize the importance of good grades.

c. By reminding them your students your authority over them again and again.

d. By giving your students a sense of belonging and acceptance.

69. Teacher H strives to draw participation of every student into her classroom discussion. Which student's need is she trying to address?
The need __________

a. to show their oral abilities to the rest of the class


b. to be creative

c. to feel significant and be part of a group

d. to get everything out in the open

70. Which is a sound classroom management practice?

a. Avoid establishing routines; routines make your student robots.

b. Establish routines for all daily needs and tasks.

c. Apply rules and policies on a case to case basis.

d. Apply reactive approach to discipline.

71. An effective classroom manager uses low-profile classroom control. Which is a low-profile classroom technique?

a. Note to parents

b. After-school detention

c. Withdrawal of privileges

d. Raising the pitch of the voice

72. Which is one characteristic of an effective classroom management?

a. It quickly and unobtrusively redirects misbehavior once it occurs.

b. It teaches dependence on others for self-control.

c. It respects cultural norms of a limited group students.

d. Strategies are simple enough to be used consistently.

73. How can you exhibit legitimate power on the first day of school?

a. By making your students feel they are accepted for who they are.

b. By informing them you are allowed to act in loco parentis.

c. By making them realize the importance of good grade

d. By making them feel you have mastery of subject matter.

74. With-it-ness, according to Kounin, is one of the characteristics of an effective classroom manager. Which phrase goes with it?

a. Have hands that write fast.

b. Have eyes on the back of your heads.


c. Have a mouth ready to speak.

d. Have minds packed with knowledge.

75. Which is an appropriate way to manage off-task behavior

a. Make eye contact.

b. Stop your class activity to correct a child who is no longer on task.

c. Move closer to the child.

d. Redirect a child's attention to task and check his progress to make sure he is continuing to work.

76. Referring to Teacher S, Nicolle describes her teacher as "fair, caring and someone you can talk to." Which power or leadership does
Teacher S have?

a. Referent power c. Reward power

b. Legitimate power         d. Expert power

77. Research tells that teachers ask mostly content questions. Which of the following terms does NOT refer to content question?

a. Closed c. Concept

b. Direct d. Convergent

78. Read the following then answer the question:

TEACHER: IN WHAT WAYS OTHER THAN THE PERIODIC TABLE MIGHT WE PREDICT THE UNDISCOVERED ELEMENTS?

BOBBY: WE COULD GO TOTHE MOON AND SEE IF THERE ARE SOME ELEMENTS THERE WE DON'T HAVE.

BETTY: WE COULD DIG DOWN TO THE CENTER OF THE EARTH AND SEE IF WE FIND ANY OF THE MISSING ELEMENTS.

RICKY: WE COULD STUDY DEBRIS FROM THE METEORITES IF WE CAN FIND ANY. 

TEACHER: THOSE ARE ALL GOOD ANSWERS BUT WHAT IF THOSE, EXCURSIONS TO THE MOON, TO THE CENTER OF THE
EARTH, OR TO FIND METEORITES WERE TOO COSTLY AND TIME CONSUMING? HOW MIGHT WE USE THE ELEMENTS
WE ALREADY HAVE HERE ON EARTH TO FIND SOME NEW ONES?

Question: Which questioning strategy/ies does/do the exchange of thoughts above illustrate?
a. Funneling         c. Nose-dive

b. Sowing and reaping d. Extending and lifting

79. Which questioning practice promotes more class interaction?

a. Asking the question before calling on a student.

b. Focusing on divergent questions.

c. Focusing on convergent questions

d. Asking rhetorical questions.

80. Which technique should a teacher use to encourage response if his students do not respond to his question?

a. Ask a specific student to respond, state the question, and wait a response.

b. Tell the class that it will have detention unless answer are forthcoming.

c. Ask another question, an easier one.

d. Wait for a response.

81. Teacher P wants to develop the skill of synthesizing in her pupils. Which one will she do?

a. Ask her students to formulate a generalization from the data shown in graphs.

b. Ask her students to answer questions beginning with "What if ... "

c. Tell her pupils to state data presented in graphs.

d. Directs her students to ask questions on the parts of the lesson not understood.

82. The following are sound specific purposes of questions EXCEPT

a. to call the attention of an inattentive student

b. to teach via student answers

c. to stimulate leaders to ask questions

d. to arouse interest and curiosity

83. For maximum interaction, a teacher ought to avoid __________ questions.

a. informational c. leading

b. rhetorical         d. divergent
84. If teacher has to ask more higher-order questions, he has to ask more __________ questions.

a. closed c. concept

b. fact         d. convergent

85. Which is NOT a sound purpose for asking questions?

a. To probe deeper after an answer is given.

b. To discipline a bully in class.

c. To remind students of a procedure.

d. To encourage self-reflection.

86. After giving an input on a good paragraph, Teacher W asks her students to rate a given paragraph along the elements of a good
paragraph. The students' task is in level of __________

a. application

b. analysis

c. evaluation

d. synthesis

87. Read the following then answer the question

TEACHER: IN WHAT WAYS OTHER THAN THE PERIODIC TABLE MIGHT WE PREDICT THE UNDISCOVERED ELEMENTS?

BOBBY:  WE COULD GOTO THE MOON AND SEE IF THERE ARE SOME ELEMENTS THERE WE DON'T HAVE.

BETTY: WE COULD DIG DOWN INTO THE CENTER OF THE EARTH AND SEE IF WE FIND ANY OF THE MISSING ELEMENTS

RICKY: WE COULD STUDY DEBRIS FROM THE METEORITES IF WE CAN FIND ANY

TEACHER: THOSE ARE ALL GOOD ANSWERS. BUT WHAT IF THOSE EXCURSIONS TO THE MOON, TO THE CENTER OF THE
EARTH, OR TO FIND METEORITES WERE TOO COSTLY AND TIME CONSUMING? HOW MIGHT WE USE THE ELEMENTS
WE ALREADY HAVE HERE ON EARTH TO FIND SOME NEW ONES?

Question: The Teacher's questions in the above exchange are examples of __________ questions

a. fact         c. direct
b. concept d. closed

88. Read this question: "How will you present the layers of the earth to your class?" This is a question that

a. directs

b. leads the student to evaluate

c. assesses cognition

d. probes creative thinking

89. The teacher's first task in the selection of media in teaching is to determine the ______.

a. choice of the students

b. availability of the media

c. objectives of the lesson

d. technique to be used

90. Based on Edgar Dale's Cone of Experience, which activity is closest to the real thing? 

a. View images c. Watch a demo

b. Attend exhibit d. Hear

91. Based on Edgar Dale's Cone of Experience, which activity is farthest from the real thing?

a. Read c. View images

b. Hear d. Attend exhibit

92. Which criterion should guide a teacher in the choice of instructional devices? 

a. Attractiveness c. Novelty

b. Cost                 d. Appropriateness

93. To elicit more student's response, Teacher G made use of covert responses. Which one did she NOT do?

a. She had the students write their response privately.

b. She showed the correct answers on the overhead after the students have written their responses.

c. She had the students write their responses privately then called each of them.

d. She refrained from judging on the student's responses.


94. Teacher W wants to review and check on the lesson of the previous day? Which one will be most reliable?

a. Having students identify difficult homework problems.

b. Having students correct each other's work.

c. Sampling the understanding of a few students

d. Explicitly reviewing the task-relevant information necessary for the day's lesson.

95. Teacher M's pupils are quite weak academically and his lesson is already far behind his time table. How should Teacher M proceed
with his lesson?

a. Experientially c. Logically

b. lnductively d. Deductively

96. Which activity should a teacher have more for his students if he wants them to develop logical-mathematical thinking?

a. Problem solving          c. Drama

b. Choral reading d. Storytelling

97. Which guideline must be observed in the use of prompting to shape the correct performance of your students?

a. Use the least intrusive prompt first.

b. Use all prompts available.

c. Use the most intrusive prompt first.

d. Refrain from using prompts.

98. To promote effective practice, which guideline should you bear in mind? Practice should be

a. done in an evaluative atmosphere

b. difficult for students to learn a lesson

c. arranged to allow students to receive feedback

d. take place over a long period of time

99. Which is one role of play in the pre-school and early childhood years?

a. Develops competitive spirit.

b. Separates reality from fantasy.

c. Increases imagination due to expanding knowledge and emotional range.


d. Develops the upper and lower limbs.

100. Teacher T taught a lesson denoting ownership by means of possessives. He first introduced the rule, then gave examples, followed
by class exercises, then back to the rule before he moved into the second rule. Which presenting technique did he use? 

a. Combinatorial c. Part-whole

b. Comparative d. Sequential

Answer Key:

51. c

52. b

53. b

54. a

55. b

56. d

57. a

58. d

59. b

60. c

61. c

62. b

63. d

64. d

65. c

66. b

67. a

68. a

69. c

70. b

71. d

72. a

73. d

74. d
75. d

76. b

77. a

78. d

79. b

80. a

81. a

82. a

83. c

84. d

85. b

86. b

87. c

88. d

89. c

90. b

91. a

92. d

93. b

94. c

95. a

96. a

97. a

98. a

99. c

100. c

Professional Education Drill 9 - Part 3

- December 09, 2018

101. The burnout malady gets worse if a teacher doesn't intervene to change whatever areas he or she can control. Which one can renew a
teacher's enthusiasm?
a. Stick to job
b. Initiate changes in jobs
c. Judge someone else as wrong
d. Engage in self-pity

102. Which Filipino trait works against the shift in teacher's role from teacher as a fountain of information to teacher as facilitator?
a. Authoritativeness c. Hiya
b. Authoritarianism d. Pakikisama

103. Which method has been proven to be effective in courses that stress acquisition of knowledge?
a. Socratic method         c. Mastery learning
b. Cooperative learning d. Indirect instruction

104. Direct instruction is for facts, rules, and actions as indirect instruction is for __________, __________, __________.
a. hypotheses, verified data and conclusions
b. concepts, patterns and abstractions
c. concepts, processes and generalizations
d. guesses, data and conclusions

105. For which may you use the direct instruction method?
a. Become aware of the pollutants around us.
b. Appreciate Milton's Paradise Lost.
c. Use a microscope properly
d. Distinguish war from aggression.

106. I want to teach concepts, patterns and abstractions. Which method is most appropriate?
a. Indirect instruction c. Direct instruction
b. Discovery         d. Problem solving

107. What should a teacher do for students in his class who are not on grade level?
a. Give them materials on their level and let them work at a pace that is reasonable for them, trying to bring them up to a grade level.
b. Give them the same work as the other students, because they will absorb as much as they are capable of
c. Give them the same work as the other students, not much, so that they won't    feel embarrassed.
d. Give them work on the level of the other students and work a little above the classmates level to challenge them.

108. By what name is Indirect instruction the Socratic method also known?
a. Mastery learning         c. Morrison method
b. Indirect Method d. Questioning method

109. Teacher B is a teacher of English as a Second Language. She uses vocabulary cards, fill-in-the-blank sentences, dictation and writing
exercises in teaching a lesson about grocery shopping. Based on this information, which of the following is a valid conclusion?
a. The teacher is reinforcing learning by giving the same information in, a variety of methods.
b. The teacher is applying Bloom's hierarchy of cognitive learning.
c. The teacher wants to do less talk.
d. The teacher is emphasizing listening and speaking skills.

110. Which is a form of direct instruction?


a. Discovery process           c. Programmed instruction
b. Problem solving d. Inductive reasoning

111. Which does NOT belong to the group of alternative learning systems?
a. Multi-grade grouping       c. Graded education
b. Multi-age grouping          d. Non-graded grouping

112. Teacher H gave her first-grade class a page with a story in which pictures take the place of some words. Which method did she use?
a. The whole language approach
b. The Spaulding method
c. The rebus method
d. The language experience approach
113. Teacher B uses the direct instruction strategy. Which sequence of steps will she follow?
I.  Independent practice
II.  Feedback and correctiveness
III.  Guided student practice
IV.  Presenting and structuring
V.  Reviewing the previous day's work

a. V-II-IV-III-I c. V-lV-III-II-I
b. III-II-IV-I-V d. I-V-II-III-IV

114. Why should a teacher NOT use direct instruction all the time?
a. It requires much time.
b. It requires use of many supplementary materials.
c. It is generally effective only in the teaching of concepts and abstractions.
d. It reduces students engagement in learning.

115. Teacher A is a teacher of English as a Second Language. She uses vocabulary cards, fill-in-the-blank sentences, dialogues, dictation and
writing exercises in teaching a lesson about grocery shopping. Based on this information, which of the following is a valid conclusion?
a. The teacher is applying Bloom's hierarchy of cognitive learning.
b. The teacher is teaching in a variety of ways because not all students learn in the same manner.
c. The teacher wants to make her teachirig easier by having less talk.
d. The teacher is emphasizing reading and writing skills.

116. I combined several subject areas in order to focus on a single concept for inter-disciplinary teaching. Which strategy/method did I use?
a. Problem-entered learning
b. Thematic instruction
c. Reading-writing activity
d. Unit method

117. Teacher E discussed how electricity flows through wires and what generates the electric charge. Then she gave the students wires, bulbs,
switches, and dry cells and told the class to create a circuit that will increase the brightness of each bulb. Which one best describes the
approach used?
a. It used a taxonomy of basic thinking skills
b. It was contructivist
c. It helped students understand scientific methodology
d. It used cooperative learning

118. With indirect instruction in mind, which does NOT belong to the group?
a. Problem solving c. Inductive reasoning
b. Lecture-recitation d. Discovery

119. I drew learners into several content areas and encouraged them to solve a complex question for inter-disciplinary teaching. Which strategy
did I use?
a. Problem-centered learning
b. Unit method
c. Reading-writing activity
d. Thematic instruction

120. In self-directed learning, to what extent should a teacher's "scaffolding" be?


a. To a degree the student needs it.
b. None, to force the student to learn by himself.
c. To the minimum, to speed up development of student's sense of independence.
d. To the maximum, in order to extend to the student all the help he needs.

121. Which is a major advantage of a curriculum-based assessment?


a. It is informal in nature.
b. It connects testing with teaching.
c. It tends to focus on anecdotal information on student progress.
d. It is based on a norm-referenced measurement model.

122. Which are direct measures of competence?


a. Personality tests                    c. Paper-and-pencil tests
b. Performance tests                  d. Standardized test

123. "What is most likely to happen to our economy when export continuously surpasses import" is a thought question on __________.
a. creating                         c. synthesizing
b. relating cause-and-effect d. predicting
124. The test item "Group the following items according to shape" is a thought test item on __________.
a. creating                          c. generalizing
b. classifying                  d. comparing
125. In the context on the theory on multiple intelligences, what is one weakness of the paper-pencil test?
a. It is not easy to administer.
b. It puts the non-linguistically intelligent at a disadvantage
c. It utilizes so much time.
d. It lacks reability.

126. With synthesizing skills in mind, which has the highest diagnostic value?
a. Essay test               c. Completion test
b. Performance test               d. Multiple choice test

127. Which one can best evaluate students' attitudinal development?


a. Essay test               c. Observation
b. Portfolio                       d. Short answer test

128. With specific details in mind, which one has (have) a stronger diagnostic value?
a. Multiple choice test
b. Non-restricted essay test
c. Restricted essay test
d. Restricted and non-restricted essay tests

129. Teacher A discovered that his pupils are very good in dramatizing. Which tool must have helped him discover his pupils' strength?
a. Portfolio assessment
b. Performance test
c. Journal entry

130. Which can effectively measure students' awareness of values?


a. Projective techniques              c. Likert scales
b. Moral dilemma              d. Anecdotal record
e. Paper-and-pencil test

131. Teacher F wanted to teach the pupils the skill to do cross stitching. Her check up quiz was a written test on the steps of cross stitching.
Which characteristic of a good test does it lack?
a. Scorability                       c. Objectivity
b. Reliability                       d. Validity

132. If your Licensure Examination Test (LET) items sample adequately the competencies listed in the syllabi, it can be said that the LET
possesses __________ validity.
a. concurrent                       c. content
b. construct                               d. predictive

133. "In the light of the facts presented, what is most likely to happen when ... ?" is a sample thought question on
a. inferring                               c. synthesizing
b. generalizing                       d. justifying
134. In a criterion-referenced testing, what must you do to ensure that your test is fair?
a. Make all of the questions true or false.
b. Ask each student to contribute one question.
c. Make twenty questions but ask the students to answer only ten of their choice.
d. Use the objectives for the units as guide in your test construction.

135. Which test has broad sampling of topics as strength?


a. Objective test                    c. Essay test
b. Short answer test                       d. Problem type

136. Which is the first step in planning an achievement test?


a. Define the instructional objective.
b. Decide on the length of the test.
c. Select the type of test items to use.
d. Build a table of specification.

137. The first thing to do in constructing a periodic test is for a teacher to __________
a. decide on the number of items for the test
b. go back to her instructional objectives
c. study the content
d. decide on the type of test to construct

138. In the parlance of test construction what does "TOS" mean?


a. Table of Specifics
b. Table of Specifications
c. Table of Specific Test Items
d. Team of Specifications

139. Shown a picture of children in sweaters inside the classroom, the students were asked this question: "In what kind of climate do these
children live?" This is a thought question on __________
a. inferring                         c. creating
b. applying                         d. predicting

140. Which guideline in test construction is NOT observed in this test item Jose Rizal wrote __________.
a. The central problem should be packed in the stem.
b. There must be only one correct answer.
c. Alternatives must have grammatical parallelism.
d. The alternates must be plausible.

141. Quiz is to formative test while periodic is to __________


a. criterion-reference test                  c. norm-reference test
b. summative test               d. diagnostic test

142. If teacher wants to test students' ability to organize ideas, which type of test should she formulate?
a. Multiple-choice type               c. Essay
b. Short answer                       d. Technical problem

143. Out of 3 distracters in a multiple choice test item, namely B, C, and D, no pupil chose D as answer. This implies that D is __________
a. an ineffective distracter
b. a vague distracter
c. an effective distracter
d. a plausible distracter

144. Study this group of tests which was administered with the following results, then answer the question
Subject Mean SD Ronnels's Score
Math 56          10 43
Physics 41            9 31
English 80         16 109

In which subject(s) did Ronnel perform best in relation to the group's performance?
a. Physics and Math     c. Physics
b. English         d. Math

145. Study this group of tests which was administered with the following results, then answer the question.
Subject    Mean       SO   Ronnel's Score
Math    56 10    43
Physics       41 9    31
English    80 16    109

In which subject(s) did Ronnel perform most poorly in relation to the group's performance?
a. English c. Math
b. English and Math d. Physics

146. What can be said of Peter who obtained a score of 75 in a Grammar objective test?
a. He answered 75 items in the test correctly.
b. He answered 75% of the test items correctly.
c. His rating is 75.
d. He performed better than 5% of his classmates.

147. In his second item analysis, Teacher H found out that more from the lower group got the test item # 6 correctly. This means that the test
item __________.
a. has a negative discriminating power
b. has a lower validity
c. has a positive discriminating power
d. has a high reability

148. NSAT and NEAT results are interpreted against set mastery level. This means that NSAT and NEAT fall under __________.
a. intelligence test                       c. criterion-referenced test
b. aptitude test                             d. norm-referenced test

149. Teacher Y does norm-referenced interpretation of scores. Which of the following does she do?
a. She describes group performance in relation to a level of mastery set.
b. She uses a specified content as its frame of reference.
c. She compares every individual students' scores with others' scores.
d. She describes what should be their performance.

150. Test norms are established in order to have a basis for __________.
a. establishing learning goals
b. interpreting test results
c. computing grades
d. identifying pupils' difficulties

Answer Key:
101. b
102. c
103. c
104. c
105. c
106. c
107. a
108. d
109. a
110. c
111. c
112. b
113. c
114. c
115. b
116. d
117. c
118. d
119. a
120. c
121. d
122. d
123. b
124. b
125. d
126. b
127. b
128. d
129. b
130. d
131. a
132. c
133. a
134. d
135. c
136. d
137. b
138. b
139. a
140. b
141. b
142. c
143. a
144. a
145. a
146. d
147. b
148. c
149. c
150. b

Professional Education Drill 9 - Part 4

- December 09, 2018

151. Which is most implied by a negatively skewed score distribution?


a. The scores are evenly distributed from left to the right
b. Most pupils are achievers
c. Most of the scores are low
d. Most of the scores are high

152. Which holds true to standardized tests?


a. They are used for comparative purposes
b. They are administered differently
c. They are scored according to different standards
d. They are used for assigning grades

153. Students' scores on a test were: 72, 72, 73, 74, 76, 78, 81, 83, 85. The score 76 is the __________.
a. mode c. mean
b. average         d. median

154. Are percentile ranks the same as percentage correct?


a. It cannot be determined unless scores are given.
b. It cannot be determined unless the number of examinees is given.
c. No             
d.     Yes

155. In which competency do my students find the greatest difficulty? In the item with a difficulty index of __________.
a. 0.1                  b. 0.9                      c. 0.5                 d. 1.0

156. Study this group tests which was administered wit the following results, then answer the question
Subject     Mean SD Ronnel's Score
Math      56 10 43
Physics      41 9 31
English      80 16 109
In which subject(s) were the scores most homogenous?

a. Math c. Physics
b. English d. Physics and Math

157. Which measure(s) of central tendency separate(s) the top half of the group from the ¬bottom half?
a. Median c. Median and Mean
b. Mean d. Mode

158. Which applies when skewness is zero?


a. Mean is greater than the median
b. Median is greater than mean
c. Scores have three modes
d. Scores are normally distributed

159. Standard deviation is to variability as mode to __________.


a. level of difficulty c. correlation
b. discrimination d. central tendency

160. What is the mean of this score distribution: 4, 5, 6, 7, 8, 9, 10?


a. 7             b. 6           c. 8.5      d. 7.5

161. Standard deviation is to variability as mean is to __________.


a. coefficient of correlation                  c. discrimination index
b. central tendency                               d. level of difficulty

162. What measure of central tendency does the number 16 represent in the following data: 14, 15, 17, 16, 19, 20, 16, 14, 16?
a. Mode c. Mode and median
b. Median d. Mean

163. Which one can enhance the comparability of grades?


a. Using common conversion table for translating test scores in to ratings
b. Formulating tests that vary from one teacher to another
c. Allowing individual teachers to determine factors for rating
d. Individual teachers giving weights to factors considered for rating

164. Which describes norm-referenced grading?


a. The performance of the group
b. What constitutes a perfect score
c. The students' past performance
d. An absolute standard

165. The search for related literature by accessing several databases by the use of a telephone line to connect a computer library with other
computers that have database is termed __________.
a. compact disc search c. on-line search
b. manual search         d. computer search

166. Two students are given the WISE II. One has a full scale IQ of 91, while the other has an IQ of 109. Which conclusion can be drawn?
a. The second student has significantly higher intellectual ability
b. The first student is probably below average, while the second has above average potential
c. Both students are functioning in the average range of intellectual ability
d. Another IQ test should be given to truly assess their intellectual potential

167. Which type of report refers to "on-the-spot" description of some incident, episode or occurrence that is being observed and recorded as
being of possible significance?
a. Autobiographical report
b. Biographical report
c. Value and interest report
d. Anecdotal report

168. The best way for a guidance counselor to begin to develop study skills and habits in underachieving student would be to __________.
a. have these underachieving students observe the study habits of excelling students
b. encourage students to talk about study habits from their own experiences
c. have them view film strips about various study approaches
d. give out a list of effective study approaches

169. Which illustrates a developmental approach in guidance and counseling?


a. Spotting on students in need of guidance
b. Teaching students how to interact in a positive manner
c. Acting as a mediator
d. Making the decision for the confused student

170. Who among the following needs less verbal counseling but needs more concrete and operational forms of assistance? The child who
__________.
a. has mental retardation
b. has attention-deficit disorder
c. has learning disability
d. has conduct disorder

171. The cultivation of reflective and meditative skills in teaching is an influence of __________.
a. Shintoism         c. Confucianism
b. Zen Buddhism d. Taoism

172. Helping in the development of graduates who are "maka-Diyos" is an influence of


a. naturalistic morality
b. classical Christian morality
c. situational morality
d. dialectical morality

173. The attention to the development of a deep respect and affection for our rich cultural past is an influence of __________.
a. Confucius c. Teilhard de Chardin
b. Hegel d. Dewey

174. Whose teaching is in support of "Education for All" (EFA), he asserted that in teaching there should be no distinction of social classes.
a. Sun Yat Sen c. Mencius
b. Confucius d. Lao tsu

175. We encounter people whose prayer goes like this: "O God, if there is a God; save my soul, if I have a soul" From whom is this prayer?
a. Stoic c. Agnostic   
b. Empiricist d. Skeptic

176. How would you select the most fit in government positions? Applying Confucius teachings, which would be the answer?
a. By course accreditation of an accrediting body
b. By merit system and course accreditation
c. By merit system
d. By government examinations

177. Whose influence is the education program that puts emphasis on self-development. through the classics, music, and rituals?
a. Buddha         c. Confucius
b. Mohammed d. Lao tsu

178. Your teacher is of the opinion that the world and everything in it are ever changing and so teaches you the skill to cope with change. What
is his governing philosophy?
a. Idealism         c. Experimentalism
b. Existentalism d. Realism

179. Value clarification as a strategy in Values Education classes is anchored on which philosophy?
a. Existentialism c. Idealism
b. Christian philosophy d. Hedonism

180. A guest speaker in one graduation rites told his audience: "Reminder, you are what you choose to be." The guest speaker is more of a/an
__________.
a. realistic         c. idealist
b. pragmatist d. existentialist

181. "All men are pretty much alike. It is only by custom that they are set apart", said one Oriental philosopher. Where can this thought be most
inspiring?
a. In a multi-cultural group of learners
b. In multi-cultural and heterogeneous groups of learners and indigenous peoples' group
c. In a class composed of indigenous peoples
d. In heterogeneous class of learners

182. From whom do we owe the theory of deductive interference as illustrated in syllogisms?
a. Plato c. Aristotle
b. Scorates         d. Pythagoras

183. Teacher A knows of the illegal activities of a neighbor but keeps quiet in order not to be involved in any investigation. Which foundational
principle of morality does Teacher A fail to apply?
a. The end does not justify the means.
b. The principle of double-effect
c. Always do what is right.
d. Between two evils, do the lesser evil.

184. Teacher A is directed to pass an undeserving student with a death threat. Which advice will a hedonist give?
a. Pass the student. Why suffer the threat?
b. Don't pass him. You surely will not like someone to give you a death threat in order to pass.
c. Don't pass him. Live by your principle of justice. You will get reward, if not in this life, in the next!
d. Pass the student. That will be of use to the student, his parents and you.

185. History books used in schools are replete with events portraying defeats and weaknesses of the Filipino as a people. How should you tackle
them in the classroom?
a. Present them and express your feelings of shame.
b. Present facts and use them as means in inspiring your class to learn from them.
c. Present them and blame those people responsible or those who have contributed.
d. Present them as they are presented, and tell the class to accept reality.

186. If you agree with Rizal on how you can contribute to our nation's redemption, which should you work for?
a. Opening our doors to foreign influence
b. Upgrading the quality of the Filipino through education
c. Stabilizing the political situation
d. Gaining economic recovery

187. Rights and duties are correlative. This means that __________.
a. rights and duties regulate the relationship of men in society
b. rights and duties arise from natural law
c. each right carries with it one or several corresponding duties
d. rights and duties ultimately come from God

188. A teacher who equates authority with power does NOT __________.
a. shame
b. develop self-respect in every pupil
c. retaliate
d. intimidate

189. Which is a true foundation of the social order?


a. Obedient citizenry
b. The reciprocation of rights and duties
c. Strong political leadership
d. Equitable distribution of wealth

190. In what way can teachers uphold the highest possible standards of quality education?
a. By continually improving themselves personally and professionally
b. By wearing expensive clothes to change people's poor perception of teachers
c. By working out undeserved promotions
d. By putting down other professions to lift the status of teaching

191. A teacher/student is held responsible for his actions because s/he __________.
a. has instincts c. has a choice
b. is mature          d. has reason

192. The typical autocratic teacher consistently does the following EXCEPT
a. encouraging students. c. ridiculing students.
b. shaming students.      d. intimidating students.

193. What should you do if a parent who is concerned about a grade his child received compared to another student's grade, demands to see
both students' grades?
a. Refuse to show either record.
b. Show both records to him.
c. Refuse to show any record without expressing permission from principal.
d. Show only his child's records.

194. Teacher Q does not want Teacher B to be promoted and so writes an anonymous letter against Teacher B accusing her of fabricated lies
Teacher Q mails this anonymous letter to the Schools Division Superintendent. What should Teacher Q do if she has to act professionally?
a. Submit a signed justifiable criticism against Teacher B, if there is any.
b. Go straight to the Schools Division Superintendent and gives criticism verbally.
c. Hire a group to distribute poison letters against Teacher B for information dissemination.
d. Instigate student activists to read poison letters over the microphone.
195. Teachers often complain of numerous non-teaching assignments that adversely, affect their teaching. Does this mean that teachers must
be preoccupied only with teaching?
a. Yes, if they are given other assignments, justice demands that they be properly compensated.
b. Yes, because other community leaders, not teachers, are tasked to leading community activities
c. NO, because every teacher is expected to provide leadership and initiative in activities for betterment of communities.
d. Yes, because teaching is enough full time job.

196. In a study conducted, the pupils were asked which nationality they preferred, if given a choice. Majority of the pupils wanted to be
Americans. In this case, in which obligation relative to the state, do schools seem to be failing? In their obligation to __________.
a. respect for all duly constituted authorities
b. promote national pride
c. promote obedience to the laws of the state
d. instill allegiance to the Constitution

197. In the Preamble of the Code of Ethics of Professional Teachers, which is NOT said of teachers?
a. LET passers
b. Duly licensed professionals
c. Possess dignity and reputation
d. With high-moral values as well as technical and professional competence

198. Teacher H and Teacher I are rivals for promotion. To gain the favor of the promotional staff, Teacher I offers her beach resort for free for
members of the promotional staff before the ranking. As one of the contenders for promotion, is this becoming of her to do?
a. Yes. This will be professional growth for the promotional staff.
b. No. This may exert undue influence ori the members of the promotional staff and so may fail to promote someone on the basis of merit.
c. Yes. The rare invitation will certainly be welcomed by an overworked promotional staff.
d. Yes. There's nothing wrong with sharing one's blessings.

199. Each teacher is said to be a trustee of the cultural and educational heritage of the nation and is, under obligation to transmit to learners
such heritage. Which practice makes him fulfill such obligation?
a. Use the latest instructional technology.
b. Observe continuing professional education.
c. Use interactive teaching strategies.
d. Study the life of Filipino heroes.

200. Teacher F is newly converted to a religion. Deeply convinced of his new found religion, he starts Monday classes by attacking one religion
and convinces his pupils to attend their religious services on Sundays. Is this in accordance with the Code of Ethics of ¬Professional Teachers?
a. Yes. What he does is values education.
b. No. A teacher should not use his position to proselyte others.
c. Yes. In the name of academic freedom, a teacher can decide what to teach.
d. Yes. What he does strengthens values education.

Answer Key:

151. c

152. c

153. d

154. c

155. a

156. d

157. c
158. d

159. c

160. a

161. b

162. c

163. a

164. a

165. c

166. a

167. c

168. b

169. c

170. b

171. b

172. b

173. a

174. d

175. c

176. c

177. a

178. b

179. b

180. d

181. b

182. a

183. c

184. b

185. b

186. b

187. c

188. b
189. b

190. a

191. c

192. a

193. b

194. a

195. c

196. b

197. a

198. b

199. d

200. b

Professional Education - Drill 8 - Part 1

- December 09, 2018

1.       In mastery learning the definition of an acceptable standard of performance is called

a.       SMART c. Criterion measure

b.       Condition d. Behavior

2.       Under what value measurement instrument does sentence/story completion fall?

a.       Opinionnaire c. Projective technique

b.       Moral dilemma d. Observation guide

3.       Teacher B clears his throat to communicate disapproval of a student’s behavior. Which specific influence technique is this?

a.       Proximity control c. Signal interference

b.       Interest boosting d. Direct appeal

4.       Which is the first step in planning an achievement test?

a.       Select the type of test items to use.

b.       Decide on the length of the test..

c.       Define the instructional objective


d.       Build a table of specification

5.       Which may NOT be a benefit derived from the use of graphic organizers?

a.       Make relationships among detail clear

b.       Enable students to identify important ideas and details

c.       Strengthen team work

d.       Represent stated information in concrete form

6.       Whitehead’s “ educated man” is the man of culture and expertise. Which one most completely describes the “ educated” man according to
Whitehead?

a.       Highly dependent and deeply involved

b.       Critical and deeply involved

c.       Productive and deeply involved

d.       Productive and obeys blindly

7.       Rights which can not be renounced or transferred because they are necessary for the fulfillment of man’s primordial obligations are called

a.       Perfect rights c. Acquired rights

b.       Alienable rights d. Inalienable rights

8.       During Spanish period the medium of instruction in schools was (were)

a.       The vernacular c. English

b.       Spanish and the vernacular d. Spanish

9.       Standard deviation is to variability as mean is to ________.

a.       Discrimination c. Correlation

b.       Level of difficulty d. Central tendency

10.   One strength of an autobiography as a technique for personality appraisal is that

a.       It can replace data obtained from other data-gathering technique

b.       It gives complete data about the author

c.       It makes possible the presentation of intimate experiences that can not be revealed in a face-to-face situation by shy students
d.       It may be read by unauthorized people

11.   If teacher wants to develop in the learner the skills to organize ideas which one should he employ?

a.       Histogram c. Graphic organizer

b.       Venn diagram d. K-W-L technique

12.   Russel thinks that some amount of constraints and discipline is indispensable to education. Therefore, it is necessary for the teacher to.

a.       Allow the child to react in any manner to these constraints

b.       Help the child see the importance of these constraints

c.       Apologize before parents for these necessary constraints

d.       Convince the child that he/she has no choice but to subject himself/herself to these constraints

13.   Which characteristic of a good test will pupils be assured of when a teacher constructs a table of specification for test construction purposes?

a.       Scorability c. Economy

b.       Reliability d. Content validity

14.   To be an effective classroom manager, teacher must be friendly but at the same time be

a.       Highly demanding c. Rigid

b.       Business-like d. Buddy-buddy

15.   In his conduct of item analysis, Teacher H found out that more from the lower group got test item #6 correctly. This means that the test item.

a.       Has a positive discriminating power

b.       Has a negative discriminating power

c.       Has high reliability

d.       Has low validity

16.   Which one will most likely increase student participation?

a.       Feeling or emotions are not permitted in the discussion

b.       The group leader allows quiet members to remain quiet

c.       The teacher models good listening habits

d.       Repeat directions over and over until everyone listens.


17.   Which is implied by a positively skewed score distribution?

a.       The mean, the median, and the mode are equal

b.       Most of the scores are high

c.       The mode is high

d.       Most of the scores are low

18.   Learning takes place best when

a.       Learning exercises are focused on the right side of the brain

b.       Learning exercises involve both sides of the brain

c.       Learning exercises are focused on cognitive objectives

d.       Learning exercises are focused on the left side of the brain

19.   A negative discrimination index means that

a.       Less from the lower group got the test item correctly

b.       More from the lower group answered the test item correctly

c.       The test item could not discriminate between the lower and upper group

d.       More from the upper group answered the test item correctly

20.   Your teacher is of the opinion that the world and everything in it are ever changing and so teaches you the skill to cope with change. Which is his
governing philosophy?

a.       Existentialism c. Idealism

b.       Realism d. Experimentalism

21.   Which is a characteristic of infused or integrated instruction?

a.       Does not allow deviation from intended objective

b.       Includes all thinking process from low level to high level

c.       Excludes related topics

d.       Confines itself to higher level thinking process

22.   Here is a performance objective:” WITH THE AID OF A PERIODIC CHART, the student will list the atomic weights of the first ten elements.” The
capitalized words are referred to as the.
a.       Performance statement c. Behavior

b.       Minimum acceptable performance d. Condition

23.   Which statement about Grade 7 is correct?

a.       Grade 7 was introduced in America but never in the Philippines

b.       Spain introduced Grade 7 in the country

c.       Grade 7 was established by the Education Act of 1940

d.       The 1901 Department of Public Instruction established Grade 7

24.   Marvin, a Grade I pupil, plays with his classmates but can not accept defeat. Based on Piaget’s theory or cognitive development, in what
developmental stage is Marvin?

a.       Concrete operation c. Formal operation

b.       Sensorimotor d. Pre-operational

25.   Which guideline in test construction is NOT observed in this test item? EDGAR ALLAN POE WROTE _____________.

a.       The length of the blank suggests the answer

b.       The central problem is not packed in the stem

c.       It is open to more than one correct answer

d.       The blank is at the end of the question

26.   In her Science class Teacher H uses the process approach. Which order of process will she follow?

I.                     Identifying the problem

II.                   Formulating the hypothesis

III.                 Setting the experiment

IV.                Drawing tentative conclusion

a.       II, III, I, IV c. I, III, II, IV

b.       II, I, III, IV d. I, II, III, IV

27. Whitehead’s “inert ideas” apply when the teacher

a.       Relates what is taught to life concerns


b.       Covers the Philippine Elementary/Secondary Learning Competencies without relating them to daily life

c.       Shows how the learner is concerned with what is taught

d.       Leads the learner to see how all that is taught hangs together in a meaningful whole

28.   For brainstorming to be effective which one should be out?

a.       Non-threatening atmosphere

b.       Teacher’s judgmental attitude

c.       Openness to idea

d.       Making use of other’s ideas shared

29.   Which is the highest level of comprehension?

a.       Critical Comprehension

b.       Critical evaluation

c.       Integration

d.       Literal comprehension

30.   Which statement on spaced and massed learning is CORRECT?

a.       Both massed learning and spaced learning are not effective

b.       Massed learning is better than spaced learning

c.       Massed learning is as effective as spaced learning

d.       Spaced learning is better than massed learning

31.   If teacher wants to elaborate on a central idea by adding details to the central idea which one should he use?

a.       Time line c. Venn diagram

b.       Cluster map d. Story map

32.   Mother equally divided the pineapple juice in two glasses for her two boys. One glass is short but stout; another long but thin. Both boys wanted
the long but thin glass believing that it contained more. In what developmental stage are the boys?

a.       Concrete operation c. Senrorimotor

b.       Formal operation d. Pre-operational


33.   A teacher is a facilitator of learning and of the development of the youth. Which practice is NOT in keeping with his role as facilitator?

a.       Keeps himself abreast with educational trends

b.       Dialogs with parents and with other members of the community

c.       Considers the multiple intelligences of learners

d.       Humiliates misbehaving pupils

34.   What can be said of Jones who obtained a score of 75 in a Grammar Test?

a.       He performed better thanh 75% of his classmates

b.       He answered 75 items in the test correctly

c.       He got a raw score of 75

d.       He answered 75% of the test items correctly

35.   Which does NOT belong to the group?

a.       Completion c. Multiple choice

b.       Matching d. Alternate response

36.   Which best indicates the effectiveness classroom activities?

a.       The utilization of varied techniques and approaches

b.       The laughter and enjoyment of students

c.       The application of concept learned in daily life

d.       The variety of instructional materials used

37.   Which feature was true to Philippine education during the Spanish regime?

a.       Establishment of a normal school

b.       Separation of Church and state

c.       Vernacular as medium of instruction

d.       Emphasis on religious instruction

38.   Q1 is to 25th percentile as median is to

a.       40th percentile c. 50th percentile

b.       75th percentile d. 60th percentile


39.   Which graphic organizer provides a visual representation of analytical thinking where the concept, the focus of analysis, is written at the center
of a circle and major characteristics are placed like the sun’s rays from the center?

a.       Sequence chair                         c. Web

b.       Story map d. Attribute wheel

40.   Which group activity is illustrated when a teacher allows the evaluation of members’ work by the group and encourage the giving of constructive
moment’s and suggestions about ways to improve the work?

a.       Jury trial c. Consensus decision

b.       Critiquing session d. Round table

41.   Which one indicates a teacher’s genuine enthusiasm and pride teaching?

a.       Telling everyone that he went to teaching for there was no other choice then

b.       Belittling the renumeration one gets from teaching

c.       Engaging himself in continuing professional education

d.       Sticking to teaching for the moment that three are no better offers

42.   It is necessary that the parts of a lesson plan from the first to the last have

a.       Completeness c. Symmetry

b.       Conciseness d. Coherence

43.   When is giving praise INEFFECTIVE? When it

a.       Provide information to students about their competence and the value of their accomplishments

b.       Uses the accomplishment of peers as the context for describing a student’s present accomplishments

c.       Shows spontaneity, variety and other signs of credibility

d.       Focuses students attention on their own task-relevant behavior

44.   In writing performance objective which word is NOT acceptable?

a.       Delineate c. Intergrate

b.       Manipulate d. Comprehend

45.   In what way does cultural alienation fail to contribute to nation-building?


a.       The Filipino will look at his culture as something superior over others

b.       The Filipino has no sense of national pride

c.       The Filipino closes himself to foreign influence

d.       The Filipino operates on a Filipino model of development

46.   Just as selected Filipino teachers today are sent abroad to study, the Americans did the same in 1903. These teacher scholars then were known
as:

a.       Insulares c. Peninsulares

b.       Pensionados d. Reformists

47.   Which according to Rizal is the main formula for national redemption?

a.       Gaining economic recovery

b.       Stabilizing the political situation

c.       Opening our doors to foreign influence

d.       Upgrading the quality of the Filipino through education

48.   An American student enrolled in the Philippines is repelled by the Filipinos’ lack of punctuality and use of euphemism. Which term refers to his
experience?

a.       Ethnocentrism c. Xenocentrism

b.       Colonial mentality d. Culture shock

49.   What is claimed to be the overall advantage of criterion-referenced over norm-referenced interpretation?

a.       An individual’s score is compared with the set mastery level

b.       An individual’s score is compared with that of his peers

c.       An individuals score is compared with the average scores

d.       An individual’s score does not need to be compared with any measure

50.   Teacher Y does norm-referenced interpretation of scores. Which of the following does she do?

a.       She uses a specified content as its frame of reference

b.       She describes group performance in relation to a level of mastery set

c.       She compares every individual student scores with others’ scores

d.       She describes what should be their performance


Answer Key:

1.     C
2.     B

3.     C

4.     C

5.     C

6.     B

7.     D

8.     D

9.     D

10.  C

11.  C

12.  B

13.  D

14.  C

15.  A

16.  C

17.  B

18.  C

19.  A

20.  B

21.  B

22.  A

23.  C

24.  D

25.  C

26.  D

27.  D

28.  B

29.  B
30.  A

31.  C

32.  D

33.  D

34.  D

35.  D

36.  B

37.  D

38.  C

39.  D

40.  B

41.  C

42.  A

43.  B

44.  D

45.  A

46.  C

47.  D

48.  C

49.  A

50.  B

Professional Education - Drill 8 - Part 2

- December 09, 2018

51.   Out of the 3 distracters in a multiple choice type of test namely, A, B, and D, no pupil chose D as answer. This implies that D is

a.       A vague alternative

b.       A plausible distracter

c.       An ineffective distracter

d.       An effective distracter


52.   The main purpose of the compulsory study of the Constitution in Philippine schools is to

a.       Prepare students for law-making

b.       Acquaint students with the historical development of the Phil. Constitution

c.       Make constitutional experts of  the students

d.       Develop the students into responsible, thinking citizens

53.   Which must be primarily considered in the choice of an instructional aid?

a.       Must be new and skillfully made

b.       Must be updated and relevant to Filipino setting

c.       Must stimulate and maintain student interest

d.       Must be suited to the lesson objective

54.   In a study pupils were asked about their preference for a mother country. Only 4.83% preferred Philippines. This finding shows that several
pupils

a.       Don’t know their country

b.       Are not sure of what they want

c.       Haven’t developed national consciousness

d.       Are not enlightened citizens

55.   All the examinees obtained scores below the mean. A graphic representation of the score distribution will be:

a.       Negatively skewed

b.       Perfect normal curve

c.       Leptokurtic

d.       Positively skewed

56.   The lesson is on the pros and cons of declaring a state rebellion in the country. Teacher B wants his students to arrive at a view of the
declaration of the state of rebellion from different perspectives. Which technique will be most appropriate?

a.       Role playing               c. Panel discussion

b.       Simulation                 d. Braistorming

57.   Which statement applies when score distribution is negatively skewed?

a.       The mean corresponds to a high value


b.       The mode corresponds to a low value

c.       The median is higher than the mode

d.       The mode and median are equal

58.   In a normal distribution curve a T-score of 70 is:

a.       Two SDs below the mean

b.       Two SDs above the mean

c.       One SD below the mean

d.       One SD above the mean

59.   STUDY THE TABLE ON ITEM ANALYSIS FOR NON-ATTRACTIVENESS AND NON-PLAUSIBILITY OF DISTRACTORS BASED ON THE RESULT OF A
TRYOUT TEST IN MATH. THE LETTER MARKED WITH AN ASTERISK IS THE CORRECT ANSWER

Based on the table which is the most effective distracter?

a.       Option D      c. Option A

b.       Option C      d. Option B

60.   Teacher T wants the group to evaluate an issue by having the group act out a jury trial. Which technique will he employ?

a.       Critiquing                c. Majority-rule decision making

b.       Panel                       d. Simulation

61.   Which activities should a teacher have more for his students if he wants them to develop logical-mathematical thinking?

a.       Storytelling                           c. Independent study

b.       Problem solving                  d.  Drama


62.   Which type of test measures higher order thinking skills?

a.       Enumeration      c. Completion

b.       Matching             d. Analogy

63.   Which is a convergent question?

a.       Did the LA SOLIDARIDAD accomplished its purpose? Why or why not?

b.       Who was the first editor of the LA SOLIDARIDAD?

c.       Why did the LA SOLIDARIDAD come about?

d.       If you were editor of the LA SOLIDARIDAD what would you do?

64.   Here is a performance objective: Given a compass and a straight edge, construct a pentagon within 5 degrees of accuracy on any of the inside or
outside angles. Which is the condition statement?

a.       Within 5 degrees of accuracy

b.       Constrcut a pentagon

c.       On any of the inside or outside angles

d.       Given a compass and a straight edge

65.   Which view on intelligence runs counter to Gardner’s MI theory?

a.       Learners possess multiple intelligence’s

b.       Learners can develop each of the eight multiple intelligence’s

c.       Learners have a single, quantifiable intelligence

d.       Learners are strong in some and weak in other intelligence’s

66.   Some students who are high in the scholastic aptitude tests have failed in college. Some who are below the standards set for admission, but
who, for various reason, were admitted, attained satisfactory standings. This proves that

a.       Aptitude tests can be perfectly relied on

b.       Human beings are certainly predictable

c.       Admission tests are not accurate, hence should not be used

d.       Aptitude tests do not measure all factors important for success

67.   The use of the process approach gives the students the opportunity to
a.       Learn how to learn

b.       Make use of laboratory apparatuses

c.       Apply the scientific method

d.       Learn on their  own

68.   Teacher H and Teacher I are rivals for promotion. To gain the favor of the promotional staff, Teacher I offers her beach resort for free for the
members of the promotional staff before the ranking. As one of the contenders for promotion, is this becoming of her to do?

a.       Yes, the rare invitation will certainly be welcomed by an overworked promotional staff

b.       Yes, this will be professional growth for the promotional staff

c.       Yes, there’s nothing wrong with sharing one’s blessings

d.       No, this may exert undue influence on the members of the promotional staff and so may fail to promote someone on the basis of merit.

69.   A child who gets punished for stealing candy may not steal again immediately. But this does not mean that the child may not steal again. Based
on Thorndike’s theory on punishment and learning, this shows that.

a.       Punishment strengthens a response

b.       Punishment removes a response

c.       Punishment does not remove a response

d.       Punishment weakens a response

70.   Standard deviation is to variability as is to central tendency

a.       Mode                                    c. Range

b.       Quantile                               d. Pearson

71.   A comprehension skill of higher level which may be inferred or implied from reading is

a.       Noting specific details

b.       Picking out the main idea

c.       Following directions

d.       Drawing conclusion

72.   One philosopher considers education as the acquisition of the art  of the utilization of knowledge. This implied that.

a.       A learner’s application of what she has learned is necessary

b.       A learner’s interest in art is commendable


c.       A learner’s acquisition of information is sufficient

d.       A learner’s acquisition of information  is not important

73.   For professional growth which source of teacher performance evaluation is considered more valid and reliable considering time spent together?

a.       Self –evaluation

b.       Supervisory evaluation

c.       Students’ evaluation

d.       Peer evaluation

74.   If you have to vote for the sake of the nation you will vote for

a.       A candidate  who has the making of a public servant

b.       A neighbor who is closet to you in times of need

c.       A candidate who helped you concretely by giving cash

d.       A Ninong from whom you owe your father’s job

75.   Teacher F wanted to teach the pupils the skill to do cross stitching. Her check up quiz was a written test on test on steps of cross stitching. Which
characteristic of a good test does it lack?

a.       Scorability           c. Validity

b.       Objectivity          d. Reliability

76.   The search for related literature by accessing several data bases by the use of a telephone line to connect a computer in a library with other
computers that have database is termed:

a.       Computer search

b.       Compact disc  search

c.       On-line search           

d.     Manual search

77.   The strongest disadvantage of the alternate-response type of  test is:

a.       The demand for critical  thinking

b.       The absence of analysis

c.       The encouragement of rote memory

d.       The high possibility of guessing


78.   Teacher wants his pupils to master the skill in adding unlike fractions. Which teaching method should he use?

a.       Discovery method                   c. Unit  method

b.       Type method                            d. Drill method

79.   If the teacher’s pattern in questioning consist of calling on a student then asking the question.

a.       The student called to answer may be able to think well of his answer

b.       The rest of the class may just dictate the answer

c.       The rest of the class may not engage themselves in thinking of the answer

d.       All students may be encouraged to participate

80.   A student was asked to respond to a series of pictures, ink blots and to similarly ambiguous stimuli for personality appraisal. To what tool was he
subjected?

a. Anecdotal record                  c. Projective technique

b. Sociometry                            d. Problem checklist

81.   The guidance counselor as a social worker means that the guidance counselor will

a.       Spend much time planning, giving, the interpreting tests

b.       Give vocational guidance

c.       Do tutoring

d.       Do counseling with children who are disturbed emotionally

82.   The “brain” of the computer is the

a.       Monitor                                 c. Keyboard

b.       UPS                                        d. CPU

83.   A class is composed of academically poor students. The distribution will most likely to be

a.       Leptokurtic                   c. Skewed to the left

b.       Skewed to the right    d. Very normal

84.   In the Preamble of the Code of the Ethics of Professional Teachers, which is not mentioned about teachers?
a.       Possess dignity and reputation

b.       With high moral values as well as technical and professional competence

c.       LET passers

d.       Duly licensed professionals

85.   According to Confucius, what is the best way to rule a people and attain harmony?

a.       By allowing people to do as they please

b.       By consulting the governed

c.       By force

d.       By moral example

86.   Which technique gives a group an opportunity to listen to other groups’ comments or suggestions regarding the group’s work?

a.       Majority-rule decision-making

b.       Critiquing session

c.       Panel

d.       Forum

87.   Which factor hampers mastery of developmental tasks?

a.       Creativity

b.       Lack of motivation

c.       Absence of physical defect

d.       High level of intelligence

88.   Which is referred to as a list of instructions that the computer carries out one at a time?

a.       Diskette                               c. Computer program

b.       File                                       d. CD

89.   Which statement about Median is CORRECT?

a.       It is measure of variability

b.       It is the most stable measure of central tendency

c.       It is the 50th percentile


d.       It is significantly affected by extrema scores

90.   Why should learning activities be carefully planned?

a.       This is required of a teacher

b.       The ability of the teacher to plan is tested

c.       The accomplishment of objectives is dependent on the plan

d.       This is expected by pupils

91.   In Russel’s negative theory of education”, for which reason must impositions on the learner be minimized?

a.       To allow the learner to do what is required

b.       To train the learner to live within limits

c.       To allow the learner to discover and to inquire

d.       To direct the learner to do the right thing

92.   In order to avoid disgrace, a pregnant, unmarried woman takes drugs to induce abortion. Is she morally justified to do that?

a.       No. The unborn child can  not made to suffer the consequences of the sins of  his parents

b.       Yes, It can save her and the child from disgrace when he grows up

c.       No. The act of inducing abortion is bad in self

d.       No. It is better to prevent the child from coming into the world who will suffer very much due to the absence of a father

93.   In which way does heredity affect the development of the learner?

a.       By compensating for what environment fails to develop

b.       By placing limits beyond which the learner can not develop

c.       By blocking the influence of environment

d.       By providing equal potential

94.   When a teacher teaches the idea that it is wrong to think that Filpino lifestyle, products and ideas are inferior to those of other nationalities he
fights against feelings of

a.       Culture shock                  c. Xenocentrism

b.       Ethnocentrism                d. Acculturation


95.   Which is the best indicator of a well-managed class?

a.       The learners pursue their task without inhibition

b.       The learners are controlled by the teacher

c.       The learners  blindly obey teacher’s instructions

d.       The learners are earnestly engaged in an activity that leads them to realize the staged goal

96.   Which measure(s) of central tendency is (are) most appropriate when the score distribution is not badly skewed?

a.       Median and mode     c. Mode

b.       Mean                           d. Median

97.   Which of the following teacher behaviors my NOT enhance the developmental of high-level thinking skills?

a.       Encouraging credibility as a criterion

b.       Asking convergent question

c.       Making students aware  of their mental process

d.       Teaching for meaning

98.   What does a skewed score distribution mean?

a.       The scores are normally distributed

b.       The mean and the median are equal

c.       The mode, the mean, and the median are equal

d.       The scores are concentrated more at one end or the other end

99.   Which is implied a negatively skewed score distribution?

a.       Most pupils are underachievers

b.       Most of the scores are high

c.       Most of the scores are low

d.       The scores are evenly distribution from the left  to the right

100.   The principle of individual differences requires teaches to

a.       Treat all learners alike while in the classroom

b.        Provide for a variety of learning activities


c.       Give greater attention to gifted learners

d.       Prepare modules for slow learners in class

Answer Key:

51.     B
52.     D

53.     C

54.     C

55.     A

56.     D

57.     D

58.     B

59.     A

60.  B

61.  B

62.  D

63.  C

64.  B

65.  C

66.  D

67.  C

68.  D

69.  D

70.  A

71.  D

72.  A

73.  D

74.  A

75.  C

76.  –

77.  D
78.  D

79.  D

80.  C

81.  D

82.  D

83.  D

84.  C

85.  D

86.  D

87.  B

88.  C

89.  C

90.  C

91.  C

92.  A

93.  C

94.  C

95.  D

96.  C

97.  A

98.  D

99.  B

100.  B

Professional Education - Drill 8 - Part 3

- December 09, 2018

101.   Which activity is are meant for the bodily-kinesthetically intelligent pupils?

a.       Cooperative learning

b.       Individualized instruction

c.       Independent study


d.       Dance

102.   The three-level approach to teaching includes the teaching of facts and skills, concepts, and values. Which teaching is in highest level?

a.       Skills c. Facts

b.       Values d. Concepts

103.   Which statement is NOT true of guidance?

a.       Guidance is concerned with the entire individual

b.       Guidance is an integral part of education

c.       Guidance helps individuals develop the ability to make intelligent choices

d.       Guidance makes choices for individuals

104.   Which on stifles student’s initiative?

a.       Rationalism

b.       Extreme authoritarianism

c.       “Utang-na-loob

d.       “Bahala na”

105.   Which does NOT belong to the group?

a.       Short answer

b.       Restricted-response essay

c.       Completion

d.       Multiple choice

106.   The following are trends in making and reporting system, EXCEPT:

a.       Indicating strong points as well as those needing improvement

b.       Conducting parent-teacher conferences as often as needed

c.       Raising the passing grade from 75 to 80

d.       Supplementing subject grades with checklist on traits


107.   A teacher discovers that a product of a certain bottling company brings about damage to teeth. Much as he wants to share the products of his
research, he could not because of harassment from all sides. Which teacher’s right is violated?

a.       Right to make a livelihood

b.       Academic freedom

c.       Right to one’s honor

d.       Right to property

108.   Which is a type of graph in which lines represent each score or set of scores?

a.       Scattergram c. Frequency polygon

b.       Histogram d. Scatterplot

109.   Which term refers to a model which is constructed so as to emphasize a particular part or function?

a.       Simulation c. Audio recording

b.       Mock up d. Realia

110.   Why can the calculator do arithmetic? Because

a.       A TV inside shows it

b.       A typewriter inside does it

c.       A computer inside the calculator tells it how

d.       A watch inside directs it

111.   On which constitutional provision is the full or partial integration of capable deaf and blind students in the classroom based? The provision on

a.       Creating scholarship for poor and deserving students

b.       Academic freedom

c.       Providing citizenship and vocational training to adult citizens and out-of-school youths

d.       Protecting and promoting the right of all citizens to quality education

112.   Which of the following is (are) the least stable measure(s) of central tendency?

a.       Median c. Mode

b.       Mean d. Mode and median


113.   Teaching in the cognitive, psychomotor, and affective domains is based on the concept that the learner is a

a.       Material and an acting being

b.       Moral and a feeling being

c.       Spiritual and material being

d.       Thinking, feeling, and acting being

114.   Which statement on responsibility is CORRECT?

a.       A person’s degree of responsibility is fixed at birth

b.       A person’s sense of responsibility decreases as he grows in age

c.       A person’s sense of responsibility can not be developed

d.       A person’s sense of responsibility increases as he grows in age

115.   Which may NOT help develop attentive listening in the students?

a.       Conducting oral tests frequently

b.       Assigning student to give a summary of the lesson

c.       Repeating direction, comments, and questions

d.       Asking a student to paraphase a previous student’s response

116.   Which assumption underlies teachers’ use of performance objectives?

a.       Performance objectives assure the learner of learning

b.       Not every form of learning is observable

c.       The success of the learner is based on others’ performance.

d.       Learning is defined as a change in the learner’s observable performance

117.   Which is unethical for teachers to do?

a.       Obeying the legitimate policies of the school administration

b.       Maintaining cordial relation with parents

c.       Refusing to serve in worthwhile neighborhood activities as these will adversely affect her teaching

d.       Conferring with the next of kin about the problems and needs of a student

118.   Which statement is TRUE to guidance?


a.       Guidance is aimed merely at the amelioration of trauma

b.       Guidance is concerned with the maximum development of the individual

c.       Guidance is confined to one type of life situation like social

d.       Guidance is making intelligent choices for confused individuals.

119.   In time of war, soldiers must be ready to give up their lives in defense of the nation. Which principle is applied?

a.       More common good-Public safety before private safety

b.       Closer relationship-Friends before strangers

c.       Wider social order-Family before the individual

d.       Nobler person-God before man

120.   “ What I hear, I forget. What I see, I remember. What I do, I understand.” This means that pupils learn best when they

a.       Learn independently

b.       Work with groups

c.       Watch TV

d.       Take active part in the learning process.

121.   Both Muslim and Christian value marriage but the Muslim practices polygamous marriage while the Christian practices monogamous marriage.
What is this called?

a.       Ethical relativism c. Acculturation

b.       Enculturation d. Cultural relativism

122.   Which of the following is true of the brainstorming technique?

a.       Quality is given the same stress as quantity

b.       Picking up others’ ideas is encouraged.

c.       Only practical ideas are accepted.

d.       Each suggested idea is scrutinized.

123.   Research found out that children learn visual discrimination tasks more rapidly if they talk to themselves. This shows that

a.       Visual discrimination can not be learned without vocalization.

b.       Visual discrimination may be learned without vocalization.


c.       Vocalization assists visual discrimination.

d.       Vocalization does not enhance visual discrimination.

124.   Which one should Teacher Y use if he wants to teach to pupils the relationship among details?

a. Journal entries c. Graphic Organizers

b. Story frame d. Learning log

125.   If teacher M wants to build teamwork among his students which should he use more.

a. Lectures c. Independent study

b. Cooperative learning d. Brainteasers

126.   Teacher C wants to compare 2 concepts. With which technique can he accomplish this best?

a.       Histogram c. Venn diagram

b.       K-W-L technique d. Spider web

127.   The problem method can have an incalculable value for children because

a.       Learning problem-solving skills will eliminate all life problems.

b.       Problem-solving skills are needed for lifetime.

c.       Learning problem solving skills will mean no more misbehaved children.

d.       Life is full of problems.

128.   Test norms are established in order to have a basis for

a.       Establishing learning objectives.

b.       Identifying pupils difficulties

c.       Planning effective instructional devices.

d.       Comparing test scores.

129.   Which is an example of a perfect duty?

a.       Supporting a poor but deserving student to school

b.       Paying the worker the wages agreed upon

c.       Donating an amount for a noble project


d.       Giving alms to the needy

130.   The test item “Group the following items according to shape” is a thought question on

a.       Creating c. Comparing

b.       Generalizing d. Classifying

131.   Marvin, a Grade I pupil, is happy when he wins in a game but sulks when he doesn’t. Which does Marvin’s behavior indicate?

a.       Egotism c. Rigidity of thought

b.       Egocentrism d. Sernilogical reasoning

132.   A melody may sound sad to one and yet when the individual notes are played separately there is nothing sad about it. This viewpoint is based
on the doctrine that

a.       The whole of experience is equal to the sum of its parts.

b.       The whole of experience is more than the sum of its parts.

c.       The whole experience is less than the sum of its parts.

d.       The whole of experience is not in any way related to the sum of its parts.

133.   On which theory is the logical sequencing of curriculum based?

a.       Perennialism c. Essentialism

b.       Progressivism d. Reconstructionism

134.   In what way can instructional aids enhance learning?

a.       Hold students in the classroom

b.       Entertain students

c.       Take the place of the teacher

d.       Reinforce learning

135.   To build a sense of pride among FILIPINO young which should be done?

a.       Re-study our history from the perspective of our colonizers

b.       Replace the study of folklores and myths with technical subjects that make youths globally competitive

c.       Re-study our history and stress on our achievements as a people


d.       Set aside the study of local history

136.   Teachers often complain of numerous non-teaching assignments that adversely affect their teaching. Does this mean that teachers must be
preoccupied only with teaching

a.       Yes, if they are given other assignments justice demands that they be properly compensated.

b.       No, because every teacher is expected to provide leadership and initiative in activities for betterment of communities

c.       Yes, because other community leaders, not teachers, are tasked to lead in community activities

d.       Yes, because teaching is enough full time job

137.   When necessary conditions are present, the use of inductive method is preferred because

a.       It needs only a few instruction materials

b.       There is greater active participation on the part of pupils

c.       It gives the teacher more time to rest

d.       Academic time is used wisely

138.   Which is the best reason why Mr. Cruz begins a lesson in Math by checking and reviewing on the previous day’s assignment and provides
practice and drills?

a.       Prepare the students for the mastery test

b.       Make learning interesting and enjoyable for students

c.       Check if parents guide their children in the making of assignment

d.       Make sure that the students understand the pre-requisite skills of the lesson

139.   Here is a test item: “Jose Rizal was born in Calamba, Laguna on June _________, 1861.” What is wrong with the item?

a.       The blank is very short

b.       It is concerned with trivia

c.       The blank is near the end

d.       It is open to more than one correct answer

140.   How students learn may be more important than what they learn. From this principle, which of the following is particularly important?

a.       Solving a problem within time allotted

b.       Getting the right answer to a word problem

c.       Knowing how to solve a problem


d.       Determining the givens

141.   Authority comes from GOD and is meant to:

a.       Distinguish the powerless from the powerful

b.       Be lorded over others

c.       Be used to exploit others

d.       Be used to serve others

142.   With which function is the left hemisphere of the brain involved?

a.       Intuitive c. Nonverval

b.       Visual d. Logical

143.   The study type of reading exercises gives practice in

a.       Picking out the main ideas

b.       All sorts of study methods

c.       Reading skills needed in other subjects

d.       Recognizing the precise meaning of words

144.   For effective classroom management, when should a teacher undertake the task of setting up of routine activities?

a.       Every Homeroom day

b.       Everyday at the start of the session

c.       As soon as the students have adjusted to their schedules

d.       On the very first day of school

145.   The score distribution follows the normal curve. What does this mean?

a.       Most of the scores are on the –2 SD

b.       Most of the scores are on the + 2 SD

c.       The scores coincide with the mean

d.       Most of the scores pile up between –1 SD and + 1 SD

146.   The computed for English and Math scores is___75. What does this mean?
a.       The higher than scores in Math the higher the scores in English.

b.       Math and English scores are not in any way related

c.       The higher the scores in Math, the lower the scores in English

d.       The lower the scores in Math the lower the scores in English

147.   To which Filipino trait can one attribute a Filipino student’s resistance to scientific methods and unquestioning obedience to authority?

a.       Impersonalism c. Personalism

b.       Rationalism d. Non-rationalism

148.   In her conduct of item analysis, Teacher G found out that a significantly greater number from the upper group of the class got test item # 5
correctly. This means that the test item

a.       Has a negative discriminating power

b.       Is valid

c.       Is easy

d.       Has a positive discriminating power

149.   For counseling to be successful which assumption must be AVOIDED?

a.       The student is willing to participate in the process

b.       The counselor tells the student what to do

c.       The environment must provide assurance of confidentiality

d.       The counselor must be able to relate to the student

150.   Which is selective reading technique meant at getting at important facts very fast?

a.       Silent reading c. Oral reading

b.       Skim reading d. Scanning

Answer key:

101.     D
102.     B

103.     D

104.     B
105.     B

106.     C

107.     A

108.     C

109.     A

110.  C

111.  D

112.  B

113.  D

114.  D

115.  C

116.  A

117.  D

118.  B

119.  A

120.  D

121.  D

122.  D

123.  C

124.  C

125.  B

126.  B

127.  B

128.  B

129.  B

130.  D

131.  C

132.  D

133.  B

134.  D

135.  C
136.  B

137.  B

138.  D

139.  B

140.  C

141.  D

142.  D

143.  C

144.  D

145.  C

146.  B

147.  C

148.  D

149.  B

150.  B

Professional Education - Drill 8 - Part 4

- December 09, 2018

151. Which element should be present for brainstorming to be effective?


a. Lax atmosphere
b. Teacher’s non-judgmental attitude
c. Teacher’s use of    “put down” strategy
d. Threatening atmosphere

152. Which technique is most appropriate when a teacher wants a group to agree on a plan of action?
a. Agenda
b. Composite report
c. Symposium
d. Consensus decision-making

153. Which order is observed in the task analysis model of lesson organization?
a. Facts, principle, concepts, generalization
b. Facts, generalization, concepts, principle
c. Facts, concepts, generalization, principle
d. Facts, concepts, principle, generalization

154. Who are NOT covered by the Code of Ethics of Professional Teachers?
a. Teachers in the tertiary level
b. All full time or part time public and private school teacher and administrators
c. Teachers  in all educational institutions at the preschool, elementary and secondary levels
d. Teachers of academic, vocational, special, technical or non-formal institutions

155. Education in human rights starts with


a. Loving the other
b. Liking the other
c. Caring for the other
d. Respect towards the other

156. What is the advantage of standard scores over percentiles?


a. They have a zero reference point
b. They have scales of equal units
c. They indicate an individual’s relative standing in a group
d. They indicate specific points in the normal curve

157. Teacher Q shares this concept with her students: “What you do not want done to your self do not do to others. “This concept is
a. Buddhist and Confucian
b. Muslim and Christian
c. Christian and Confucian
d. Hindu and Christian

158. A teacher by reason of having gotten tired of elections resulting to no change did not exercise her right of suffrage last May 14, 2001.
Which provision in the Code of Ethics did she fail to observe?
a. Every teacher shall possess and actualize full commitment and  devotion to duty
b. A teacher shall not engage in the promotion of any political, religious, or other partisan interest
c. Every teacher shall vote and shall exercise all other constitutional rights and responsibilities
d. Every teacher shall enjoy academic freedom

159. The Continuous Progression Scheme introduced in the elementary level in the Philippines, in the school year 1970-1971 had as its
objective.
a. Mastery of the 3 r’ s
b. Pupil’s progression according to capacity
c. Mass promotion
d. Access to education

160. To educate the child for freedom is to educate him/her to


I. Respond to himself/herself
II. Respond to others
III. Do as he/she pleases
IV. Live as he/she desires

a. II, III b. I, III          c. I, II                d. I,  IV

161. Peter sees Tony being attached by a bully classmate and realizes that, unless assisted, Tony will be badly beaten up. He decides, however,
not to stop them because Peter enjoys seeing the fight. Did Peter act responsibly?
a. Yes, because by not taking part he prevented the fight from becoming worse
b. Yes, because he did not involve himself in the fight
c. No, Peter’s motive did not lead to any good
d. Yes, because anyway Tony, a bully himself, deserves to be beaten

162. In the problem solving method of teaching, which is the primary role of the teacher?
a. Director c. Lecturer
b. Classifier d. Judge

163. The schema theory asserts that


a. We learn by the process of conditioning
b. We are passive learners
c. We organize what we learn according to patterns
d. We are totally conditioned by our environment

164. Which term applies to the search for related literature by computer access of data bases on discs kept in libraries?
a. Manual research
b. On-line research
c. Compact discs computer research
d. Computer research

165. Which one can enhance the comparability of grades?


a. Individual teachers giving weights to factors considered for rating
b. Using a common conversion table for translating test scores into ratings
c. Allowing individual teaches to determine factors for rating
d. Formulating tests that vary from one teacher to another

166. Lorelie obtained a NSAT percentile rank of 80. This indicates that.
a. She surpassed in performance 80% of his fellow examinees
b. She got a score of 80
c. She surpassed in performance 20% of his fellow examinees
d. She answered 80 items correctly

167. Determining the pronunciation and meaning of words by analyzing roots,  affixes, and derived forms is called
a. Contextual attack                          c. Structure analysis
b. Phonetic analysis                         d. Blending sounds

168. The cultivation of reflective and meditative skills in teaching is an influence of


a. Confucianism c. Shintoism
b. Taoism d. Zen Buddhism

169. Which test item is in the highest level of Bloom’s taxonomy of objectives?
a. Explain how trees receive nutrients
b. Explain how a tree functions in relation to the ecosystem
c. Rate three different methods of controlling tree growth
d. List the parts of a tree

170. If the student is encouraged to develop himself to the fullest, which of Maslow’s hierarchy of needs should he satisfy?
a. Safety needs                              c. Belongingness
b. Physiological needs                  d. Self-actualization

171. When plateau is reached in a student’s learning curve, it is advisable for the learner to.
a. Continue reviewing
b. Give up learning
c. Take a rest
d. Force himself/herself to get interested

172. Based on Piaget’s theory, in what cognitive development stage in the learner who is capable of abstract, hypothetical, and logical
reasoning?
a. Concrete operation stage
b. Pre-operational stage
c. Formal operation stage
d. Sensori-motor stage

173. When teachers are convinced that it is best to teach students the skill to adapt to change since change is the only thing permanent in this
word, they subscribe to the philosophy of
a. Realism c. Pragmatism
b. Existentialism d. Idealism

174. The old adage “ Do not to others what you do not want others do to you” is a teaching of
a. Lao tsu c. Mohammad
b. Confucious d. Buddha
175. Which is the ultimate aim of classroom management?
a. To remove the physical conditions in the room that distract children’s attention
b. To set up conditions that bring about effective teaching and learning
c. To secure conformity to rulers with ease
d. To make children realize that they can not do everything they want

176. Which violates the spirit of Filipinization of educational institution?


a. An educational institution owned by a corporation of  which 40% of the capital is owned by Filipino citizens
b. An educational institution owned by a religious order
c. An American President serving as President of the educational institution
d. An educational institution owned by a charitable institution

177. Which is an element of norm-referenced grading?


a. The student’s past performance
b. An absolute standard
c. The performance of the group
d. What constitutes a perfect score

178. Which can effectively measure students awareness of values?


a. Anecdotal record c. Likert scales
b. Projective techniques d. Moral dilemma

179. Teacher K delegates some of her responsibilities to the whole class. She separate student monitors for attendance, classroom cleaning,
and distribution of learning materials. In which aspect is she good?
a. Instructional planning c. Classroom management
b. Value development d. Classroom teaching

180. The Second World War has caused too much destruction to life and property and the Allied forces consisting of the United States, Great
Britain and France wanted to end it but Japan consistently refused. To force Japan’s surrender the Allied forces dropped the first atomic bomb
on Hiroshima and Nagasaki, Japan on August 9, 1945. Which principle of morality applies in this situation?
a. The end justices the means
b. Always do what is right
c. The end does not justify the means
d. Between two evils, do the lesser evil

181. Of the following methods used to discover the interests of people, which does NOT belong to the group?
a. Analyzing activities that an individual performs
b. Using interests tests and inventories
c. Using anecdotal records
d. Asking individuals what they like to do

182. A child refuses to obey orders or displays negativism as a development trait. How may you best handle him?
a. Insist on compliance to the same degree required of pupils.
b. Take every opportunity to praise him for every positive attitude displays
c. Avoid giving him orders or if you do and he objects take back the order
d. Detain him after office hours for him to do what he has been ordered to do.

183. Which process enhances the comparatibility of grades?


a. Determining the level of difficulty of the test
b. Using a table of specifications
c. Giving more high level questions
d. Constructing departmentalized examinationsm for each subject area

184. Royalty rights acquired through the authorship of a book are examples of
a. Imperfect rights                                    c. Inalienable rights
b. Alienable rights                                   d. Acquired rights
185. For grades to be made valid indicators of students achievements which process should be observed?
a. Adopting letter grades such as A,B,C and D
b. Defining the course objectives as intended learning outcomes
c. Explaining the meaning of marks or grades
d. Giving objective type of tests

186. Using the six descriptions of the elements of a good short story, IDENTIFY IN WRITING THE SIX ELEMENTS IN THE SHORT STORY BY O
HENRY with complete accuracy. The capitalized words are referred to as the
a. Minimum acceptable performance   
b. Performance statement          
c. Criterion success
d. Condition

187. For comparing and contrasting, which graphic organizer is most appropriate?
a. Story map c. Web
b. Venn diagram d. Cycle

188. Under which teaching strategy does a School’s Division practice of assigning a Girl Scout to serve as Superintendent of the Day or Major of
the Day for leadership training fall?
a. Symposium c. Panel discussion
b. Simulation d. Dramarization

189. Which is characteristic of the country’s educational system during the American regime?
a. Religious c. Filipinistic
b. Centralized d. Decentralized

190. Which term refers to the collection of student’s products and accomplishments for a period for evaluation purposes?
a. Anecdotal record                              c. Observation report
b. Portfolio                                          d. Diary

191. With Republic Act 7836 the licensure exams for teachers is with
a. Commission on Higher Education
b. Professional Regulation Commission
c. Department of Education, Culture, and Sports
d. Civil Service Commission

192. Which one helps produce an environment conducive for learning?


a. Cooperative learning
b. Long assignments
c. Individual competition
d. Excessive praise

193. With which goals of educational institutions as provided for by the Constitution is the development of work skills aligned?
a. To teach the duties of citizenship’s
b. To develop moral character
c. To inculcate love of country
d. To develop vocational efficiency

194. Which human right does libel violate?


a. The right to one’s own life
b. The right to one’s honor
c. The right to make a livelihood
d. The right to property

195. Teacher U wants to make use of a structured format where two positions on a controversial issue are presented formally and each speaker
is a given a center amount of  time to state a position. Which technique should he use?
a. Simulation c. Debate
b. Symposium d. Critiquing session

196. A test is considered relievable if


a. It is easy to score
b. It serves the purpose for which it is constructed
c. It is consistent and stable
d. It is easy to administer

197. A class is composed to bright students. The distribution will most likely be
a. Platykurtic c. Skewed to the left
b. Skewed to the right                        d. Very normal

198. An arsonist is seen preparing to burn a public market. Can he be shot to death in the name of self-defense?
a. Yes, but the act of killing can not be called self-defense
b. Yes, the good to be defended is of great importance and is proportionate to the evil done by the aggressor
c. No, shooting him to death is not proportionate to the arson he is preparing to do
d. No, in no way is killing justified, only God can take away life

199. If a student’s appropriate behavior is mild and that it appears that the misbehavior will not spread to others, it is sometimes best for the
teacher not to make a fuss of it. Which influence technique is this?
a. Antiseptic bouncing
b. Planned ignoring
c. Proximity control
d. Signal interference

200. Which materials consist of instructional units that cater to the different needs and the varying mental levels of pupils?
a. Multi-level materials
b. Multi-grade materials
c. Minimum learning competencies
d. Plantilla

Answer Key:

151. B

152. D

153. C

154. D

155. D

156. D

157. C

158. A

159. A

160. C

161. C

162. D
163. C

164. C

165. D

166. A

167. B

168. D

169. B

170. D

171. C

172. C

173. A

174. B

175. B

176. A

177. C

178. D

179. C

180. D

181. C

182. B

183. D

184. D

185. B

186. D

187. B

188. B

189. B

190. B

191. B

192. –

193. D
194. B

195. C

196. C

197. B

198. C

199. –

200. A

Professional Education - Drill 7 - Part 1

- December 09, 2018

1.    “Do not teach too many subjects. What you teach, teach thoroughly”, said one philosopher. The primary reason behind the advice is to ensure
that teachers?

a.         Cover all the PELCs / PSLCs.

b.         Relate their lessons to their students daily life

c.         Avoid failing grades

d.         Avoid spoon feeding their students

2.    A student passes a book report poorly written but ornately presented in a folder to make up in the poor quality of the book report content.
Which Filipino trait does this practice prove? Emphasis on

a.         “ Porma” over substance

b.         art over academics

c.         substance over “porma”

d. art over science

3.    Extreme authoritarianism in the home reinforces learners to

a.         work creatively

b.         depend on others for directions

c.         direct themselves

d.         do things on their own initiative

4.    Which did the Americans teach the Filipinos which the Spaniards did NOT?

a.         Their government c. Their language


b.         Their art d. Their religion

5.    The back-to-basic curriculum is in essence a (an) ___________ curriculum

a.         Existentialist c. essentialist

b.         progressivist d. perennialist

6.    In the most recent curriculum introduced in 2002, Makabayan as a subject

a.         Singles out values education

b.         Serve as a practice environment for holistic learning

c.         Compartmentalizes subjects excluding English, Filipino, Science and Math

d.         Integrates English, Filipino, Science and Math

7.    “ Approach every pupil as she is/he is without allowing yourself to be influenced by your foreknowledge of his/her home background” is an
advice from a (an)

a.         essentialist b. rationalist c. povitivist

d. existentialist-phenomenologist

8.    The Filipino learner envisioned by the Department of Education is one who is imbued with the desirable values of a person who is

a.         makabayan, makasarili, makakalikasan, and maka-Diyos

b.         makabayan, makatao, makahalaman, at maka-Diyos

c.         makabayan, makakaragatan, makatao, at maka-Diyos

d.         makabayan, makatao, makakalikasan at maka-Diyos

9.    Sex education in schools implies that

I.      man is educable

II.         main is free in the choice of his sexual expression

III.  man has pure sexual instinct

a. I,II,III b. I,II c. II,III d. I,III

10.  How is the latest curriculum introduced in the elementary and secondary schools called?

a.         The Re-structured Basic Education and Curricula


b.         2002 Elementary and Secondary Education Curricula

c.         Millennium Curriculum

d.         2002 Basic Education Curriculum

11.  “ Sometimes it may be better to make wrong decisions when a decision is urgent than to make a “ right” decision too late” is a thought
encouraged by the

a.         rationalist c. realist

b.         progressivis d. existentialist

12.  Which does pataasan ng ihi mentality negate?

a.         Initiative c. Teamwork

b.         Flexibility d. Religiosity

13.  To whom do Filipinos owe the widespread Philippine educational system today?

a.         The First Filipino political leaders’

b.         The Americans

c.         The Japanese

d.         Spaniards

14.  Which is NOT a characteristic of the 2002 Basic Education Curriculum?

a.         More integrated

b.         More holistic

c.         More compartmentized

d.         Make use of interdisciplinary approach

15.  All of these prevent the emergence of the truth which learners and teachers are in search of EXCEPT a ( an)

a.         Mindset c. open mind

b.         “mask” d. defense

16.  Teaching health in schools is backed up by the concept that

a.         man has non-tangible and transcending dimensions


b.         man’s body is the expression of his soul

c.         man has a dual nature d. man is nothing but matter

17.  Before the Thomasites arrived in the Philippines, who served as teachers to the colonials during the American regime?

a.         The Elementary Teacher’s Certificate graduates of the normal school

b.         The American Soldiers d. The government officials

c.         The selected Grade 7 graduates

18.  Who were Thomasites?

a.         The soldiers who doubted the success of the public educational system to be set in the Philippines

b.         The first American teacher recruit to help establish the public educational system in the Philippines

c.         The first religious group who came to the Philippines on board the US transport Thomas

d.         The devotees to St. Thomas Aquinas who came to evangelized

19.  The American teachers who were recruited to help set the public educational system in the Philippines during the American regime were called
Thomasites because:

a.         They arrived in the Philippines on the feast of St. Thomas

b.         They first taught at the University of Sto. Tomas

c.         They were devotees of St. Thomas Aquinas

d.         They disembarked from the CIS transport called Thomas

20.  A group of Filipino children served as respondents in a research conducted. The children were asked they tell what they wanted to be, if given
the choice. None of them said “ to be an American, to be a Japanese, to be a Korean, etc. What does this finding show?

a.         Inferiority of other nationalities

b.         Superiority of the Filipino

c.         Superiority of other nationalities

d.         Filipino lack of a sense of national pride

21.  An “ I –thou” relationship exist between teacher and students when there is

a.         Favoritism b. pretension c. respect d. prejudice


22.  Which educational trend is occurring in all modern societies as a result of knowledge explosion and rapid social, technological and economic
changes?

a.         International education

b.         Lifelong learning

c.         Nuclear education

d.         Team teaching

23.  One learns by association and also by insight. This shows that the association and cognitive theories of learning are

a.         diametrically opposed

b.         complementary

c.         partly wrong

d.         partly correct

24.  A pupil who has developed a love for reading keeps on reading for her enjoyment. His motivation for reading is

a.         both intrinsic and extrinsic

b.         extrinsic c. intrinsic d. insufficient

25.  Which is the most effective way(s) of retaining learned material?

I.                    hearing it

II.                  seeing it I

III.                notes complication

a.      I and II c. all of these

b.      II only d. II and III

26.  Where are multi-media resources and computers, which is the most favorable result of the optimal use of educational media technology?

a.         Increased learning

b.         Speed learning

c.         Interactive learning

d.         More interesting learning

27.  The right hemisphere of the brain is involved with the following functions EXCEPT
a.         nonverbal functions

b.         intuitive functions

c.         detail-oriented functions

d.         visual functions

28.  What functions are associated with the right brain?

a.         Visual, intuitive, logical

b.         Visual, intuitive, non-verbal

c.         Visual, non-verbal, logical

d.         Visual, logical, detail oriented

29.  John Watson said: “Men are built not born.” What does this statement point to?

a.         The ineffectiveness of training on a person’s development

b.         The effect of heredity

c.         The absence of genetic influence on a person’s development

d.         The effect of environmental stimulation on a person’s development

30.  Under which type of guidance service does the concern of schools to put students into their most appropriate courses fall?

a.         Research service

b.         Individual inventory service

c.         Placement service

d.         Information service

31.  Which is an advantage of teacher made tests over those of standardized tests?

a.         highly reliable

b.         better adapted to the needs of the pupils

c.         highly valid d. more objectively

32.  With the number of senses to be stimulated to criterion, which one should be first in the list?

a.         Audio aid c. Visual aid

b.         Audio-visual aid d. Multi-sensory aid


33.  Which refers to the Filipino trait of practicing conflicting values in different venues with different social groups?

a.         crab mentality

b.         “kanya-kanya” mentality

c.         Procrastination

d.         Split personality

34.  Richard excels in classifying different types of leaves and rocks. He loves to collect specimens and catalogs them. His strength is

a.         naturalistic intelligence c. spatial intelligence

b.         intrapersonal intelligence d. existential intelligence

35.  Students C says “ bahala na” and braves the storm for a test. In this situation which is expressed by the students “ bahala na”?

a.         lack of self-reliance c. Lack of foresight

b.         Willingness to take risks d. Indolence

36.  It is easy for children to learn language because each person as a Language Acquisition Device that predisposes one to acquire language. This
theory is espoused by________.

a.         Watson b. Gardener c. Chomsky d. Piaget

37.  Miss Cortez is teaching a three-year-old boy how to put on his shirt. She might first reward him for placing his right arm in the right sleeve, then
the left arm min the left sleeve, then buttoning the front of the shirt, then tucking the shirt into his pants. This technique is called.

a.         conditioning c. chaining

b.         fading d. reinforcement

38.  According to Erikson’s theory, the child aged three to five is largely

a.         Mischievous c. lazy

b.         ego-centric d. altruistic

39.  Teacher S wants to show to the class a magnified picture of Mayon Volcano mounted on a bond paper. Which one will she use?

a.         Overhead projector c. Slide

b.         Opaque projector d. Filmstrip


40.  Teacher V wants to check prior knowledge of his pupils about water pollution. She writes the main topic water pollution in the center of the
chalkboard and encircles it. Then she asks the pupils to provide information that can be clustered around the main topic. Which technique did
the teacher employ?

a.         Deductive teaching c. Semantic mapping

b.         Demonstration d. Vocabulary building

41.  After having been humiliated by his teacher, student B evaluates that teacher very poorly despite teacher’s excellent performance. Which trait
is illustrated by student B’s behavior?

a.         Rationalism c. Personalism

b.         Impersonalism d. Particularism

42.  In her desire to help the children build their vocabulary, she maintains a small board at the corner of her classroom for the Word of Day to
teach the children one new word each day. Which method of vocabulary- building does she employ?

a.         Incidental attention to building meaning

b.         All of these

c.         Direct vocabulary instruction

d. Wide reading

43.  Conducting follow up studies of graduates and drop out is guidance service that falls under

a.         individual inventory service

b.         research service

c.         counseling service

d.         placement service

44.  Teacher F helps pupils determine the pronunciation and meanings of words by analyzing roots, affixes, and derived forms. This process is called

a.         contextual attack c. blending sounds

b.         phonetic analysis d. structural analysis

45.  Researches conducted show that teachers expectancies of students often become self-fulfilling prophecies. What is this phenomenon called?

a.         Ripple effect c. Pygmalion effect

b.         Hawthrone effect d. Halo effect


46.  Age two (2) is usually described as the “ terrible 2’s” because, according to Erikson, at this stage the child tends to be

a.         Inquisitive c. playful

b.         Sickly d. assertive in words and in actions

47.  Peter is able to gain a lot of friends. He is well-linked and popular among other students. He has a natural flare for making people laugh and is
almost always ready to help them. He possesses what is called:

a.         naturalistic intelligence

b.         spatial intelligence

c.         intra personal intelligence

d.         interpersonal intelligence

48.  Jones, age 7, was removed from her class frequent fights with other children. She refused to be disciplined and was to focus in class only for
very short time. She also has frequent tantrums. She is suffering from

a.         mental retardation

b.         down syndrome

c.         attention-deficit hyperactivity disorder

d. learning disability

49.  Which ones are two-dimensional representations of earth’s geographic and /or political features?

a.         Mock ups b. Models c. Globes d. Maps

50.  Studies in the area of neurosciences disclosed that the human brain has limitless capacity. What does this imply?

a.         Pupils can possibly reach a point where they have learned everything

b.         Every child is a potential genius

c.         Every pupil has his own native ability and his learning is limited to this native ability

d.         Some pupils are admittedly not capable of learning

Answer key:

1.   A

2.   A
3.   B

4.   A

5.   C

6.   C

7.   B

8.   D

9.   B

10.D

11.B

12.C

13.B

14.C

15.B

16.C

17.C

18.B

19.B

20.D

21.B

22.C

23.A

24.B

25.C

26.C

27.C

28.B

29.D

30.B

31.C

32.B

33.C
34.B

35.A

36.B

37.A

38.B

39.B

40.A

41.C

42.D

43.A

44.D

45.C

46.C

47.C

48.C

49.D

50.B

Professional Education - Drill 7 - Part 2

- December 09, 2018

1.    Through the enactment of republic Act 7836, the regulation and licensing of teachers is now with the:

a.      Professional regulation commission

b.      Commission on Higher education

c.       Department of education, Culture and Sport

d.      Civil Service Commission

2.    Which characterizes the perfections type of student?

a.         Often anxious, fearful, or frustrated about quality of work


b.         Does not volunteer or initiate

c.         Gives up easily

d.         rarely completes tasks

3.    Which of the following statements is NOT TRUE with respect to physical force or violence?

a.         Scold a hungry student for his lack of attention

b.         Believe in their students

c.         Critisize the misbehavior not the student who misbehaves

d.         Structure the classroom environment to make it supportive

4.    Which of the following group activities provides an open environment in which group members can discuss their opinions without being judged
as wrong?

a.         Panel c. Buzz session

b.         Forum d. Philips 66

5.    Results in NEAT and NSAT for the past years show that achievement has highest in:

a.         Science

b.         Heograpiya, Kasaysayan at Sibika

c.         English

d.         Filipino

6.    Which is the least authentic mode of assessment?

a.         experiments in science to assess skill in the use of scientific method

b.         oral performance to assess student’s spoken communication skills

c.         paper-and-pencil test in vocabulary

d.         artistic production for music or art subject

7.    The Philosophers that maintain that “ Truth exists in an objective order that is independent of the knower are the:

a.         pragmatists c. existentialists

b.         idealists d. realists


8.    The following are four levels of computer use. You are able to use the computer as a tool for particular purposes. In what level are you?

a.         computer hacker

b.         computer expertise

c.         Computer literacy

d.         computer competency

9.    A teacher who equates authority with power usually does the following, Except:

a.         shames

b.         retaliates

c.         develops self respect in every pupil

d.         intimidates

10.  Which type of test is least useful in educational diagnosis?

a.         Multiple choice item c. Matching item

b.         True-false item d. Short answer item

11.  In a microcomputer system the actual machine itself is called the:

a.         CPU c. hardware

b.         software d. peripheral

12.  Which interactive teaching should be AVOIDED?

a.         using multiple-response strategy

b.         using “ put-down” strategy

c.         asking more divergent questions

d.         asking more evaluative questions

13.  Teacher C wants to develop in his pupils the skill to organizes ideas. Which activity will fit?

a.         simulation c. Brainstroming

b.         Game d. concept mapping

14.  The process whereby a culture is transmitted from members of one society to another is called:
a.         integration c. acculturation

b.         assimilation d. enculturation

15.  One aim of La Liga Filipina, i.e., to encourage popular education is in line with the present Phil. Government in education to promote

a.         lifelong and quality

b.         effectiveness & efficiency

c.         relevance and quality

d.         access and equity

16.  Teacher T wants group members to give their full attention to what one individual wants to express. This is done by the whole group seated in a
circle while 2 chairs are placed in the center of the circle for two members to converse while the others listen. What is this technique called?

a.    Round table c. Forum

b.    Symposium d. Fish bowl technique

17.  Confucius asserted that in teaching there should be no distinction of classes. Confucius’ teaching is in support of

a.      Moral recovery program

b.      Back-to –the-basics

c.       Education for all (EFA)

d.      Values Education

18.  The first American teachers in the Philippines were

a.         Elementary graduates

b.         Graduates of the normal school

c.         Missionaries

d.         Soldiers

19.  Which instructional aid requires pupils to perform?

a.      mock up c. pantomime

b.      film d. comic books

20.  Which among the following objectives in the psychomotor domain is highest in level?
a.      To contract a muscle

b.      To run a100 meter dash

c.       To distinguish distant and close sounds

d.      To dance the basic steps of the waltz

21.  The project method is considered valuable for the following reasons EXCEPT one:

a.Pupils develop alertness, open-mindedness and tolerance

b.Pupils are trained to be responsible

c.  Motive for learning is developed

d.Movement among pupils is not free and free discussion is almost nil

22.  The use of drills in the classroom is rooted on Thorndike’s law of

a.         Belongingness c. Exercise

b.         Readiness d. Effect

23.  For wholistic learning, it is urgent to utilize both sides of the brain; the effective as well as the cognitive, the logical along with the

a.         Behavioral c. Musical

b.         Intuitive d. Rational

24.  Which instructional aid requires pupils to verbalize?

a.         Diorama c. Sociodrama

b.         Brainstorming d. Graph

25.  Education during the revolutionary period in the Philippines then was focused on the

a.         Mastery of the Spanish language

b.         Strengthening of the moral fiber of the people

c.         Mastery of the basic skills

d.         Establishment of nationhood

26.  Gagne delineates 9 external events in sequential instructional planning. Which are the first and the last in the sequence?

a.      Informing students of the lesson objectives-enhancing the retention and transfer of learning
b.      Informing students of the lesson objectives-assessing student performance

c.       Gaining attention- enhancing the retention and transfer of learning

d.      Stimulating recall of previous learning- assessing student performance

27.  Students non-verbal behavior affects student-teacher interaction. An example is a student’s choice of seat at the beginning of the year
influences the teacher’s impression of how responsive the student is. Under which category of non-verbal communication does the example
fall?

a.         Attentiveness c. Timing

b.         Location/proximity d. Disruptive behavior

28.  In instructional planning, which among these three; unit plan, course plan, lesson plan is (are) most specific? _______ plans.

a.      Course b.      Unit           c. Lesson             d. Course and lesson

29.  If a student thinks about thinking, he is involved in the process called:

a.      creative thinking c. higher-order thinking

b.      metacognition d. critical thinking

30.  The following are some drill techniques EXCEPT:

a.      Assigning exercises from a workbook

b.      Challenging students to be above the level of the class

c.       Giving short quiz and having students grade papers

d.      Asking papers to repeat answers

31.  For instructions to be effective it must be interactive. Which then should be avoided by teachers?

a.      Teacher episode c. Non-verbal cues

b.      Teacher asking questions                  d. Teacher monologue

32.  Which is NOT a basic component of a lesson plan?

a.      Evaluation                   b. Assignment c. Resources                 d. Content

33.  To improve students’ attentive behavior, which should be vigilant about?

a.      Raising a hand to volunteer a response


b.      Turning around to listen to a student who is speaking

c.       Maintaining eye contact with the teacher

d.      Doodling with a pencil

34.  One criterion for content to be developed in a lesson is LEARNABILITY. What does the capitalized letter word mean?

a.      The content should be worthwhile

b.      The content should be within the capacity of the students to learn

c.       Resources and materials for the development of the content are available

d.      The content should be useful or practical in some situation

35.  Learning is enhanced when

a.      the learner competes with his classmates

b.      the teacher expresses his enlargement with low scores

c.       the expectation of success is high

d.      the expectations is under stress and is challenged

36.  Teacher V teaches the pupils to find the main idea, sequence events, find the details and read creatively and critically. What skills does Teacher
V teach?

a.      Utilization skills                     c. Comprehension skills

b.      Study skills d. Word recognition skill

37.  Vygotsky claimed that social interaction is important for learning. What does this imply?

a.      Children are independent problem solvers

b.      Children learn from adults and other children

c.       Since they are not capable of interaction, children in the crib has no learning yet

d.      Children learn well by passive presentation of information

38.  Teacher Z has a variety of instructional materials at her disposal for a lesson on marine life. Which one should she consider first and foremost?

a.      Pupil’s ability c. Pupil’s interest

b.      lesson objective d. Topic


39.  Which were features of Educational Decree of 1863?

I.     The church’s active involvement in the public educational system

II.   Segregation of boys and girls in the school

III.Few benefits and privileges for teachers

IV.Absence of a system of teacher promotion

a.         II,III b. I,II c. I,IV d. I,III

40.  Which one explains curriculum alignment correctly?

a.      Teachers emphasize different learning experiences based upon their skills and interests.

b.      Teachers plan activities considering learner’s multiple intelligences and varied learning styles

c.       Objectives match with assessment

d.      Objectives match instruction and assessments

41.  Positive interdependence as an element of collaborative learning means that the students must

a.         help one another in the individual test for everyone to pass

b.         depend on the diligent student

c.         be grouped heterogeneously

d.         learn to depend on each other to achieve a goal

42.  Teacher E wants to encourage collaborative learning in her class. Which one should do to heighten positive interdependence

a.      Emphasize the idea that the success of the group depends on the diligent student

b.      Works for homogeneous grouping

c.       Provide each student with one set of resources

43.  Which one can help the student develop the habit of critical thinking?

a.    asking low level questions

b.    a willingness to suspend judgment until sufficient evidence id presented

c.    Asking convergent questions

d.    Blind obedience to authority

44.  Which materials consists of instructional units that cater to the different needs and the varying mental levels pupils?
a.      Minimum learning competencies

b.      Multi-level materials

c.       Plantilia

d.      Multi-grade materials

45.  Which is a teaching approach for kindergartens that makes the real world experiences of the child the focal point of educational stimulation?

a.    Montessori approach

b.    Traditional approach

c.    Eclectic approach

d.    Situation approach

46.  Gray defines reading a four-step process. In which order do the fur steps come?

I.           Integration II. reaction

II.         Perception IV. Comprehension

a. I-II-III-I b. II-III-IV-I c. III-IV-I-II d. II-IV-III-I

47.  Teacher M wants to develop in her pupils comprehension skills. What order of skills will she develop?

I.                    literal comprehension

II.                  interpretation

III.                critical evaluation

IV.                integration

a.II-III-IV-I                    b. IV-III-II-I c. III-IV-I-II                    d. I-II-III-IV

48.  An integrative, conceptual approach introduced by Roldan has as its highest level is the development of _______ thinking skills

a.    Interpretative c. Creative

b.    Literal d. Critical

49.  The use of mnemonics helps a pupil _______ information

a.      Analyze c. Apply

b.      Understand d. Remember


50.  which schools are subjects to supervision, regulation and control by the state?

a.      public, private sectarian and non-sectarian schools

b.      public schools

c.       private schools

d.      sectarian and non-sectarian schools

Answer key:

1.   A
2.   A

3.   A

4.   C

5.   D

6.   C

7.   B

8.   C

9.   C

10.B

11.C

12.B

13.D

14.C

15.A

16.D

17.C

18.D

19.C

20.D

21.D

22.C

23.B
24.B

25.B

26.C

27.B

28.C

29.D

30.C

31.D

32.C

33.D

34.B

35.C

36.D

37.B

38.B

39.B

40.D

41.D

42.C

43.B

44.B

45.A

46.D

47.D

48.D

49.D

50.B

Professional Education - Drill 7 - Part 3

- December 09, 2018


1.    Which graphic organizers are used to show event in chronological order?

a.      Series of events chart and story map

b.      Time line and cycle

c.       Time line and series of events chart

d.      Time line and story map

2.    Which program was adopted to provide universal access to basic education to eradicate illiteracy?

a.“ Paaralan –sa-bawat-Barangay”

b.Education for all

c.  Values Education Framework

d.Science and Education Development Plan

3.    Teacher M instructs her class to refer to the dictionary in correcting their spelled words. Which practice is exemplified?

a.      curriculum idigenization   

b.      computed-assisted instruction

c.       multi-level instruction    

d.      self-evaluation

4.    Which note-taking technique includes the use of various sizes of circle to indicate the degree of the importance and connectedness of ideas?

a.    Venn diagram                c. Comparison matrix

b.    Fishbone diagram        d. Webbing

5.    The free public elementary and secondary education in the country is in line with the government effort to address educational problems of

a.      effectiveness and efficiency                   c. productivity

b.      relevance and quality                             d. access and equity

6.    Classroom discussion should end with a final summary also known as

a.    internal summaries                   c. organizers

b.    medial summaries                    d. postorganizers

7.    Teacher wants his pupils to master the skill in adding unlike fractions. Which method should he use?
a.         Unit method                          c. Laboratory method

b.         Drill method                          d. Discovery method

8.    Which is correct statement about teaching method?

a.         There is one best method for teaching

b.         Indirect instructional method is teacher-centered

c.         There is a best method of teaching

d.         No one teaching method can accomplish every goal

9.    Science Teacher C introduces a topic on earthquakes, then help her students determine what they need to know about earthquakes, then help
her students determine what they need to know about earthquakes and the activities and resources which will help them learn. Which
technique does Teacher C employ?

a.         Individualized instruction

b.         Computer assisted instructions

c.         Self-directed learning

d.         Collaborative learning

10.  Teacher X assesses her pupil’s needs then cerates that everyone has something to share and may learn from one another. What is the teacher
preparing the class for?

a.         Buzz grouping                                         c. Peer tutoring

b.         Remedial instructions                           d. Ability grouping

11.  Teacher asked the pupils these questions after reading A New Home for Ruben. “ What glimpses of rural life   of rural life did you get from the
story? How do you compare them with yours?”

a.         Critical evaluation                                  c. Interpretation

b.         Application                                              d. Literal comprehension

12.  In the K-W-L technique K stands for what the pupil already knows what he really wants to know and L for what he

a.         failed to learn                                         c. needs to learn

b.         he likes to learn                                     d. learned

13.  Teacher Z shows a histogram on Filipino population growth form 1990 to 2001. Seeing the pattern of growth, pupil is asked to tell what will
happen in the next two years. What skill is the pupil asked to display?

a.         Inferring                                                c. Extrapolating


b.         Predicting                                             d. Drawing conclusion

14.  Which theory stresses the importance of using a pre-reading strategy that encourages students to use their own experiences to predict and
evaluate the problems an action of characters in the stories they read?

a.         Metacognition theory 

b.         Text structure theory  

c.         Vocabulary theory

d.         Scheme theory

15.  Which questioning technique would be appropriate for inductive lessons?

a.         Use questions requiring only memory responses

b.         Expect participation only among the more motivated students

c.         Involve students actively in the questioning process

d.         As teacher, you ask no questions

16.  The Philippine Constipation directs the teaching of religion in public schools on the following conditions EXCEPT

a.         option is expressed in writing

b.         without cost to the government

c.         with cost shouldered by the parents or guardians       

d.     given only  at the option

17.  What type of lesson takes place when teacher dwells on the same lesson but present it in a different way?

a.         Developmental                                     c. Review

b.         Deductive                                             d. Inductive

18.  Marvin has had difficulty getting right solution to a problem in algebra. Suddenly he “saw” how to solve the problem. Which of the following
explains this situation?

a.         Revelation              b. Insight                 c. Retention                  d. Memory

19.  Which one will likely contribute to the effectiveness of small discussion?

a.         repeating directions over and over

b.         Developing a plan for getting non-volunteers to respond


c.         Threatening atmosphere

d.         Volunteerism is the rule of contributing to the discussion

20.  Which one is benefit derived from the use of performance objectives?

a.         A panacea to class management problems

b.         Makes it easier to measure student achievement

c.         Makes possible value integration

d.         Makes selection of instructional materials easier

21.  In research, the definition of key terms used is

a.         Literal                                            c. operational

b.         Conceptual                                  d. figurative

22.  After a lesson AIDS, the students are asked  to summarize the lesson in one long sentence by answering : Who?; Does what or whom?; When?;
Where?; How?; Why?; This assessment technique evaluates students ability to

a.         Evaluate                                        c. synthesize

b.         Analyze                                         d. recall

23.  On the first day of class after initial introductions, the teacher administered a Misconception/ Preconception check. She explained that she
wanted to know what the class as a whole already knew about the Philippines before the Spaniards came. This misconception/Preconception
check is a form of

a.         diagnostic test                               c. criterion-referenced test

b.         placement test                              d. achievement test

24.  In research which is the dependent variable?

a.         Stimulus                                           c. attribute

b.         Input                                                d. predictor

25.  Which types of statistics give (s) information about the sample being studied?

a.         Inferential and correlational

b.         Inferential

c.         Descriptive
d.         Correlational

26.  Which happens when score distribution is positively skewed?

a.         The median will be higher than the mean and the mode

b.         The mode corresponds to a low value

c.         The mean will have a high value

d.         The mode corresponds to a high value

27.  Teacher A wrote of Henry: “ When Henry come to the class this morning, he seemed very tired and slouched into his seat. He took no part in
class discussion and seemed to have no interest in what was being discussed. This was very unusual, for he has always been eager to participate
and often monopolized the discussion time.” What Teacher A wrote is an example of a (an)

a.         incidence report                             c. personality report

b.         anecdotal report                            d. observation report

28.  What is a simple frequency distribution? A graphic representation of

a.         Means           

b.         standard deviations 

c.         lowest and highest

d.         score raw scores

29.  About what percent of the cases falls between +2 and –1 in a normal curve?

a.         43.1%                b. 95.4 %                 c. 99.8%                     d. 68.2 %

30.  Which error do teachers commit when they tend to  overrate the achievement student identified by aptitude tests as gifted because they
expect achievement and giftedness to go together?

a.         Generosity error                                  c. Severity error

b.         Central tendency error                       d. Logical error

31.  What is the mean of this score distribution: 4,5,6,7,8,9,10?

a. 8.5                    b. 6                         c. 7.5                            d. 7

32.  Which statement correctly applies to a student who got score of 72 in the test?
a.         He surpassed the score of 72 students

b.         He correctly answered 72% of the items in the test

c.         He obtained a raw score of 72 items in the test

d.         He answered only 72 items in the test

33.  Which measure (s) of central tendency can be determine by inspection?

a. Median                        b. Mode                   c. Mead                   d. Mode & Median

34.  When a significantly greater number form the lower group gets a test item correctly, this implies that the test items

a.         is very valid   

b.         is not very valid        

c.         is not highly reliable

d.         is highly reliable

35.  Teacher C adds the number of cases 1 over 2 obtain

a.         Mode                                          c. median and mode

b.         Median                                      d. mean

36.  Like a typewriter, a computer has a

a.         desk drive                                  c. screen  

b.         keyboard                                   d. cursor

37.  The following demand criterion –referenced tests, EXCEPT

a.         Personalized System of Instruction

b.         Outcome –based education

c.         Collaborative learning

d.         Mastery learning

38.  Read this test item then answer the questions below, WHAT IS THE END PUNCTUATION FOR AN INTERROGATIVE SENTENCE?

a.         Quotation marks       

b.         Comma

c.         Exclamation point


d.      Semicolon   

e.      Questions marks

39.  It is not wise to laugh at a two-year-old child when he utters bad word because in his stage he is learning to

a.         consider other views

b.         distinguish right and wrong

c.         socialize   

d.     distinguish sex differences

40.  In which subjects can learning outcomes be best evaluated by criterion-reference tests?

a.         In subjects where speed and accuracy of learner’s response are emphasized

b.         In all subjects

c.         In subjects where the acquisition of cognitive and psychomotor abilities are emphasized

d.         In subjects where objects are hierarchically arranged

41.  In a Science class test, one group had a range within  the top quarter of 15 points  and another group on the same measurement had a range of
30 points. Which statement applies?

a.         The first group has variability twice as great as the second group within the top quarter

b.         The first group is more varied than the second group

c.          The second group has a variability twice as a great as the first group within the top quarter

d.         the second group does not differ from the first group of variability

42.  Which applies when all high frequencies are not adjacent?

a.         The score distribution will neither be bimodal nor multimodal

b.         There is no point of score  concentration

c.         The score distribution will be concentrated

d.         There will only be one point of score concentration

43.  Mothers who demand their 3 to 5 children to spend their time in serious academic stud forget that early childhood is the.

a.         Questioning age                              c. initiative age

b.         Pre-school age                                d. Toy age


44.  Teacher A wants to make use of the most stable measure of variability? Which one(s) should you recommend?

a.         external range and quartile range

b.         standard deviation

c.         quartile range      

d.     external range

45.  Which refers to a single word or phrase that tells the computer to do something with a program or file?

a.         computer program   

b.         command      

c.         password

d.         Computer language

46.  To determine student’s entry knowledge and skills which test should be given?

a.         Aptitude                                            c. diagnostic

b.         standard                                           d.  placement

47.  Which is (are) signs of the student with Attention deficit Disorder?

a.         Cares for his/personal things

b.         Impatient while waiting for his/her turn during games

c.         Completes work before shifting to another

d.         Excessively quite

48.  After scoring , Teacher G got the difference of the highest and the lowest scores in each class. What did she compute?

a.         range of the lowest fourth

b.         standard deviation    

c.         range of the middle 50%

d.     external range

49.  After scoring, teacher G got the difference of the highest and the lowest scores  in each class. What did she compute?

a.         Porfolio assessment is dynamic assessment


b.         Assessment should stress the reproduction of knowledge

c.         An individual learner is inadequately characterized by a test score

d.         An individual learner is adequately characterized by a test score

50.  What is the mode in the following score distribution: 96,97,98,97,93,90,89,97,81,80?

       a. 96                             b. 98                           c. 97                           d. 83

Answer Key:

1.   C
2.   B

3.   D

4.   A

5.   D

6.   C

7.   B

8.   C

9.   D

10.A

11.B

12.D

13.C

14.D

15.C

16.B

17.C

18.B

19.B

20.B

21.C

22.C

23.A

24.A
25.C

26.B

27.D

28.D

29.B

30.D

31.D

32.C

33.B

34.C

35.D

36.B

37.C

38.E

39.B

40.D

41.C

42.B

43.D

44.B

45.B

46.C

47.B

48.D

49.–

50.C

Professional Education - Drill 7 - Part 4

- December 09, 2018

1.    If a teacher is concerned with the development of students’ higher order thinking skills, his lesson objectives must go beyond.
a.         Analysis c. comprehension

b.         recall d. application

2.    Which statement about guidanc4 is FALSE?

a.         A guidance program is inherent in every school

b.         Guidance is a function of the entire school

c.      The classroom teacher is not part of the school guidance program since she/he is not trained to be a guidance counselor

d.      Guidance embraces curriculum, teaching, supervision and all other activities of the school guidance

3.    Which statement on the counseling is FALSE?

a.         The ultimate goal of counseling is greater happiness on the part of counselee.

b.         For counseling to be successful, the counselee is willing to participate in the process

c.         The school counselor is primarily responsible for counseling

d.         Counseling is the program that includes guidance

4.    A test item has a difficulty index of .81 and discrimination index of.13. What should the test constructor do?

a.         Retain the item c. Revise the item

b.         Make it a bonus a item           d. Reject the item

5.    Continuous evaluation of the guidance program in one service of guidance programs. Under which guidance service does it fall?

a.         Individual inventory service

b.         Research service

c.         Placement service

d.     Educational service

6.    Shown a picture of children in sweaters inside the classroom, the students were asked this question: “ In what kind of climate of this children
live?” This is a thought question on

a.         Creating c. predicting

b.         Interfering d. applying

7.    Which is the final, indispensable component of a lesson plan?


a.         Assignment c. Activity

b.         References d. Evaluation

8.    Which multiple choice type of test has options that are based on a graphical presentation of data or on a paragraph or picture ?

a.         setting and options test

b.         structured –response test

c.         contained options test

d.         stem and options test

9.    For mastery learning which type of testing will be most fit?

a.         Formative testing

b.         Criterion-reference testing

c.         Aptitude testing

d.         Norm reference testing

10.  Which is a characteristics of an imperfect type of matching test?

a.         An item may have no answer at all

b.         The items in the right and left columns are equal in number

c.         An answer maybe repeated

d.         There are two or more distracters

11.  The following are necessary for successful counseling ECEPT:

a.         counselee willingness to participate

b.         atmosphere of confidentiality

c.         professionalism

d.         buddy-buddy relation between counselor and counselee

12.  Which is included in item analysis?

a.         Determining the percentage equivalent of the cut off score

b.         Identifying the highest score

c.         Determining the cut off score


d.         Determining the effectiveness of distracters

13.  Teacher Z wants to check whether or not his students learned what he intended to teach and so formulated a quiz that should be based on his

a.         topic(s) c. students ability

b.         extra readings d. instructional objectives

14.  If teacher wants to test student’s ability to organize ideas, which type of test should she formulate?

a.         technical problem type c. Short answer

b.         Essay d. Multiple-choice type

15.  Teacher should not be a slave of his lesson plan. This means that

a.         a teacher must be ready to depart from her lesson plan, if she remembers something more interesting than what she earlier planned

b.         teacher is the best lesson plan designer

c.         a lesson plan must be followed by a teacher no matter what

d.         a teacher must be willing other than her intended lesson

16.  The first thing to do in constructing a periodic test is for a teacher to

a.         decide on the type of test to construct

b.         go back to her instructional objective

c.         decide on the number of items for the test

d.     study the content

17.  The criterion for success in Teacher D’s objective is that “ the pupils must be able to spell 90% of the words correctly” Ana and 19 others
correctly spelled 40 words only out of 50. This means that Teacher D

a.         attained his objective because of her effective spelling drill

b.         attained his lesson objective

c.         failed to attain his lesson objective as far as the twenty pupils are concerned

d.         did not attain his lesson objective because of the pupil’s lack of attention

18.  A positive discrimination index means that

a.         the test item could not discriminate between the lower and upper groups
b.         more from the upper group got to he item correctly

c.         more from the lower group of the item correctly

d.         the test item has low reliability

19.  Why are test norms established? To have a basis for

a.         computing grades

b.         establishing learning goals

c.         identifying pupil’s difficulties

d.         interpreting test results

20.  When points in the scattergram are spread evenly in all directions this means that

a.         the correlation between twp variables is positive

b.         the correlation between two variables is low

c.         the correlation between two variables is high

d.         there is no correlation between two variables

21.  The more the learner feels what he is learning the better the learning. Therefore the teachers should take up lessons that

a.         are easy to comprehend

b.         appeal to the learner’s imagination

c.         portray complex ideas

d.         have significance for the learner

22.  The whole child of education is contrary to the

a.         focus on the disadvantaged approach to education

b.         the mental discipline approach to education

c.         progressivist approach to education

d.         academic essentials approach to education

23.  The strengthening of liberal education which includes classical literature in the curriculum is based on the thought of the

a.         Rationalist c. humanists

b.         Hedonists d. stoics


24.  Which one is rejected by communism but is accepted by fascism?

a.         Dictatorship c. No opposition

b.         Private ownership d. Personal liberty

25.  To provide for individual differences how is curriculum designed?

a.         Some degree of flexibility is provided

b.         Realistic and meaningful experiences are provided

c.         Minimum learning competencies are included

d.     Social skills are emphasized

26.  The emphasizes given on respect and care for the aged of Values Education classes goes along with the teachings of an Oriental philosopher by
the name of _______.

a.         Hsun tsu c. Kung-fu-tsu

b.         Mo Tsu d., Lao tsu

27.  Which philiosphy approves of a teacher whole lectures most of the time and requires his students to memorize the rules of grammar?

a.         Pragmantism c. Realism

b.         Existentialism d. Idealism

28.  The current emphasizes on the development of critical thinking by the use of philosophic methods that emphasize debate and discussion began
with

a.         Confucius c. Aristotle

b.         Socrates d. Plato

29.  The inclusion of Logic the curriculum is perhaps an influence of the importance of logic that _______ stressed

a.         St. Augustine c. The hedonists

b.         The humanists d. The scholastics

30.  You observe that pupils answer even when not called, shout MA’AM to get your attention, and laugh when someone commits mistakes. What
should you do?

a.         Send the misbehaving pupils to the guidance counselor


b.         Involve the whole class in setting rules of conduct for the whole class

c.         Make a report to the parents about their children’s misbehavior

d.         Set the rules for the for the class to observe

31.  A political boss builds a school in a distant barrio in order to get the votes for an unworthy and corrupt candidate. Is the action of the political
boss moral?

a.         No, the candidate is undeserving

b.         Yes, the votes were in exchange for the whole class.

c.         Yes, it was his duty to strategize for his candidate to win

d.         No, his motive was not meant for a good effect

32.  Can an insane person be blamed for killing a stranger?

a.         Yes, because an insane person possesses a little degree of voluntariness

b.         No, because the one killed is a stranger, not in any way related to hi.

c.         Yes, because an insane person because is not totally ignorant

d.         No, because of his ignorance and lack voluntariness

33.  My right ends where the right of the other begins. What does this mean?

a.         Rights are absolute

b.         Rights are inalienable

c.         Rights are alienable

d.         Rights are not absolute

34.  The teacher’s first task in the selection of media in teaching is to determine the

a.         availability of the media

b.         technique to be used

c.         objectives of the lesson

d.         choice of the students

35.  which criterion should guide a teacher in the choice of instructional devices?

a.         Attrcativeness c. Appropriateness


b.         Novelty d. Cost

36.  Which one does NOT illustrate the principle that rights and duties are correlative?

a.         The right a living wage involves the duty of the employer to give the salary agreed upon and the duty of the employee to give a fair amount of
work

b.         The right of the state to compel citizens to military service is reciprocated by the duty of the state to protect the citizens

c.         The right of an unmarried pregnant woman to abort her baby in relation to hr duty to protect her name

d.         The right to life on the part of one person corresponds to the obligation on the part of other persons to respect such a right

37.  Which economic system upholds the unlimited right to private property and pursuit of profit to the exclusion of other considerations?

a.         Moderate capitalism

b.         Moderate capitalism

c.         Exaggerated capitalism d. Communism

38.  Which statement on human rights is CORRECT?

a.         Human rights are unlimited

b.         Human right is a moral power not a physical power

c.         All human rights are inalienable

d.         Human right is might

39.  Which is TRUE foundation of the social order?

a.         Equitable distribution of health

b.         The reciprocation of rights and duties

c.         Obedient citizenry

d.         Strong political leadership

40.  Which is an example of an alienable right?

a.         Right to possess a house through inheritance

b.         Right to life

c.         Right to alms

d.         Right to marry


41.  The worker’s right to form unions or to strike can be suppressed in times of national emergency. On what norm is this based?

a.         Clearer title- the certain before the little

b.         Higher law-inalienable rights before alienable

c.         Wider social order- the family before the individual

d.         Nobler person person-God before man

42.  with which is true authority equated?

a.         Power c. Coercion

b.         Suppression d. Service

43.  Why should the teacher take the obligation upon himself to study and understand the custom and traditions of the community where he
works? In order to

a.         identify the weaknesses of the culture of the community

b.         have a sympathetic attitude for the people of the community

c.         please the people of the community

d.         change the culture of the community

44.  Which one should teacher AVOID to produce an environment conducive for learning?

a.         Tests c. Individual competition

b.         Seat plan d. Games

45.  In a study conducted, the pupils were asked which nationality they preferred if given a choice. Majority of the pupils wanted to be Americans. In
this case, in which obligation relative to the state are schools seemed to be failing? In their obligation to

a.         promote obedience to the to the laws of the state

b.         instill allegiance to the Consultation

c.         respect for all duly constituted authorities

d.         promote national pride

46.  Which skills should be taught if teacher wants to equip his students with the skill to organize information gathered?

a.         Note taking, outlining, using library

b.         Summarizing note taking using the library


c.         Note taking, outlining, summarizing

d.         Outlining, summarizing, using the card catalogue

47.  Which practice negates teacher’s role as facilitator of learning?

a.         Teacher does more talk so learners talk less

b.         Teacher make use of interactive teaching strategies

c.         Teacher caters to multiple intelligences in the classroom

d.         Teacher does less talk for learners to talk more

48.  Annual medical check up required of teachers is done in the interest of

a.         parents

b.         school administrators

c.         Filipino medical doctors

d.         The state and of every teacher

49.  Which activity is most fit if teacher his students to appreciate the value of teamwork?

a.         Brainstorming c. Field trip

b.         Group game d. Buzz group

50.  The information processing psychology asserts that

a.         the learner is a passive receiver of stimuli

b.         the learner is totally conditioned by environment

c.         learning is purely a conditioning process

d.         learning is an interactive process between the learner and the environment

Answer Key:

1.   B
2.   C

3.   C

4.   C

5.   B
6.   B

7.   A

8.   B

9.   B

10.D

11.D

12.D

13.D

14.B

15.A

16.B

17.C

18.B

19.D

20.D

21.D

22.C

23.B

24.B

25.D

26.D

27.B

28.D

29.D

30.D

31.D

32.C

33.D

34.C

35.C

36.C
37.D

38.C

39.B

40.A

41.C

42.A

43.B

44.B

45.D

46.C

47.A

48.D

49.B

50.D

Professional Education - Drill 6 - Part 1

- November 29, 2018

1.        Devices can make a lecture more understandable and meaningful. What is the most important thing a teacher should consider in
the selection and utilization of instructional materials?

A.       Objectives of the lessons C. Attractiveness of instructional materials


B.       Availability of instructional materials D. Degree of interest on the part of the students

2.        Teacher E asks student A to identify and analyse events, ideas or objects in order to state their similarities and difference. In which part
of the lesson does said activity take place?

A.       Preparation B. Generalization C. Application D. Comparison & abstraction

3.        Which part of the lesson is involved in giving of situation or activities based on the concepts learned?

A.       Preparation B. Generalization C. Application D. Comparison & abstraction

4.        Teacher F wants the class to find out the effect of heat on matter. Which method will help him accomplish his objective?

A.       Project method B. Laboratory method C. Problem method D. Expository method

5.        In Math, teacher G presents various examples of plane figures to her class. Afterwards, she asks the students to give the definition of
each. What method did she use?

A.       Inductive B. Laboratory C. Deductive D. Expository


6.        Teaching Tinkling to I – Maliksi becomes possible through the use of

A.       Inductive method B. Expository method C. Demonstration method D. Laboratory method

7.        What is the implication of using a method that focuses on the why rather than the how?

A.       There is best method


B.       A typical one will be good for any subject
C.        These methods should be standardized for different subjects
D.       Teaching methods should favour inquiry and problem solving

8.        When using problem solving method, the teacher can

A.       Set up the problem C. Propose ways of obtaining the needed data
B.       Test the conclusion D. Help the learners define what is to be solved

9.        Which part of the lesson does the learner give a synthesis of the things learned?

A.       Motivation B. Application C. Evaluation D. Generalization

10.    The strategy of teaching which makes of old concept of “each-one-teach-one” is similar to

A.       Peer learning B. Independent learning C. Partner learning D. Cooperative learning


11.    When using instructional material, what should the teacher primarily consider?

A.       The material must be new and skilfully made C. The material must stimulate and maintain students’ interest
B.       It must be suited to the lesson objective D. It must be updated and relevant to Filipino setting

12.    Which is NOT a provision for the development of each learner in a good curriculum?

A.       Extensive arrangements are made for the educational diagnosis of individual learners
B.       Self-directed, independent study is encouraged wherever possible and advisable
C.        Self-motivation and self-evaluation are stimulated and emphasized throughout the learning opportunities of the school
D.       The program provides a wide range of opportunities for individuals with the same abilities , needs and interests

13.    In the elementary level, English literature and Social studies relate well. While history is being studied, different literary pieces during the
historical period is being studied as well. What curriculum design is shown here?

A.       Separate Subject design B. Discipline design C. Correlation design D. Broad field design

14.    Ms. Mateo, a History teacher considers the element of time in arranging the content of her lessons in World History. What way of
establishing sequence is given emphasis by Ms Mateo?

A.       Simple to complex B. Part to whole C. Concrete to abstract D. Chronological

15.    Teacher Dominguito believes that a new respect for the child is fundamental in curriculum. Thus, all activities in the classroom are
geared towards the development of the child- the centre of the educative process. To which approach in curriculum does Teacher
Dominguito adhere?

A.       Learner-centered B. Subject-centered C. Problem-centered D. Pragmatic

16.    Which curriculum design element is taking place when Eduardo, a 4 th year student can connect the lessons he learned in a subject area to
a related content in another subject area?

A.       Articulation B. Balance C. Continuity D. Integration

17.    The following curricular changes too place in what particular period? Restore Grade VII, double-single session was abolished and more
textbooks were written by Filipino authors
A.       American period B. Philippine republic C. Japanese occupation D. New society

18.    What process is being undertaken by curriculum developers when they enrich or modify certain aspects of a particular program without
changing its fundamental conceptions?

A.       Curriculum improvement C. Curriculum design


B.       Curriculum change D. Curriculum implementation

19.    What refers to the authenticity of the content selected by the curriculum developer?

A.       Feasibility B. Learn ability C. Significance D. Validity

20.    Which is NOT a component of curriculum designing?

A.       Objective B. Learning content C. Learning experiences D. Diagnosis of needs

21.    What do you call the curriculum when the teacher puts into action all the different planned activities planned activities in the classroom?

A.       Recommended curriculum C. Taught curriculum


B.       Written curriculum D. Supported curriculum

22.    Prof. Delos Santos is thinking of an online learning approach by which content provides links to information at other locations and serves
as a focal point for a distance education experience. Which of the following should she use?

A.       Teleconferencing B. Self-paced program C. Web-based instruction D. Computer-aided instruction

23.    With the increasing use of educational technology inside the classroom, what role is expected of the teacher?

A.       Facilitator B. Researcher C. Knowledge giver D. Source of information

24.    It is impractical to bring real objects to the classroom s Aaron constructed a three-dimensional visual instead. Which of the following did
he construct?

A.       Chart B. Cartoon C. Model D. Graphic organizer

25.    Prof. Mandanas would like to use an audio compact disc in teaching a lesson in Filipino. In which activity in the teaching-learning process
is it very effective?

A.       In developing listening skills B. In teaching creative writing C. In composing poems D. In building concepts.

26.    If I have to use the most authentic method of assessment, which of these procedures should I consider?

A.       Traditional Test C. Written test


B.       Performance-based assessment D. Objective assessment

27.    After doing the exercise on verbs, Ms. Borillo gave a short quiz to find out how well the students have understood the lesson. What type
of assessment was done?

A.       Summative assessment B. Formative assessment C. Diagnostic assessment D. Placement


assessment

28.    Ms. Ortega tasked her students to show how to play basketball. What learning target is she assessing?

A.       Knowledge B. Reasoning C. Skills D. Products

29.    What type of validity does the pre-board examination possesses if its results can explain how the student will likely perform in their
licensure examination?
A.       Concurrent B. Predictive C. Construct D. Content

30.    Which term refers to the collection of student’s product and accomplishments in a given period for evaluation purposes?

A.       Diary B. Portfolio C. Anecdotal record D. Observation report

31.    Mrs. Pua is judging the worth of the project of the students in her Science class based in a set of criteria. What process describes what she
is doing?

A.       Testing B. Measuring C. Evaluating D. Assessing

32.    Ms. Ricaforte uses alternative methods of assessment. Which of the following will her NOT likely use?

A.       Multiple choice test B. Reflective journal writing C. Oral presentation D. Developing portfolios

33.    Mrs. Nogueras is doing an assessment OF learning. At what stage of instruction should she do it?

A.       Before instruction B. After instruction C. Prior to instruction D. During the instructional process

34.    In a positively skewed distribution, the following statements are true EXCEPT

A.       Median is higher than the mode C. Mean is lower than the mode
B.       Mean is higher than the median D. Mean is not lower them the mode

35.    If quartile deviation is to median, what is to mean?

A.       Standard deviation B. Mode C. Range D. Variance

36.    Miss Cortez administered a test to her class and the result is positively skewed. What kind of test do you think Miss Cortez gave to her
pupils?

A.       Post-test B. Pertest C. Mastery test D. Criterion-referenced test

37.    The result of the test given by teacher A showed a negatively skewed distribution. What kind of test did teacher a give?

A.       The test is difficultB. It is not too easy or too difficult C. It is moderately difficult D. It is easy

38.    Standard deviation is to variability as mode to _______________

A.       Correlation B. Discrimination C. Central tendency D. Level of difficulty

39.    Goring perform better than 65% of the total number of examinees in the district achievement test. What is his percentile rank?

A.       P35 B. P65 C. P66 D. P75

40.    Which is a guidance function of a test?

A.       Identifying pupils who need corrective teaching C. Assigning marks for courses taken
B.       Redacting success in future academic and vocational education D. Grouping pupils for instruction within a class

41.    Mr. Labanga, an elementary school teacher in Science found out that many of his pupils got very high scores in the test. What measure of
central tendency should he use to describe their average performance in the subject?

A.       Mean B. Median C. Mode D. Range

42.    Which of the following indicates how compressed or expanded the distribution of scores is?
A.       Measure of position B. Measure of central tendency C. Measures of correlation D. Measure of variability

43.    Mr. Gringo tried to correlate the scores of his pupils in Social studies test with their grades in the same subject last 3 rd quarter. What test
validity is he trying to establish?

A.       Content validity B. Construct validity C. Concurrent validity D. Criterion-related validity

44.    In his second item analysis, Mr. Gonzales found out that more from the lower group got the test item 15 correctly. What does this mean?

A.       The item has become more valid C. The item has a positive discriminating power
B.       The item has become more reliable D. The item has a negative discriminating power

45.    Mr. Lorenzo would always give the chapter test on a Friday. What schedule of reinforcement is used by Mr. Lorenzo?

A.       Fixed Interval B. Variable Interval C. Fixed Ratio D. Variable Ratio

46.  Here is a test item: “The improvement of basic education should be the top priority of the Philippine Government. Defend or defy this
position.” What type of question is this?

A.       Analysis B. Convergent C. Evaluative D. Low level

47.  Student A wishes to write a lesson plan. Which question s/he asks herself first?

A.       What material will I need? C. What do I want to accomplish


B.       How will I get things started? D. What exercises will I give my students?

48.  Which of the following characterizes best an effective classroom manager? One who is friendly yet

A.       Rigid B. demanding C. business-like D. buddy-buddy

49.  Which of the classroom activity below is effective?

A.       The concept learned is applicable to daily life C. The techniques and approaches used are varied
B.       The variety of instructional materials used is evident D. The laughter and enjoyment of students are contagious

50.  Which of the following characterizes beat a well-managed class? Which learners?

A. Are controlled by the teacher C. Blindly obey teacher’s instructions


B. Pursue their task without inhibition D. Are engaged in an activity that leads them to realize
the set goal

Professional Education - Drill 6 - Part 2

- November 29, 2018

1.     Which of the following belongs to a lower-order thinking skills?

A. Teaching for meaning C. Encouraging creativity


B. Asking convergent question D. Making the students aware of their mental processes

2.     When should Teacher M undertake the task of setting up routine activities?

A.       Every homeroom period C. On the very first day of school


B.       Every day at the start of the session D. As soon as the students have adjusted to their schedule

3.     Which of the following marks a conducive environment?

A.       Excessive praise B. long assignments C. Individual competition D. cooperative learning

4.     Which of the following helps develop critical thinking?

A. Asking low level questions C. Blind obedience authority


B. Asking convergent questions D. Willingness to suspend judgement until sufficient evidence is presented

5.     What design element established the vertical linkage form level to level to avoid getting gaps and wasteful overlaps?

A.       Articulation B. Balance C. Scope D. Sequence

6.     What refers to the authenticity of the content selected by the curriculum developer?

A.       Feasibility B. Learnable C. Significance D. Validity

7.     What do we call the allocation of content to a definite grade capable of learning?

A.       Time allotment B. Grade Level C. Grade replacement D. Maturity level

8.     Which pattern of experience centered curriculum centers on the normal activities of children and is based on each child’s needs,
interests and potentials?

A.       Child centered B. Activity C. Social function D. Specific competencies

9.     Which curriculum development phase focuses on the change which will take place in certain aspects of the curriculum without changing
the fundamental conceptions?

A. Curriculum planning B. Curriculum improvement C. Curriculum design D. Curriculum


evaluation

10.  Which is NOT a component of curriculum designing?

A.       Objective B. Learning content C. Learning experiences D. Diagnosis of needs

11.  Who controls the subject centered curriculum?

A.       Learner B. Teacher C. Parent D. Separate subjects

12.  To ensure success in curriculum development, which of the following specific actions should a curriculum leader avoid?

A.       Work with people not over them C. Use your status frequently to establish discipline
B.       Keep channels of communication open D. Show that you too desire to improve

13.  Which of the following is a reason for the continuous appraisal of the existing curriculum in all levels?

A.       New national policies in government C. Changing needs and condition of society
B.       Economic status of the people D. Political trust of the country

14.  Which of the following best defines curriculum development?

A. The total mental phenomenon directly received at any given time


B. The planning of learning opportunities intended to bring about certain desired changes in pupils and the assessment of the extent
To which these changes have taken place
C. A continuous cycle of activities in which all elements of curriculum are considered
D. Education is aiding each child to be socially creative individuals

15.  What do you call the curriculum when the teacher puts into action all the different planned activities in the classroom?

A.       Recommended Curriculum B. Taught Curriculum C. Written Curriculum D. Supported Curriculum


16.  All the reading theories recognize the role of the reader and the text in the comprehension process. However, only the interactive model
accounts for the role of the reading situation in the meaning-making process. What factors does the interactive model consider in the
reading process?

A.       Outcome B. task C. context D. purpose

17.  Reading in the content area aims to help students make sense of the text and negotiate meanings as readers actively interact with the
text. Which of the following activities will nest achieve this goal?

A.       Have reading of the text be done at home C. Make them read silently
B.       Allow students to ask questions D. Practice oral reading fluency

18.  Ronald is about to buy a book. After taking a book form the display shelf, he looked at the title, opened it and looked at the table of
contents, then the summary found at the back cover. He realized that what he took from the shelf is not what he needs. Which of the
following strategies do you think id Ronald do to decide why he does no need it?

A.       Scanning B. Close reading C. Skimming D. Careful slow reading

19.  After M. Rivas planned her lesson in English, she found out that the materials at hand do not match her objectives. Which is the best thing
she can do?

A. Modify the available materials C. Teach the lesson the following day
B. Change the objectives to match with available materials D. Carry out the lesson as planned and use the materials at hand

20.  With the increasing use of the educational technology inside the classroom, what role is expected of the teacher?

A.       Facilitator B. Knowledge giver C. Researcher D. Source of information

21.  It is impractical to bring real objects to the classroom so Aaron constructed a three-dimensional visual instead. Which of the following
did he construct?

A.       Chart B. Cartoon C. Model D. Graphic organizer

22.    Standard deviation is to variability as mode to _______________

A.       Correlation B. Discrimination C. Central tendency D. Level of difficulty

23.    Goring perform better than 65% of the total number of examinees in the district achievement test. What is his percentile rank?

A.       P35 B. P65 C. P66 D. P75

24.    Which is a guidance function of a test?

A. Identifying pupils who need corrective teaching C. Assigning marks for courses take
B. Redacting success in future academic and vocational education D. Grouping pupils for instruction within
a class

25.    Mr. Labanga, an elementary school teacher in Science found out that many of his pupils got very high scores in the test. What measure of
central tendency should he use to describe their average performance in the subject?

A. Mean B. Median C. Mode D. Range


26.    Which of the following indicates how compressed or expanded the distribution of scores is?

A. Measure of position B. Measure of central tendency C. Measures of correlation D. Measure of variability

27.    Mr. Gringo tried to correlate the scores of his pupils in Social studies test with their grades in the same subject last 3 rd quarter. What test
validity is he trying to establish.

A. Content validity B. Construct validity C. Concurrent validity D. Criterion-related


validity

28.    In his second item analysis, Mr. Gonzales found out that more from the lower group got the test item 15 correctly. What does this mean?

A. The item has become more valid C. The item has a positive discriminating power
B. The item has become more reliable D. The item has a negative discriminating power

29.    Q1 is 25th percentile as median is to what percentile?

A.       40th percentile B. 60th percentile C. 50th percentile D. 75th percentile

30.    Mrs. Del Salvatier would like to find out how well her students know each other, what assessment instruments would best suit her
objective?

A.       Self-report instrument B. Sociometric technique C. Guess-who technique D. All of the above

31.    Which of the following assessment procedures/tools is useful in assessing social relation skills?

A.       Anecdotal record B. Attitude scale C. Peer appraisal D. Any of the above

32.    Which educational level/s/provide/s for free and compulsory as stipulated in Article Iv, section 2 of the Philippine constitution?

A.       Elementary level B. Secondary level C. Elementary & secondary levels D. Tertiary level

33.    Who among the following is in the category of non- academic personnel as provided for under Education Act 1982?

A.       Guidance counsellors B. School principals C. School nurse D. School librarian

34.    How is gradual progression of teacher’s salary form minimum to maximum done?

A.       Regular increment every year C. Regular increment every 3 years


B.       Increment after ten years of service D. Increment after 5 years

35.    What appointment can be given to Teacher a who possesses the minimum qualifications but lacks the appropriate civil service eligibility?

A.       Contractual basis B. Permanent C. Provisional D. Substitute

36.    Which of the following is true about human development?

A.       Human development consider both maturation and learning


B.       Development refers to the progressive series of changes of orderly coherent type toward the goal maturity.
C.        Development is the gradual and orderly unfolding of the characteristics of the individuals as they go through the successive stages of
growth
D.       All of the above

37.    What do you call the quantitative increase in terms of height and weight as observed but the school physician during the physical
examination of the students?
A.       Development B. Learning C. Growth D. Maturation

38.    Mrs. Alvarez conducts research on the psychological domain of development. In what particular area of the child’s development is Mrs.
Alvarez mostly like to be interested with?

A.       Perpetual abilities B. Emotions C. Brain-wave patterns D. Use of language

39.    Which of the following is the correct order of psychosexual stages proposed by Sigmund Freud?

A.       Oral stage, anal stage, phallic stage, latency stage, genital stage
B.       Anal stage, oral stage, phallic stage, latency stage, genital stage
C.        Oral stage, anal stage, genital stage, latency stage, phallic stage
D.       Anal stage, oral stage, genital stage, latency stage, phallic stage

40.    What is the best description of Erickson’s psychological theory of human development?

A.       Eight crises all people are thought to face


B.       Four psychological stages in the latency period
C.        The same number of stages as Freud’s but with different names
D.       A stage theory that is not psychoanalytic

41.    In Erickson’s theory, what is the unresolved crisis of an adult who has difficulty establishing a secure, mutual relationship with a life
partner?

A.       Initiative vs. Guilt B. Intimacy vs. Isolation C. Autonomy vs. Shame and Doubt D. Trust vs. Mistrust

42.    Alyssa is eight years old, and although she understand some logical principles, she still has troubles in understanding hypothetical
concepts. According to Piaget, Alyssa belongs to what particular stage of cognitive development?

A.       Sensorimotor B. Concrete operational C. Preoperational D. Formal operational

43.    Which of the following provides the best broad description of the relationship between heredity and environment in determining height?

A.       Heredity is the primary influence, with environment affecting development only in severe situations.
B.       Heredity and environment contribute equally to development
C.        Environment is the major influence on physical characteristics.
D.       Heredity directs the individual’s potential and development determines whether and to what degree the individual reaches that
potential.

44.    What is the correct sequence of prenatal stages of development?

A.       Embryo, germinal, fetus C. Germinal, embryo, fetus


B.       Germinal, fetus, embryo D. Embryo, fetus, germinal

45.    When a baby realized that a rubber duck which has fallen out the tub must be somewhere on the floor, he is likely to achieved what
aspect of cognitive development?

A.       Object permanence C. Mental combination


B.       Differed imitation D. Goal-directed behavior

46.    Which of the following will be Freud’s description of the child’s behavior if he has a biting, sarcastic manner?

A.       Anally explosive C. Fixated in the oral stage


B.       Anally retentive D. Experiencing the crisis of trust vs. mistrust

47.    What is Freud’s idea about a young boy’s guilt feeling brought about by jealousy of his father’s relationship with his mother?

A.       Electra complex C. Phallic complex


B.       Oedipus complex D. Penis envy complex

48.    When a little girl who says she wants her mother to go on vacation so that she can marry her father, Freud believes that he is voicing a
fantasy consistent with?

A.       Oedipus complex C. Theory of the mind


B.       Electra complex D. Crisis of initiative vs. Guilt

49.    Which of the following can best describe the prescribe the preschooler’s readiness to learn new task and play activities?

A.       Emerging competency and self-awareness C. Theory of the Mind


B.       Relationship with parents D. Growing identification with others

50.    Erickson’s noted that when preschoolers eagerly begin many new activities but are vulnerable to criticism and feelings failure, they are
experiencing what particular crisis?

A.       Identity vs. role confusion C. Basic trust vs. mistrust


B.       Initiative vs. Guilt D. Efficacy vs. helplessness

Professional Education - Drill 6 - Part 3

- November 29, 2018

1.        Teacher P, the English coordinator was assisted by Teacher Q throughout the celebration of English Week. What should Teacher P
do to acknowledge Teacher Q’s assistance?

A.  Buy her a gift


B.  Keep quiet about the assistance received
C.  Mention formally to the principal the assistance received
D.  Make an announcement giving due recognition of the assistance received

2.        Is holding a rally to protest the delay of benefits due a person ethically acceptable?

A.  Yes, when hold while on official time


B. Yes, when hold with the approval of the principal
C. Yes, when hold outside the official time
D. Yes , when hold together with parents and students

3.        What should a teacher do when he/she falls in love with/her student?

A.  Court the student at home


B.  Propose and marry the student
C.   Wait till the student is no longer under his/her tutelage
D.   Act normally as if nothing happens and the student does not exist

4.        When principal starts to exercise his/her powers over making and promoting students, is his/her action acceptable?

A.   Yes, when the teacher cannot make decision on time


B.  Yes, when there is abuse of judgement on the part of the teacher
C.   No, teachers are more knowledgeable of their student’s performance
D.  No, grading and promoting students are exclusive functions of teachers
5.        Teacher R was asked by her principal to teach pre-school class in addition to her regular grade one class, what will be for her additional
compensation?

A.  Her basic salary


B. Number of years of service
C. Performance rating
D. Her regular salary + 25% of her basic pay

6.        Which of the following shows responsiveness of public officials and employees?

A. Avoiding wastage in public funds


B.  Providing public information
C. Formulating rules regarding work
D. Encouraging appreciation of government services

7.        Teacher S, a Science teacher has been accused of sexual harassment by one her students. What should the school principal do?

A.   Ask the teacher to surrender to the police


B.   Advice the teacher to transfer to other school
C. Tell the teacher to stop reporting to school
D. Create a committee to investigate the accusation

8.        Teacher T receives a love letter form one of her third year high school student in English. What should Mr. Martin do?

A.    Read her letter to the class


B.    Kept the student express her feelings through letters
C.   Return the letter to the student and tell her to not do it again
D.   Surrender the letter to the parent of the student

9.        Mr. Nico, a Social Science teacher is advocating reforms which the principal failed to recognize. What should the principal do?

A.   Subject Mr. Nico to a disciplinary measure


B.   Just keep quiet about the behavior of Mr. Nico
C.   Call Mr. Nico to the office and clarify things out with him
D.  Send Mr. Nico a memo requiring him to explain his behavior

10.    Which of the following manifests “Commitment to democracy” as explained in R.A. 6713

A.  Maintaining the principle of accountability


B.   Committing to democratic values and ways of life
C.   Manifesting by deed the supremacy of civilian authority over the military
D.   All of the above

11.    Teacher U was ordered by her principal to come to school on four Consecutive Saturdays for the training of the students ‘editorial staff of
their school paper. Is this allowed under R.A. 4670

                  A.  Yes, provided the teacher is compensated


                  B.   No, because it’s not within the regular functions of the classroom teacher
                  C.   Yes, because it’s part of the teachers other duties
                  D.   No, because it’s not clearly indicated in the law

12.    In observation and imitation learning. What should be the learner’s response when the teacher initially models the behaviour?

A.  Reproduce and match


B.  Pay attention
C. imitate and practice
D. shows satisfaction
13.    What is the correct sequence of the information processing?

A.  Sensory register –STM-LTM


B.  STM-sensory register-LTM
C. Sensory register LTM-STM
D. LTM-sensory register- STM

14.    What should be the hierarchy of the type’s pf learning according to the cumulative learning theory?

1.   Problem solving learning 3. Rule learning


2.   Discrimination learning 4. Concept learning

A.       2-1-3-4
B.       2-1-4-3
C. 2-3-4-1
D. 2-4-3-1

15.    Which is essential in meaningful reception learning?

A.  Concepts are presented to learner and received by them


B.  Concepts are related to one another
C. Concepts are discovered by  the learner
D. Concepts are solicited form learners

16.    Grace is bilingual. She speaks both English and Filipino fluently. She begins to Study Spanish and immediately recognizes many
similarities between Spaniard and Filipino languages and uses this information to acquire the new language faster. What kind of transfer
was Grace able to use?

A.       Lateral transfer


B. Specific transfer
C. General transfer
D. Vertical transfer

17.    Cristina has been staring at the match stick puzzle problem. She figuring out how to solve it. Suddenly, a bright idea flashes in her mind
and excitedly, successfully solves the puzzle problem. What type of learning is exhibited?

A. Analytic learning
B. insight learning
C. Discovery learning
D. trial and error learning

18.    Marko excels in adding numbers. He learned this skill in his Math class. He is now able to apply this skill in his Music class. What type of
transfer was used?

A.  Lateral transfer
B. Specific transfer
C. General transfer
D. Vertical transfer

19.    Mr. Lorenzo would always give the chapter test on a Friday. What schedule of reinforcement is used by Mr. Lorenzo?

A.  Fixed Interval
B. Variable Interval
C. Fixed Ratio
D. Variable Ratio

20.    To remember the six digits 8,4,3,9,4,5, the Math teacher grouped the number in twos 84,39,45 or in threes, 843,945. What control
process of retaining information is referred to?
A. Chunking
B. Rehearsing
C. Interfering
D. Remembering

21.    Here is a test item: “The improvement of basic education should be the top priority of the Philippine Government . Defend or refute this
position.” What type of question is this?

A.  Analysis
B. Convergent
C. Evaluative
D. Low level

22.    What can help achieve the relevant quality education?

A.  Strong curriculum
B. Competent instruction
C. School-community relations
D. Competent administrators

23.    Which of the following provisions under the Magna Carta for public School teachers will most likely promote teachers’ welfare and
defend their interests?

A.   Be promoted in rank and salary


B.  Regulate their social involvement
C. Undergo and participate in professional development
D. Establish, join and maintain professional & self – regulating organization

24.    What kind of tension is referred to when people prefer to have quick answers and ready solution to many problems even if it calls for a
patient, concerted, negotiated strategy of reform?

A.  Tension between modernity and tradition


B.  Tension between long term and short term considerations
C. Tension between spiritual and material
D. Tension between individual and the universal

25.    In what strands of the four pillars of education implies a shift from skill to competence, or a mix of higher –order skills to each individual?

A.  Learning to know
B. Learning to do                   
C. Learning to live together                  
D. Learning to be

26.    Which pillar of education of J. Delor (UNESCO) focuses on voc-tech relevant to people-centered human development?

A.  Learning to know
B. Learning to do
C. Learning to live together
D. Learning to be

27.    Which of the following is NOT a characteristic of Multicultural Education?

A.  Personally empowering
B. Socially transformative
C. Pedagogically humanistic
D. Culturally discriminating
28.    What is the kind of education that manifests democratization of access and inclusivity?

A.  Relevance
B. Sustainability
C. Quality
D. Equity

29.    Which among the following rights manifests rule of law and good governance?

A.  Right to education
B. Right to environmental protection
C. Right of participation
D. Right to work

30.    Which among is NOT a core principle of human rights?

A.       Human dignity


B. Non-discrimination
C. universality
D. Independency

Professional Education - SET E - Part 1

- November 22, 2018

Professional Education - SET E - Part 1

1.    Which is true of LET passers?

A.   Every LET passer shall be required to take the Professional Oath before practicing as a Professional Teacher
B.   Taking the Professional Oath before practicing as a Professional Teacher is optional for LET passer
C.   The Professional Oath can be taken before any Professional Teacher
D.   The taking of oath must be within one month from passing the LET

2.    Can a failed examinee qualify for a position of para-teacher?

A.   No
B.   Yes
C.   Yes, if his LET rating is below 75
D.   Yes, if his LET rating is 70 and above

3.    In order to assist new teachers, which is the most effective way to clarify the school’s goals and responsibilities early in the
first year?

A.   School curriculum


B.   Student’s handbook
C. Principal’s memorandum
D. Orientation

4.    How do orientation sessions help the new teacher?

I.             Be attuned to the school environment


II.            Learn guidelines to follow
III.          Develop good relations

A.   II and III B. I and II C. I only D. I, II, and III


  
5.    A teacher introduces herself as teacher only. What does this imply?

A.   The teaching profession is not a very significant one


B.   The teaching profession is the lowest paid profession
C.   She must have been forced to pursue a career in teaching
D.   She takes no pride in the teaching profession

6.    When should teachers support one another

A.   At all times for whatever cause


B.   When they dialogue with administration
C.   When there is rift among teachers
D.   When the best interest of learners is at stake in a controversy

7.    Name the skills relevant to the teachers looking at problems as opportunities:

I.             Responsiveness to problem solving


II.            Alertness for possibilities
III.          Strong defense for tradition
IV.          Capacity to forget problems

A.   I, II, III, and IV C. II, III, and IV


B.   I and II D. I, II, and III

8.    Which of the following is not relevant as immediate expectations for BEED and BSEd graduates?

A.   Acting as an agent of change


B.   Motivation for continuing professional growth
C.   Principled understanding of the learning process
D.   High literacy and critical thinking

9.    Among the following, which directly qualifies the teacher candidate to be registered at par with other professions like
medicine, law, nursing, and the like?

A.   Teaching job placement


B.   Licensure examination
C.   Continuing professional education
D.   Membership in professional education

10.What does “once a teacher, forever a student” imply for the professional teacher?

A.   Teacher’s affinity with the student


B.   Student-centered instruction
C.   Post-graduate studies for teachers
D.   Lifelong learning

11.What is the overall stream for education, growth and fulfillment in the teaching profession?

A.   Merit and promotion C. In-service


B.   Lifelong career D. Pre-service

12.In essence, what is the new Performance Appraisal System for Teachers (PAST)?

A.   Competency-based C. Practice-based


B.   Skill-based D. Knowledge-based

13.What is known as a self-appraisal for professional growth that is acceptable and useful for recognizing weakness and
strengths for a new beginning teacher?

A.   Self-evaluation C. Master teacher’s evaluation


B.   Student’s evaluation D. Principal’s evaluation

14.Self-evaluation can be done in various ways, but this is not one of them.

A.   Written reflection


B.   Self-videotape of class performance
C.   Use of an evaluation instrument
D.   Peer feedback session

15.What problem is given focus by research studies which show low rate of work absorption among technical/vocational
graduates?

A.   Highly protected formal sector


B.   Training-job mismatch
C.   Too long vocational courses
D.   Penalized informal sector

16.Under R.A. 7722 or Higher Education Act of 1994, which agency has direct supervision and management of teacher
education institutions or colleges of education in the country?

A.   Commission on Higher Education


B.   Professional Board of Teachers
C.   Teacher Education Council
D.   Department of Education

17.Under R.A. 7722 or Higher Education Act of 1994, which agency identifies supports and develops centers of excellence
among higher education institutions in the country?

A.   Department of Education


B.   Professional Regulation Commission
C.   Commission on Higher Education
D.   Teacher Education Council

18.Which of the following has greatly influenced the introduction of K-to-12 reform in basic education?

A.   Need to employ more teachers


B.   Need to conform to international standards
C.   Need to allot for more funding
D.   Need to upgrade facilities of schools

19.From the Constitution, what is the condition for allowing students to be taught religion in public schools?

A.   Catechists paid by government


B.   Extra pay for teachers
C.   Outside of school hours only
D.   Upon written permission by parents

20.What is the constitutional exemption for prohibition to establish schools solely for aliens in the Philippines?

A.   Schools for Asians only


B.   Schools for special alien children
C.   Religious or mission schools
D.   Schools for children of diplomats and their dependent

21.To whose responsibility is assigned by Philippine Constitution to the establishment, maintenance and support of complete,
adequate, and integrated system of education?

A.   The citizens


B.   The legislators
C.   The state
D.   The local government

22.Among rights of the schools, which is not provided by the law?

A.   Right for basic education to subjects of study


B.   Right to provide proper governance
C.   Right to enforce administrative systems
D.   Right for institutions for higher learning to determine academic grounds for admission

23.Among categories of time in classrooms and schools, which refers to the number of days and hours in the school calendar
specified by the school and government agencies?

A.   Allocated time C. Engaged time


B.   Mandated time D. Instruction time

24.Of the following, which needy and qualified students are most deserving of government funding assistance (scholarships,
grants-in-aid, etc.)?

A.   Students from cultural communities


B.   Students who show high learning skills
C.   Poor transferees from private schools
D.   Students from sub-urban areas

25.Former DepEd Director described her humble early schooling: a silong (ground floor of a house) converted into a
classroom. Which best characterizes the situation?

A.   Church-school C. School-home connection


B.   City school facility D. Rural village school

26.The world has become a global village. What sound practice is expected of teachers?

A.   Ethnocentrism C. Ethnic cleansing


B.   Respect of diverse culture D. Xenocentrism

27.What observation tests to the fact that the student’s motivation vary according to socio-cultural background?

A.   Females mature earlier than boys


B.   Genetic endowments may show gifted endowments among the young
C.   Brains of boys are bigger and better than those of females
D.   Children from low-income household meet more obstacles in learning

28.Which of the following displays one’s care for the welfare of others?

A.   Accepting responsibility for one’s choices


B.   Fulfilling commitments
C.   Developing self-esteem
D.   Taking pleasure in helping others

29.What best describes “puwede na” mentality vs. excellence in work/service?

A.   Arduous preparation C. Committed work


B.   Resignation to mediocrity D. Striving to be the best

30.What is the cultural trait of conflicting values that aims to please people in different venues and situations rather than abide
by principles?

A.   Bahala na mentality C. Split-personality


B.   Crab mentality D. Kanya-kanya system

31.Among components of a morally mature person, which demonstrates that a teacher seeks peaceful resolution of conflicts?

A.   Fulfilling commitments


B.   Working for peace/reconciliation
C.   Doing community work
D.   Accepting responsibility for choices
  
32.Inculcating moral maturity among students, which of the following relates to beliefs and ideals?

A.   Referring for prejudiced action


B.   Respecting freedom of conscience
C.   Promoting human equality
D.   Avoiding deception and dishonesty

33.Among active participation of school officials and teachers in the community, which of the following is not appropriate due
to prevailing religious sentiments?

A.   Literacy assistance for out-of-school children/youths


B.   Household campaign for healthful practice
C.   Introducing cooperative thrift practices
D.   Promoting contraceptives for Planned Parenthood

34.What is the degree of moral certitude of Alegre who entered into marriage only out of obedience to her parents, but certain
whether she wanted marriage at all?

A.   Lax B. Probable C. Doubtful D. Certain

35.In the poem, “When in Distance with Fortune and Men’s Eyes” Shakespeare felt depressed looking at his sad fate and
wishing he were someone else more rich, with many friends, with greater skills and future prospects in life. What is the
best antidote to such depressed feelings?
A.   Deny that others have more than you have
B.   Count your own blessings
C.   Strive to surpass others
D.   Think of others poorer than you are

36.Among mistaken goals in the Acceptance Approach to discipline, what happens when students seek to hurt others to make
up being hurt or rejected?

A.   Revenge seeking C. Withdrawal


B.   Power seeking D. Attention getting
  
37.Among mistaken goals in the Acceptance Approach to discipline, what happens when students are not getting the
recognition they desire, continually seek help, and refuse to work unless the teacher hovers over them?

A.   Attention getting C. Revenge seeking


B.   Withdrawal D. Power seeking

38.Among mistaken goals in the Acceptance Approach to discipline, what happens when students feel helpless and rejected so
that they remove themselves rather than confront the situation?

A.   Withdrawal C. Revenge seeking


B.   Power seeking D. Attention getting

39.To manage behavior, the teacher needs to be able to identify the mistaken goals of the students. What is the hidden goal of
students who become violent?

A.   Goal is to seek power C. Goal is to isolate self


B.   Goal is to get attention D. Goal is to get revenge

40.To what problem type do students belong, who have short attention span, unable to sustain concentration by being easily
affected by sights, sounds or speech?

A.   Withdrawn C. Low achiever


B.   Defiant D. Distractible

41.Of the approaches to classroom management, which involves a variety of techniques and methods of ranging simple
rewards to elaborate reinforcement, e.g. praise or rewards?

A.   Behavioral modification approach


B.   Group management approach
C.   Business-academic approach
D.   Assertive approach 

42.Among styles of classroom management, which expects teachers to specify rules of behavior, consequence for disobeying
them, and to communicate these rules and consequences clearly?
A.   Business-academic approach
B.   Behavioral modification approach
C.   Group managerial style
D.   Assertive style

43.Of the approaches to classroom management, which emphasizes the organization and management of studentsas they
engage in academic work?

A.   Assertive approach


B.   Group managerial approach
C.   Behavioral modification approach
D.   Business-academic approach

44.If threat or punishment is necessary on erring students, how should this be done?

A.   First a warning before the threat


B.   Make the threat with immediate punishment
C.   Make the threat and reinforce with warning
D.   Warn and threat at the same

45.If threat or punishment is necessary, which of the following expressed the threat with LEAST severity?

A.   “I will work for your removal from class and you are a shame to your classmates.”
B.   “I will send you to principal’s office, or else also ask your parents to confer with the principal.”
C.   “You will get shortened recess or receive more severe punishment.”
D.   “You will report early to school tomorrow or you will be marked absent.”

46.Of the following, which is a non-threatening style of disciplining unruly students?”

A.   Stand under heat of the sun


B.   Do push-ups
C.   Send to guidance office
D.   Squat before the class

47.Judging the relative severity of common classroom management strategies, which of the following is least severe if applied
on erring students?

A.   Surprise quiz


B.   Lowering school mark
C.   Throwing students out of class
D.   Summons to principal’s office

48.For problems of discipline that cannot be resolved in class, which of the following should not be done as it reduces the
authority of the teacher?

A.   Rely on others to resolve your classroom problems


B.   Analyze your methods
C.   Give students a chance to redeem himself
D.   Communicate with parents

49.Which of the following is the best option to deal with chronic behavior problems in the classroom?

A.   Indicate consequences of behavior


B.   With gained confidence and skills
C.   Be calm and consistent
D.   Establish positive expectations for students

50.Teacher Liza acts like a classroom manager, not a disciplinarian. Which of the following did she avoid doing?

Professional Education - SET E - Part 2

- November 22, 2018

Professional Education - SET E - Part 2

51.  From classroom management strategies applied on erring on students, which of the following should not be done?

A.   Surprise quiz


B.   Parent-principal conference
C.   Communicating problem to parents
D.   Shaming erring student before the class
  
52.    As preventive measure for classroom discipline, the teacher may restructure the program. How is this not be done?

A.   Teach again lessons difficult to understand


B.   Remove tension level before processing with lesson
C.   Modify lesson
D.   Skip whole lesson unit altogether

53.    Teacher Jose talks to students about their interests, what they did over the weekend, their progress in school work, etc.
What positive approach to classroom management did Teacher Jose apply?

A.   Trusting students


B.   Being fair and consistent
C.   Expressing interest for students
D.   Being positive

54.    Each teacher has a role as a behavior reinforcing agent. Which of the following should not be encouraged among
teachers?
A.   Positive relationship
B.   Threats
C.   Reward appropriate behavior
D.   Constant reinforcement

55    How and when should teachers most effectively deal with misdemeanor for class management and control?

A.   At the beginning of the class


B.   By end of each class
C.   Early and promptly
D.   When heads have control

56.    Which of the following best characterize the Business-Academic Approach in classroom management?

A.   Persist, enforce, don’t give up


B.   Calmness, consistency, confidence
C.   Work, organization, procedures
D.   Control, persistency, sanctions
  
57.    Which of the following classroom climate is less threatening?

A.   Cooperative C. Individualistic


B.   Competitive D. Autocratic

58.    In the 5’S Formula to create a conducive learning environment, which S stands cleanliness of the work place?

A.   Shine B. Sweep C. Systematized D. Sort

59    Among the components in the instructional framework for learning strategies, which is demonstrated by teacher Ana who
assigns homework for students to do on their own?

A.   Independent practice C. Consolidation


B.   Review D. Modeling

60.What mistake is teacher Benny trying to avoid when he never drops a topic before it is completed?

A.   Dangling topic C. Bounded topic


B.   Non-directional topic D. Non-assessed topic

61.Of subcategories of teacher movement behavior, what is happening when the teacher bursts into activities without
assessing student readiness and gives orders, statements, or questions that confuse students?

A.   Stimulus-bounded C. Thrust


B.   Truncation D. Dangle
62.The class has become uncontrollable, and so teacher yen decided to have some undisciplined students out of the room to
the counselor’s office. What did teacher Yen apply by way of preventive approach to discipline?

A.   Removing seductive objects C. Antiseptic bouncing


B.   Direct appeal D. Physical restraint

63.Teacher Anna dealt effectively with a minor infarction of whispering a student to a neighbor during class. Which of the
following did she do?

A.   Use non-verbal signals (gesture or facial expression)


B.   Continue to teach and ignore infarction
C.   Reprimand student after class
D.   Reprimand quietly

64.What is being shown by teachers who ignore slow-learners and take pride in the work of fast-learners?

A.   Bullying C. Cursing


B.   Hurling invectives D. Discrimination

65.The following are non-verbal behavior by teachers. Which one is an encouraging behavior?

A.   Voice volume shows arrogance


B.   Teacher grabs, slaps students
C.   Stands or sits that shows alertness
D.   Teacher frowns, sneers

66.The following are examples of non-verbal behavior. Which one is restricting or not acceptable mannerism by teachers?

A.   Teacher suggests approval for students and continues with “um-hm”
B.   Teacher moves toward student
C.   Teacher slouches to suggest being carefree
D.   Teacher’s intonation shows support

67.Among teaching styles, how do you classify teachers who clearly and fairly communicate standards for discipline and
performance to student?

A.   Authoritarian C. Permissive


B.   Uninvolved D. Authoritative

68.Among teaching styles, how do you classify teachers who lack confidence in enforcing and demanding compliance to
rules?

A.   Permissive C. Uninvolved


B.   Authoritative D. Authoritarian

69.Among teaching styles, how do you classify teachers who are control oriented and impose rules without explanation?
A.   Authoritative C. Permissive
B.   Authoritarian D. Uninvolved

70.Values are subjective. Your value may not necessarily be my value. What subscribe to this thought?

I.             Progressivist II. Pragmatists III. Essentialist

A.   II and III C. I and III


B.   I, II, and III D. I and II

71.Students must be taught self-responsibility is the desire of the ____________ teacher.

A.   Existentialist C. Constructivist


B.   Pragmatic D. Utilitarianist

72.Of the following, which is the most powerful source of values like beauty and goodness outside the school system?

A.   Government propaganda C. Commercial advertising


B.   Nature D. Mass media

73.Of the following, which is the best way parents can support the learning development of their children?

A.   Buying materials for project


B.   Making calls on teachers
C.   Interest and assistance in class assignments
D.   Checking report card grades

74.Teacher Ernie makes sure that he covers the essential subject content, while treating them sufficiency or in-dept. What
guiding principle is he following for lesson preparation?

A.   Balance C. Significance


B.   Feasibility D. Self-sufficiency
   
75.Which of the following is a quality of an engaging learning environment most reflective of Education For All (EFA)?

A.   Improvised to enhance learning for all types of learners


B.   Decentralized governance
C.   Student-centered learning activities
D.   Open communication for all-teachers, parents, students

76.What is the main focus of the Schools Mean Success(SMS) concept for lifelong learning?

A.   School recognition and accreditation for success


B.   Tapping student’s potential for success
C.   Pillars of instruction, research and outreach for success
D.   Mission-vision of schools for success

77.Of the following interventions, which is directly aimed at responding to the transitional gap between academic achievement
and employment?

A.   Deregulation of tuition fees


B.   Voluntary accreditation of schools
C.   School networking with business and industry
D.   Identification of centers of excellence

78.Learning is influenced by social interaction and interpersonal relations. What must a teacher do?

A.   Make students work collaboratively


B.   Motivate students to reflect on how they learn
C.   Make students feel good about themselves
D.   Give more independent study

79.Which of the following refer to teamwork among teachers in classroom teaching?

A.   Peer evaluation C. Peer discussion


B.   Peer teaching D. Peer assessment

80.Of goals of education, which relates to the strengthening of our society’s sense of belonging and identity?

A.   Autonomy C. Moral character


B.   Enculturation D. Citizenship

81.In the implementation of the curriculum at the classroom level, ineffective strategies are called “Red”. Which of the following
belongs to the Red Flag?

A.   Integration of problem solving


B.   Available enrichment activities
C.   Content applied to real-life situations
D.   Overemphasis on drill and practice

82.What does the traditional curriculum and mass education lack that the progressive curriculum can deliver?

A.   Authoritarian role of the teacher


B.   Collective discipline
C.   Value for individualization
D.   Rigid system of grading

83.Among unit plan components, to what cluster do models, replicas, charts, graphs, and specimens belong?

A.   Resources and materials C. Evaluation procedures


B.   Learning activities D. Objectives

84.Among the following terms in technology, which has the broadest concept of technology for the whole educative system?

A.   Computer technology C. Instructional technology


B.   Audiovisuals D. Educational technology

85.In the process of integrating technology in instruction, which of the following points to the highest kind of cognitive skills
desired?

A.   Nurturing of class teamwork


B.   Engendering meaningful learning
C.   Introducing social learning
D.   Developing metacognition

86.While there is modernity and progress through the use of technology in instruction, what is the greatest danger along the
domain of learning that is posed by technology?

A.   Using technology to fill in lesson gaps


B.   Using technology to lighten up lessons
C.   Developing passivity and uncritical minds
D.   Developing visual and audial orientation

87.Of the following, which is the most traditional example of “technology” in the limited sense of tools for education?

A.   Moving film C. CD-ROM


B.   Chalkboard D. Graphs and photos

88.Which of the following is the most appropriate use of television in the classroom?

A.   TV viewing for the whole one-hour class


B.   TV viewing to replace absent teacher
C.   TV viewing with cuts and explanation
D.   TV viewing to entertain students

89.In e-learning, which is the motivational aspect of learning through the electronic media?

A.   Virtual imaging


B.   Technology integration
C.   Enthusiasm to use technology
D.   Digital platforms

90.What is aroused among viewing learners by dramatic educational films in what is known as “cliff hangers”?

A.   Relevance C. Satisfaction


B.   Significance D. Expectancy
91.Of the following effects on learning, what is the effect of assigning various sections of the newspaper, and allowing choice
depending on the learner’s choice?

A.   Changing attitudes and feelings


B.   Reinforcing learning
C.   Allowing different interests
D.   Encouraging participation

92.According to Nagel’s Acronym PPFF, what is important to Follow Up in the proper use of instructional materials?

A.   Student interest C. Lesson objective


B.   Student expectations D. Media and materials 

93.How can the teacher best begin in developing a lesson plan design?

A.   High level open-ended question


B.   General learning goals
C.   Vision-mission statement of the school
D.   Clearly defined outcomes

94.Among unit plan components, to what cluster do projected competencies-outcomes belong?

A.   Resources and materials C. Evaluation procedures


B.   Objectives D. Content

95.Classroom routines from the backbone of daily school activities, but which of the following is not covered by such routines?

A.   Submission of work C. Distribution of papers


B.   Lesson content D. Seating arrangement

96.Which of the following is an acceptable teacher-effectiveness practice?

A.   Setting of objectives rather than outcomes


B.   Directions given sparingly and delivered casually
C.   Classroom routines done early in the school year
D.   Planning done following monitoring of learning

97.In the History class, teacher Naomi uses a current events IQ contest to determine champions in identifying people, places,
and events. What learning objective/outcome does she aim to achieve?

A.   Application C. Responding


B.   Knowledge or recall D. Perpetual

98.Which of the following best relate to basic literacy to learners?


A.   Drill C. Dramatize experiences
B.   Verbal symbols D. Demonstration 

99.When classes began, Teacher Dan asked student in pairs to interview and introduce each other to the class. How will this
activity be classified according to Edgar Dale’s cone of experience?

A.   Purposeful experience C. Dramatize experience


B.   Verbal symbols D. Demonstration

100.With closeness to direct experience as standard, which one should a teacher choose?

A.   Contrived experiences C. Dramatize experiences


B.   Study trip D. Demonstration

Professional Education - SET E - Part 3

101.    What provides the most direct type of learning, but is difficult to supply in the traditional classroom?

A.   Real-life experience


B.   Computer-assisted learning
C.   Verbal symbolism
D.   Multisensory instructional aids

102.    Of the following, which is a form of direct learning experience?

A.   Internet research


B.   Role playing
C.   Village interview
D.   Preparation of learning materials

103    On clusters of meaningful learning activities, which does not belong to bodily-kinesthetic learning activities?

A.   Story telling C. Dance


B.   Hands-on demonstration D. Role-playing

104    How can lessons be made more meaningful by way of learning continuity?

A.   Add value to content


B.   Introduce new content
C.   Stress retention of facts
D.   Relate present with past lesson

105.    How can a teacher introduce new information to students?

A.   Valuing new knowledge


B.   Relating it to knowledge they already know
C.   Demonstrating novelty of new knowledge
D.   Increasing retention of new knowledge

106.    Of the following classroom activities, which best relate to child visual learning and development?

A.   Drawing and coloring C. Textbook reading


B.   Singing and acting D. Games and sports

107.    In cognitive learning, what are firmly established and thoroughly tested principles or theories?
A.   Hypothesis B. Facts C. Laws D. Concepts

108.    Among cognitive objectives, what refers to the transfer of knowledge from one concept to another?

A.   Synthesis C. Analysis


B.   Application D. Comprehension

109.    In the taxonomy of learning objectives or outcomes, which relates to judging in terms of internal evidence or logical
consistency, or external evidence consistent with fact?

A.   Evaluation B. Receiving C. Valuing D. Application

110.Under the domains of learning, to what domain do comprehension, analysis, and evaluation belong?

A.   Reflective B. Cognitive C. Affective D. Psychomotor

111.Among cognitive levels of thinking, what cognitive level is stressed by the Essay question: “In the light of the facts
presented, what is most likely to happen when…?”

A.   Inferring B. Classifying C. Applying D. Analyzing


  
112.Of skills teachers should understand and students need to acquire, which is the ability to monitor and modify thinking, as
well as deal with ambiguous or unclear information without getting frustrated?

A.   Focusing B. Sharpening C. Tolerance D. Narrowing

113.Among categories of thinking skills, which is the ability to breakdown complex information into parts for understanding?

A.   Focusing B. Narrowing C. Sharpening D. Analysis

114.Which aspect of multi-intelligence is enhanced by asking students to work on a physical model of the atom after a
teacher’s discussion on the subject of the atom?

A.   Interpersonal C. Mathematical


B.   Kinesthetical D. Linguistic

115.In his Literature class, teacher Ted gives stress to one’s own ability to write poetic verse, drawn from sensible images,
thoughts and perceptions. What creative thinking behavior is he trying to develop in students?

A.   Curiosity B. Awareness C. Originality D. Elaboration

116.What is the primary advantage of active learning techniques (discussion, role play, gaming, etc.) in instruction?

A.   Higher order thinking


B.   Student engagement
C.   Problem solving
D.   Focus on questions, not answers

117.Among thinking skills in learning, what includes fluent, flexible, original and elaborate thinking?

A.   Divergent thinking C. Critical thinking


B.   Convergent thinking D. Problem solving
  
118.In his speech class, teacher Rod uses poetry dramatic reading by students, using movements (gestures, facial
expressions, creative movements, etc.). What learning objectives or outcome is he looking for?

A.   Non-discursive communication C. Skilled movements


B.   Reflex movements D. Characterization

119.Under the affective domain of learning, what includes active attention to stimuli such as acquiescence, willingness to
answer, and feelings of satisfaction?

A.   Organization C. Responding


B.   Valuing D. Characterization

120.Of the following, which is the effective way to advance practical oral proficiency of students?

A.   Give more group work compared with individual work


B.   Request the construction of a school speech laboratory
C.   Engage students in conversational activities
D.   Give stress to correct grammar

121.What kind of literary works are very creative by way of descriptive language and dramatic episodes?

A.   Fiction novels C. Biography


B.   Historical account D. Autobiography

122.Where can students find critical views on current event in newspapers?

A.   Entertainment page C. Ads section


B.   Opinion page D. News page

123.What is an example of a book or writing intended for moral knowledge of the very young readers?

A.   Romeo and Juliet C. Jack and the Beanstalk


B.   Diary of Anne Frank D. Les Miserables

124.What desirable thinking behavior is aimed at in poetry in writing activity?

A.   Interest B. Originality C. Awareness D. Curiosity

125.Among models of reading strategies, what did student Lina adopt when she read back and forth, attending to both what is
in her mind and what’s on the page?

A.   Down-top B. Interactive C. Top-down D. Bottoms-up

126.Sequence the following events on the historical development of reading:

I.             Greek letters and the Roman alphabet were developed


II.            Through the Semite’s ingenuity, sounds and symbols gave rise to the Phoenician alphabet
III.          People used pictures and characters to convey messages
IV.          Researchers showed the process of reading, comprehension, and interpretation.

A.   I, II, III, and IV C. III, II, I, and IV


B.   I, II, IV, and III D. IV, II, I, and III

127.What is known as learning a new word by studying its roots and affixes?

A.   Morphology C. Structural analysis


B.   Phonetic analysis D. Contextualization

128.What is known as getting the meaning of a new unfamiliar word by looking for hints from surrounding words in a sentence
or paragraph?

A.   Context clues C. Semantic clues


B.   Syntactic clues D. Graphophonic clues

129.In Grace Godell’s Reading Skills Ladder, which is at the very bottom and needed to move up to the sequential ladder of
reading skills?

A.   Vocabulary building C. Basic sight words


B.   Phonetic analysis D. Structural analysis

130.In Grace Godell’s Skills Ladder, which step should come last and only when needed?

A.   Using contextual clues C. Using the dictionary


B.   Using parts of a book D. Using phonetic analysis
  
131.How will you make a child memorize the table of multiplication from the point of view of Pavlov?

A.   Make them understand the meaning of multiplication


B.   Call the child’s attention every time you recite the table for them
C.   Model the recitation of the table of multiplication
D.   Reward the child with a candy for every correct recitation

132.What field of education manages children with behavioral problems like attention-deficit disorders?

A.   Distance education


B.   Special education
C.   Skills and technology training
D.   Early childhood education

133.Of the following, which is the truest of adolescents?

A.   Hormonal changes C. Last splurge of dependence


B.   Defiance of peer group D. Unruly behavior

134.In Piaget’s stages of Cognitive Development, what is the pre-school age in which the child’s intelligence is intuitive, able to
make mental representation, and close to the use of symbols?

A.   Formal operational stage C. Sensory motor stage


B.   Concrete operational stage D. Preoperational stage

135.In Piaget’s Concrete-Operational Stage in cognitive development, which refers to the ability to order or arrange things
logically on dimensions such as weight, volume, or size?

A.   Conservation C. Decentering


B.   Reversibility D. Seriation

136.In Piaget’s Concrete-Operational Stage in cognitive development, which refers to the ability of the child to perceive
different features of objects and situations, e.g. toughness in stone, elasticity in rubber, etc.?

A.   Decentering C. Reversibility


B.   Seriation D. Conservation

137.In Piaget’s Formal-Operational Stage in cognitive development, which of the following is not among the characteristics of
the child’s thinking ability?

A.   Logical thinking


B.   Abstract problem solving
C.   Hypothesis making
D.   Knowing properties (number, mass, volume, etc.)

138.In Erickson’s Stages of Theory of Development, which affirmation belongs to the stage of Identity versus Role Confusion?

A.   When people look at what I have done, I feel embarrassed


B.   I get a good deal of pleasure from working
C.   I wonder what sort of person I am
D.   I feel competent

139.In Erickson’s Stages of Theory of Development, which of the following needs to be developed from the child (1 to
years old)?

A.   Autonomy B. Initiative C. Industry D. Trust

140.Following Kohlberg’s Theory of Moral Development, what is the moral reasoning or perspective of joy who allows her
classmate to copy her test work, so that she will be her friend?

A.   Social approval C. Obedience


B.   Punishment D. Law and order

141.Of the following questions, which relate to metacognition?

A.   How is this done? C. What facts did I know?


B.   Am I learning well enough? D. When did this event occur?

142.Among the following questions, which is not applicable to metacognition?

A.   How will I know if I am learning at an appropriate rate?


B.   Did I understand what I just hear, read or saw?
C.   Do I know what I need to now?
D.   What factual information have I retained? 

143.Which of the following does not belong to the metacognitive reading activities?

A.   Predicting B. Testing C. Sampling D. Perceiving


144.Among examples listed, which does not represent teaching strategies to develop metacognition?

A.   Have the students develop questions about what is going on around them
B.   Have teachers the free hand to monitor and rate learning achievement
C.   Have students to monitor their own learning and thinking
D.   Have students make predictions about what is to be represented

145.Among components in the instructional framework for learning strategies, which is demonstrated by teacher Jiny who
worked together with her students on a skill or task and figure out how to apply the skill?

A.   Independent practice C. Consolidation


B.   Guided practice D. Review

146.Principal Ted thinks uses free and open dialogue, event confrontation, so that teachers and learners can learn to discard
old habits and attitudes. What is his general view about learning?

A.   It requires school resources


B.   It is at times a painful process
C.   It is a consequence of experience
D.   It is primarily a fun experience

147.John Dewey said, “An ounce of experience is better than a ton of theory.” To which does this statement point?

A.   The need for theory C. The primacy of experience


B.   The need for experience D. The primacy of theory
   
148.Someone said, “Experience without theory is blind, but theory without experience is mere intellectual play.” This means
that:

A.   Theory and experience must go hand-in-hand


B.   Theory is more important than experience
C.   Experience is more important than theory
D.   One can stand without the other

149.What dependent variable can be used to test the hypothesis, “The more a teacher knows about a specific subject matter,
the better she can teach it”?

A.   Teacher’s yearly performance rating


B.   Personality traits of the teacher
C.   Motivation from the school head
D.   Incentives offered to teachers

150.In the school, who is the key to the success or failure in teaching-and-learning, being the link between the child and the
curriculum?

A.   The teacher C. The student


B.   The parent D. The principal

Professional Education - SET E - Part 4

- November 22, 2018

Professional Education - SET E - Part 4

151.    In outcomes based education, which of the following are not considered outcomes?

A.   Products C. Portfolios


B.   Procedures D. Performances
152.    What is the erroneous concept of outcomes in Outcomes-Based Education?

A.   Performance B. Products C. Results D. Content

153.    In a Problem-Based Learning Group, who take notes, join discussion and review materials?

A.   Facilitator C. Team members


B.   Team leader D. Recorder
  
154.    In assessment for Problem-Based Learning, who are responsible in judging student performance?

A.   Teacher-and-peers
B.   Teacher-and-supervisors
C.   Teacher-and-students
D.   Teacher-students-supervisors

155.    Following a model procedure, how is the process in Problem-Based Learning sequenced?

I.             List action for what we need to know


II.            Explore the issues
III.          Write solution with supporting document
IV.          Develop and write problems statement

A.   I, IV, II, and III C. II, IV, I, and III


B.   II, I, IV, and III D. III, I, IV, and II

156.    Which is not an aim in having partners in team learning?

A.   Focused attention C. Cooperative work


B.   Sharing ideas D. Lighter work

157.    What is the active learning strategy to promote friendly competition and an atmosphere of fun in learning?

A.   Games B. Fishbowl C. Debate D. Case study

158.    Among development activities, which does not belong to data-gathering activities which can allow active participation by
students?

A.   Library research C. Case study


B.   Solving real-world problems D. Panel discussion

159.    Of the following, which is the most conducive for authentic assessment?
A.   Complex multiple-choice test
B.   Performance drill
C.   De-contextualized setting
D.   Real world application

160.Which of the following are alternative assessments?

A.   Portfolio, exhibits, journals


B.   Paper-and-pencil test, demonstration, reports
C.   Multiple choice, structured observation, sentence completion
D.   Self-assessment, authentic assessment, surveys

161.In order for assessment to have clarity of learning targets, outcomes must be stated in behavioral terms so that this can be
measured. Which of the following shows this desired clarity?

A.   To practice reading with competence


B.   To improve the speed of reading competence
C.   To appreciate reading competence
D.   To understand the concept of reading competence

162.Among recording devices during assessment, which can the student use to describe behavior and context which
observations may be forgotten or remembered incorrectly?

A.   Portfolio C. Anecdotal record


B.   Checklist D. Self-assessment

163.What is a scoring guide that lists the criteria and their levels of quality on which evaluation will be focused?

A.   Portfolio C. Stem


B.   Journal D. Rubric

164.Which is an alternative assessment tool that consists of a collection of work artifacts or in progress accomplishment by a
targeted clientele?

A.   Portfolio C. Achievement test


B.   Evaluation instrument D. Rubric

165.Which of the following is the most reliable tool for determining student’s ability to write?

A.   Portfolio assessment C. Interview of students


B.   Scoring rubric D. Self-assessment

166.Using Rubrics to assess Powerpoint presentations of students, which criteria from the following is most significant as it is
the essence of presentation?

A.   Presenting design C. Oral delivery


B.   Content value D. Orderly sequencing

167.What is being assessed by the Rubric that seeks to assess high, moderate, or low imaginative thinking?

A.   Aesthetic B. Creativity C. Appeal D. Craftsmanship

168.Which of the following best describes creativity?

A.   Imaginative B. Positive C. Resourceful D. Inspired

169. Among written categories of assessment, what instruments did Science teacher Julian use when he prepared a list of
behavior that make up a certain type of performance in the use of microscope?

A.   Observation of self-report C. Product raring skill


B.   Written response instrument D. Performance test

170.Using extrinsic motivational assessment, what could be the noblest motive in students pursuing a lifetime work and
mission for the teaching profession?

A.   Promise of higher rank and prestige


B.   Social service to upcoming generations
C.   Economic security and welfare
D.   Respected position in society

171.Of the following, which is not a characteristic of the Response Journal that reflects observations and interpretations of the
student, such as on a reading material or a field visit?

A.   Encourage risk taking C. Open and factual


B.   Literary and grammatical D. Non-threatening 

172.What is the criterion-referenced test designed to determine?

A.   Performance level on a specific skill


B.   Performance on a wide content of coverage
C.   Attaining a personal target
D.   Performance higher than other members of the group

173.In interpreting assessment results, which is described when the results are consistent?

A.   Reliability C. Validity


B.   Subjectivity D. Objectivity

174.Since Teacher Alvin wants his test be more reliable, which of the following should he adopt?
A.   Make a test that requires students the need for more time to complete
B.   Make the test difficult
C.   Increase number of test items
D.   Restrict range abilities in the test

175.Which measure of central tendency is most reliable when scores are extremely high and low?

A.   Median
B.   Mode
C.   Mean
D.   Cannot be identified unless individual scores are given

176.Which is a characteristic of the following score distribution: 98, 93, 93, 93, 90, 88, 87, 85, 85, 70, 51, 34, 34, 34, 20, 18, 15, 12,
9, 8, 6, 3, 1.

A.   No discernible pattern C. Skewed to the right


B.   Tri-modal D. Bimodal

 177.. What is the normal curve in the grade distribution of students in a class?

A.   All have average grades


B.   Most get average grades, few high and low
C.   Most get high grades
D.   Most get low grades

178. What can be said of student performance in a negatively skewed class score distribution?

A.   Most students performed well


B.   All students had average performance
C.   Few students performed well
D.   Most students performed poorly

179. Which is true when the standard deviation is big?

A.   Scores are spread out


B.   Scores are not extremes
C.   The bell curve shape is steep
D.   Scores are concentrated

180. What does it mean to state the difficulty index of a test item is one (1)?

A.   The test item is extremely easy


B.   The test item is not valid
C.   The test item is very good and should be retained
D.   The test item is very difficult

181 What does the test mean if the difficulty index is 1?

A.   Missed by everyone C. Very difficult


B.   Very easy D. A quality item

182 What does it mean to state that the determination index of a test item is -0.35?

A.   More from the upper group got the item correctly
B.   The test is quite reliable
C.   More from the lower group got the item correctly
D.   The test item is valid

183 What does it mean if student Pete got a 60% percentile rank in a class?

A.   He scored better than 60% of the class


B.   He got 60% of the test items correctly
C.   He scored less than 60% of the class
D.   He got 40% of the test wrong
  
184A positive correlation is to direct relation; to what relation is negative correlation?

A.   Indefinite relation C. Definite relation


B.   Inverse relation D. Neutral relation

185.What is the meaning of a negative correlation between amount of practice and number of error in tennis?

A.   The decrease in the amount of practice sometimes affects the number of errors
B.   The increase in the amount of practice does not at all affect the number of errors
C.   Decrease in the amount of practice goes with decrease in the number of errors
D.   As the amount of practice increases, the number of errors decreases

186What is the graphic illustration for the relationship between two variables?

A.   Normal curve C. Frequency polygon


B.   Histogram D. Scatter diagram

187.Among standardized tests, which reveals strength and weaknesses for purposes of placement and formulating an
appropriate instructional program?

A.   Achievement tests C. Diagnostic tests


B.   Competency tests D. Personality tests
188.What kind of assessment helps teachers determine gaps in learning a subject matter?

A.   Formative assessment C. Summative assessment


B.   Diagnostic test D. Placement examination

189.An entering student in college would like to determine which course is best suited for him. Which test is appropriate for
this purpose?

A.   Achievement test C. Intelligence test


B.   Diagnostic test D. Aptitude test
  
190.Tests should be non-threatening, but which of the following actual threatens students?

A.   Self-record of test scores C. Scheduled exam


B.   Surprise quiz D. Submission of report

191.In preparing classroom tests, which of the following checklist is the last among the steps in tests preparation?

A.   How are the test scores to be tabulated?


B.   Have I prepared a Table of Specifications?
C.   How are the test results to be reported?
D.   How is the objective item to be scored?

192.With what should the assessment of test items be aligned?

A.   Monitoring and evaluation


B.   Instructional objectives
C.   Instructional media
D.   Content and process

193.Principal Rodriguez introduced machine-scored tests in his school. What factor in scoring does this add to make tests
more reliable?

A.   Objective scoring C. Ease in checking


B.   Cost-saving D. Time-economics

194.The tests questions in Teacher Dante’s test were confusing and were subject to wrong understanding, especially to poorer
students. What was wrong with the test?

A.   Test items inappropriate for outcomes being measured


B.   Inappropriate level of difficulty of items
C.   Unclear directions
D.   Ambiguity 

195.For easier correction of his test, Teacher Edward prepared the True and False items in alternating 1, 3, 5, 7 etc. True, and 2,
4, 6 etc. False. What is wrong with the test in terms of Usability?
A.   Improper arrangement of items
B.   Identifiable pattern of answers
C.   Ambiguity
D.   Poorly constructed test items

196.What is wrong with test items that give clues to answers to questions?

A.   Unclear directions C. Ambiguity


B.   Tests brevity D. Poor construction

197.In her tests, Teacher Dorie made tests that were either too difficult or too easy. What was wrong with her tests?

A.   Inappropriate level of difficulty of the test items


B.   Identifiable pattern of answers
C.   Unclear directions
D.   Ambiguity

198.In using Essay questions, what of the following are these tests susceptible to, such as to make them less reliable measure
broad knowledge of the subject matter?

A.   Bluffing C. Guessing


B.   Time consuming D. Cheating

199.What primary response factor is considered by Essay questions?

A.   Wide sampling of ideas


B.   Factual information
C.   Originality
D.   Less time for constructing and scoring

200Among the opportunities for Essay questions, which of the following does not provide high-level thinking opportunity?

A.   Analysis C. Synthesis


B.   Evaluation D. Memorization

Professional Education - SET D - Part 1

- November 22, 2018

Professional Education - SET D - Part 1

1. Which is one role of play in the pre-school and early childhood years?
A. Develops competitive spirit.

B. Separates reality from fantasy.

C. Increases imagination due to expanding knowledge and emotional range.

D. Develops the upper and lower limbs.

2. Student Z does not study at all but when the Licensure Examination for Teachers (LET) comes, before he takes the LET,
he spends one hour or more praying for a miracle, i.e. to pass the exam. Which attitude towards religion or God is
displayed?

A. Religion as fake

B. Religion as magic

C. Religion as authentic

D. Religion as real

3. As a teacher, you are a rationalist. Which among these will be your guiding principle?

A. I must teach the child that we can never have real knowledge of anything.

B. I must teAch the child to develop his mental powers to the full.

C. I must teach the child so he is assured of heaven.

D. I must teach the child every knowledge, skill, and value that he needs for a better future.

4. All men are pretty much alike. It is only by custom that they are set apart, said one Oriental philosopher. Where can this
thought be most inspiring?

A. In a multi-cultural group of learners

B. In multi-cultural and heterogeneous groups of learners and indigenous peoples' group

C. In a class composed of indigenous peoples

D. In heterogeneous class of learners

5. Teacher A discovered that his pupils are very good in dramatizing. Which tool must have helped him discover his pupils'
strength?

A. Portfolio assessment
B. Performance test

C. Journal entry

D. Paper-and-pencil test

6. NSAT and NEAT results are interpreted against set mastery level. This means that NSAT and NEAT fall under
__________.

A. intelligence test

B. aptitude test

C. criterion-referenced test

D. norm-referenced test

7. In a social studies class, Teacher I presents a morally ambiguous situation and asks his students what they would do.
On whose theory is Teacher I's technique based?

A. Kohlberg

B. Bandura

C. Piaget

D. Bruner

8. Which is a sound classroom management practice?

A. Avoid establishing routines

B. Establish routines for all daily needs and tasks.

C. Apply rules and policies on a case to case basis.

D. Apply reactive approach to discipline.

9. A sixth grade twelve-year old boy comes from a dysfunctional family and has been abused and neglected. He has been
to two orphanages and three different elementary schools. The student can decode on the second grade level, but he can
comprehend orally material at the fourth or fifth grade level. The most probable cause/s of this student's reading problem
is/are __________.

A. emotional factors

B. poor teaching
C. neurological factors

D. immaturity

10. Teacher U teaches to his pupils that pleasure is not the highest good. Teacher's teaching is against what philosophy?

A. Realism

B. Hedonism

C. Epicureanism

D. Empiricism

11. With which goals of educational institutions as provided for by the Constitution is the development of work skills
aligned?

A. To develop moral character

B. To teach the duties of citizenship

C. To inculcate love of country

D. To develop vocational efficiency

12. Direct instruction is for facts, rules, and actions as indirect instruction is for __________, __________, __________.

A. hypotheses, verified data and conclusions

B. concepts, patterns and abstractions

C. concepts, processes and generalizations

D. guesses, data and conclusions

13. To elicit more student's response, Teacher G made use of covert responses. Which one did she NOT do?

A. She had the students write their response privately.

B. She showed the correct answers on the overhead after the students have written their responses.

C. She had the students write their responses privately then called each of them.

D. She refrained from judging on the student's responses.


14. What should you do if a parent who is concerned about a grade his child received compared to another student's
grade, demands to see both students' grades?

A. Refuse to show either record.

B. Show both records to him.

C. Refuse to show any record without expressing permission from principal.

D. Show only his child's records.

15. John Watson said:Men are built not born.What does this statement point to?

A. The ineffectiveness of training on a person's development.

B. The effect of environmental stimulation on a person's development.

C. The absence of genetic influence on a person's development

D. The effect of heredity.

16. A guest speaker in one graduation rites told his audience: "Reminder, you are what you choose to be." The guest
speaker is more of a/an __________.

A. realistic

B. pragmatist

C. idealist

D. existentialist

17. The best way for a guidance counselor to begin to develop study skills and habits in underachieving student would be
to __________.

A. have these underachieving students observe the study habits of excelling students

B. encourage students to talk about study habits from their own experiences

C. have them view film strips about various study approaches

D. give out a list of effective study approaches

18. Principal C shares this thought with his teachers:Subject matter should help students understand and appreciate
themselves as unique individuals who accept complete responsibility for their thoughts, feelings, and actions.From which
philosophy is this thought based?
A. Perennialism

B. Essentialism

C. Existentialism

D. Progressivism

19. The search for related literature by accessing several databases by the use of a telephone line to connect a computer
library with other computers that have database is termed __________.

A. compact disc search

B. manual search

C. on-line search

D. computer search

20. Teacher W wants to review and check on the lesson of the previous day? Which one will be most reliable?

A. Having students identify difficult homework problems.

B. Having students correct each other's work.

C. Sampling the understanding of a few students.

D. Explicitly reviewing the task-relevant information necessary for the day's lesson.

21. During the Spanish period, what was/were the medium/media of instruction in schools?

A. The Vernacular

B. English

C. Spanish

D. Spanish and the Vernacular

22. With indirect instruction in mind, which does NOT belong to the group?

A. Problem solving

B. Lecture-recitation
C. Inductive reasoning

D. Discovery

23. I combined several subject areas in order to focus on a single concept for inter-disciplinary teaching. Which
strategy/method did I use?

A. Problem-entered learning

B. Thematic instruction

C. Reading-writing activity

D. Unit method

24. Read the following then answer the question

TEACHER: IN WHAT WAYS OTHER THAN THE PERIODIC TABLE MIGHT WE PREDICT THE UNDISCOVERED ELEMENTS?

BOBBY: WE COULD GOTO THE MOON AND SEE IF THERE ARE SOME ELEMENTS THERE WE DON'T HAVE.

BETTY: WE COULD DIG DOWN INTO THE CENTER OF THE EARTH AND SEE IF WE FIND ANY OF THE MISSING
ELEMENTS.

RICKY: WE COULD STUDY DEBRIS FROM THE METEORITES IF WE CAN FIND ANY.

TEACHER: THOSE ARE ALL GOOD ANSWERS. BUT WHAT IF THOSE EXCURSIONS TO THE MOON, TO THE CENTER OF
THE EARTH, OR TO FIND METEORITES WERE TOO COSTLY AND TIME CONSUMING? HOW MIGHT WE USE THE
ELEMENTS WE ALREADY HAVE HERE ON EARTH TO FIND SOME NEW ONES?

Question: The Teacher's questions in the above exchange are examples of __________ questions.

A. fact

B. concept

C. direct

D. closed
25. In his second item analysis, Teacher H found out that more from the lower group got the test item # 6 correctly. This
means that the test item __________.

A. has a negative discriminating power

B. has a lower validity

C. has a positive discriminating power

D. has a high reability

26. For maximum interaction, a teacher ought to avoid __________ questions.

A. informational

B. rhetorical

C. leading

D. divergent

27. The teacher's first task in the selection of media in teaching is to determine the ______.

A. choice of the students

B. availability of the media

C. objectives of the lesson

D. technique to be used

28. Principal B tells her teachers that training in the humanities is most important. To which educational philosophy does
he adhere?

A. Existentialism

B. Perennialism

C. Progressivism

D. Essentialism

29. Who among the following needs less verbal counseling but needs more concrete and operational forms of assistance?
The child who __________.
A. has mental retardation

B. has attention-deficit disorder

C. has learning disability

D. has conduct disorder

30. After giving an input on a good paragraph, Teacher W asks her students to rate a given paragraph along the elements
of a good paragraph. The students' task is in level of __________

A. application

B. analysis

C. evaluation

D. synthesis

31. In the Preamble of the Code of Ethics of Professional Teachers, which is NOT said of teachers?

A. LET passers

B. Duly licensed professionals

C. Possess dignity and reputation

D. With high-moral values as well as technical and professional competence

32. Study this group of tests which was administered with the following results, then answer the question.

Subject            Mean   SD       Ronnels's Score

Math    56        10        43

Physics           41        9          31

English            80        16        109

In which subject(s) did Ronnel perform most poorly in relation to the group's performance?

A. English

B. English and Math

C. Math
D. Physics

33. Teacher G's lesson objective has something to do with the skill of synthesizing? Which behavioral term is most
appropriate?

A. Test

B. Assess

C. Appraise

D. Theorize

34. Standard deviation is to variability as mean is to __________.

A. coefficient of correlation

B. central tendency

C. discrimination index

D. level of difficulty

35. If you agree with Rizal on how you can contribute to our nation's redemption, which should you work for?

A. Opening our doors to foreign influence

B. Upgrading the quality of the Filipino through education

C. Stabilizing the political situation

D. Gaining economic recovery

36. Teacher T taught a lesson denoting ownership by means of possessives. He first introduced the rule, then gave
examples, followed by class exercises, then back to the rule before he moved into the second rule. Which presenting
technique did he use?

A. Combinatorial

B. Comparative

C. Part-whole

D. Sequence
37. Theft of school equipment like tv, computer, etc. by teenagers in the community itself is becoming a common
phenomenon. What does this incident signify?

A. Prevalence of poverty in the community.

B. Inability of school to hire security guards.

C. Deprivation of Filipino schools.

D. Community's lack of sense of co-ownership.

38. As a teacher, what do you do when you engage yourself in major task analysis?

A. Test if learning reached higher level thinking skills.

B. Breakdown a complex task into sub-skills.

C. Determine the level of thinking involved.

D. Revise lesson objectives.

39. With-it-ness, according to Kounin, is one of the characterestics of an effective classroom manager. Which phrase goes
with it?

A. Have hands that write fast.

B. Have eyes on the back of your heads.

C. Have a mouth ready to speak.

D. Have minds packed with knowledge.

40. Ruben is very attached to his mother and Ruth to her father. In what developmental stage are they according to
Freudian psychological theory?

A. Oedipal stage

B. Latent stage

C. Anal stage

D. Pre-genital stage

41. Teacher Y does norm-referenced interpretation of scores. Which of the following does she do?
A. She describes group performance in relation to a level of mastery set.

B. She uses a specified content as its frame of reference.

C. She compares every individual students' scores with others' scores.

D. She describes what should be their performance.

42. As a teacher, you are a reconstructionist. Which among these will be your guiding principle?

A. I must teach the child every knowledge, skill, and value that he needs for a better future.

B. I must teach the child to develop his mental powers to the full.

C. I must teach the child so he is assured of heaven.

D. I must teach the child that we can never have real knowledge of anything.

43. From whom do we owe the theory of deductive interference as illustrated in syllogisms?

A. Plato

B. Scorates

C. Aristotle

D. Pythagoras

44. Teacher B engages her students with information for thorough understanding for meaning and for competent
application. Which principle governs Teacher B's practice?

A. Contructivist

B. Gestalt

C. Behaviorist

D. Cognitivist

  

45. What was the most prominent educational issue of the mid 1980s?

A. Bilingual Education

B. Values Education

C. Accountability
D. Mainstreaming

46. A teacher/student is held responsible for his actions because s/he __________.

A. has instincts

B. is mature

C. has a choice

D. has reason

47. For which may you use the direct instruction method?

A. Become aware of the pollutants around us.

B. Appreciate Milton's Paradise Lost.

C. Use a microscope properly.

D. Distinguish war from aggression.

48. The concepts of trust vs. maturity, autonomy vs. self-doubt, and initiative vs. guilt are most closely related with the
works of __________.

A. Erikson

B. Piaget

C. Freud

D. Jung

49. What measure of central tendency does the number 16 represent in the following data: 14, 15, 17, 16, 19, 20, 16, 14, 16?

A. Mode

B. Median

C. Mode and median

D. Mean

50. The principle of individual differences requires teachers to __________.


A. give greater attention to gifted learners

B. provide for a variety of learning activities

C. treat all learners alike while in the classroom

D. prepare modules for slow learners in class

Professional Education - SET D - Part 2

- November 22, 2018

Professional Education - SET D - Part 2

51. Which of the following propositions is attributed to Plato?

A. Truth is relative to a particular time and place.

B. Human beings create their own truths.

C. Learning is the discovery of truth as Iatent ideas are brought to consciousness.

D. Sense perception is the most accurate guide to knowledge.

  

52. Which is one characteristic of an effective classroom management?

A. It quickly and unobtrusively redirects misbehavior once it occurs.

B. It teaches dependence on others for self-control.

C. It respects cultural norms of a limited group students.

D. Strategies aresimple enough to be used consistently.

53. Out of 3 distracters in a multiple choice test item, namely B, C, and D, no pupil chose D as answer. This implies that D
is __________

A. an ineffective distracter

B. a vague distracter
C. an effective distracter

D. a plausible distracter

54. I drew learners into several content areas and encouraged them to solve a complex question for inter-disciplinary
teaching. Which strategy did I use?

A. Problem-centered learning

B. Unit method

C. Reading-writing activity

D. Thematic instruction

55. Value clarification as a strategy in Values Education classes is anchored on which philosophy?

A. Existentialism

B. Christian philosophy

C. Idealism

D. Hedonism

56. Teacher A is a teacher of English as a Second Language. She uses vocabulary cards, fill-in-the-blank sentences,
dialogues, dictation and writing excercises in teaching a lesson about grocery shopping. Based on this information, which
of the following is a valid conclusion?

A. The teacher is applying Bloom's hierachy of cognitive learning.

B. The teacher is teaching in a variety of ways because not all students learn in the same manner.

C. The teacher wants to make herteachirig easier by having less talk.

D. The teacher is emphasizing reading and writing skills.

57. If teacher has to ask more higher-order questions, he has to ask more __________ questions.

A. closed

B. fact

C. concept

D. convergent
58. Are percentile ranks the same as percentage correct?

A. It cannot be determined unless scores are given.

B. It cannot be determined unless the number of examinees is given.

C. No

D. Yes

59. Which method has been proven to be effective in courses that stress acquisition of knowledge?

A. Socratic method

B. Cooperative learning

C. Mastery learning

D. Indirect instruction

60. A teacher's summary of a lesson serves the following functions, EXCEPT

A. it links the parts of the lesson

B. lt brings together the information that has been discussed

C. it makes provisions for full participation of students.

D. it clinches the basic ideas or concepts of the lesson.

61. Which is a true foundation of the social order?

A. Obedient citizenry

B. The reciprocation of rights and duties

C. Strong political leadership

D. Equitable distribution of wealth

62. Which one can enhance the comparability of grades?


A. Using common conversion table for translating test scores in to ratings

B. Formulating tests that vary from one teacher to another

C. Allowing individual teachers to determine factors for rating

D. Individual teachers giving weights to factors considered for rating

63. Who among the following puts more emphasis on core requirements, longer school day, longer academic year and
more challenging textbooks?

A. Perennialist

B. Essentialist

C. Progressivist

D. Existentialist

64. Which test has broad sampling of topics as strength?

A. Objective test

B. Short answer test

C. Essay test

D. Problem type

65. Read this question:How will you present the layers of the earth to your class?This is a question that

A. directs

B. leads the student to evaluate

C. assesses cognition

D. probes creative thinking

66. In Krathwohl's affective domain of objectives, which of the following is the lowest level of affective behavior?

A. Valuing

B. Characterization

C. Responding
D. Organization

67. Rodel is very aloof and cold in his relationships with his classmates. Which basic goal must haye not been attained by
Rodel during his developmental years, according to Erikson's theory on psychological development?

A. Autonomy

B. Trust

C. Initiative

D. Generativity

68. Two students are given the WISE II. One has a full scale IQ of 91, while the other has an IQ of 109. Which conclusion
can be drawn?

A. The second student has significantly higher intellectual ability

B. The first student is probably below average, while the second has above average potential

C. Both students are functioning in the average range of intellectual ability

D. Another IQ test should be given to truly assess their intellectual potential

69. To come closer to the truth we need togo back to the things themselves.This is the advice of the

A. behaviorists

B. phenomenologists

C. idealists

D. pragmatists

70. Which guideline in test construction is NOT observed in this test item: Jose Rizal wrote __________.

A. The central problem should be packed in the stem.

B. There must be only one correct answer.

C. Alternatives must have grammatical parallelism.

D. The alternates must be plausible.


71. In what way can teachers uphold the highest possible standards of quality education?

A. By continually improving themselves personally and professionally

B. By wearing expensive clothes to change people's poor perception of teachers

C. By working out undeserved promotions

D. By putting down other professions to lift the status of teaching

72. Each teacher is said to be a trustee of the cultural and educational heritage of the nation and is, under obligation to
transmit to learners such heritage. Which practice makes him fulfill such obligation?

A. Use the latest instructional technology.

B. Observe continuing professional education.

C. Use interactive teaching strategies.

D. Study the life of Filipino heroes.

73. Studies in the areas of neurosciences disclosed that the human brain has limitless capacity. What does this imply?

A. Some pupils are admittedly not capable of learning.

B. Every pupil has his own native ability and his learning is limited to this nativeabilty.

C. Every child is a potential genius.

D. Pupils can possibly reach a point where they have learned everything.

74. In a treatment for alcoholism, Ramil was made to drink an alcoholic beverage and then made to ingest a drug that
produces nausea. Eventually, he was nauseated at the sight and smell of alcohol and stopped drinking alcohoL Which
theory explains this?

A. Operant conditioning

B. Social Learning Theory

C. Associative Learning

D. Attribution Theory

75. The following are used in writing performance objectives, EXCEPT


A. delineate

B. diagram

C. integrate

D. comprehend

76. Which guideline must be observed in the use of prompting to shape the correct performance of your students?

A. Use the least intrusive prompt first.

B. Use all prompts available.

C. Use the most intrusive prompt first.

D. Refrain from using prompts.

77. On whose philosophy was A. S. Neil's Summerhill, one of the most experimental schools, based?

A. Rousseau

B. Pestalozzi

C. Montessori

D. John Locke

78. "In the light of the facts presented, what is most likely to happen when ... ?" is a sample thought question on

A. inferring

B. generalizing

C. synthesizing

D. justifying

79. Which applies when skewness is zero?

A. Mean is greater than the median

B. Median is greater than mean

C. Scores have three modes


D. Scores are normally distributed

80. Whose influence is the education program that puts emphasis on self-development. through the classics, music, and
rituals?

A. Buddha

B. Mohammed

C. Confucius

D. Lao Tsu

81. Which is an appropriate way to manage off-task behavior?

A. Make eye contact.

B. Stop your class activity to correct a child who is no longer on task.

C. Move closer to the child.

D. Redirect a child's attention to task and check his progress to make sure he is continuing to work.

82. In Krathwohl's taxonomy of objectives in the affective, which is most authentic?

A. Characterization

B. Organization

C. Responding

D. Valuing

83. Study this group of tests which was administered with the following results, then answer the question.

Subject Mean SD Ronnels's Score

Math 56 10 43

Physics 41 9 31

English 80 16 109
In which subject(s) were the scores most homogenous?

A. Math

B. English

C. Physics

D. Physics and Math

84. We encounter people whose prayer goes like this: "O God, if there is a God; save my soul, if I have a soul" From whom
is this prayer?

A. Stoic

B. Empiricist

C. Agnostic

D. Skeptic

85. The test item "Group the following items according to shape" is a thought test item on __________.

A. creating

B. classifying

C. generalizing

D. comparing

86. Teacher B is a teacher of English as a Second Language. She uses vocabulary cards, fill-in-the-blank sentences,
dictation and writing exercises in teaching a lesson about grocery shopping. Based on this information, which of the
following is a valid conclusion?

A. The teacher is reinforcing learning by giving the same information in, a variety of methods.

B. The teacher is applying Bloom's hierachy of cognitive learning.

C. The teacher wants to do less talk.

D. The teacher is emphasizing listening and speaking skills.

87. What is the mean of this score distribution: 4, 5, 6, 7, 8, 9, 10?


A. 7

B. 6

C. 8.5

D. 7.5

88. Teacher P wants to develop the skill of synthesizing in her pupils. Which one will she do?

A. Ask her students to formulate a generalization from the data shown in graphs.

B. Ask her students to answer questions beginning withWhat if ...

C. Tell her pupils to state data presented in graphs.

D. Directs her students to ask questions on the parts of the lesson not understood.

89. Bruner's theory on intellectual development moves from enactive to iconic and symbolic stages. In which stage(s) are
diagrams helpful to accompany verbal information?

A. Enactive and iconic

B. Symbolic

C. Symbolic and enactive

D. Iconic

90. Which holds true to standardized tests?

A. They are used for comparative purposes

B. They are administered differently

C. They are scored according to different standards

D. They are used for assigning grades

91. An effective classroom manager uses low-profile classroom control. Which is a low-profile classroom technique?

A. Note to parents

B. After-school detention

C. Withdrawal of privileges
D. Raising the pitch of the voice

92. Which type of report refers toon-the-spotdescription of some incident, episode or occurrence that is being observed
and recorded as being of possible significance?

A. Autobiographical report

B. Biographical report

C. Value and interest report

D. Anecdotal report

93. By what name is Indirect instruction the Socratic method also known?

A. Mastery learning

B. Indirect Method

C. Morrison method

D. Questioning method

94. Teacher A knows of the illegal activities of a neighbor but keeps quiet in order not to be involved in any investigation.
Which foundational principle of morality does Teacher A fail to apply?

A. The end does not justify the means.

B. The principle of double-effect

C. Always do what is right.

D. Between two evils, do the lesser evil.

95. What should a teacher do for students in his class who are not on grade level?

A. Give them materials on their level and let them work at a pace that is reasonable forthem, trying to bring them up to a
grade level.

B. Give them the same work as the other students, because they will absorb as much as they are capable of.

C. Give them the same work as the other students, not much, so that they won't feel embarrassed.

D. Give them work on the level of the other students and work a little above the classmates level to challenge them.
96. A child who gets punished for stealing candy may not steal again immediately. But this does not mean that the child
may not steal again. Based on Thorndike's theory on punishment and learning, this shows that __________

A. punishment strengthens a response

B. punishment removes a response

C. punishment does not remove a response

D. punishment weakens a response

97. Helping in the development of graduates who aremaka-Diyosis an influence of

A. naturalistic morality

B. classical Christian morality

C. situational morality

D. dialectical morality

98. Teacher H gave her first-grade class a page with a story in which pictures take the place of some words. Which method
did she use?

A. The whole language approach

B. The Spaulding method

C. The rebus method

D. The language experience approach

99. Soc exhibits fear response to freely roaming dogs but does not show fear when a dog is on a leash or confined to a
pen. Which conditioning process is illustrated

A. Generalization

B. Extinction

C. Acquisition

D. Discrimination

100. Teachers often complain of numerous non-teaching assignments that adversely, affect their teaching. Does this mean
that teachers must be preoccupied only with teaching?
A. Yes, if they are given other assignments, justice demands that they be properly compensated.

B. Yes, because other community leaders, not teachers, are tasked to leadin community activities.

C. NO, because every teacher is expected to provide leadership and initiative in activities for betterment of communities.

D. Yes, because teaching is enough full time job.

Professional Education - SET D - Part 3

- December 12, 2018

101. The social science share many commodities like the ______
a.faces on understanding and explaining human behavior
b.systematic collection and application data
c.use of scientific method
d.all of the above

102. When s person shows the ability to understand and appreciate the similarities and differences in customs, values, and beliefs in one's
culture, he/she is said to be
A. Possess social literacy
B. Socio-cultural awareness
C. Possess multi-cultural literacy 
D. Possess global awareness

103. It is a type of political culture where citizens view their active involvement in politics as both desirable and effective. Citizens see their
active involvement in politics as crucial to the life of community
A. Subject culture
B. Civic culture
C. Participant culture
D. Parochial culture

104. Rote learning in the teaching-learning process is best describe as_____.

A. Motivated
B. Discovery
C. In-depth
D. Automated

105. By way of language medium, which is best for student of diverse cultural background ?

A. Regional dialect
B. National language 
C. English and Filipino
D. Multi- lingual teaching

106. With the kindergarten act RA 10157 which applies?

A. No child can go to grade 1 without completing kindergarten


B. A child beyond 5 years of age may go to grade 1
C. A child who is more than 5 years of age won't qualify for kindergarten
D. The kindergarten child learns subjects as Math, English, Science and Filipino

107. The use of technology in learning has been highly associated with the application of the_______.
A. Essentialist learning philosophy
B. Behaviorist learning philosophy
C. Progressivist learning philosophy
D. Contructivist learning philosophy

108.What visual aid is useful in showing the trend in the temperature change month-to-month?
A. Bar graph
B. Flow chart
C. Map
D. Drawing

109.Teacher H asks one of her students, “What do you want to become when you grow up?” This
question is an indication of what kind of philosophy?
A. Progressivism
B. Existentialism
C. Naturalism
D. Idealism

110. How many percent is given to written work in Language, AP, and ESP for Grade 1-10?
a. 30% 
b. 40%
c. 20%
d. 50%

111. How many percent is given to written work in Math and Science for Grade 1-10?
a. 30%
b. 40%
c. 20%
d. 50%

112. How many percent is given to written work in MAPEH, EPP, and TLE for Grade 1-10?
a. 30%
b. 40%
c. 20%
d. 50%

113. How many percent is given to performance task in MAPEH, EPP, and TLE for Grade 1-10?
a. 40%
b. 50%
c. 60%
d. 30%

114. How many percent is given to performance task in Math and Science for Grade 1-10?
a. 40%
b. 50%
c. 60%
d. 30%

115. How is so-called colonial mentality manifested?


A. Cultural relativism
B. Ethnocentrism
C. Cultural diversity
D. Xenocentrism

116. It illustrates how a portion of the data relates with the whole.
A. Pie graph
B. Bar graph
C. Line graph
D. Rectangle graph
117. Which thought activity or cognitive ability is tested in the essay question: Using christian ethics, how can population control be managed?

A. Relating cause and effect


B. Application
C. Justifying
D. Summarizing

118.The teacher begins to use technology tools to deliver curriculum content to the system.
a. Transformation
b. Adoption
c. Active
d. Entry

119.The student uses technology tools to collaborate with other rather than working individually at all times.
a. Collaborative
b. Authentic
c. Goal directed
d. Infusion

120. .Students use technology tools to set goals, plan activities, monitor progress, and evaluation result rather than simply completing
assignments without reflection.
a. Goal directed
b. Constructive
c. Adoption
d. Entry

121.The teacher direct students in conventional and procedural use of technology tool.
a. Infusion
b. Constructive
c. Entry
d. Adoption

122.The teacher encourage the innovative use of technology tools.


a. Active
b. Transformation
c. Adaptation
d. Adoption

123. Technology tools are used to facilitate higher order learning activities that may not have been possible without the use of technology.
a. Adaptation
b. Adoption
c. Active
d. Transformation

124.The teacher provides the learning context and the student choose the technology tools to achieve outcomes.
a. Adoption
b. Adaptation
c. Infusion
d. Entry

125.The teacher facilitates students independently using technology tools.


a. Entry
b. Infusion
c. Adaptation
d. Adoption
126. Students are actively engaged in using technology as a tool rather than passively receiving information from the technology.
a. Goal directed
b. Authentic
c. Active
d. Collaborative

127. .Students use technology tools to link learning activities to the world beyond the instructional setting rather than working on
discontextualized assignments.
a. Entry
b. Infusion
c. Authentic
d. Goal directed

LET Reviewer : Professional Education part 1

- November 02, 2018

LET Reviewer : General Education

1. As a parent and at the same time a teacher, which of the following will you to do to show your cooperation to a PTA project in your school
to be financed with the proceeds of the sales of the school canteen where food prices are a little bit higher? 

A. Bring food for you and your children, but always make it a point to buy in the school canteen. 

B. Buy all your food in the school canteen but request for a discount. 

C. Bring food enough for you and your children but do not eat in the canteen. 

D. Buy all your food from the school canteen even if you cannot afford to do every day.

2. How can you help a habitual borrower of money get rid of his habit?

A. Let him do something for you in return for the money you lent him.
B. Direct him to others.
C. Do not lend him anymore. 
D. Ask for a collateral for the cash he is loaning.

3. Periodic checks on student seatwork with a smile and pat on the should effectively reinforce good study habit is an example of ____.

A. Discrimination reinforcement           
B. Continuous reinforcement
C. Variable-ratio schedule                          
D. Fixed interval and variable interval schedule

4. A person strives to work at a given task because of a need. Which of the following situations can make a person strive to meet his needs?
(needs of the learners)

A. Minimize the unpleasant consequences of student involvement.


B. Utilize your own opinion as teacher in making final decisions in the classroom.
C. Use unfamiliar materials as examples in order to initially arouse their curiosity.
D. Ask pupils to submit test questions or reactions from which you can select topics.  

5. The singing of the National Anthem in schools in an offshoot of the philosophy of ____.

A. Nationalism                                       
B. Naturalism
C. Pragmatism                                       
D. Socialism

6. The environment in order to facilitates, learning must be interactive. Which of the following best typifies this kind of environment?

A. The child goes out and discovers for himself some rock or fossil. 
B. The child listens to a lecture on fossils given by the teacher.
C. The child summarizes the section on fossils in his science textbook.
D. The child copies a list of fact concerning fossils on the blackboard.

7. Social development means that acquisition of the ability to behave in accordance with ____.

A. Social expectation                                           


B. Social insight
C. Stereotyped behavior                                       
D. Universal norms

8. When an adolescent combines ability to use deductive and inductive reasoning in constructing realistic rules that he can respect and live
by, how does he perceive his environment?

A. He views the world from his own perspective 


B. He interprets events from a limited views
C. He views events apart from himself and other people
D. He views the world and himself through the eyes of other people

9. Who introduced the technique of using the drawing of a man as a measure of intelligence? 

A. Aristotle           
B. Herbert                   
C. Goodenough                         
D. Benet

10. Which Republic Act provides government assistance to students and teachers in private education? 

A. RA 7784           
B. RA 6728                   
C. RA 7836                           
D. RA 6675

11. The authoritarian setting in the Filipino home is reinforced by a classroom teacher who:

A. Encourage pupils to ask questions                        B. Is open to suggestions


C. Prescribes what pupils should do                          D. Ask open ended questions

12. Who among the following believes that learning requires disciplined attention, regular homework, and respect for legitimate authority?

A. Essentialist             


B. Perennialist               
C. Progressivist                 
D. Reconstructionist
13. The Constitutional provision on language has the following aims, EXCEPT:

A. To make the regional dialect as auxiliary media of instructions in regional school


B. To maintain English as a second language
C. To make Filipino the sole medium of instruction (no-against bilingual; multilingual policy)  
D. To make Filipino the national language & medium of instruction & communication

14. The tendency to emphasize so much on school beautification to the detriment of pupils performance illustrates the:

A. Filipino’s lack of seriousness                           


B. Filipino’s sense of humor
C. Filipino’s lack of reflection                                 
D. Filipino’s love for “porma”

15. Which is NOT a characteristic of democratic discipline? 

A. Child has opportunity to express his/her opinion


B. Child given punishment is related to the misdeed
C. Child understands the meaning of rules
D. Child obeys blindly 

16. Who among the following stressed the processes of experience and problem solving?

A. Dewey                 B. Aristotle                     C. Hegel                         D. Plato

17. Which of the following reasons for measuring student achievement is NOT valid? 

A. To prepare feedback on the effectiveness of the learning process

B. To certify that students have attained a level of competence in a subject area

C. To discourage students from cheating during test and getting high scores 

D. To motivate students to learn & master the materials they think will be covered by the achievement test.

18. Which characterizes the perfectionist type of students? 

A. Does not volunteer or initiate

B. Give up easily

C. Rarely complete tasks

D. Often anxious, fearful or frustrated about quality work

19. When a school decides to work on a thematic curriculum which should be out of the picture? 
A. Peer collaboration   

B. Integration   

C. Team teaching 

D. Competition 

20. Teacher wants to teach his pupils the technique on reading for information. Which technique should be use?

A. Text structure 

B. Prior knowledge

C. Story map

D. SQ3R

21. In instructional planning, which among these three: unit plan, course plan, lesson plan is (are) most specific? ____plans

A. Course and lesson

B. Course

C. Lesson 

D. Unit

22. The use of drills in the classroom is rooted on Thorndike’s law of: 

A. Readiness

 B. Exercise         

C. Effect

D. Belongingness

23. Positive interdependence as an element of collaborative learning means that the students must: 

A. Learn to depend on each other to achieve a goal

B. Depend on the diligent students


C. Help one another in the individual test for everyone to pass

D. Be grouped heterogeneously

24. Which of the following measures should a teacher do to a principal whom she would like to file a case of sexual harassment w/o
violating the relationship of the teacher to her superiors? 

A. Present the case before competent authority & prepare to prove the charge 

B. Write an anonymous letter to a higher school official to denounce the superior

C. Call a parent-teacher meeting and denounce the superior

D. Encourage the other teachers & students to hold a demonstration to oust the superior

25. Pick out the situation that illustrates the duty of a new teacher to the state: 

A. Take a long vacation which she firmly believes she deserves after four years of diligent study before taking the examination for teachers

B. Apply for teaching job where eligibility is not required to gain teaching experience before taking the teachers board examination.

C. Prepare for the wedding she and her boyfriend have long planned to be able to raise a family with children which they plan to rear as good
citizen of our country

D. Take the licensure examination for teacher and an oath to do her best to help carry out the policies of the state

26. Parents are up in arms on the telephone bills that pay for sex calls. What is the solution to this problem? 

A. The telephone company is to blame for this (not a solution)

B. The government restriction have no teeth (not a solution)

C. Parents allow this to make their children modern (inconsistent to the situation)

D. Parents, school and students should discuss this openly

27. Cooperatives are encouraged in as many groups as possible. What agency controls the different cooperatives? 

A. Security and Exchange Commission

B. Commission on Audit

C. Department of Local Government

D. Bureau of Cooperative (Cooperative Development Authority)


28. Society and media know drinking starts off drug addiction. What should be discussed in schools? 

A. Drug addiction has been traced to drinking wine

B. Nobody drinks at home except father

C. TV ads show drinking is a source of fellowship (the only choice that includes media & society) 

D. High taxes on liquor will be a deterrent to eventual drug use

29. Cooperatives have branched out to consumers cooperative. Schools have included the concepts of cooperatives. Where is it practiced? 

A. School book stores

B. Schools uniform purchases

C. School canteen 

D. Class stores

30. A student collapsed in her social class. It was found out that he did not eat her lunch. What principle is shown in the situation?

A. Psychological need

B. Physiological need 

C. Somatotonic

D. Safety need

31. The main function of a philosophy of education is to: 

A. Aid the learner to build his own personal philosophy 

B. Reconsider existing educational goals in the light of society’s needs 

C. Provide the academic background prerequisite to learning 

D. Define the goals & set the direction for which education is to strive

32. Which technique(s) enable(s) a teacher to identify and eventually assists students with interpersonal difficulties? 
A. Anecdotal record 

B. Cumulative record 

C. Personal inventory 

D. Sociogram 

33. Teachers and students can participate in these levels of computer use. Give the order of computer use from simplest to complex? 

A. Computer competency, computer literacy, competency expertise 

B. Computer literacy, computer expertise, computer competency 

C. Computer competency, computer expertise, computer literacy 

D. Computer literacy, computer competency, computer expertise 

34. Which one is considered the “Brain” of the microcomputer? 

A. CPU 

B. Software 

C. Video screen 

D. Keyboard 

35. A group activity wherein one group representative presents the output to the bigger group rather than individual pupils presenting the
output is known as: 

A. Consensus decision 

B. Composite report 

C. Jury trial 

D. Agenda 

36. In the formulation of classroom, which of the following should a teacher refrain from doing? 

A. State classroom regulation as clearly as possible 

B. Enlist student aid in the formulation of classroom regulation 


C. Enforce classroom regulations consistently and fairly 

D. Teacher and the class should make as many regulations as possible 

37. Zero standard deviation means that: 

A. The students’ scores are the same 

B. 50% of the score obtained is zero 

C. More than 50% of the score obtained is zero 

D. Less than 50% of the scores obtained is zero 

38. Which is the least authentic mode of assessment? 

A. Paper-and-pencil test in vocabulary 

B. Oral performance to assess student’s spoken communication skills 

C. Experiments in science to assess skill in the use of scientific methods 

D. Artistic production for music or art subject 

39. In what period of a child is physical growth fastest? 

A. Prenatal period 

B. Early childhood 

C. Early adolescence 

D. Pubescence 

40. How does fear affect the voluntariness of an act? 

A. Makes the act involuntary 

C. Increase voluntariness 

B. No effect at all 

D. Lessens but not destroy voluntariness (fear- modifier of human act) 

41. A group of people asserts that their culture is superior to another. This exemplifies: 
A. Cultural gap 

B. Norm conflict 

C. Cultural conflict 

D. Ethnocentrism (equivalent to superiority complex) 

42. A test consists of a graph showing the relationship between age and population. Following it is a series of true-false items based on the
graph. Which type of test does this illustrate? 

A. Laboratory exercise 

B. Performance 

C. Problem solving 

D. Interpretative 

43. Which curricular move served to strengthen spiritual and ethical values? 

A. Integration of creative thinking in all subject 

B. Reducing the number of subject areas into the skill subject 

C. Introduction of Value Education as a separate subject area 

D. Re-introducing Science as all subject in Grade 1 

44. Which computer seems to have the most potential for the classroom? 

A. Mainframe computer 

B. Microcomputer 

C. Minicomputer 

D. LPC 

45. A teacher notices glaring wrong pronunciation of vowel sounds among her students necessitating more practice. Which of the following
activities would be at most help? 

A. Dictionary use 
B. Assignments 

C. Review 

D. Drill

46. Which storage device is significantly more efficient in holding information 

A. Hard disk 

B. Software 

C. Floppy disk 

D. Audio cassette 

47. The Filipino tendency to resort to the easy way out from a term paper as a course requirement by hiring a ghost writer or by passing a
photocopied term paper provide which Filipino traits? 

A. Anticipation 

B. Ambivalence 

C. Pakikisama 

D. Lack of discipline 

48. “No pain, no gain”. This means that: 

A. Only those willing to carry the crosses that like impose can share the joy of life 

B. One should be a penitent every good Friday by carrying his cross(not logical) 

C. The more suffering in this life, the more one is assured of heaven (fanatism) 

D. One should look for suffering to save himself/herself (not logical) 

49. Which trust on value formation is meant to help the students make use of their thinking and scientific investigation to decide on topics
and questions above values? 

A. Value inculcation 

B. Value clarification (investigation) 

C. Analysis 
D. Moral development 

50. Which interactive teaching should be AVOIDED? 

A. Using “put down” strategy 

B. Asking more divergent questions 

C. Using multiple response strategy 

D. Asking more evaluative questions

- November 02, 2018

51. Rights which cannot be renounced or transferred because they are necessary for the fulfillment of man’s primordial obligations are
called: 

A. Alienable rights 
B. Inalienable rights 
C. Perfect rights 
D. Acquired rights 

52. In mastery learning the definition of an acceptable standard of performance is called 

A. Condition 
B. Behavior 
C. SMART 
D. Criterion measure 

53. A negative discrimination index means that: 

A. More from the lower group answered the test items correctly 
B. The items could not discriminate between the lower and upper group 
C. More from the upper group answered the test item correctly 
D. Less from the lower group got the test item correctly 

54. Your teacher is of the opinion that the world and everything in it are changing and so teaches you the skill to cope with the changes.
Which in his governing philosophy? 

A. Experimentalism (progressivism) 
B. Existentialism 
C. Realism 
D. Idealism 

55. For brainstorming to be effective which one should be out? 

A. Making use of the other’s ideas shared 


B. Non-threatening atmosphere 
C. Teacher’s judge mental attitude 
D. Openness to idea 

56. Which statement on spaced and massed learning is CORRECT? 

A. Massed learning is better than spaced learning 


B. Spaced learning is better than massed learning (teach less, teach well) 
C. Massed learning is as effective as spaced learning 
D. Both massed learning & spaced learning are not effective 

57. A teacher is a facilitator of learning and of the development of the youth. Which practice NOT keeping with his role as facilitator? 

A. Considers the multiple intelligences of learners


B. Humiliates misbehaving pupils 
C. Dialogs with parents and with other members of the community 
D. Keeps himself abreast with educational trends

58. Which one indicates a teacher’s genuine enthusiasm and pride in teaching? 

A. Sticking to teaching for the moment that there are no better offers 
B. Telling everyone that he went to teaching for there was no other choice them 
C. Engaging himself in continuing professional education 
D. Belittling the re-numeration one gets from teaching 

59. In writing performance objective which word is NOT acceptable? 

A. Manipulate 
B. Integrate 
C. Delineate 
D. Comprehend 

60. When is giving praise INEFFECTIVE? When it: 

A. Uses the accomplishment of peers as the context for describing a student’s present accomplishment. 
B. Provides information to student’s about their competence and the value of their accomplishments C. Focuses students attention on her own
task-relevant behavior 
D. Shows spontaneity, variety and other signs of credibility

61. Which statement applies when scores distribution is negatively skewed? 

A. The mode corresponds to a low value


B. The median is higher than the mode 
C. The mode and median are equal
D. The mean corresponds to a high value

62. The use of the process approach gives the students the opportunity to: 

A. Learn to their own


B. Make use of laboratory apparatuses
C. Apply the scientific method
D. Learn how to learn 

63. A comprehension skill of higher level which may be inferred or implied from reading is:

A. Picking out the main idea 


B. Following direction
C. Nothing specific details
D. Drawing conclusion (synthesis)

64. In the Preamble of the Code of Ethics of Processional Teachers, which is not mention about teachers? 

A. Dully licensed professionals


B. LET passers 
C. Possess dignity and reputation
D. With high moral values

65. What does a skewed score distribution mean? 

A. The scores are concentrated more at one end or the other end 
B. The mode, the mean, and the median are equal
C. The mean and the median are equal
D. The scores are normally distributed

66. Which is implied by a negatively skewed score distribution? 

A. The scores are evenly distributed from the left to the right
B. Most pupils are underachievers
C. Most of the scores are high 
D. Most of the scores are low

67. A teacher discovers that a product of certain bottling company brings about damage to teeth. Much as he wants to share the products
of his research, he could not because of harassment from all sides. Which teacher’s right is violated? 
A. Right to property
B. Right to one’s honor
C. Academic freedom 
D. Right to make a livelihood

68. Why can the calculator do arithmetic? Because 

A. A computer inside the calculator tells it how 


B. A watch inside direct it
C. A typewriter inside does it
D. A TV inside shows it

69. On which constitutional provision is the full or partial integration of capable deaf and blind students in the classroom based> the
provision on 

A. Providing citizenship and vocational training to adult citizen


B. Protecting and promoting the right of all citizen to qualify education 
C. Academic freedom
D. Creating scholarship for poor and deserving students

70. Teaching in the cognitive, psychomotor and effective domains is based on the concept that the learner is a: 

A. Moral and feeling being


B. Thinking, feeling and acting being 
C. Material and an acting being
D. Spiritual and material being

71. Both Muslim and Christian value marriage but the Muslim practices polygamous marriage while the Christian practices
monogamous marriage. What is this called? 

A. Cultural relativism
B. Ethical relativism 
C. Acculturation
D. Enculturation

72. Teacher wants to compare 2 concepts. With which technique can accomplish this best? 

A. K-W-L technique
B. Spider-web
C. Venn diagram 
D. Histogram

73. To build a sense of pride among Filipino youth which should be done? 

A. Re-study our history and stress on our achievements as a people 


B. Set aside the study of local history
C. Re-study our history from the perspective of our colonizers
D. Replace the study of folklores and myths with technical subjects

74. When necessary conditions are present, the use of inductive method is preferred because 

A. There is greater active participation on the part of pupils


B. It gives the teacher more time to rest
C. It needs only a few instruction materials
D. Academic time is used wisely

75. Which is the best reason why teacher begins a lesson in Math by checking and reviewing on the previous day’s assignment and
provides practice and drills? 

A. Check if parents guide their children in the making of assignment


B. Make sure that the students understand the pre-requisite skills of the lesson 
C. Prepare the students for the mastery test
D. Make learning interesting and enjoyable for students

76. Which is selective reading technique meant at getting at important facts very fast? 

A. Skim reading
B. Scanning 
C. Oral reading
D. Silent reading
77. For counseling to be successful which assumption must be AVOIDED? 

A. The environment must provide assurance of confidentiality


B. The student is willing to participate in the process
C. The counselor must be able to relate to the student
D. The counselor tells the student what to do 

78. Which technique is most appropriate when a teacher wants a group to agree on a plan of action?

A. Composite report
B. Symposium
C. Agenda
D. Consensus decision making 

79. Which term applies to the search for related literature by computer access of data bases on discs kept in libraries? 

A. On-line research 
B. Compact discs computer research
C. Manual research
D. Computer research

80. Which best indicates the effectiveness classroom activities? 

A. The laughter and enjoyment of students


B. The application of concept learned in daily life 
C. The utilization of varied techniques and approaches
D. The variety of instructional materials used

81. The main purpose of the compulsory study of the Constitution in Philippine schools is to 

A. Develop the students into responsible, thinking citizens 


B. Acquaint students w/ the historical development of the Philippine Constitution
C. Prepare students for law-making
D. Make constitutional experts of the students

82. Some students who are high in the scholastic aptitude tests have failed in collage. Some who are below the standards set for
admission, but who, for various reasons, were admitted, attained satisfactory standings. This provide that 

A. Human beings are certainly predictable


B. Admission tests are not accurate, hence should not be used
C. Aptitude tests do not measure all factors import for success 
D. Aptitude tests can be perfectly relied on

83. If the teacher’s pattern in questioning consists of calling on a student then asking the question 

A. All students may be encouraged to participate


B. The student called to answer may be able to think well of his answer
C. The rest of the class may just dictate the answer
D. The rest of the class may not engage themselves in thinking of the answer 

84. In order to avoid disgrace, a pregnant, unmarried woman takes drugs to induce abortion. Is she morally justified to do that? 

A. Yes, it can save her and the child from disgrace when he grows up
B. No, the unborn child cannot be made to suffer the consequences of the sins of his parents
C. No, the act of inducing abortion is bad in itself 
D. No, it is better to prevent the child from coming into the world who will suffer very much due to the absence of a father

85. In which way does heredity affect the development of the learner? 

A. By placing limits beyond which the learner cannot develop (maturity and readiness) 
B. By compensating for what environment fails to develop
C. By blocking the influence of environment
D. By providing equal potential to all

86. The cultivation of reflective & meditative skills in teaching is an influence of 

A. Taoism
B. Shintoism
C. Confucianism
D. Zen Buddhism 

87. A child refuse to obey orders or displays negativism as a development trait. How may you best handle him? 

A. Take every opportunity to praise him for every positive attitude display 
B. Detain him after office hours for him to do what he has been ordered to do
C. Insist on compliance to the same degree required of pupils
D. Avoid giving him orders or if you do he objects take back the order

88. Which term refers to the collection of student’s products and accomplishment for a period of evaluation purposes? 

A. Portfolio 
B. Observation report
C. Anecdotal record
D. Diary

89. For comparing and contrasting, which graphic organizer is most appropriate? 

A. Cycle
B. Web
C. Story map
D. Venn diagram 

90. Laging UMUUKILKIL sa isipan ang nasirang pangako ng anak. 

A. Sumasapi
B. Gumugulo
C. Bumubuhay
D. Sumasapi

91. Huwag kang maniniwala sa bulaklak ng kanyang matamis na dila. 

A. Ito’ypanunukso
B. Ito’ypagbibiro
C. Ito’ypambobola
D. Ito’ypagsisinungaling

92. If a resilient child with superior intelligence is reared in a poor environment the probable outcome would be:

A. No change in IQ because environment deprivation has nothing to do w/ intelligence


B. Slight change in IQ although he can overcome frustration and obstacles
C. Mental retardation in IQ because he is culturally deprived

93. Which of the following is usually considered the most important factor in a child’s observable classroom behavior? 

A. Intelligence
B. Heredity
C. Self-concept
D. Cultural background

94. Section 5, article XIV, of the Constitution states that academic freedom shall be enjoyed in 

A. Public assemblies
C. All levels of learning
B. State colleges and universities
D. All institution of higher learning

95. A teacher who subscribes to the pragmatic philosophy of education believes that experience should follow learning in her teaching,
she therefore exerts effort in 

A. Encouraging learners to memorize factual knowledge


B. Equipping learners with the basic abilities and skills
C. Requiring learners full mastery of the lesson
D. Providing learners opportunities to apply theories and principles

96. Freud expounded that there is a period when young boys experience rivalry with their father for their mother’s affection. This is 

A. Oedipus complex
B. Achilles syndrome
C. Electra complex
D. Cassandra syndrome

97. Education is a life long process. This simply means that education

A. May take place formally or informally to enable the individual to grow


B. May take place anywhere and anytime the individual so desires
C. Is a continuous process of experiencing and reorganizing experiences
D. Take place in the school where the individual is exposed, self-contained experiences

98. The tendency to imitate elders is very strong in the early childhood stage. Teachers should therefore be very good

A. Counselors
B. Role models
C. Disciplinarians
D. Facilitators of learning

99. How is Values Education offered in the National Secondary Education Curriculum? 

A. Emphasized in Science and Technology


B. As a separate subject
C. Integrated in all subject areas
D. Integrated with Technology and Home Economics

100. The NSEC orients secondary education to

A. The teaching of the national symbols


B. The development of competencies and values for social living
C. Health values development
D. National development requirements & reflects search based direction

LET Reviewer : MODULE DRILLS 1 - Professional Education part 3

- November 02, 2018

101. The child cannot distinguish abstracts during the sensory motor period of development. Which of these techniques should a teacher
apply to accommodate learning? 

A. Make use of individualize instruction

B. Explain the lesson very well

C. Utilize concrete objects to clarify concept

D. Provide variety of educational toys (sensorimotor-0 to 2)

102. Which of these systems of learning includes ways and methods which are used in preserving and building certain
traditions within cultural communities? 

A. Non-formal learning

C. Cultural learning

B. Multi-level learning

D. Indigenous learning
103. Which of the following statements is TRUE in the use of experiments and demonstrations in teaching science 

A. It is valuable if used in the context of a lesson that relates observation to other information

B. It should be encouraged in elementary school since the concepts they encompass are difficult for your children

C. It is as valuable as teaching by lecturing

D. It is less valuable than teaching through inquiry and discussion

104. Identical twins are more alike than fraternal twins. Which of the following statements/principles supported by this? 

A. Environment affects both fraternal and identical twins

B. Intelligence hinges in physical structure

C. Heredity has a part in determining intelligence

D. Intelligence is determined partly by pre-natal nutrition (b-c-d about intelligence)

105. Which of these philosophers is reflective of that of Dewey’s which stresses the development of an individual capable
of reflective thinking specifically that of being able to solve the problem be faces individually or collectively. 

A. Disciplinarianism

B. Experimentation

C. Developmentalism

D. Rationalism

106. Which of the following abilities is stressed by humanistic education? 

A. Learn the different philosophies of education 

B. Develop man into a thinking individual 

C. Enjoy the great works of man such as the classics 

D. Make man distinctly civilized, educated and refined 

107. An appreciation lesson is one that is designed to lead the class to conduct and enjoy something. Which of the following statements
closely approximate the meaning of the above? 
A. An appreciation lesson should be a lesson in values 

B. Appreciation lessons help pupils weigh and clarify values 

C. One cannot fully appreciate what one does not understand or enjoy 

D. A teacher should plan lessons that will guide children to appreciate what is beautiful 

108. Which of the following is the best time for a teacher to set up routine activities that will contribute to effective classroom
management? 

A. As soon as the students have established 

B. During his homeroom days 

C. Daily at the start of the session 

D. On the very first day of school 

109. In large classes where little of the work can be individualized the most effective and practical ways to individualize instruction to 

A. Devise group activities w/c afford every pupils an opportunity to work at his own 

B. Give the pupils freedom to launch individual projects 

C. Assign homework and check it regularly 

D. Assigned program material for out-of-class hours 

110. Which of these is the MOST important principle that a teacher should follow in initiating a program of positive reinforcement? 

A. Make sure the reward comes immediately after the appropriate behavior 

B. Punish negative behavior and reward positive behavior 

C. Provide regular opportunity for socially acceptable behavior 

D. Consider peer approval and recognition 

111. The trend of focusing attention on the child’s interests, abilities and needs and on the improvement of community living necessitate the
use of the 

A. Discovery approach 

B. Integrative approach 
C. Conceptual technique 

D. Project Method 

112. The best way the teacher can be sure of the appropriateness of an instructional material is to 

A. Consider its technical quality 

B. Consider its availability 

C. Try it out before using it in class 

D. Consider its cost 

113. Task analysis involves the breaking down of a learning task into subtasks or sub-skills. Given a task to retell a story, which of the
following skills is NOT needed? 

A. To disseminate information 

B. To identify topic sentences 

C. To outline a selection 

D. To arranged events in sequence 

114. You are assigned to teach students with varied abilities. You want to teach a more homogeneous grouping. Which type of grouping will
tend to benefit your students? 

A. Mixed ability grouping 

B. With-in class ability grouping 

C. Low ability grouping 

D. High ability grouping 

115. Which of the following examples illustrate the use of questions to focus pupil attention on the key points of the lesson? 

A. Why are machine made goods cheaper than those made by hand? 

B. What is Rizal Park known for? 

C. Have you ever enjoyed watching the clouds on a bright day? 

D. Who came while I was writing on the blackboard? 


116. The new teacher entered a noisy classroom. She shouted immediately at the students desperately trying to get order and discipline.
Since then the teacher has not controlled the class. Which is the most probable cause of the teacher’s failure? 

A. The students’ reaction to the teacher is the consequence of her behavior 

B. Rules are not defined & procedures to sustain order is not into place 

C. The new teacher wants to show the class who is the authority 

D. The class wants to test the ability and patience of the teacher 

117. The educational implementation of research findings relative to the ability of dull learners and bright learners to organize and
generalize is for the teacher 

A. To make the bright learners guide the dull ones in learning to generalize 

B. To make the bright learners to generalize and the dull ones to memorize 

C. To give the dull learners to more concrete experiences to serve as basis for generalizing 

D. To give both the dull and bright learners concrete and abstract experiences to serve as basis for generalizing 

118. Which of the following will you do FIRST to establish good class management? 

A. Discuss the required rules for proper class behavior 

B. Discuss the work plan for the year 

C. Prepare a seat plan 

D. Train the class in the distribution of materials 

119. A student was diagnosed to have a high IQ but is failing in his academic subject. What should the teacher do to help him? 

A. Talk to his parents 

B. Talk the student & find out his problem 

C. Examine his study habits 

D. Refer him to the guidance counselor 

120. When do test, inventories and career information become effective for counseling service? 
A. When the data generated are interpreted on time by professionally competent person 

B. The psychological test result are still valid and reliable 

C. When the records are updated 

D. When the records are kept for ready reference when needed

121. When do test, inventories and career information become effective for counseling service?

A. When the data generated are interpreted on time by professionally competent person

B. The psychological test result are still valid and reliable.

C. When the records are updated.

D. When the records are kept for ready reference.

122. Which of the following is a major advantage in using arithmetic mean?

A. It is more commonly used than other measures

B. It discriminates between the lowest and the highest

C. It is simple to computer

D. It is more than stable than the median

123. In preparing a multiple choice test how many options would be ideal? 

A. Five

B. Three

C. Any

D. Four

124. Learners often find it much easier to fit into a new social situation when given encouragement and support. How can this be done?

A. By giving him room responsibility

C. By giving him special help

B. By assigning peers or Buddies to him


D. By discovering his new interest

125. Who expounded on the need to study the child carefully for individualized instruction?

A. Da Feltre

B. Erasmus

C. Boccacio

D. Ascham

126. Which of the following should a teacher do if she cannot pay the monthly installment of an appliance she got from a department store
in their town?

A. Reject any notice of demand for payment to make the impression that she did not receive

B. Move to another neighborhood to escape payment

C. Inform the manager of the store personally and make a satisfactory arrangement of payment on or before the due date of payment

D. Offer to return the used on the monthly installment she paid

127. Which of the following will you recommended to a senior high school scholar who is impregnated by a fellow student?

A. Tell her parents about her condition

B. Direct her to an abortion clinic

C. Stop schooling all after she gives birth

D. Force her boyfriend to marry her

128. The government prescribes a higher percentage on the administration of educational institution to filipino citizens in order to

A. Minimize the unemployment problem

B. Protect the rights of the citizen

C. Procedure globally competitive graduates

D. Ensure the teaching of Filipino

129. Teachers should bear in mind that the period of greatest mental development is from:
A. 9 t0 12 years

B. 12 to 15 years

C. 6 to 9 years

D. 3 to 6 years

130. Which of the following is the best situation wherein you can balance responsibility and accountability?

A. A teacher paid on an hour basis, takes her time with the subject matter till end of period

B. A teacher paid on an hour basis, teaches as much as she could for duration of the period

C. A teacher paid on an hour basis, spends most of the time on the latest gossips in showbiz

D. A teacher paid on an hour basis, entertain her students w/ stories till the end of the period

131. You have a pupil who is so talkative, naughty and aggressive that he is a burden to the entire members of the class. How would you
remedy this problem?

A. Call the parents for a dialogue

C. Reprimand him always

B. Report the case to the principal

D. Talk to him seriously

132. What should a teacher do before constructing items for a particular test?

A. Prepare the table of specifications

C. Determine the length of time for answering it

B. Review the previous lessons

D. Announce to students the scope of the test

133. Under which of the multiple choice type of test can this question be classified? Which of the following statement expresses this concept
in different forms?

A. Association
B. Definition

C. Difference

D. Cause

134. Of the following types of test which is the most subjective in scoring?

A. Matching type

B. Simple call

C. Multiple choice

D. Essay

135. In which of these research methods can the researcher control certain variable?

A. Experimental

B. Ex post facto

C. Descriptive

D. Historical

136. During the first grading period, a student obtained failing marks in five academic subjects. Which of the following tests would best
explain his performance? 

A. Mental ability 

B. Personally

C. Attitude

D. Aptitude

137. Measuring the work done by a gravitational force is a learning task. At what level of cognitions is it? 

A. Application 

B. Knowledge

C. Evaluation

D. Comprehension
138. Setting up criteria for scoring essay tests is meant to increase their ____. 

A. Objectivity 

B. Reliability

C. Validity

D. Usability

139. Which of the following will you do to an examinee you caught cheating and who offered you a certain sum of money to keep quiet. 

A. Motion him to keep quiet & watch for him after the examination

B. Confiscate his test paper & report him to the examination supervisor

C. Announce to all examinees the name of the cheater

D. Ignore him but let him feel that you saw him (deal with him after the test) 

140. Which of the following is the best situation wherein you can balance rights and authority? 

A. Allow all their only daughter’s suitor to come and go as she pleases

B. Censor all their only daughter’s suitors

C. Choose a life-partner for their only daughter

D. Caution their only daughter’s choice of a boyfriend 

141. In testing which of the following is referred to as cultural bias?

A. Test items are more familiar with some culture

B. Some culture do better on tests than others

C. Test will show who is more cultured

D. Cultured people do better on tests 

142. Which is the most obvious and familiar way of reporting variability?

A. Range between highest and lowest scores


B. Standards deviation 

C. Standard error of the mean

D. Distribution of raw scores

143. The theory of identical elements in learning holds that transfer is facilitated when the ____. 

A. Teacher uses different teaching devices

B. Learner has a mastery of specific responses

C. Development task is easily identifies

D. Experience is similar to the application situation 

144. If this is not met the adolescent tends to be critical and always tries to find fault. This is the need. 

A. For recognition 

B. For adventure

C. To belong

D. For material security

145. The way a child talks, walks of manifest, gestures may have been learned from models he had been exposed. This explains ____.(Social
L. Theory-learning by observation) 

A. Affective

B. Insight

C. Social 

D. Cognitive

146. Audio-visual aids are in classroom teaching to ____? 

A. Help make learning more permanent

B. Help clarify important concept

C. Arouse & sustain student’s interest


D. All of these

147. Which of the following is the MOST important purpose for using achievement test? To measure the ____. 

A. Quality and quantity of previous learning 

B. Educational & vocational aptitude

C. Quality & quantity of previous teaching

D. Capacity for failure learning

148. Which of the different types of test covers a wide variety of objectives? 

A. True-false

B. Multiple choice

C. Matching

D. Essay 

149. In a multiple choice test, keeping the options brief indicates ____. 

A. Inclusion in the item irrelevant clues such as the use in the correct answer

B. Non-inclusion of opinion that mean the same

C. Plausibility & attractiveness of the item 

D. Inclusion in the item any word that must otherwise repeated in each response

150. Which of these criteria is the most important in test constructions?

A. The stem should contain the central problem

B. Items should be congruent with the objectives 

C. A table of specification should be prepared

D. Options should be almost the same length

LET Reviewer : MODULE DRILLS 2 - Professional Education part 1


- November 02, 2018

1. Under which category will a globe as an instructional material fall?

a. Picture
b. Model 
c. Mock up
d. Realia

2. Which developmental stage is sometimes called the pre-school years?

a. Middle Childhood
b. Late Infancy
c. Early Childhood
d. Early Infancy

3. Which is (are) NOT supportive of the development of creative thinking?

a. open classroom atmosphere


b. authoritative teacher
c. divergent questions
d. authoritarian teacher

4. The following are features of the Re-structures Basic Education Curriculum EXCEPT:

a. stronger integration of competencies and values within and across the learning areas
b. greater emphasis on content, less on the learning process
c. increased time for tasks to gain mastery of competencies
d. interdisciplinary modes of teaching

5. Below is a list of methods used to established the reliability of the instrument. Which method is questioned for its reliability due to
practice and familiarity?

a. split half
b. equivalent forms
c. test-retest
d. Kuder Richardson

6. Which one holds true to anecdotal records?

a. combining facts with interpretation


b. ease in writing objective, descriptive behavior
c. describing behavior in natural settings
d. describing behavior in a laboratory setting

7. As a teacher you are a rationalist. Which among these will be your guiding principle?

a. I must teach the child that we can never have real knowledge of anything
b. I must teach the child every knowledge, skill, and value that he needs for a better future
c. I must teach the child to develop his mental powers to the full
d. I must teach the child so he is assured of heaven

8. Which is a characteristics of DEVELOPMENTAL INAPROPRIATE PRACTICE type of education?

a. Contextual learning
b.
c. Experiential learning
d. Integrated learning skills

9. With knowledge explosion, which image of the teacher is unacceptable?

a. questioner 
b. “guide on the side”
c. facilitator of learning
d. omniscient teacher

10. These are also known as “combination classes” organized in barrios/ barangays where the required number of pupils of the same grade
level have not met the required number of pupils of the same grade level has not met the required number to make up a separate class thus
the teacher apportions class time for instruction to every grade level within class. These are:

a. extension classes
b. homogenous classes
c. heterogeneous classes
d. multi-grade classes

11. What is a possible effect of an overcrowded curriculum?

a. Lifelong learning skills tend to be fully developed.


b. Lack of mastery of essential competencies.
c. In depth learning tends to be given greater emphasis.
d. There is greater concept understanding.

12. Roberto who is in Grade IV has the following grade norm equivalents: MATH-4; Reading-8; and Science-5. What can be said of Roberto?
Compared to the average Grade IV pupil, Roberto is

a. behind in Math
b. average in Science
c. behind in Math and average in Science
d. advanced in Science

13. Rights and duties are correlative. This means that

a. right and duties regulate the relationship of men in society


b. each right carries with it one or several corresponding duties
c. rights and duties ultimate come from GOD.
d. rights and duties arise from the natural law.

14. Which of the following activities is the LEAST brain-compatible?

a. Explaining how the CPU can function like the brain


b. Detecting error in the computer program
c. Making a flowchart of a computer process
d. Encoding a paragraph with speed and accuracy

15. Which physical arrangement of chairs contributes to effective classroom management?

a. that which makes it easier to clean the room


b. that which distinguishes teacher from students
c. that which sticks to the traditional chair arrangement in the classroom
d. that which enhances classroom interaction

16. For mastery of learning and in line with the Outcome-Based Evaluation model which element should be present?
a. inclusion of non-performance objectives
b. construction of criterion-referenced tests
c. construction of norm-referenced tests
d. non-provision of independent learning

17. As a naturalist, Jean Jacques Rousseau claimed that everyone is essentially good. This view can help the teacher best when

a. dealing with misbehaving students


b. helping the slow learners in class
c. teaching the students some values
d. knowing the students potentials

18. With specific details in mind, which one (s) has (have) a stronger diagnostic value?

a. Restricted essay test


b. Non-restricted essay test
c. Restricted and non-restricted essay test
d. Multiple choice test

19. Teacher U teaches to his pupils that pleasure is not the highest good. Teacher’s teaching is against what philosophy?

a. empiricism
b. Epicureanism
c. hedonism
d. realism

20. Teacher C teaches in a remote multi-grade class. She is seldom visited by her principal and supervisors. She teaches when she feels like
teaching and not when she does not feel like teaching. What trait of a good teacher does she lack?

a. Emotional intelligence
b. Integrity and accountability
c. Competence
d. Intelligence

21. Which may work against collaborative teaching?

a. Establishment of mutual goals


b. Joint rewards
c. Homogenous grouping
d. “knee-to-knee” seat arrangement by group

22. Which may work about standard deviation is CORRECT?

a. The lower the standard deviation the more spread the scores are.
b. The higher the standard deviation the less spread the scores are.
c. The higher the standard deviation the more spread the scores are.
d. It is a measure of central tendency.

23. Which one of the following is NOT a measure of central tendency?

a. median
b. mean
c. variance
d. mode

24. What type of test is given to determine the admission or non-admission of a student to the program?
a. placement
b. diagnostic
c. aptitude
d. achievement

25. What is an advantage of point system of grading?

a. it does away with establishing clear distinctions among students.


b. It is precise.
c. It is qualitative.
d. It emphasizes learning not objectivity of scoring.

26. Which according to Jocano is a strength of the Filipino people?

a. sense of commitment to nationalistic ideals


b. sense of historical or national identity
c. sense of humor
d. sense of national pride

27. Which is a sound classroom management practice?

a. Apply rules and policies on a case to case basis.


b. Apply reactive approach to discipline.
c. Avoid establishing routines; routines make your students robots.
d. Establish routines for all daily needs and tasks.

28. What is the possible effect of an overcrowded curriculum?

a. in-depth learning tends to be given greater emphasis.


b. There is greater concept understanding.
c. Lifelong learning skills tend to be fully developed.
d. There is lack of personal analysis and reflection on major concepts.

29. Matthew Arnold’s ideal person was “the person whose powers were all in balance”. The following enumerate which have to be
developed for a person to become Arnold’s ideal person EXCEPT.

a. head, heart, hands, health


b. knowledge, skills, attitudes
c. brain, mind, reason
d. cognitive, psychomotor, and affective powers

30. How student learn maybe more important than what they learn. From this principle, which of the following is particularly important?

a. knowing how to solve problem


b. getting the right answer to a world problem
c. determining the given
d. solving the problem within time limit

31. If you want a child to eliminate an undesirable behavior, punish him. This in accordance to Thorndike’s law of ___.

a. Reward
b. Exercise
c. Punishment
d. Effect

32. In which cognitive developmental stage is a child unable to distinguish between own perspective and someone else’s?
a. sensorimotor stage
b. concrete-operational stage
c. formal-operational stage
d. pre-operational stage

33. With which will the existentialist agree? The school is a place where individuals

a. can reflect on ideas.


b. can observe by using their senses to the maximum.
c. can meet to pursue dialogue and discussion about their own lives and choices.
d. listen and accept what teachers say.

34. A person is held responsible for his actions because

a. he is mature.
b. he has a choice.
c. he has instincts.
d. he has reason.

35. Teacher Q does not want Teacher B to be promoted and so writes an anonymous letters against Teacher B accusing her of fabricated lies.
Teacher Q mails this anonymous letter is the School Division Superintendent. What should Teacher Q do if she has to act professionally?

a. Hires a group to distribute poison letters against Teacher B for information dissemination.
b. Submit a signed justification criticism against teacher B, if there is any.
c. Instigate student activities to read poison letters over the microphone.
d. Go straight to the schools Division Superintendent and gives criticism verbally.

36. What psychological principle is invoked when a teacher connects the new lesson to the one just completed so that the student may gain
a holistic view of the subject?

a. stimulation
b. conceptualization
c. recognition
d. apperception

37. Which prescribes the abolition of private property by force?

a. moderate capitalism
b. socialism
c. communism
d. exaggerated capitalism

38. Under which category does the MPS that we talk about in the measurement and evaluation of learning fall?

a. Mode
b. Mean 
c. Median
d. Ratio

39. Which attitude is exemplified by a Boy Scout who says “Bahala Na!” and drives into a pool to save a drowning boy?

a. A daring attitude
b. A “segurista” attitude
c. A daring attitude combined with” pagmalasakit”
d. An “I dont-care attitude’’

40. My lesson is on methods of family planning. I wanted my class to gain a holistic understanding of family planning methods. So I invited a
priest to talk on morality of family planning methods, a doctor on the physiological aspect of family planning methods, an officer of
Population Commission for his experiences as officer, and a couple who practices family planning to talk before my class and to answer
questions raised by the class. What technique did I employ?

a. Forum
b. A panel discussion
c. Debate
d. Symposium

41. “ The State shall protect and promote the right of all citizens to qualify education at all levels.” Which government program is in support
of this?

a. Exclusion of children with special needs from the formal system


b. Free elementary and secondary education
c. Deregulated tuition fee hike
d. Re-introduction of the NEAT and NSAT

42. Which is the Magna Carta for Public School Teachers?

a. RA 7836
b. RA 4670 
c. RA 7722
d. P.D. 1011

43. Which is an indicator of teacher’s effectiveness in instilling discipline among children?

a. Teacher’s presence is needed for discipline.


b. Students have developed concern for one another.
c. Students behave for fear of punishment.
d. Students behave because of a promised reward.

44. Based on Mager’s approach in writing lesson objectives, which lesson objective is written correctly?

a. At the end of the period, 80 of the class is able to compose a seven-to-ten sentence paragraph that observes unity and clarity.
b. At the end of the period, 80 of the class is able to solve the 5 word problems correctly.
c. At the end of the period, the student is able to compose a seven-to-ten sentence paragraph that observes unity and clarity.
d. At the end of the period, 80 of the class is able to solve the 5 words problems with 90 accuracy.

45. Which items do not affect the variability of test scores?

a. Test items that are a bit easy


b. Test items that are moderate in difficulty
c. Test items that are a bit difficulty
d. Test items that every examinee gets correctly

46. The attention to the development of a deep respect and affection for our rich cultural past is an influence ____.

a. Hegel’s
b. Confucius 
c. Dewey’s
d. Teilhard de Chardin’s

47. As reported by the Committee on Information Technology, Science, Mathematics Education and other technologies of the 2000
Presidential Commission on Educational Reform, the elementary curriculum is _____.

a. reclustered
b. integrated 
c. overcrowded
d. innovative

48. Which is a feature of exaggerated capitalism?

a. Abolition of private property


b. Sense of social justice
c. Individualistic concept of wealth
d. Social ownership of some means of production

49. In the perlance of test construction what does TOS mean?

a. Term of Specifications
b. Table of Specifics
c. Table of Specific Test Items
d. Table of Specifications

50. STUDY THIS MATCHING TYPE OF TEST THEN ANSWER THE QUESTION:

                           Column A                                          Column B
                 _____ 1. Equilateral                            a. with 3 equal sides
                 _____ 2. Right                                    b. with 5 equal sides
                 _____ 3. Octagon                               c. has a 90-degree angle
                 _____ 4. Pentagon                             d. means many
                 _____ 5. Heptagon                            e. with 7 sides
                 _____ 6. Poly                                    f. with 8 sides

QUESTION: WHICH DEFECT DOES THE MATHCING TEST ABOVE HAVE?

a. An obvious pattern is followed in the answering.


b. It is an imperfect type of matching test.
c. The items are quite easy.
d. The items are NOT homogenous.

LET Reviewer : MODULE DRILLS 2 - Professional Education part 2

- November 02, 2018

51. An industrial Arts teachers is very strict in the classroom, yet he smokes inside the room whenever he wants to. His pupils even see him
with his :barkada” drinking liquor in public places. If you were the principal, how would you deal with him?

a. Explain to the pupils that smoking is bad for young people but not to adults.
b. Be tolerant about his behavior for it is only a manifestation of “ pakikisama”.
c. Express disapproval in writing the unbecoming behavior of the teacher.
d. Talk with him about the importance of being consistent on actions and in the rules of discipline enforced in the classroom.

52. Teacher B wants to diagnose in which vowel sound (s) her students have difficulty. Which tool is most appropriate?

a. Portfolio assessment
b. Journal entry
c. Performance test
d. Paper-and-pencil test

53. Which one is inimical to nationhood?

a. The new politics movement


b. Political will to institute national reforms
c. A sense of historical identity
d. Chronic clan mentality

54. The index of difficulty of a particular test is .10. What does this mean? My students ___.

a. gained mastery over that item


b. performed very well against expectation
c. found that test item was neither easy nor different
d. were hard up in that item

55. To develop scientific thinking problem-solving skills which activity will be most appropriate?

a. Brainstorming
b. Role playing
c. Buzz groups
d. Inquiry group discussion

56. A catalyst teacher is good at the following skills, EXCEPT;

a. makes effective use of cooperative learning techniques.


b. ask thought-provoking questions.
c. require uniform reports
d. do inquiry teaching

57. Which of the 3-id, ego, and superego, consist of instincts?

a. Ego & superego


b. Ego
c. Id
d. Superego

58. A good classroom manager has “eyes on the back of her head.” This means that the effective teacher ____.

a. has eye contact with his students


b. gives penetrating looks to his students
c. looks at his students from head to toe when he expresses legitimate anger
d. is aware of all actions and activities in the classroom

59. Which method is “hands-on, minds-on” learning?

a. Unit method
b. Demonstration method
c. Project method
d. Integrative method

60. What is the role of the learner in the Re-structured Basic Education Curriculum?

a. passive recipients of knowledge


b. rival of classmates in learning
c. active partner in learning
d. object of pedagogy

61. Study this group of tests which was administered with the following results, then answer the question.

         Subject                      Mean                      SD                     Ronnel’s score


        Math                            56                          10                              43
        Physics                        41                           9                               31
        English                        80                          16                              109

In which subject (s) did Ronnel perform best in relation to the group’s performance?

a. Physics and Math


b. English
c. Math
d. Physics

62. Which is also called a bar graph?

a. Lithograph
b. Hectograph
c. Holograph
d. Histogram

63. Which is a good practice that a teacher uses in correcting student’s errors?

a. Encourage the students to read books and magazines to improve their spelling.
b. Readily correcting the error upon utterance to distract the students flow of though
c. Nothing students errors and dealing with them after the class activities
d. Ignore the mistake, anyway she will commit the same error in the future.

64. With knowledge explosion, which image of teacher is unacceptable?

a. questioner
b. facilitator of learning
c. “guide on the side”
d. almighty teacher

65. I want my students to show historical ages graphically which will be most appropriate?

a. Fishbone diagram
b. Continuous scale
c. Series if events chain
d. Flow chart

66. In Piaget’s concrete operational stage, what is it that a child can NOT do?

a. Doing mentally what was done just physically done


b. Reasoning applied to specific example sets
c. Classifying objects into different sets
d. Imagining the sets necessary to complete an algebraic equation

67.Which is supportive of the development of creative thinking?

a. Judgment atmosphere
b. Brainstorming technique
c. Authoritarian teacher
d. Convergent questions

68. With social development in mind, which is most effective?

a. Computer-assisted instruction
b. Games
c. Cooperative learning
d. Puzzle

69. Which is a characteristics of skill-based instruction for language?

a. It is discovery-based learning
b. It is student-centered
c. It is school learning
d. Students take part in planning lessons.

70. Which question demands the highest level of thinking?

a. How should you present a report in class?


b. What condition must be met for reporting method to be effective?
c. What steps are followed in class reporting?
d. Was the student reporting well done? Support your answer.

71. Which is more of a spatial task?

a. Read a book then write a response.


b. Examine a statistical chart then write a response.
c. Watch a movie then write a response.
d. Go on a field trip then write a response.

72. Which principle is observed by Ausabel’s schema theory?

a. Learners have stock knowledge of things based on background information and experiences.
b. There is no need to provide background information.
c. Children can be taught how to study.
d. Teachers must presume that learners know everything.

73. Which one uses a projector?

a. Model
b. Slides
c. Mock up
d. Realia

74. Perrenialism advocates for the development of the cognitive faculties. Which of the following teacher’s objective taps higher mental
functioning?

a. Label the parts of the butterfly on the paper


b. Sequence the figures of the butterfly’s life cycle.
c. Name the different stages a butterfly goes through.
d. Compare and contrast butterflies from moths.

75. According to Piaget’s theory in which developmental stage, can the child do symbolic thinking and go beyond the connection of sensory
information and physical action?

a. Sensorimotor
b. Concrete operational
c. Formal operational
d. Pre-operational

76. The computer is user-friendly. This means that

a. the computer lets the user win.


b. the computer program has menus.
c. a touch screen is used for input.
d. the computer program is easy to use.

77. I am reminded of my Grade I teacher every time I see Miss Valdez because their mannerisms are alike. Which principle of association
explains this?

a. Contiguity
b. Similarity
c. Frequency
d. Contrast

78. Bullying among kids in school is rampant. What needs to be taught to eradicate it?

a. Full development of talents


b. Athletic skills
c. Respect for the dignity of persons
d. Full development of physical powers

79. Under no circumstances shall a teacher be prejudiced nor discriminatory against any learner, says the Code of Ethics. When is a teacher
prejudiced against any learner?

a. When he refuses a pupil with a slight physical disability in class


b. When he makes a farsighted pupil sit at the back
c. When he makes a nearsighted pupil sit in front
d. When he considerate multiple intelligences in the choice of his teaching strategies.

80. In what developmental stage is growth most rapid?

a. Adolescence
b. Middle childhood
c. Infancy
d. Early childhood

81. A woman kills the man who had raped her by reason of self-defense. Is this right?

a. Yes, rape is very serious act of aggression and is proportionate to killing.


b. Yes, it is done to defend her reputation.
c. Yes, this is the only time the woman can ably defend herself.
d. No, the killing is no longer self-defense because the rape is already over.

82. In which type of grading do teachers evaluate student’s learning not in terms of grade but by evaluating the students in terms of
expected and mastered skills?

a. Point grading system


b. Relative grading
c. Mastery grading
d. Grade contracting

83. If we teach our students to think creatively, what do we encourage them to do?

a. To question the illogical


b. To criticize the unreasonable
c. Think “within the box”
d. Do “outside-the-box” thinking

84. Which learning activity is most appropriate if teacher’s focus is attitudinal change?
a. Exhibit
b. Field trip
c. Game
d. Role playing

85. Where in the three-level teaching strategy does the teacher pose the question: can we say that the root cause of water pollution is our
very own indifference? How can we fight our indifference to the water pollution problem around us?

a. Concept-level
b. Facts and concept level
c. Value-level
d. Concepts and value level

86. Which practices speak of education of the human spirit?

I. Familiarizing the learner with the world’s heritage of art all its forms
II. Encouraging the learner to be practical
III. Studying the biographies of heroes and martyrs
IV. Teaching the learner how to learn

a. I, II
b. I, IV
c. II, III
d. I, III

87. The teacher prepares a minimum of five questions in her daily lesson plans. In what phase of teaching-learning activities does she take
up these questions?

a. Evaluation 
b. Motivation
c. Assignment
d. Recitation

88. To be rationalist which is the highest faculty of man?

a. Emotion
b. Will
c. Senses
d. Reason

89. Teacher F teaches the rest of the class while one group works in a study area on a special task. Students share materials help one
another, and assess each ideas and assignment. What approach does the teacher use?

a. Cooperative learning
b. Integrative learning
c. Adaptive instruction
d. Independent study

90. The concluding part of the three-level approach is the

a. Concepts level
b. Value level 
c. Experiential level
d. Facts level

91. The Venn diagram is most fit for lessons on


a. Comparison
b. Contrast
c. Categorization
d. Analogy

92. “ Man may understand all about the rotation of the earth but he may still miss the radiance of the sunset”, said Dr. Martin Luther King.
Which is an educational implication of this statement? Stress on

a. science education
b. skill education
c. liberal education
d. technical education

93. A son put a time bomb in the luggage of his mother who took it abroad a Philippine airline. The bomb exploded while the airplane was in
flight killing the mother and forty other passengers. Although the movie of the criminal act was never revealed by the son, he aroused
suspicion, when he named himself beneficiary to an insurance policy he had previously taken out on his mother’s life. Was the son
accountable for the death of his mother and other passengers?

a. No. He did not directly the death of his mother and the other passengers.
b. Yes, if he got the insurance money after the death.
c. Yes, he may have been directly interested only in the insurance money but indirectly as a foreseen consequence, he willed the death of all
passengers.
d. No, if he refused to get the insurance money after the incident.

94. My lesson is on methods of family planning. I would like my class to have a holistic and a comprehensive understanding of family
planning methods. Which technique will me most appropriate?

a. Symposium
b. Panel discussion
c. Debate
d. Forum

95. With the linguistically intelligent group in mind, which activity is least effective?

a. Concept maps
b. Debates
c. Manipulatives
d. Deductive reasoning

96. The mode of a score distribution is 25. this means that

a. Twenty-five is the score that occurs least.


b. Twenty-five(25) is the score that occurs most.
c. Twenty-five is the average of the score distribution.
d. There is no score of 25.

97. What does a positively skewed score distribution imply?

a. The students must be highly intelligent.


b. The scores are concentrated on the right side of the normal distribution curve.
c. The scores are evenly distributed.
d. The scores are concentrated on the left side of the normal distribution curve.

98. To have a test with a coverage and with power to test analytical thinking and case scoring?
Which type should the teachers use?
a. Alternate response
b. Short answer
c. Completion
d. Multiple choice

99. Who was a strong supporter of inclusive education and “education for all” concept?

a. Rousseau
b. Kung-fu-tsu
c. Dewey
d. Hegel

100. Which visual display is a result of student’s comprehension of a selection read proven by his ability to organize and integrate concepts
and information gleaned from the selection?

a. Journal
b. Story map 
c. Venn diagram
d. Semantic web

LET Reviewer : MODULE DRILLS 2 - Professional Education part 3

- November 02, 2018

101. After showing the chart on families of different sizes and their corresponding water consumption, Teacher L asked pupils this question:
What can you say about the size of families and their average water consumption? To answer the question which specific skill is demanded
of the pupils?

a. Drawing inferences
b. Recall
c. Evaluation
d. Stating generalization

102. Each teacher is said to be a trustee of the cultural and educational heritage of the nation and is under obligation to transmit to learners
such heritage. Which practice makes him fulfill such obligation?

a. Use of interactive teaching strategies


b. Observing continuing professional education
c. Use of the latest instructional technology
d. Study of the life of Filipino heroes

103. Ms. Cho gives a quiz to her class after teaching a lesson. What does she give?

a. Diagnostic test
b. Summative test
c. Performance test
d. Formative test

104. Teacher K believes that every learner can be helped to achieve his full potential and so functions effectively in society when we satisfy
his needs. On what theory(ies) is Teacher K’s belief based?

a. Cognitive psychology
b. Humanistic psychology 
c. Behaviorist theory
d. S-R theories
105. Which is a characteristics of a child with ADHD?

a. Inability to read
b. Inattention
c. Poor health
d. Inability to spell

106. With the gradations of objectives in the affective domain in mind, under what level does “developing a consistent philosophy of life”
fall?

a. Organization
b. Responding
c. Characteristics
d. Valuing

107. My students are dominantly a kinesthetic group. Which activities should I have more?

a. Ecological field trips


b. Hand-on demonstration
c. Rapping
d. Journal writing

108. A test item with a discriminating index of .10 is ___.

a. neither good or bad


b. poor
c. reasonability good
d. very good

109. Who of the following considers music as the highest form of aesthetic creation because it cuts across the heart of the Absolute?

a. Existentialist
b. Idealist
c. Realist
d. Pragmatist

110. One group of psychologist said: “Wait until the child is ready”. But who said “one can help the child to get ready by developing pre-
requisite skills in an interesting and meaningful way”?

a. Gagne
b. Bruner
c. Ebbinghaus
d. Kohler

111. Which is a characteristics of whole language instruction?

a. It is formal and systematic instruction.


b. It is not supportive of risk-taking.
c. It is natural learning.
d. It is learning from part to whole.

112. The following characterize a child-centered kindergarten EXCEPT

a. academic-oriented
b. importance of play in development
c. emphasis on individual uniqueness
d. focus on the education of the whole child.

113. Concurrent validity requires

a. correlation study 
b. item analysis
c. item difficulty
d. peer consultation

114. What is a possible effect of an overcrowded curriculum?

a. Lifelong learning skills tend to be fully developed.


b. There is greater concept understanding.
c. The tendency is to give greater emphasis on in-depth learning
d. There is lack of mastery of essential competencies.

115. I want my students to master the concept of social justice. Which series of activities will be most effective?

a. Review-pretest-teaching-posttest
b. Pretest-teaching-posttest
c. Pre-test on social justice-teaching=posttest-re-teaching for unlearned concepts-post test
d. Teaching-posttest

116. Instead of describing the steps in opening a document on file in the computer, I ask my students to do graphically. What should be
used?

a. Continuous scale
b. Fishbone diagram
c. Concept pattern organizer
d. Flow chart

117. When is median the appropriate measure of central tendency?

a. Variance is heterogeneous.
b. Distribution is normal; and number of samples is less than 30.
c. Variance is homogeneous.
d. Distribution is skewed; and a number of samples is less than 30.

118. The last year of pre-service education of Education is student teaching.


Student teaching is _____.

a. simulation
b. hand-on 
c. demonstration
d. role-playing

119. I use the gumamela to teach the parts of a complete flower. Later, the flowers will be studied in comparison to gumamela. What
teaching method did I use?

a. Problem-solving method
b. Discovery method
c. Type-study method
d. Laboratory method

120. What functions are associated with the left brain?

a. Verbal, logical, intuitive


b. Verbal, visual, intuitive
c. Verbal, intuitive, detail-oriented
d. Verbal, logical, detail-oriented

121. Fear of the dentist from a painful experience, fear of heights from falling off a high chair when we were infants are learned through

a. insight
b. classical conditioning
c. operant conditioning
d. imitation

122. Which one describes the percentile rank of a given score?

a. The percent of cases of a distribution below and above a given score


b. Classical conditioning
c. Operant conditioning
d. Imitation

123. Which may help an adolescent discover his identity?

a. Decision to follow one path only.


b. Relating to people.
c. Parents pushing in to follow a specific path.
d. Exploring many different roles in a healthy manner.

124. Which process does NOT form part of creative thinking?

a. Curiosity
b. Imagination
c. Originality
d. “Inside the box” thinking

125. “Using the principles of ..as a guide, describe how you would solve the following problem situation” is a sample though question on
_____.

a. summarizing
b. synthesizing
c. applying
d. analyzing

126. Which is (are) supportive of the development of creative thinking?

a. Authoritarian teacher
b. Judgmental atmosphere
c. Open-ended questions
d. Convergent questions

127. The variance, standard deviation, and range are all measures of

a. variability
b. central tendency
c. grouping
d. partition values

128. Which term refers to a teacher helping a colleague grow professionally?

a. Independent study
b. Facilitating
c. Peer mentoring
d. Technology transfer

129. Ramil helps his classmate carry heavy books if their teacher sees him and so praises Ramil. In which level of moral development
according to Kohlberg’s theory?

a. Conventional
b. Non-conventional
c. Pre-conventional
d. Post conventional

130. The teacher gives indigenous achievement test to his 25 students. The test consist of 50 items. He wants to classify his student’s
performance based on the test result.
What is the appropriate measure of central tendency?

a. Any of the above


b. Mode
c. Median
d. Mean

131. Here is a sample TRUE-FALSE test item: ALL WOMEN HAVE A LONGER LIFE-SPAN THAN MEN. What is wrong with the test item?

a. The test item is quoted verbatim from a textbook.


b. The test item contains trivial details.
c. A specific determiner gives cue to the answer.
d. The test item is vague.

132. “The State shall protect and promote the right of all citizens to quality education at all levels.” Which government program is in support
of this?

a. Deregulated tuition fee hike


b. Re-introduction of the NEAT and the NSAT
c. Primary schools for indigenous peoples
d. Exclusion of children with special needs from the formal system.

133. In what way can feminism contribute to nationhood?

a. Supporting the perpetuation of a family dynasty.


b. Educating relatives of those in power that in the spirit of familism they should not do anything that would embarrass their relatives
c. Stressing the idea that “walang kuwentang tao kung ‘yong sariling kamag-anak ay hindi matutulungan”.
d. Choosing candidates on the basis of blood relationships.

134. Which question is in the highest level of Bloom’s taxonomy of objectives in the cognitive domain?

a. What two groups are present in an experimental design?


b. What could have happened if there was no control group?
c. Distinguish between control and experimental group.
d. Judge the validity of the experimental design.

135. Which describes learning that5 students get when they go into community immersion?

a. Symbolic
b. Simulated
c. Experiential
d. Theoretical
136. Teacher R helped his students recall that stalagmites grow on the ground while stalactites grow on the “ceiling” of a cave by associating
“G” in stalagmites with ground and “C” with ceiling. What did teacher R makes use of?

a. Visual aid
b. Audio-visual aid
c. Meaning-maker device
d. Mnemonic device

137. What primary criterion should guide a teacher in the choice of instructional devices?

a. Attractiveness
b. Novelty
c. Cost
d. Appropriateness

138. Makabayan as a subject in the Re-structured Basic Education Curriculum is the “laboratory for life”. What does this mean? It is in this
subject where the learner ____.

a. the biographies of heroes who are makabayan will be taught


b. will be taught the true concept of being pagkamakabayan
c. will be taught the Filipino strengths and weakness
d. will demonstrate practical knowledge and skills that have been gained in the other subjects.

139. Referring to the characteristics of a progressivist curriculum, which does NOT belong to the group?

a. Flexible
b. Child-centered
c. Relevant
d. Subject-matter centered

140. The “self-expressive learner” learns best from ___.

a. lectures
b. activities that enlighten and enchant like myths and, human achievement.
c. Drills
d. Personal expression and personal encounters

141. Which individualized teaching method makes use of workbooks, teaching machines or computers?

a. Inquiry
b. Project method
c. Programmed instruction
d. Discovery method

142. With the re-structured Basic Education curriculum, which teaching-learning feature does not apply?

a. Interactive
b. Teacher monologue
c. Interdisciplinary
d. Creative thinking

143. In the faculty room everyone is talking about Teacher W who is tutoring for a free her own pupil who is vying for honors. What is the
professional thing for the other teachers to do?

a. As a group report her to the principal


b. Leave her alone, she might accuse you of meedling in her personal life.
c. Talk to the parents of the tutee. Tell them what Teacher W. is doing is unprofessional.
d. Correct her and remind her tutoring one’s own pupil for a fee is unethical.

144. For facts to be useful in research, it is best to ___.

a. See if they withstand the test of time.


b. Gather additional facts for comparison
c. Use all original facts
d. Select, classify, and summarize them.

145. Which a type of statistics is (are) meant to draw out implications about the population from which the sample is taken?

a. descriptive and inferential


b. inferential
c. co relational
d. descriptive

146. When Teacher I presents sets of data then asks the students to enter a conclusion, generalization or a pattern of relationship which
method does she use?

a. process approach
b. unit method
c. inductive inquiry method
d. type method

147. Which curriculum (a) was (were) in place before the 2002 Basic Education Curriculum?

a. 2-2 plan curriculum and college preparatory curriculum


b. 2-2 plan curriculum
c. new elementary/secondary education curriculum
d. college preparatory curriculum

148. Which is (are) supportive of the development of creative thinking?

a. Judgment atmosphere
b. Convergent questions
c. Authoritarian teacher
d. Brainstorming technique

149. Teacher L is a typical idealist teacher. Which belief about knowledge is he likely to advocate?

a. Knowledge through trial and error


b. Knowledge through hypothesis testing
c. Knowledge from direct, concrete experience
d. Knowledge through reasoning

150. What is the role of learner in the Re-structures Basic Education Curriculum?

a. Passive recipient of knowledge


b. Object of pedagogy
c. Rivals of classmates in learning
d. Constructor of meaning

LET Reviewer : MODULE DRILLS 2 - Professional Education part 4

- November 04, 2018


151. Which does NOT hold true to collaborative learning?

a. Each student is held accountable for what is to be learned?


b. The members of the group have face-to-face interaction.
c. Student’s social skills are necessary.
d. Success of the group depends on one diligent student.

152. The “mastery learner” learns best from ____.

a. dramas
b. drill
c. debates
d. role-playing

153. In what way can teachers uphold the highest possible standards of quality education?

a. By putting down other professional to lift the status of teaching


b. By working out underserved promotions
c. By continually improving them selves personally and professionally.
d. By wearing expensive clothes to change people’s poor perception of teachers

154. What recent educational development realities with perennialism?

a. Exercising the students higher order thinking skills (HOTS)


b. Having drills and rote activities that develop learning habits
c. Suiting the strategies to the students learning styles.
d. Individualizing the learning experience and activities.

155. Who among the following is genuinely nationalistic? The citizen who

a. works overseas for the educational of his/her children


b. sings the national anthem with all his/her heart
c. salutes the flag as it is raised during generate jobs in the countryside

156. The steps of the goal-oriented instructional model arranged ion order include

a. pre-assessment, specification of objectives, instruction, evaluation


b. specification of objectives, instruction, pre-assessment, evaluation
c. pre-assessment, specification of objectives, motivation, instruction
d. specification of objectives, pre-assessment, instruction, evaluation

157. Nila reads WAS for SAW or D fro P or B. From her reading behavior, one can say that Nila suffers from _____.

a. dysphasia
b. dyslexia
c. dysgraphia
d. dyspraxia

158. What will you do if someone confided to you an anomy which will implicate people close to you?

a. Tell him to go to the authorities and report the matter


b. Encourage him to keep quit so as not to harm your friends.
c. Encourage the one who confined to you to seek the services of a lawyer.
d. Tell him to call a radio broadcaster to air it.
159. I want my students to identify the strengths and weakness of an event. Which one will I direct them to use?

a. Cycle
b. Compare/contrast matrix
c. Plus/delta evaluation
d. Affinity diagrams

160. Students B claims: “ I cannot see perfection but I lone for it so it must be real.” Under which group can he be classified?

a. Realist
b. Pragmatist
c. Idealist
d. Empiricist

161. Which is a characteristic of dyslexic children?

a. It lacks focus
b. Hyperactive
c. Attentio-getter
d. Hard up in writing and spelling

162. A grade II teacher wanted to show the parts of a seed by using a large, wooden seed visual aid with detachable cotyledons and tiny
seed. Under what classification does this wooden structure fall?

a. Replica
b. Model
c. Mock up
d. Realia

163. With individualized teaching strategies as point of reference, which does not belong to the group?

a. Journal writing
b. Independent study
c. Computed-assisted instruction
d. Demonstration method

164. Some of your students don’t seem to like you as their teacher. If you will regard the situation on the level of the ego, what will you
most likely think about?

a. What wrong have done to deserve this?


b. To hell with them
c. What is it about me that they do not like
d. Why should I care they like me or not?

165. Which teaching method is intended primarily for skill and concept mastery by way of practice?

a. Review
b. Drill
c. Project
d. Supervised study

166. I want to diagnose my student’s difficulties in written communication skills. Which one should I use?

a. Short written response


b. Teacher observation
c. Oral reports
d. Performance task
167. The teacher who nurtures the student’s positive psychological nature is likely to

a. trace all students background and coordinate with the guidance center in case the
students have problems.
b. be in class and listen to reasons of justification when students get into conflict.
c. provide some practical learning activities that will develop or foster harmonious relationships in classroom.
d. remind the students to develop a peaceful classroom atmosphere so that everyone will be safe and happy.

168. Which instructional material is closest to direct experience?

a. Film showing
b. Simulation
c. Field trip
d. Dramatization

169. Which of the following is considered a peripheral device?

a. Keyboard
b. CPU
c. Monitor
d. Printer

170. After reading “ A NEW HOME for Ruben”, teacher asked the pupils this questions: “Who was Ruben”? Under what level of
comprehension does this question fall?

a. Application
b. Literal comprehension
c. Interpretation
d. Critical evaluation

171. “Once a teacher, forever a student.” What is an implication of this statement to teachers?

a. A teacher is teacher only in relation to the student.


b. A teacher’s pre-service preparation is wanting of quality.
c. The competent and committed teacher is a race commodity nowadays.
d. A teacher must go through a continuing professional in order to be effective.

172. Which applies when the score distribution is concerned on the left side of the curve?

a. Bell curve
b. Positively skewed
c. Bimodal
d. Negatively skewed

173. Which software allows teachers and students to write, edit, and polish assignments and reports?

a. Database
b. Word processing
c. Spread sheets
d. Graphics

174. Rights can not be unduly trampled upon or suppressed without moral quilt because they are

a. inalienable
b. inviolable
c. perfect
d. adventitious

175. Helping is the development of graduates who are “maka-Diyos” is an influence ____.

a. naturalistic morality
b. situational morality
c. dialectical morality
d. classical Christian morality

176. Which is TRUE end of teachers authority in the classroom?

a. To coerce the child to do what is good


b. To motivate the child to internalize self-discipline
c. To sow fear in every child for orders sake
d. To make the child obey orders

177. The mechanism that is used to get data into the Main Memory of a computer is called a (n)

a. entry device
b. input device
c. temporary device
d. output device

178. Which does NOT go with an integrated teaching strategy?

a. Isolated bits of information


b. Multidisciplinary
c. Interconnected
d. Interdisciplinary

179. Which statement on IQ test is WRONG?

a. Can be advanced by maturational changes


b. Measures fixed potential
c. Measures current performance
d. Can be enriched by environmental experiences.

180. Which curriculum (curricula) was (were) implemented before the re-structed basics education curriculum?

a. NSEC
b. NSEC and NSEC
c. PRODED
d. NSEC

181. Which holds true to norm-reference testing?

a. Comparing individual’s performance to the average performance of a group.


b. Constructing test items in term of instructional objective.
c. Identify an acceptable level of mastery in advance
d. Determining tasks that reflect instructional objectives.

182. What does a conservative Filipino student experience when she migrates to the United States and witness for herself public display of
affection?

a. Colonial mentality
b. Culture shock
c. Acculturation
e. Enculturation

183. As part of their lesson, the music teacher scheduled the young beginners to vocalize for 5 to 10 minutes daily three times per week. In
terms of psychomotor learning, such teacher’s action illustrates.

a. skill perfecting
b. mass practice
c. distributed practice
d. skilled performance

184. Which of the following has as its strength immediate feedback?

a. Process approach
b. Laboratory method
c. Group dynamics
d. Computer-assisted instruction

185. In which competency do my students find more class interaction?

a. 1.0
b. .50
c. .90
d. .10

186. Which questioning practice promotes more class interaction?

a. Asking the question before calling on a student to answer


b. Focusing on convergent functions
c. Asking theoretical questions
d. Focusing on divergent questions

187. About what percent of the cases falls between + 1 and –1 in a normal curve?

a. 64.2%
b. 63.2%
c. 65.5%
d. 68.2%

188. “Describe the reasoning errors in the following paragraph” is a sample thought question on ____.

a. synthesizing
b. applying
c. analyzing
d. summarizing

189. In a research conducted by Herome Kagan, almost one-third of a group of children who had an inhibited temperament at 2 years of age
were not unusually shy or fearful when they were four years old. What does this prove?

a. Development ends after infancy


b. Latter experience do not change the impact of early experiences.
c. Development continues after infancy.
d. Early experiences are the sole determiners in the development or persons.

190. Which is the most authentic proof of nationalism on the part of teachers?

a. Winning the award “Most Outstanding Teacher in the District”


b. Utilizing every minute of the academic time for competent teaching
c. Earning graduate degree for promotion
d. Membership in the national organization for professional Teachers.

191. A sufficient wait time can achieve the following EXCEPT

a. Decreases the amount of guessing or wrong inferences


b. Discourages slow and fast thinking students to respond
c. Increase the number of correct responses.
d. Provides time for teachers to evaluate the answers given

192. Which role (s) does the teacher play when he sets desirable learning activities for the individual learner and takes the responsibility of
matching available resources with the needs of each learner?

a. Manager of the teaching-learning process


b. Scholar and director of learning
c. Abstractor and supervisor of learning
d. Counselor and consultant of learning

193. Which is (are) a more reliable measure (s) of a central tendency in a positively skewed distribution like personal income?

a. Mode
b. Mean
c. Median
d. Mode and median

194. Teachers D banged the door when she entered the classroom while the pupils waited nervously to greet her. She was tense and so were
the pupils. Later, she tried to encourage them to participate but the children remained quit in their seats. Which can best explain the
situation?

a. The teacher’s verbal and non-verbal messages are contradictory.


b. The impact of the non-verbal and verbal messages of the behavior did not change.
c. The pupil’s perceptions of the teacher’s non-verbal behavior did not change.
d. The teacher’s verbal communication has not been interpreted correctly by the pupils.

195. Which item is learned most by my students? In the item with a difficulty index of

a. .50
b. .10
c. .90
d. 1.0

196. In research which is another term for independent variable?

a. Response
b. Outcome
c. Criterion
d. Input

197. The lesson is about the brain. Teacher D likes to share some facts about the brain by the use of a graphic organizer. Which graphic
organizer is most fit?

a. Generalization pattern organizer


b. Episode pattern organizer
c. Descriptive pattern organizer
d. Time sequence pattern organizer

198. Which questioning practice will promote more class interaction?


a. Rejecting wrong answer
b. Asking the questions before calling on a student answer
c. Asking theoretical questions
d. Focusing on convergent questions

199. The claim of a benefactor to the gratitude of his protégé is an example of a (an)

a. alienable right
b. acquired right
c. perfect right
d. imperfect right

200. A student’s dislike for Math is due to a traumatic experience with a Math teacher in the past. When law explains this?

a. Analogy
b. Vividness
c. Disposition or mind set
d. Partial activity

LET Reviewer : MODULE DRILLS 1 - Professional Education part 1

- November 02, 2018

1. The world has become a global village. What sound practice is expected of teachers?

A. Ethnocentrism

B. Xenocentrism

C. Cultural diversity

D. Culture shock

2. Which of the following contributes to the provision of quality education?

A. Construct classroom and laboratory rooms.

B. Hire the best teacher applicants from the Division pool.

C. Provide adequate textbooks and other instructional materials.

D. Maintain sanitary, safe, child-friendly and conducive environment to learning.

3. The following characterize a learning environment that promotes fairness among learners of various cultures, family background and
gender, EXCEPT:
A. Inclusive

B. Gender-sensitive

C. Exclusive

D. Safe and conducive

4. Which of the following BEST reflects a professional teacher’s conviction of the nobility of the teaching profession?

A. Timely feedback

B. Mastery learning

C. Community involvement

D. Lifelong learning

5. Education plays a crucial role in achieving sustainability. This can be done by promoting__________.

A. gender bias

B. bullying

C. environmental protection

D. ethnocenterism

6. What is the best way for a new teacher to blend into your school’s unique culture?

A. Orientation by peers.

B. Memorizing the Code of Conduct.

C. Understanding the schools’ vision, mission, and goals.

D. Involvement in planning activities.

7. Are vocational teachers required of professional license?

A. Yes.

B. No.

C. Only if they teach in the regular school


D. Only if they have less than 5 years of teaching experience.

8. Which of the following scenarios demonstrate a globally aware teacher?

I. He/she studies and tries to understand other nations, cultures and languages.

II. He/she conscientiously studies her own area of expertise.

III. He/she works collaboratively with his diverse learners in a spirit of honesty, mutual respect and

open dialogue.

A. I and II 

B. I and III

C. I and III 

D. All of them

9. Which aspect of work needs to be taught to future employers and employees in the 21st century workplace?

A. Wages and incentives

B. Human rights issues

C. Industry skills and orientation

D. Loyalty for company and work

10. What could be one of the challenges of multicultural education?

A. High social skills of learners

B. High language proficiency

C. Multi-cultural school setting

D. Content integration

11. After being observed, Teacher Belen received a comment from Principal Calugtong that she has a mastery of the content yet cannot
make her students interested and engaged in the discussion. Principal Calugtong asked her to read an article entitled, “Teachers of the 21st
Century.” From her readings, which do you think can help Teacher Belen?
A. Entrepreneurial skills

B. Interdisciplinary skills

C. Cultural skills

D. Information and technology skills

12. Which of the following has greatly influenced the introduction of K-12 reform in basic education?

A. need to allot for more funding

B. need to upgrade facilities of schools

C. need to employ more teachers

D. need to conform international standards

13. Other than para-teachers, who else are entitled to a special permit?

A. A person who has all the qualities of an effective teacher.

B. A person who has gained international recognition.

C. A person who is widely acknowledged expert in his/her specialization.

D. Both B and C.

14. Which of the following is a quality of an engaging learning environment most reflective of Education For All (EFA)?

A. Decentralized governance

B. Student-centered learning activities

C. Innovations to enhance learning for all types of learners

D. Open communications for all-teachers, parents, students

15. Which alternative delivery mode frees children from the confines of the four corners of the classroom as it allows pupils to learn even
while at home or in the community?

A. Multi-grade Program

B. Adopt-A-School Program
C. MISOSA

D. GASTPE

16. Of the following, which is the most powerful source of values like beauty and goodness outside the school system?

A. Government propaganda

B. Nature

C. Commercial advertising

D. Mass media

17. The Department of Education authorizes contributions in schools but not during enrolment. Which is NOT an authorized contribution?

A. Boy/Girl Scouts

B. Philippine National Red Cross

C. School Organization

D. School Publication

18. Why was Science NOT included as a separate subject in Grades 1 and 2 under the Basic Education Curriculum?

A. To decongest the curriculum

B. To make learning more enjoyable

C. Science concepts are embedded in learning areas

D. All of the above

19. “Barrio High School Movement”:__________; “Iloilo Experiment”:__________

A. Pedro Orata; Jose Aguilar 

C. Francisco Benitez; Ramon Magsaysay

B. Jose Aguilar; Pedro Orata 

D. Ramon Magsaysay; Francisco Benitez


20. Martin Luther King’s philosophy is rooted in the concept of non-violence. This adheres with the teachings of_________

A. Buddhism 

C. Taoism

B. Hinduism 

D. Islam

21. A school principal wants to emphasize the pillar “Learning to Do” in his school. Which of the following can help her teachers achieve this
objective?

A. Art of Empathy

B. Art of Dialogue

C. Artistic Skills

D. Occupational Training

22. Among the four pillars, the Delor’s Commission emphasized one. Which of the following is consistent with this important pillar?

A. Technical-vocational skills

B. Analytical skills

C. Interpersonal skills

D. Joint projects

23. A certain public school is occupying an area inside a subdivision, while its students live in the fringes of that subdivision. The well-heeled
residents often complain about the noise, litter and petty crimes allegedly done by the students. As concerned teachers, what should you
do?

A. Engage the students in regular clean-up activities around the streets surrounding the school.

B. Call the students and their parents in weekly conferences to let them know about the grievances of

the residents.

C. Together with the principal, have a dialogue with the Homeowners Association on ways that will mutually benefit the school and the
subdivision .

D. Ask the principal to bring the matter to the local government since it is beyond your school’s
jurisdiction.

24. On the first day of school, you immediately notice a transferee in your class. As days go by, you also notice the student is shy to recite
due to her provincial accent and lacks fluency in Filipino. Which of  the following is the LEAST effective way to ease her discomfort and pave
the way for the class to take her in?

A. Talk to her outside of class everytime you have the chance.

B. Ask her to tell something about herself and the place she came from in front of her classmates.

C. Treat her like one of your regular students, but make it known she can approach you anytime.

D. Ask your students to be friendly and welcome her to the class.

25. In a PTA meeting where contribution for a PTA fund raising project was discussed, a parent with three children enrolled in the school
pleaded that she may not be charged the total P900 (since each child is asked P300) because it was beyond her financial capacity. The
teacher adviser answered, “Marami kasi kayong anak, wala na tayong magagawa dyan!” To maintain your cordial relations with parents,
which could be the BEST answer?

A. “We understand your plight. Let’s find out how we can help. Do you have any proposal?”

B. “This was agreed upon by all of you. This amount did not come from me nor from any one. Unless

everyone in this meeting now changes his/her mind.”

C. “We feel for you. But agreement is agreement unless we repeal what we agreed upon from the

very start.”

D. “It may be quite late to raise the question. This should have been raised in the first meeting.”

26. A school’s academic coordinator has been found to engage in gambling, which has caused him to be absent most of the time. Can his
certificate of registration as a teacher be revoked?

A. No, unless he’s proven guilty.

B. No, because he’s protected by his rights as a teacher.

C. Yes, because habitual gambling is a dishonorable conduct and is against the practice of teaching.

D. Yes, because he’s incompetent.

27. A student asks a question to which you don’t know the answer. What should you do?
A. Ignore the question and entertain another one.

B. Tell them honestly that you don’t know the answer.

C. Give that as an assignment to be answered the following day.

D. Tell them that you have studied it in the past, but that you have forgotten about it now.

28. You are always short of money. The mayor's son is failing in your class. One day, Mrs. Mayor visits you and offers a loan payable when
able, in exchange for a favorable mark for her son. What should you do?

A. Thank the mayor's wife and decline the offer.

B. Sincerely thank the mayor's wife and explain that regretfully you cannot do it because of the Code

of Ethics for Teachers.

C. Accept the offer but fail the mayor's son just the same.

D. Accept the offer and give a passing mark to the mayor's son.

29. Teachers often complain of numerous non-teaching assignments that adversely affect their teaching. Does this mean that teachers must
be preoccupied only with teaching?

A. Yes, because teaching is more than enough full time job.

B. No, because to lead in community activities is the job of elected officials.

C. Yes, if they are given other assignments justice demands that they be properly compensated.

D. No, because every teacher is expected to provide leadership and initiative in activities for the

betterment of communities.

30. How do you characterize values that apply to the whole of humanity as taught by great teachers, such as Buddha, Lao Tzu, Jesus and the
like?

A. Secular values

B. Sectarian values

C. Universal values

D. Values of organized churches

31. Which statement/s describe/s old, time-tested teaching methods?


I. They are teacher-directed.

II. Mastery of subject matter is of utmost importance.

III. Procedures are well-established.

A. II and III 

B. I and III

C. I and II 

D. I, II and III

32. The list of LET competencies upon which your licensure examination is based is a product of five consultative workshops with the
academe in the entire country. What approach to LET competency development was used?

A. Interactive 

B. Rational

C. Objectivist 

D. Collaborative

33. Kurt loves to design and build things. At the age of 10, he has already designed and built his own tree house. He has designed and built
elaborate buildings as projects for book reports. He has built a raft that actually floated down the local creek. What would Howard Gardner
say about Kurt?

A. Kurt is high in visual spatial and logical mathematical skills.

B. Kurt is high in naturalist skills.

C. Kurt is high in verbal skills.

D. Kurt is unlikely to be successful in school.

34. If you learn best through listening to lectures, discussions, and to what others say, what type of learner are you?

A. Auditory 

B. Tactile

C. Visual 
D. Kinesthetic

35. Which is a classroom application of Vgotsky’s idea of scaffolding?

A. Give the learner the necessary assistance until she can be on her own.

B. From the start leave the learner to herself because she has the power for self-learning.

C. Don’t spoil the learner by doing what she ought to do.

D. Give the learner a task that challenges her ability.

36. Which is TRUE of the integrative approach to lesson planning?

I. Multidisciplinary

II. Teacher-centered

III. Highly structured lessons

A. II and III 

B. I only

C. II only 

D. III only

37. The framework for creative thinking includes the production of a great number of ideas or alternative solutions to a problem. Secondly,
the ideas produced must show a variety of possibilities and different points of views. Together they are considered effective in developing
creativity among students. What does the framework include?

A. Problem solving and enhancing

B. Variety and strategy

C. Different approaches and strategies

D. Fluency and flexibility

38. OBE stands for outcomes-based education. It is a student-centered approach that focuses

on intended learning outcomes resulting from instruction. All except one of the following may be considered as outcomes or end targets of
an OBE. Which one?
A. Knowledge gained from attending classes

B. Products which reflect a learner’s competence in using content.

C. Demonstrations of actual learning

D. Applications of lessons learned

39. In direct instruction, the role of the teacher is more of a _______.

A. Proctor 

C. Lecturer

B. Parent 

D. Guide

40. Ms. Amores wanted to inculcate the love for learning Mathematics among her students. Which technique should she use?

A. Enable students to have meaningful learning experiences to apply their Math lessons.

B. Provide students with easy exercises that they can readily answer.

C. Assign fast learners to teach slow learners.

D. Give slow learners homework to do with their Math class buddy.

41. Which is one justification of the use of the lecture method?

A. When subject is very new and there are no references yet

B. When students are poor motivated

C. When teacher is substituting for the regular teacher

D. When subject is very broad in scope

42. For empathy and for value change and development, which is best?

A. Simulation 

B. Role playing
C. Game 

D. Drills

43. Ms. Toledo wants to develop the social skills of her students as they learn different phases of matter.How can she accomplish this?

A. Group students with equal abilities together and let them perform an experiment on phases of

matter.

B. Group students with different abilities together and let them perform an experiment on phases of matter.

C. Ask them to group pictures of solids, liquids, and gases.

D. Show them how to distinguish solids from liquids and gases.

44. The second year students were discussing climate change. Each group was expected to come up with a solution that would protect the
environment. Which is the best method of delivery to attain tangible results?

A. Demonstration method 

B. Problem method

C. Project method 

D. Laboratory method

45. For values education, which technique is most appropriate?

A. Conflict story 

B. Brainstorming

C. Simulation 

D. Panel discussions

46. Ausubel recommends the use of visuals designed to bridge the gap between what the learners  already know and what they need to
know. Which visual is referred to?

A. Pictures 

B. Comparison patterns

C. Concept maps 
D. Overviews

47. Students learn best when they discover things by themselves. What materials will be most effective in conducting experiments?

A. Posters and charts 

B. Models and real objects

C. Film and slides 

D. Manuals and illustrations

48. To keep students focused in the discussion, what must be done especially if students are unruly and noisy?

A. Employ innovative strategies.

B. Explore various means of giving sanctions.

C. Ignore the students who are misbehaving.

D. Refer the unruly and noisy pupils to the Guidance Office.

49. Which objective in the psychomotor domain is in the highest level?

A. To play the piano skillfully

B. To endure fatigue in the entire duration of a game

C. To discriminate shapes by touching

D. To move hands and legs in a coordinated fashion

50. Why is there a need to write lesson plans?

A. To comply with the requirements set by the school.

B. To guide the teacher in the learning activities

C. To get a good rating in the performance of appraisal

D. To be able to show something during classroom observations

LET Reviewer : MODULE DRILLS 1 - Professional Education part 3


- November 02, 2018

101. Mrs. Gorospe wants to apply gestalt principles in the classroom. Which of the following learning activities did she avoid?

A. She relates a new topic with something the student already knows.

B. She teaches topics with commonalities next to each other.

C. She uses bolder fonts for important words in the paragraph.

D. She rewards desirable behaviors.

102. What is an application of Vygotsky’s idea of scaffolding?

A. Give the learner a task that challenges his/her ability.

B. From the start leave the learner to himself/herself because she has the power for self-learning.

C. Don’t spoil the learner by doing what he/she ought to do.

D. Give the learner the necessary assistance until he/she can be on his/her own.

103. A girl lost her toy and asked her father for help. The father asks her where she last saw the toy; the child says “I can’t remember.” He then
asks a series of questions - “did you have it in your room? Outside? Next door?” To each question, the child answers, “no.” When he says “in
the car?” 

she says “I think so” and goes to retrieve the toy. Which of the following statements can be deduced from the above situation?

A. Remembering and problem solving are co-constructed.

B. A child functions independently to solve the problem.

C. Modeling can aid learning and development.

D. Both A and B

104. When parents are overly protective, the child becomes overly trusting who cannot believe anyone would mean them harm. According
to Erikson’s theory, this is called____________.

A. maladaptation 

C. fixation

B. malignancy 
D. virtue

105. A group of values education teachers agree that they will focus on value internalization when teaching. Which level of morality should
they help their students attain?

A. Post-conventional morality

B. Pre-conventional morality

C. Conventional morality

D. Between conventional and post-conventional morality.

106. Your brother confessed to you that he killed his girlfriend six months ago due to an alleged third  party. Although you were convinced
that it was a “crime of passion”, you planned to report the crime and/or personally bring your brother to the police. According to Kohlberg’s
theory of moral development, you are under what stage of moral development ?

A. Instrumental relativist orientation

B. Law and order orientation

C. Interpersonal Concordance orientation

D. Universal ethical principles orientation

107. Which educational issue can be clarified by understanding Maslow’s Needs Theory?

A. Sex education issues in school

B. Delinquency in the public schools

C. The effects of different classroom structures

D. The effect of poverty on academic achievement

108. Which of the following demonstrates cephalocaudal trend of development?

A. An infant first produces an endogenous smile, then an exogenous smile, then a laugh.

B. An infant coos, then babbles, then speaks single words, then uses language.

C. An infant obtains visual skills, then olfactory skills, then auditory skills.

D. An infant first raises his or her head, then sits up, then stands up.
109. What is mainly addressed by early intervention program for children with disabilities, ages 0 to 3 years old?

A. Preventing labeling of disabled children

B. Identifying strengths and weaknesses in special children

C. Ensuring inclusion for special children

D. Early growth development lag

110. It is a language-based disability in which a child has trouble understanding words, sentences, or paragraphs.

A. Dyslexia

B. Dyscalculia

C. Dysgraphia

D. Auditory and Visual Processing Disabilities

111. Who is best equipped to lead in the formulation of an individual Educational Plan for Special Children?

A. Child therapists

B. Special education teachers

C. Parents of special children

D. Child psychologists

112. Millet has difficulty working independently. When asked to sit in her seat for long periods of time, she frequently gets up to sharpen her
pencil, look out the window, or flips through books on the bookshelf. She rarely completes her homework. Millet most likely has which of
the following conditions?

A. Mental retardation 

B. Down's syndrome

C. Attention deficit/hyperactivity disorder 

D. Dyslexia

113. Ms. Lumasac is concerned about one of her students. The student, Conrad, has serious and
persistent problems concerning his relationships with other students; he displays aggression and is often out of control. Conrad is most likely
to be suffering from:

A. Depression

B. Attention deficit/hyperactivity disorder

C. Emotional and behavioural disorders

D. Sensory disorders

114. Which of the following is authentic assessment?

A. Make students recite the multiplication table to check for mastery.

B. Make students dance cha cha to determine if they learned how to dance cha cha.

C. Make students draft a flow chart to demonstrate a process.

D. Make students check their own answers after a Math quiz.

115. How are the results of a summative assessment utilized?

A. To check attainment of lesson objectives

B. To diagnose learning difficulties

C. To check on student understanding from time to time.

D. To report learning outcomes to parents and administrators.

116. Teacher Jenny wants to monitor student’s performance on the topic they are discussing. Which type of test should she use?

A. Placement Test 

C. Diagnostic Test

B. Formative Test 

D. Summative Test

117. Interpreting assessment results considers consistency. Which is described when the results are consistent?
A. Validity 

B. Subjectivity

C. Reliability 

D. Objectivity

118. What type of validity is used to determine if the assessment measure can predict future

performance?

A. Content 

B. Construct

C. Criterion 

D. Consequential

119. Which method should be done to make sure that the test items within the test measure the same thing?

A. Inter-rater Method 

B. Equivalent-forms Method

C. Test-retest Method 

D. Internal Consistency Method

120. Thea was careless in marking her answer in the test paper. Which error did she commit?

A. Content Sampling Error 

B. Systematic Error

C. Time Sampling Error 

D. Random Error

121. The best way to assess learning is to use real-life situations, objects and materials existing in the environment. Hence, teachers are
encouraged to use ___________.

A. rating scale 

C. observation technique

B. pencil- and-paper test 

D. authentic assessment

122. Which of the following are alternative assessments?

A. Portfolio, exhibits, journals

B. Paper-and-pencil test, demonstration, reports


C. Student self-assessment, authentic assessment, surveys

D. Multiple-choice, structured observations, sentence completion

123. Mr. Diaz asked his students to explain the difference between norm-referenced and criterion referenced tests. In which level is the
student’s task?

A. Analyzing 

B. Synthesizing

C. Evaluating 

D. Remembering

124. Which of the following does not belong to the group?

A. True-False Item B. Multiple-choice TypeC. Interpretive Exercise D.Extended-response Item

125. Teacher Neri’s lesson objective is “to perform operations on integers”. Which is a valid test for this particular objective?

A. Solve the following using PEMDAS.B. What are the four basic operations?C. How do you add numbers with different signs?

D. What is the product when a negative number is multiplied to another negative number?

126. What is defective in this test item?

Which type of assessment is concerned with the entry performance of


I. Checklist

III Questionnaires

II Interview 

IV Placement Test

A. The stem of the item is poor.B. The distracters are not plausible.C. The item provides clue to the answer.

D. The alternatives are not equal in length.

127. To determine if his students have attained the intended learning outcomes, Teacher Jay is planning to give a year-end exam that will
cover all the topics they have discussed. Which type of test should he use?

A. Essay Test B. Formative TestC. Objective Test D. Problem Solving Test

128. One advantage of essay type over multiple-choice type is that they:

A. measure from simple to complex learning outcomes.B. allow students to express their own ideas.

C. require less time in preparing the test.D. make learning more meaningful.

129. Which statement is/are TRUE of the given rubric below?


Scoring Rubric

5 - Demonstrate complete understanding of the problem.

     All requirements of task are included in response.

4 -  Demonstrate considerable understanding of the problem.

      All requirements of tasks are included.

3 - Demonstrate partial understanding of the problem.

      Most requirements of task are included.

2 - Demonstrate little understanding of the problem.

     Many requirements of task are missing.

1 -  Demonstrate no understanding of the problem.

0-  No response/ task not attempted.

A. It is holistic 

B. It is developmental.

C. It is analytical. 

D. It is both holistic and developmental.

130. The discrimination index of a test item is –0.35. What does this mean?

A. More from the upper group got the item correctly.

B. More from the lower group got the item correctly.

C. The test is quite reliable.

D. The test item is valid.

131. A test item has a difficulty index of 0.89 and a discrimination index of 0.44.

What should the teacher do?


A. Reject the item. B. Revise the item.C. Retain the item. D. Make it a bonus item.

132. If no student got the item correctly, what should the teacher do with the item?

A. Ask the students why the item is still difficult for them.

B. Determine if the item is a miskeyed item or ambiguous.

C. Administer a diagnostic test to determine student’s difficulty.

D. Determine why the strengths and weaknesses of the students.

For numbers 133 to 134, refer to the item analysis below.

                      Item No. 5. The correct answer is C.

                         Group                       A              B              C*                 D

                      Upper Group               9               1               7                    8 

              

                      Total                            13             8              13                   16

133. Which was the MOST effective distracter?

A. Option A B. Option BC. Option C D. Option D

134. Which was the LEAST effective distracter?

A. Option A B. Option BC. Option C D. Option D

135. Students’ scores were as follows: 82, 82, 83, 84, 86, 88, 84, 83, 85.

The score 84 is the _______________.

A. Mode B. medianC. Mean  D. average

            For numbers 136 to 137, refer to the score distribution below.

               98, 93, 93, 93, 90, 88, 87, 85, 85, 85, 70, 51, 34, 34, 34, 20, 18, 15, 12, 9, 8, 6 3, 1

136. What is the range of this score distribution?

A. 34  B. 93 C. 85  D. 97

137. Which measure of central tendency is most reliable for this score distribution?
A. Mode  B. Median C. Mean  D. None

138. Lilia’s score in a test has a t-score of 70. To which of the following groups does she belong?

A. Average  B. Above Average c. Below average  D. Needs Improvement

139. Which is true about z-score?

A. It is a positive numerical value. B. It is affected by the variations in scores.

C. It shows the difference between the test scores. D. It is used to determine if more student got low marks.

140. Which of the following divides a score distribution in half?

A. Range B. Median C. Deciles D. Percentiles

141. Which of the following indicates a relationship between two sets of data?

A. Z-score B. Kurtosis C. Variance D. Correlation

142. How will you interpret a student’s 80% percentile score? The student scored_______________.

A. Higher than 80% of the members of the group B. Better, relative to the competencies targeted

C. High in all the skills being tested D. 80% of the specified content

143. What can be said of student performance in a positively skewed score distribution?

A. Almost all students had average performance. B. A few students performed excellently

C. Most students performed poorly D. Most students performed well

144. Which is one measure of attitude that consists of a value statement where you are asked to express your degree of agreement or
disagreement of a statement?

A. Checklist B. Likert scale C. Interview schedule D. Semantic differential

145. Which test will the Guidance Office of school give if it wants to help students in predicting their probable success in specific educational
and vocational fields?

A. Diagnostic test B. Achievement test C. Aptitude Test D. IQ Test

146. With assessment of affective learning in mind, which does NOT belong to the group?

A. Diary entry B. Cloze Test C. Moral dilemma D. Reflective writing

147. Which of these can measure awareness of values?

A. Sociogram B. Rating scales C. Moral dilemma D. Projective Techniques

148. Obtaining the ranking of students is of major concern when using _________________.

A. Percentile Test B. Summative Test C. Achievement Test D. Norm-referenced Test

149. Which of the following grading schemes provide the least information to parents about the

performance of their child in class?


A. Letter grade

B. Pass-fail grade

C. Numerical grade

D. Supplemental system

150. Which error do teachers commit when they consider the character of the students?

A. Halo effect

B. Severity error

C. Personal biases

D. Reliability decay

LET Reviewer : MODULE DRILLS 1 - Professional Education part 2

- November 25, 2018

DOWNLOAD
Click here to Download

51. Teacher Kim, a Physical Education teacher wrote this objective in her lesson plan “to execute the four fundamental dance steps.” When
observed by the school principal, she was showing her class the basic dance steps. Why did the teacher use a demonstration method to
implement her objective?

A. It is a chance to show teacher’s expertise.


B. It is easier to imitate a teacher who shows the steps.
C. No student knows how execute the steps.
D. Class time is limited to ask student to execute.

52. Which is a classroom application of this principle: “Students learn more effectively when they elaborate on new information.” Ask your
students to _________.

A. write the principle five times


B. commit the principle to memory
C. identify an application of the principle
D. print the principle in big letters then put it in a place where you can read it time and again

53. Mrs. Santos wants to determine immediately the learning difficulties of her students. Which of the following do you expect her to
undertake?

A. Require her students to prepare a portfolio.


B. Administer an achievement test.
C. Administer a diagnostic test.
D. Interview her students.

54. Which statement(s) is/are NOT true about teaching, learning and assessment?
I. Curriculum and assessment are a little but related.
II. Assessment is separate from curriculum and instruction.
III. Assessment is a basis for planning instruction.
IV. The process of learning is as important as the content of the curriculum

A. I only
B. I and II
C. II only
D. II and III

55. Which of the following is something that should almost always be discussed with students when they are given an assignment that may
be new to them?

A. What the student can expect to learn from the given assignment.
B. Whether the student will be tested on the material covered by the assignment.
C. Whether the assignment will be graded according to the same criteria as other assignments.
D. What kind of prior experience the teacher has had with the same type of assignment.

56 Why are computers increasingly becoming pervasive in schools nowadays?

A. Anybody can operate computers without formal training.


B. Computers are the most effective educational technology.
C. Computers increase efficiency and productivity.
D. Students have access to computers in school and at home.

57. How can a teacher make effective use of technology to improve her lessons?

A. Allow students to bring their personal tablets in school.


B. Use internet resources to research on additional inputs to improve her lessons.
C. Transfer lessons from textbooks to PowerPoint presentation.
D. Allow students to use Facebook to comment on one another’s projects.

58. When using questions in the classroom, all of the following strategies are recommended, EXCEPT which one?

A. Asking leading questions that suggest the desired answer


B. Avoiding questions that can be answered with yes or no
C. Using fact-based questions to lead into thinking-based questions
D. Allowing plenty of time for students to think about answers

59. How can a teacher help students with different learning and thinking styles continue to learn more effectively?

A. Allow sufficient time for processing different types of information


B. Provide a general overview of the lesson
C. Use a variety of reflection strategies
D. Use questions of all types to stimulate various levels of thinking and valuing.

60. Which of the following is best for a teacher to do when establishing classroom rules?

A. Mention the rules once at the beginning of the school year.


B. State the rules in a forceful way to establish authority.
C. Explain why the established rules are necessary for student cooperation.
D. Create as many rules and procedures as possible to guarantee order and control.

61. All except one of the following techniques may be used for effective classroom
management. Which one?
A. Setting limits on accepted and nonaccepted action in the classroom
B. Assigning extra work to students for misbehaving in class
C. Monitoring what is happening in the classroom
D. Removing a positive stimulus for inappropriate behavior

62. A sound advice for classroom managers is “Nip the problem in the bud.” What does this mean?

A. Assume a reactive posture in classroom management.


B. Be proactive in management approach.
C. Reinforce positive behavior.
D. Treat minor disturbance calmly

63. Movement management is all but one of the following. Which one?

A. Pace of the lesson is modulated to keep student's attention.


B. Momentum of a lesson is consistent, monitored, and adjusted to prevent slow-downs and
stoppages.
C. Teacher answers the same question multiple times.
D. Happenings and disruptions are handled smoothly.

64. A teacher’s nonverbal coping skills in the classroom, like planned ignoring, signal interference, proximity control, and touch interference
have the following advantages except one. Which one?

A. These produce the least disturbance in the classroom.


B. Other students can help the teacher discipline the problem student.
C. Off-task behavior is managed while the teaching-learning processes continues.
D. The likelihood of hostile confrontation is minimized because there is no public intervention.

65. What mistaken goal does a student aim for when he/she expresses defiance by teasing, tmpertanrums, and low-level hostile behavior to
get the teacher to argue or fight with him/her?

A. Attention getting
B. Revenge seeking
C. Power seeking
D. Withdrawal

66. When Teacher Joanne manages time in the classroom, she does all except one of the following. Which one?

A. Establish class routines.


B. Maintain lesson momentum and avoid interruptions.
C. Set many class rules with students to prevent misbehavior.
D. Use the “overlapping” technique

67. When does praise become INEFFECTIVE? When it _____________.

A. is given after the process is completed.


B. specifies the particulars of the accomplishment
C. attributes the student’s success to ability alone, or to factors such as luck or easy task.
D. makes the student appreciate his own task-related behavior and think about problem-solving.

68. When curriculum content is fairly distributed in each area of discipline this means that the curriculum is __________.

A. sequenced
B. integrated
C. balanced
D. continue
69. Which pair of the components of a lesson plan does NOT parallel curriculum components?

A. Behavioral objectives and Evaluation


B. Teaching Methods and Instruction
C. Subject matter and Content
D. Assessment and Evaluation

70. The K to 12 curriculum hinges on the following ideals except one. Which one?

A. Curriculum is contextualized and global.


B. Curriculum is subject matter-centered.
C. Curriculum is inquiry-based, reflective, and collaborative.
D. Curriculum aligns with the framework of the MTB-MLE.

71. Central to the design approach in UBD is the need to ______ that evoke in-depth understanding.

A. focus on alternative methods of assessing


B. use alternative methods of teaching
C. go against traditional testing.
D. design lessons and assessment.

72. What is referred to as Bruner’s structure for teaching that introduces the fundamental structure of all subjects in the school years, and
then revisits the subjects in more and more complex forms over time?

A. Hidden curriculum
B. Sustaining expectations
C. Assessment
D. Spiral curriculum

73. The Mother Tongue-Based Multilingual Education (MTB-MLE) of the K + 12 curriculum is a ____ method since it starts from the known to
unknown.

A. inductive
B. deductive
C. inductive first then deductive
D. deductive first then inductive

74. Why are there more illustrations in elementary books than in the upper levels?

A. Elementary pupils are slow in the reading process.


B. High school students are more mature and imaginative.
C. Elementary pupils are less imaginative yet, they need more exposure.
D. There is no need to motivate the elementary pupils all the time.

75. A big story breaks in your local newspaper. You want to use the headlines as an inquiry device. You might begin by ______________.

A. explaining what you believe to be the underlying causes


B. asking the class to infer connotations and denotations from the headline
C. describing the background of the story as you know it
D. showing the headline and then asking for a volunteer to read the story to the class and
interpret the meaning

76. Which is a planning tool that helps the reader analyze the story so that the questions during
discussion will create a coherent framework for understanding text?

A. Story map
B. Semantic web
C. Graphic organizer
D. Tree outline

77. Which is an effective technique in teaching children in the emergent literacy stage?

A. Imposing the use of correct spelling when writing


B. Giving of isolated word drills and memorization of rules
C. Providing activities that let then enjoy
D. Having a print-rich classroom

78. If you use the Pygmalion effect to explain why children coming from depressed areas cannot read, to which would you attribute the poor
reading performance of this particular group of children?

A. Poor expectations from depressed areas


B. Lack of motivation
C. Ineffective teaching methods
D. Lack of reading materials

79. Multiple intelligences can be used to explain children’s reading performance. Which group tends to be good readers?

A. Linguistically intelligent group


B. Existentially intelligent group
C. Spatially intelligent group
D. Kinesthetically intelligent group

80. With which of the following is poor reading performance associated?

I. Use of rote memorization as reading strategy


II. Relatively low self-esteem
III. Deprived home environment

A. I, II and III
B. I and II
C. II and III
D. II

81. How can teachers encourage students to become readers?

A. By giving them plenty of reading materials


B. By exposing them to expensive and varied reading materials
C. By giving them reading materials within their level of interest
D. By giving them reading materials to supplement their knowledge in different subject areas

82. The benefit of reading aloud is that children learn___________.

A. New vocabulary in meaningful contexts


B. To value the presence of their friends as they read together
C. To make predictions by examining pictures and listening for clues
D. To use their imaginations to explore new ideas as they listen to stories being read.

83. What is the BEST move that teachers can do to encourage students to become good readers?

A. Give them sufficient reading materials suitable to their interest and level of development.
B. Require them to read all the books about “How to Develop Your Reading Skills”.
C. Give them reading materials to supplement their knowledge of various subjects.
D. Ask them to buy the latest books released by popular authors.
84. John Dewey said, “An ounce of experience is better than a ton of theory.” To which does this
statement point?

A. The need theory


B. The need of experience
C. The primacy of experience
D. The primacy of theory

85. Learning is influenced by social interaction and interpersonal relations. What must a teacher do?

A. Make students work collaboratively


B. Motivate student to reflect on how they learn.
C. Make students feel good about themselves
D. Give more independent study

86. Schools should look into every child's environmental systems in order to understand more about the characteristics of each child, each
learner. What are the important responsibilities of teachers and schools?

I. Welcome and nurture families.


II. Value work done on behalf of children at all levels.
III. Replace relationships in the home.

A. I and III
B. II only
C. III only
D. I and II

87. Which VIOLATES this brain-based principle of teaching-learning: “Each child’s brain is unique and vastly different from one another.”

A. Employing MI teaching approaches.


B. Giving ample opportunity for a pupil to explore even if the class creates “noise”
C. Making a left-handed pupil writes with his right hand as this is better
D. Allowing open dialogue among students

88. Social exposure to various cultures expands a child’s pool of knowledge. Which statements go/es with this sentence?

I. The less experiences a child has, the more disciplined he/she becomes.
II. The more experiences a child has, the richer his/her world becomes.
III. The more selective parents in the exposure of their child, the more challenged the child becomes.

A. I only
B. I and II
C. II only
D. III only

89. Pilar processes information linearly. Her style is a specialty of which part of the brain:

A. Global
B. Both A and B
C. Analytic
D. None of the above

90. Which of the following characterize/s a constructivist approach?

I. Teacher makes meaning


II. Less teacher talk
III. With teachers facilitating, students create knowledge

A. I and II
B. II and III
C. I and III
D. I, II, and III

91. Applied behavior analysis is important in managing serious behavior problems. Which of the following is an example?

A. Giving constant verbal reminders.


B. Praising students who are behaving.
C. Allowing students to be busy on meaningful tasks.
D. Selecting punisher criteria for punishment and reinforcer criteria for reinforcement.

92. The teacher notices that Student A cannot solve a problem on his own, but can when he is given calculated hints. This is an example of

A. Scaffolding
B. The zone of proximal development
C. Pre-operational thinking
D. Lateralization

93. Teacher Arielle asks his students to see the connection of their new lesson to their own personal experiences and share the same with
the class. Teacher Arielle believes in which principle of learning?

A. Learning is an active process.


B. Learning is the discovery of the personal meaning of ideas.
C. Learning begins with setting clear expectations and learning outcomes.
D. Learning is a cooperative and a collaborative process.

94. Learning is meaningful when it is connected to students’ everyday life. What is an application of this principle in learning?

A. Teacher asks the class to show application of what they learned to their daily life.
B. Teacher makes everyday life the starter of his/her lesson.
C. Teacher narrates true-to-life stories.
D. Teacher asks students to come up with clippings of Filipino life.

95. Teacher Myleen talked all period. She taught the class the steps in conducting a descriptive research. She also showed a poorly done
research, discussed why it is poorly done and finally showed a model of a descriptive research. Was teacher Mylene’s approach learner-
centered?

A. Yes, she taught for all the learners.


B. Yes, she made things easy for the learners.
C. No, she was more subject matter-centered and teacher centered.
D. No, because her subject matter was highly technical.

96. Learning is an active process. Which one is an application of this principle?

A. Let students learn the steps in opening a computer by making them follow the steps.
B. Group students for work on project, that way project becomes less expensive.
C. Teach your content from a multidisciplinary perspective.
D. Avoid drills which are out of context.

97. Visual imagery helps people store information in their memory more effectively. Which is one teaching implication of this principle?

A. Tell them to read more illustrated comics.


B. You will not object when your students daydream in class.
C. Instruct students to take notes while you lecture.
D. Encourage your students to imagine the characters and situations when reading a story.

98. You will understand when a pre-school boy asserts that two rows of five coins similarly spaced have equal amounts; but when one row is
spread out so that it is longer than the other, he says it has more coins. Based on Piaget’s theory, which ability does he lack?

A. Multiple classification
B. Reversibility
C. Perspective taking
D. Conservation

99. A child asked why his cousins call his father “Uncle” instead of “Daddy”. Which characteristic was shown by the child?

A. Egocentrism
B. Conservation
C. Lack of Object Permanence
D. Centration

100. A mother told his son that “ANIMAL” and “AMINAL” are not the same. In Piaget’s term, this process is called _______.

A. Assimilation
B. Accommodation
C. Decentration
D. Centration

LET Reviewer : Professional Education - SOCIAL DIMENSIONS Questionnaire Part 1

- October 22, 2018

DOWNLOAD
Click here to Download

1. The Philippine Elementary School Curriculum gives greater emphasis on the development of basic skills like reading, writing, and
arithmetic. What is the philosophical basis for this?

A. Pragmatism 

B. Perennialism 

C. Essentialism

D. Existentialism
RATIONALIZATION: C. Essentialism. Essentialism - is an educational theory rooted in classical realism and idealism which advocates curricular
reform stressing the essentials of the basics like the 3R's.

2. Teacher H asks one of her students, “What do you want to become when you grow up?” This question is an indication of what kind of
philosophy?

A. Progressivism 

B. Naturalism 

C. Existentialism

D. Idealism

RATIONALIZATION: D - Idealism. Idealism is the answer because it stresses the existence of ideas independent from the material world. Ideas
that which exist in the mind are the only reality.

3. Teacher X has not only explained the concept of Philosophy of Education but also imparted this to her students. This demonstrates what
kind of philosophy?

A. Naturalism 

B. Idealism 

C. Realism

D. Perennialism

RATIONALIZATION: C - Realism. Realism concerns with what is real, actual. For ideas to be realized, they must be transferred or demonstrated.

4. In his class, Teacher M always presents principles and values so as to encourage his students to examine them and decide for themselves
whether to accept them or not. What kind of philosophy does he practice?

A. Idealism 

B. Essentialism 

C. Humanism

D. Existentialism

RATIONALIZATION: D - Existentialism. Existentialism is a philosophy that emphasizes subjectivity, freedom and responsibility.
5. When a teacher emphasizes that man’s sense should be trusted because they are the only way to acquire knowledge, the teacher can be
regarded as____.

A. Naturalist 

B. Realist 

C. Empiricist

D. Pragmatist

RATIONALIZATION: C -Empiricism. Empiricism upholds that the only source of knowledge is the senses and sense-based experience.

6. Teacher K views her pupils as unique, free choosing, and responsible individuals. She plans activities where the pupil can develop his
unique personality. What theory underlies this nature of the pupil?

A. Realism 

B. Essentialism 

C. Existentialism

D. Progressivism

RATIONALIZATION: C. Existentialism. Existentialism is the philosophy of subjectivity of self hood whose fundamental doctrine proclaims man’s
freedom in the accomplishment of his destiny. In this case, emphasis is given on the students rather than on the curriculum content.

7. "Learning is the process of retrieving prior learning", this is a statement from ____.

A. Constructivist 

B. Reconstructivist 

C. Progressivist

D. Empiricist

RATIONALIZATION: A. Constructivist. As defined, Constructivism is a philosophy of learning which asserts that reality does not exist outside of
the human conceptions. It is the individual that construct reality by reflecting on his own experience and gives meaning to it. It is founded on the
premise that by reflecting on our own experiences, we construct our own understanding of the world we live in. Therefore, learning is simply the
process of adjusting our mental modes to accommodate new experiences.
8. Teacher U teaches to his pupils that pleasure is not the highest good. Teacher's teaching is against what philosophy?

A. Realism 

B. Hedonism 

C. Epicureanism

D. Empiricism

RATIONALIZATION: B.Hedonism. Epicureanism is just a form of Hedonism. Hedonism is the general, Epicureanism is the specific. Always select
the general answer.

9. After listening to the homily of the Priest about fidelity, Catherine has a moment of reflection. Her understanding of the value of fidelity
has become deeper as she related this to her past experience. This typifies what kind of philosophy?

A. Constructivism 

B. Reconstructivism 

C. Humanism

D. Existentialism

RATIONALIZATION: A.Constructivism. Constructivism is a philosophy of learning which asserts that reality does not exist outside the human
conceptions. It is the individual who construct reality by reflecting on his own experience and gives meaning to it.

10. After studying the Principle of Identity, Teacher W asks her students to determine which among the given set of problems conforms to
the said identity. This shows that Teacher W upholds what kind of Philosophy?

A. Perennialism 

B. Progressivism 

C. Essentialism

D. Naturalism

RATIONALIZATION: B. Progressivism. Progressivism is a philosophy of learning by doing things.


11. Thea listened to the advice given by her sister to end the relationship that she has with Gilbert. However, her sister learned that the
advice she has given was not followed and Thea decided to continue the relationship. This action of Thea is a manifestation of what kind of
philosophy?

A. Essentialism 

B. Existentialism 

C. Perennialism

D. Humanism

RATIONALIZATION: B. Existentialism. Existentialism is the philosophy of subjectivity of selfhood and proclaims man’s freedom in the
accomplishment of his destiny.

12. After finishing the degree in Education, Teacher M learns that learning never stops. In fact, she accumulates more knowledge after
leaving the portal of her alma mater. This typifies what kind of philosophy?

A. Constructivism 

B. Perennialism 

C. Progressivism

D. Humanism

RATIONALIZATION: C.Progressivism. Progressivism connotes change, growth and development. It is an active form of philosophy for it continues
to evolve.

13. Matilda is an advocate of the principle “making the most of your life”. She is indeed an advocate of what kind of philosophy?

A. Humanism 

B. Perennialism 

C. Realism

D. Existentialism

RATIONALIZATION: A.Humanism. Humanism is a philosophy that stresses to “live life to the fullest”.
14. Teacher X is a very dedicated teacher in the nursery. Her foremost concern is for students to learn how to adapt themselves in the
environment. This shows that Teacher X upholds what kind of philosophy?

A. Naturalism 

B. Pragmatism 

C. Progressivism
D. Rationalism

RATIONALIZATION: A.Naturalism. Naturalism aims to unfold the child’s potential, not to prepare him for a definite vocation or social position
but to prepare him to adapt to the changing times and needs.

15. Teacher J serves as an inspiration to his students because of his efficiency and effectiveness as a teacher. The mind set of his students
towards him/her is an instance of what kind of philosophy?

A. Realism 

B. Idealism 

C. Nationalism

D. Constructivism

RATIONALIZATION: B.Idealism. Idealism is a philosophy which asserts that what’s in the mind is the only reality. In this situation, the teacher
inspires his/her students through the values that he/she possessed. Values are absolute are told to belong in realm of ideas. Hence, these are
considered as ideas that exist only in the mind.

16. John learns well through active interplay with others. His learning increases when engaged in activities that has meaning in him. Which
philosophy can describe this?

A. Progressivism 

B. Idealism 

C. Realism

D. Empiricism

RATIONALIZATION: A.Progressivism. Progressivism is a philosophy of hands of learning or experiential learning. “Learning by doing”. Learning is
based from the questions of one’s experience of the world. Hence, it is the learner himself who thinks, solves, and gives meaning through his
individual experiences.
17. Your teacher is of the opinion that the world and everything in it are ever changing and so teaches you the skill to cope with the changes.
Which in his governing philosophy?

A. Experimentalism 

B. Existentialism 

C. Idealism

D. Realism

RATIONALIZATION: A.Experimentalism. Experimentalism believes that things are constantly changing. It is based on the view that reality is
what works right now and that goodness comes from group decisions. As a result, schools exist to discover and expand the society we live in.
Students study social experiences and solve problems.

18. Principal C shares this thought with his teachers: "Subject matter should help students understand and appreciate themselves as unique
individuals who accept complete responsibility for their thoughts, feelings, and actions." From which philosophy is this thought based? /
Teacher V teaches her students to appreciate themselves as unique individuals and are to accept the responsibility to their feelings, actions
and thoughts. She upholds the philosophy of _______.

A. Perennialism 

B. Essentialism 

C. Existentialism

D. Progressivism

RATIONALIZATION: C.Existentialism. Existentialism is the philosophy of subjectivity of selfhood and proclaims man’s freedom in the
accomplishment of his destiny.

19. Teacher H class is engage in problem solving activity which in a way is a reflection of the personal and social experiences. The purpose of
this activity is for the students to acquire the skills that can help them in solving their own real-life problems. What philosophy can best
describe this?

A. Realism 

B. Idealism 

C. Progressivism

D. Existentialism
RATIONALIZATION: C.Progressivism. Progressivism connotes growth and development. In this manner learning comes best when things are
being experienced, that is learning by doing things. It involves participation of the students and allows them to exercise freedom.

20. In the Social Science class of Teacher G, students identify the various social and economic problems that require urgent solutions. They
not only discuss the ways to address it but also agreed to participate in solving them. What kind of philosophy does this class uphold?

A. Naturalism 

B. Constructivism 

C. Progressivism

D. Reconstructivism

RATIONALIZATION: D.Reconstructivism. Reconstructivism covers the underlying factors that constitute reality or society. In this regard, students
are encourage to become involved in the problems whether political, social, or economical that confront the society and be able to arrive at
solutions in order to reconstruct society.

21. A curriculum should only include those that have survived the test of time and combine the symbols of literature, history, and
mathematics. Thus, curriculum like this contains values that are constant and universal. What philosophy describes this kind of curriculum?

A. Idealism 

B. Perennialism 

C. Humanism

D. Essentialism

RATIONALIZATION: B.Perennialism. Perennialism maintains that education involves confronting the problems and questions that have
challenged people over the centuries. Thus there is a need to study classical tradition of great books.

22. Who among the following puts more emphasis on core requirements, longer school day, longer academic year and more challenging
textbooks?

A. Perennialist 

B. Essentialist 

C. Progressivist

D. Existentialist
RATIONALIZATION: B. Essentialist. Essentialism is an educational theory rooted in classical realism and idealism which advocates curricular
reform stressing the essentials of the basics like the 3R's.

23. “Education is a continuous process of experiencing and of visiting or reorganizing experiences “according to a Progressivist. What does it
mean?

A. Education begins and ends in school.

B. Education takes place anytime and anywhere.

C. Education happens formally and informally.

D. Education goes on throughout life.

RATIONALIZATION: D. Education goes on throughout life. Progressivism connotes growth and development. It is an active form of philosophy
for it continues to evolve.

24. Mr. Cruz exemplifies an ideal teacher and son. He is competent in his field based on the various recognitions that he received from his
school. As a dutiful son, he assumes all the responsibilities of raising his family (this includes his parents, siblings, and their own families) –
giving all their needs and wants. The fulfilment of these is the neglect of the responsibilities to himself. This attitude of Mr. Cruz is against
what kind of philosophy?

A. Existentialism 

B. Constructivism 

C. Pragmatism

D. Humanism

RATIONALIZATION: D.Humanism. Humanism stresses personal culture, individual freedom, and development (the best way towards full and rich
lives).

25. Teacher R ensures to put a certain amount of his monthly earnings in the bank. At the end of the school year, he used all his savings in
visiting places rich of cultural heritages rather than of buying expensive clothes, jewellery, and latest gadgets. Favoring the former action
over the latter exhibits that Teacher R follows what kind of philosophy?

A. Existentialism 

B. Essentialism 

C. Behaviorism

D. Progressivism
RATIONALIZATION: A. Existentialism. Existentialism is a philosophy that gives a person the freedom to choose. It is a philosophy of subjectivity.

26. Teacher R ensures to put a certain amount of his monthly earnings in the bank. At the end of the school year, he used all his savings in
visiting places rich of cultural heritages rather than of buying expensive clothes, jewellery, and latest gadgets. This shows that Teacher R is
an advocate of what kind of philosophy?

A. Pragmatism 

B. Perennialism 

C. Naturalism

D. Essentialism

RATIONALIZATION: B. Perennialism. Perennialism views education as a recurring process based on eternal truths. Good education involves a
search for and understanding of truth which can all be found in the great works of civilization.

27. Teacher X rates the art work of her students not just on the merit of its appeal to the senses but also considers its uniqueness and the
responsibility that every student has given in accomplishing the task. This practice shows how Teacher X upholds this kind of philosophy:

A. Realism 

B. Reconstructivism 

C. Idealism

D. Existentialism

RATIONALIZATION: D. Existentialism. Conceives philosophy as something that is felt by an individual because it’s concrete in itself or based on
what is concrete. There is also personal involvement in this kind of philosophy.

28. In line with the philosophy of Reconstructivism, which of the following should be given emphasis in teaching?

A. To seek a better position in the society

B. To compare oneself with the less fortunate

C. To become economically self-reliant

D. To designate one’s superiority over the others


RATIONALIZATION: C. To become economically self-reliant. Reconstructivism aims to achieve social changes. This is a philosophy that entails
the identification of social problems, finding solutions for this and be a part of the solution.

29. Joining organizations or clubs in school is requisite to granting of academic distinction under DECS Order No. 65, s. 1998. This shows
school community connection reflected in activities geared towards society's needs. What philosophy is related to this? / Activities planned
by school clubs or organizations show school-community connection geared towards society's needs. What philosophy is related to this?

A. Realism 

B. Progressivism 

C. Existentialism

D. Reconstructivism

RATIONALIZATION: D.Reconstructivism. Reconstructivism aims to achieve social changes. This is a philosophy that entails the identification of
social problems, finding solutions for this and be a part of the solution.

30. Increase of time allotment in English, Math and Science in the Elementary level under the New Elementary School Curriculum (NESC) as
provided in DECS Order No. 1, s. 1993 contributes in the culturation of basic literacy by providing the needed essential skills. The related
philosophy is ______.

A. Realism 

B. Essentialism 

C. Idealism

D. Perennialism

RATIONALIZATION: B.Essentialism. Essentialism is an educational theory rooted in classical realism and idealism which advocates curricular
reform stressing the essentials of the basics like the 3R's.

LET Reviewer : Professional Education - SOCIAL DIMENSIONS Questionnaire Part 2

- October 22, 2018

DOWNLOAD
Click here to Download

31. Teacher A believes that the learner is the product of his environment. Sometimes he has no choice. He/she is determined by his
environment.
A. Rationalist 

B. Behaviorist 

C. Existentialist

D. Progressivist

32. The curriculum is viewed as a means of developing desirable habits. It is recommended that the way /means to form these habits is
through the mastery of organized subject matter. Which philosophy is behind this educational view?

A. Naturalism 

B. Idealism 

C. Realism

D. Pragmatism

33. Teachers in school perform the role and responsibility of parents in the development and education of the child. This stems from the
belief that the home is the primary agency in the education of the individual. Thus, teachers are regarded as surrogate parents along the
loco parentis principle. Which philosophy espouses this view of education?

A. Pragmatism 

B. Naturalism 

C. Idealism

D. Realism

34. This philosophy believes that students need a passionate encounter with the positive and negative phases of life like the joy and agony
of love, desirability of life, the inevitability of death, the anguish of freedom and the consequences of choices and actions. Which philosophy
advocates this view in education?

A. Existentialism 
B. Perennialism 

C. Essentialism

D. Realism

35. The teacher should provide for shared experiences in the classroom that fosters cooperative learning rather than competitive learning.
Scientific means of inquiry complements such atmosphere. Which theory advocates this view?

A. Existentialism 

B. Essentialism 

C. Perennialism

D. Progressivism

36. This philosophy believes that ideas are inherent in the invidual. This explains the nature of the Socratic dialogue in teaching, where the
teacher asks questions to wring out the ideas innate in the learner. Which school of thought has advocated this belief?

A. Naturalism 

B. Idealism 

C. Realism

D. Pragmatism

37. Teacher B believes that the learner can choose what he can become despite his environment.

A. Rationalist 

B. Behaviorist 

C. Existentialist

D. Progressivist

38. Teacher F is convinced that whenever a student performs a desired behavior, provided reinforcement and soon the student will learn to
perform the behavior on his own. On which principle is Teacher F's conviction based?

A. Cognitivism 

B. Environmentalism

C. Behaviorism

D. Constructivism

39. Changes in the T.H.E takes care of the cognitive needs of students. It also emphasizes the use of project method as implied in DECS Order
No. 91, s. 1998. The related philosophy is _____________

A. Realism 

B. Existentialism 

C. Progressivism

D. Reconstructivism
40. DECS Order No. 13 s. 1998 entitled "Revised rules and regulations on the teaching of religion" is emphasizing the development of one's
spirituality and moral. What philosophy is related to it?

A. Realism 

B. Idealism 

C. Essentialism

D. Progressivism

41. DECS Order No. 57 s. 1998 entitled "Clarification on the changes in the Social Studies programs" offers Economics in 3rd year instead of
4th year. The change hopefully will provide the needed social reform in alleviating the condition of poverty. What philosophy addresses this
concern?

A. Realism 

B. Existentialism 

C. Essentialism

D. Reconstructivism

42. DECS Order No. 54, s. 1995 entitled "War on Waste" helps students realize their role in the conservation of nation's resources. This also
helps instil discipline among them. What philosophy is related to this?

A. Realism 

B. Essentialism 

C. Perennialism

D. Reconstructivism

43. Which philosophy has the educational objective to indoctrinate Filipinos to accept the teachings of the Catholic church which is to foster
faith in God?

A. Realism 

B. Pragmatism 

C. Idealism

D. Existentialism

44. Virtue as one component in the teaching of Rizal as a course focuses on the teaching of good and beauty consistent with the good and
beauty in God. What philosophy supports this?

A. Existentialism 

B. Idealism 

C. Progressivism

D. Reconstructivism

45. Scouting and Citizens Army Training (CAT) give training in character building, citizenship training, etc. which leads to the creation of new
social order and a new society eventually. What philosophy supports this?
A. Existentialism 

B. Perennialism 

C. Progressivism

D. Reconstructivism

46. Teacher V demonstrated the technique on how to group students according to their needs and interests and how to use self-paced
instructional materials. Which philosophy is manifested in this activity?

A. Essentialism 

B. Progressivism 

C. Realism

D. Reconstructivism

47. Teacher G, a Christian Living teacher, puts so much significance on values development and discipline, what could be her educational
philosophy?

A. Idealism B. Pragmatism 

C. Progressivism

D. Realism

48. What philosophy is related to the practice of schools acting as laboratory for teaching reforms and experimentation?

A. Essentialism 

B. Existentialism 

C. Progressivism

D. Reconstructivism

49. What philosophy of education advocates that the curriculum should only include universal and unchanging truths?

A. Essentialism 

B. Idealism 

C. Perennialism

D. Pragmatism
50. Which philosophy of education influence the singing of the National anthem in schools?

A. Nationalism 

B. Naturalism 

C. Pragmatism

D. Socialism

51. Who among the following believes that learning requires disciplined attention, regular homework, and respect for legitimate authority?

A. Essentialist 

B. Progressivist 

C. Realist

D. Reconstructivist

52. Teacher G says: "If it is billiard that brings students out of the classroom, let us bring it into the classroom. Perhaps, I can use it to teach
Math." To which philosophy does Teacher G adhere?

A. Essentialism 

B. Idealism 

C. Progressivism

D. Reconstructivism
53. Which philosophy influenced the cultivation of reflective and meditative skills in teaching?

A. Confucianism 

B. Existentialism 

C. Taoism

D. Zen Buddhism

54. “Specialization is knowing more and more about less and less”. It is better to be a generalist claims Teacher R. What philosophy does she
uphold?

A. Essentialism 

B. Existentialism 

C. Progressivism

D. Pragmatism

55. “Everything changes” explains the teacher. This entails readiness to accept challenges in life and be ready to address it. What kind of
philosophy can this be attributed?

A. Realism 

B. Materialism 

C. Behaviorism

D. Rationalism
56. “Life is what you make it”, exemplifies best what __________ is :

A. Existentialist 

B. Humanist 

C. Idealist

D. Realist

57. If Teacher V believes that learning also takes place in having interaction with the environment, this teacher can be regarded as:

A. Pragmatist 

B. Empricist 

C. Realist

D. Herbalist

58. Teacher Q is concerned more on conceptual matters since reality is mental. She upholds this kind of philosophy:

A. Empiricism 

B. Realism 

C. Idealism

D. Progressivism

59. Teacher W is known in giving great importance on humanistic education. Which of the following will depict this trait?
A. Developing students into thinking individuals

B. Making students civilized, distinctly educated and refined

C. Giving the students the freedom to choose and decide for themselves

D. Teaching the students the different works of civilization

60. To come closer to the truth we need to "go back to the things themselves." This is the advice of the __________.

A. Behaviorists 

B. Phenomenologists 

C. Idealists

D. Pragmatists

LET Reviewer : Professional Education Part 1

- November 02, 2018

1.    With R.A. 9155, to which body were all the functions, programs, and activities of the Department of Education related to Sports
competition transferred?

a.    Technical Education Services Department Authority


b.    Philippine Sports Commission
c.     National Commission for Culture and the Arts
d.    Commission on Higher Education
2.    Parenting style influences children’s development. Read the following parent’s remarks for their children then, answer the question.
Parent C – Tells her child: “You should do it my way or else. There is no discussion.”
Parent D – Tells her husband: “It is 10:00 PM, do you know where your child is?”
Parent E – Tells her child: “You know, you should have not done that. Let’s talk about it so you can handle the situation better next
time.”
Parent F – Tells her child: “You may do what you want. We will always be here for you, no matter what you do.”
Which Parenting style is Authoritarian?

a.    D b. F c. E d. C

3.    Two identical beakers A and B are presented to the child. Teacher Sonny pours the liquid from B to C which is taller and thinner than
A and B but has equal capacity with B. The teacher asks if the beakers A and C have the same amount of liquid. The child says
“NO” and points to C as the beaker that has more liquid. In which cognitive developmental stage is the child?

a.    Sensorimotor stage


b.    Concrete operational stage
c.    Pre-operational stage
d.    Formal Operational stage

4.    To determine her students’ level of moral development, Teacher Evangeline presents to her class a morally ambiguous situation and
asks them what they would do in such a situation. On whose theory is Teacher Evangeline’s technique based?

a.    Bruner
b.    Kohlberg
c.     Freud
d.    Piaget

5.    According to R.A. 9155, which among the following is considered the “heart of the formal education system”?

a.    The pupil


b.    The teacher
c.     The classroom
d.    The school

6.    You arrange the rows of blocks in such a way that a row of 5 blocks is longer than a row of 7 blocks. If you ask which row has more,
Grade 1 pupils will say that it is the row that makes the longer line. Based on Piaget’s cognitive development theory, what problem is
illustrated?

a.    Assimilation problem


b.    Accommodation problem
c.    Conservation problem
d.    Egocentrism problem

7.    According to R.A. 9155, a school head has two roles, namely administrative manager and ____.
8.     
a.    Health officer
b.    Instructional leader
c.     Facilitator
d.    Guidance counselor

9.    After reading and paraphrasing Robert Frost’s Stopping by the Woods on Snowy Evening, Teacher Marko asked the class to share
any insight derived from the poem. In which domain in Bloom’s taxonomy of objectives is the term paraphrase?

a.    Analysis c. Comprehension


b.    Application d. Synthesis

10.  Which characterizes a constructivist teaching-learning process?

a.    Conceptual interrelatedness


b.    Multiple perspectives
c.     Authentic assessment
d.    Passive acceptance of information

11.  On what theory is the sequencing of instruction anchored?

a.    Gagne’s hierarchical theory


b.    B.F. Skinner’s operant conditioning theory
c.     Bandura’s social learning theory
d.    Thorndike’s law of effect

12.  A common complaint of teachers about pupils is this: “You give them assignment, the following day they come without any. You
teach them this today, asks them tomorrow and they don’t know. It is as if there is nothing that you taught them at all.” Based on the
theory of information processing, what must teachers do to counteract pupil’s forgetting?

I.      Punish every child who can’t give correct answers to questions.
II.    Work for meaningful learning by connecting lesson to what pupils know.
III.   Reward every child who remembers past lessons.

a.    III only c. II and III


b.    I and III d. II only

13.  When small children call all animals “dogs”, what process is illustrated, based on Piaget’s cognitive development theory?

a.    Assimilation c. Reversion


b.    Conservation d. Accommodation

14.  Based on Bandura’s theory, which conditions must be present for a student to learn from a model?

               I.      Attention          III. Motor reproduction


               II. Retention        IV. Motivation
a.    I and II c. I, II, III and IV
b.    I, II and III d. III and IV

15.  According to Tolman’s theory on purposive behaviorism, learning is goal-directed. What is its implication to teaching? 

a. Evaluate lessons based on your objective/s


b. Set as many objectives as you can
c. Stick to your objectives/s no matter what happens
d. Make the objective/s of your lesson clear and specific

16.  Which is the ideal stage of moral development? Stage of _____.

a.    Social contrac
b. Universal ethical principle
c. Law and order
d. Good boy/good girl

17.  Cristina’s family had a family picture when she was not yet born. Unable to see herself in the family picture, she cried despite her
mother’s explanation that she was not yet born when the family picture was taken. What does Cristina’s behavior show?

Limited social cognition

 Egocentrism

Semi-logical reasoning

Rigidity of thought

18.  To help a student learn to the optimum, Vygotsky advises us to bridge the student’s present skill level and the desired skill level by
______.

a.    Challenging c. Inspiring


b.    Scaffolding d. Motivating

19.  Based on Piaget’s theory, what should a teacher provide in the formal operational stage?

a.    Stimulating environment with ample objects to play with


b.    Games and other physical activities to develop motor skills
c.    Activities for hypothesis formulation
d.    Learning activities that involve problems of classification and ordering

20.  “Do not cheat. Cheating does not pay. If you do, you cheat yourself” says the voiceless voice from within you. In the context of
Freud’s theory, which is/are at work?

a.    Id c. Ego
b.    Id and Superego d. Superego
21.  Here are comments from School Head Carmen regarding her observations on teacher’s practice in lesson planning:
The words “identify,” “tell” and “enumerate” are overused. Many times they make use of non-behavioral terms. Often their lesson
objectives do not include value formation and inculcation.
What can be inferred from the School Head’s comments regarding teacher formulated lesson objectives?

a.    Often lesson objectives are in the low level


b.    Very often lesson objectives are in the cognitive domain
c.     Quite often lesson objectives describe teacher’s behavior
d.    Often lesson objectives are in the psychomotor domain

22.  Sassi, a Grade I pupil is asked, “Why do you pray everyday?” Sassi answered, “Mommy said so.” Based on Kohlberg’s theory, in
which moral development stage is Sassi?

a.    Pre-convention level


b.    Conventional level
c.     In between conventional and post-conventional levels
d.    In between pre- and post-conventional levels

23.  Teacher Fatima tells her students: “You must be honest at all times not only because you are afraid of the punishment but more
because you yourselves are convinced of the value of honesty.” Based on Kohlberg’s theory, which level of moral development
does the teacher want her students to reach?

a.    Conventional level


b.    Between conventional and post-conventional levels
c.     Between pre-conventional and post-conventional levels
d.    Post-conventional level

24.  Why is babyhood referred to as a “critical period” in personality development? Because:

a.    At this time the baby is exposed to many physical and psychological hazards
b.    Changes in the personality pattern take place
c.    At this time the foundations are laid upon which the adult personality structure will be built
d.    The brain grows and develops at such an accelerated rate during babyhood

25.  It is good to give students creative learning tasks because ______.

a.    Development is affected by cultural changes


b.    The development of individuals is unique
c.     Development is the individual’s choice
d.    Development is aided by stimulation

26.  According to Havighurst’s development tasks, reaching and maintaining satisfactory performance in one’s occupational career is
supposed to have been attained during ____.

a.    Middle age and Early adulthood


b.    Middle age
c.     Old age
d.    Early adulthood
27.  Student Deina says: “I have to go to school on time. This is what the rule says.” In what level of moral development is the student?

a.    Pre-conventional
b.    Post-conventional
c.    Conventional
d.    Cannot be specifically determined

28.  In planning for instruction, can a teacher begin with assessment?

a.    No, it may discourage and scare the learners


b.    Yes, determine entry knowledge or skill
c.     Yes, to make the class pay attention
d.    No, assessment is only at the end of a lesson

29.  Which among the following is closest to the real human digestive system for study in the classroom?

a.    Drawing of the human digestive system on the board


b.    Model of the human digestive system
c.     The human digestive system projected on an OHP
d.    Drawing of the human digestive system on a page of a textbook

30.  Here is a question: “Is the paragraph a good one?” Evaluate. If broken down to simplify, which is the best simplification?

a.    Why is the paragraph a good one? Prove


b.    Is the paragraph a good one? Why or Why not?
c.     If you asked to evaluate something, what do you do? Evaluate the paragraph?
d.    What are the qualities of a good paragraph? Does the paragraph have these qualities?

31.  Which one is in support of greater interaction?

a.    Probing
b.    Repeating the question
c.     Not allowing a student to complete a response
d.    Selecting the same student respondents

32.  With this specific objective, to reduce fractions to their lowest terms, this is how the teacher developed the lesson.

Step 1 – Teacher stated the rule on how to reduce fractions to their lowest term
Step 2 – Teacher wrote 2 , 3 , 4 , 5 , 6
4 6 8 10 12
and showed how to reduce them to 1
2
Step 3 – Teacher wrote 3 , 6 , 4 and
9 9 12
showed how to reduce them to their lowest term.
Step 4 – Teacher gave this written exercise to the class.
Reduce the following fractions to their lowest terms: 3 , 7 , 5 , 8 , 5 , 4
12 14 10 16 15 6

Did the lesson begin with concrete experience then developed into the abstract?

a.    No
b.    Yes, a little
c.     Yes, by way of the examples given by the teacher
d.    Yes, the pupils were involved in arriving at the rule on reducing fractions to their lowest terms

33.  I want to compare two concepts. Which technique is most appropriate?

a.    Attribute wheel


b.    K-W-L techniques
c.    Venn diagram
d.    Spider web organizer

34.  Which activity should a teacher have more for his students if he wants them to develop logical-mathematical thinking?

a.    Focus group discussion


b.    Problem solving
c.     Games
d.    Small group discussion

35.  I want to use a pre-teaching strategy that will immediately engage my students in the content and will enable me to get an insight into
how students think and feel about the topic. Which is most appropriate?

a.    K-W-L chart c.Graphic organizer


b.    Story boarding d. Document analysis

36.  For a discussion of a topic from various perspectives, it is best to hold a ______.
a.    Debate c. Panel discussion
b.    Brainstorming d. Symposium

37.  After establishing my learning objectives, what should I do to find out what my students already know and what they do not yet know
in relation to my lesson objectives in the cognitive domain?

a.    Give a pretest


b.    Study the least learned competencies in the National Achievement Test
c.     Analyze my students’ grades last year
d.    Interview a sample of my students
38.  What characterizes genuine change? Change in _____.

a.    Appearance c. Substance


b.    Form d. Physical attribute

39.  In which strategy, can students acquire information from various perspectives, and led to reflective thinking and group consensus?

a.    Debate
b.    Small group discussion
c.     Panel discussion
d.    Symposium

40.  At the end of my lesson on the role of a teacher in learning, I asked the class: “In what way is a teacher an enzyme?” With this
question, it engaged the class in _______.

a.    Allegorical thinking


b.    Concrete thinking
c.    Metaphorical thinking
d.    Symbolical thinking

41.  Which must be primarily considered in the choice of instructional aide?

a.    Must stimulate and maintain student interest


b.    Must be updated and relevant to Filipino setting
c.    Must be suited to the lesson objective
d.    Must be new and skillfully made

42.  For lesson clarity and effective retention, which should a teacher observe, according to Bruner’s theory?

a.    Begin teaching at the concrete level but go beyond it by reaching the abstract
b.    Use purely verbal symbols in teaching
c.     Start at the concrete level and end there
d.    End teaching with verbal symbols

43.  Is it advisable to use realias all the time?

a.    No, for the sake of variety of instructional materials


b.    No, only when feasible
c.     Yes, because there is no substitute for realias
d.    Yes, because it is the real thing

44.  I want my students to look at the issues on the call for President Arroyo to step down from several perspectives. Which activity is
most fitting?

a.    Cross examination c. Symposium


b.    Panel discussion d. Debate

45.  I intended to inculcate in my students the value of order and cleanliness. I begin my lesson by asking them to share their experiences
about the dirtiest and the cleanest place they have seen and how they felt about them. From there I lead them to the consequences
of dirty and clean home of surroundings. In my lesson development plan, how do I proceed?

a.    Transductively c. Deductively


b.    Inductively d. Concretely

46.  Teacher Neri wants to develop the ability of sound judgment in his students. Which of the following questions should he ask?

a.    What is the essayist saying about judging other people?


b.    With the elements of a good paragraph in mind, which one is best written?
c.     Why is there so much poverty in a country where there is plenty of natural resources?
d.    Of the characters in the story, with whom do you identify yourself?

47.  The teacher is the first audio-visual aid in the classroom. What does this imply?
a.    You take care that you follow the fashion or else students won’t listen to you
b.    Your physical appearance and voice should be such that students are helped to learn
c.     Make good use of the radio and TV in the classroom
d.    Include singing in your teaching method

48.  I used the gumamela flower, a complete flower, to teach the parts of a flower. Which method did I use?

a.    Demonstration method


b.    Type-study method
c.     Drill method
d.    Laboratory method

49.  A teacher would use a standardized test ______.

a.    To serve as a unit test


b.    To serve as a final examination
c.     To engage in easy scoring
d.    To compare her students to national norms

50.  Other than finding out how well the course competencies were met, Teacher Kathy also wants to know her students’ performance
when compared with other students in the country. What is Teacher Kathy interested to do?

a.    Formative evaluation


b.    Authentic evaluation
c.    Norm-referenced evaluation
d.    Criterion-referenced evaluation
51.  I want to help my students retain new information. Which one will I use?

a.    Questions c. Games


b.    Mnemonics d. Simulations
LET Reviewer : Professional Education Part 2

- November 02, 2018

1.    I want to use a diagram to compare the traditional and authentic modes of assessment. Which one is most fit?
a.    Affinity diagram
b.    Tree diagram
c.    Venn diagram
d.    Fishbone diagram

2.    A big story in your local newspaper. You want to use the headlines as an inquiry device. To increase student participation, you might
begin by ____.
a.    Asking one to read the news story and interpret what he read after
b.    Asking the class to infer connotations and denotations from the headline
c.     Explaining what you believe to be the underlying causes
d.    Describing the background of the story as you know it
3.    If a triadic interaction includes three (3) persons, how many persons are included in a dyadic interaction?
a.    Two
b.    Two groups composed of two persons each
c.     One, the person and himself
d.    Four
4.    When I teach skills that are critical to the learning of the next topics, what should I employ?
a.    Direct instruction
b.    Mastery learning
c.     Socratic method
d.    Cooperative learning
5.    I want my students to have mastery learning of a basic topic. Which can help?
a.    Drill
b.    Socratic method and drill
c.     Morrisonian technique and drill
d.    Socratic method
6.    Teacher Feng wanted to teach the pupils the skill of cross stretching. Her check-up quiz was a written test on the steps of cross
stitching. What characteristic of a good test does it lack?
a.    Predictive validity
b.    Objectivity
c.    Reliability
d.    Content validity
7.    In the parlance of test construction, what does TOS mean?
a.    Table of Specification
b.    Team of Specifications
c.     Table of Specifics
d.    Terms of Specifications
8.    If I favor “assessment for learning,” which will I do most likely?
I.      Conduct a pre-test results
II.    Teach based on pre-test results
III.   Give specific feedback to students
IV.  Conduct peer tutoring for students in need of help
a.    I, II and IV c. I, II and III
b.    II, III and IV d. I, II, III and IV
9.    After a lesson on the atom, the students were asked to work on a physical model of the atom to determine learning. For which group
of students is building an atom model intended?
a.    Interpersonality intelligent
b.   Kinesthetically intelligent
c.    Mathematically intelligent
d.    Linguistically intelligent
10.  If I want to develop creative thinking in my students, which one/s should I use?
I.              Problem solving
II.            Brainstorming
III.           Dramatics
a.    I and II c. III only
b.    II only d. I, II and II

11.  Which is/are effective method/s in teaching students critical reading skills?
a.    Read and interpret three different movie reviews
b.    Interpret editorials about a particular subject from three different newspaper
c.     Distinguish fiction from non-fiction materials
d.    Interpret editorials and read and interpret three different movie reviews

12.  I want to present the characteristics features of a constructivist approach. What should I use?
a.    Fishbone diagram
b.    Venn diagram
c.     Narrative frame
d.    Attribute wheel
13.  If all of your students in your class passed the pretest, what should you do?
a.    Administer the posttest
b.    Go through the unit as usual because it is part of the syllabus
c.     Go through the lesson quickly in order not to skip any
d.    Go on to the next unit
14.  Teacher Vicky shows her students a picture of people in thick jackets. Then she asks them to tell her the kind of climate when the
picture was taken. If she asks 5 questions of this kind and her students do not get them, it is safe to conclude that pupils are quite
weak in the skill of _______.
a.    Analyzing c. Synthesizing
b.    Inferring d. Generalizing
15.  Which must go with self-assessment for it to be effective?
a.    Scoring rubric
b.    Consensus of evaluation results from teacher and student
c.     External monitor
d.    Public display of results of self-evaluation
16.  Which group of scores is most varied? The group with ________.
a.    0.90 SD c. 0.10 SD
b.    0.50 SD d. 0.75 SD
17.  The main purpose in administering a pretest and a posttest to students is to _____.
a.    Measure gains in learning
b.    Measure the value of the material taught
c.     Keep adequate records
d.    Accustom the students to frequent testing
18.  Assessment is said to be authentic when the teacher ________.
a.    Consider students’ suggestion in teaching
b.    Gives valid and reliable paper-and-pencil test
c.    Gives students real-life task to accomplish
d.    Includes parents in the determination of assessment procedures
19.  The following are computed means of a hundred-item test: Physical science, 38; Math, 52; English, 33. Based on the data, which is
true?
a.    The examinees seem to be very good in Physical Science
b.    The Math test appears to be the easiest among the three
c.     The examinees seem to excel in English
d.    The English test appears to be the easiest among the three
20.  An examinee whose score is within x + 1 SD belongs to which of the following groups?
a.    Above average
b.    Average
c.     Below average
d.    Needs improvement
21.  Which is true of a bimodal distribution?
a.    The scores are neither high nor low
b.    The group has two different groups
c.     The score are high
d.    The score are low
22.  When you reach the “plateau of learning”, what should you do?
a.    Forget about learning
b.    Reflect what caused it
c.     Force yourself to learn
d.    Rest
23.  What can be said if student performance in a positively skewed scores distribution?
a.    Most students performed well
b.    Most students performed poorly
c.     Almost all students had averaged performance
d.    A few students performed excellently
24.  A number of test items in a test are said to be non-discriminating? What conclusion/s can be drawn?
I.      Teaching or learning was very good.
II.    The item is so easy that anyone could get it right.
III.   The item is so difficult that nobody could get it.
a.    II only c. III only
b.    I and II d. II and III
25.  A test item has a difficulty index of 0.51 and a discrimination index of 0.25. What should the teacher do?
a.    Revise the item
b.    Retain the item
c.     Make it a bonus item
d.    Reject the item
26.  The difficulty index of a test item is 1. This means that _____________.
a.    The test item is a quality item
b.    The test is very difficult
c.    The test is very easy
d.    Nobody got the item correctly
27.  If the compound range is low, this means that _____________.
a.    The students performed very well in the test
b.    The difference between the highest and the lowest score is low
c.     The difference between the highest and the lowest score is high
d.    The students performed very poorly in the test
28.  What is the mastery level of a school division in a 100-item test with a mean of 55?
a.    42% b. 55% c. 45% d. 50%
29.  A negative discrimination index means that ____________.
a.    The test item has low reliability
b.    More from the lower group answered the test item correctly
c.     The test item could not discriminate between the lower and upper groups
d.    More from the upper group got the item correctly
30.  In an entrance examination, student Anna’s Percentile is 25 (P25). Based on this Percentile rank, which is likely to happen?
a.    Student Anna will be admitted
b.    Student Anna will not be admitted
c.     Student Anna has 50-50 percent chance to be admitted
d.    Student Anna has 75 percent chance to be admitted
31.  What does a percentile rank of 62 mean?
a.    It is the student’s score in the test
b.    The student answered sixty-two (62%) of the items correctly
c.    The student’s score is higher than 62 percent of all students who took the test
d.    Sixty-two percent (62%) of those who took the test scored higher than the individual
32.  What does the computer have in common with the TV?
a.    Key board c. Screen
b.    File d. Disk drive
33.  Which depicts in graphic form the social relations present in a group?
a.    Interest inventory
b.    Sociogram
c.     Anecdotal record
d.    Johari’s window
34.  Planned ignoring, signal interference, and proximity control are techniques used in _____.
a.    Managing temper tantrums
b.    Managing surface behavior
c.     Operant conditioning
d.    Life-space interviewing
35.  What should you do to get the child’s attention when she/he is distracted by an object in the room?
I.      Call him by his name and tell him to pay attention
II.    Put away the distracting influence
III.   Involve him in helping with an activity
a.    I and II c. II and III
b.    I and III d. I, II and III
36.  Which practice in our present educational system is in line with Plato’s thought that “nothing learned under compulsion stays with the
mind”?
a.    Clarification of school policies and classroom rules on Day 1
b.    Presentation of standards of performance in the learner
c.    Making the teaching-learning process interesting
d.    Involving the learner in the determination of learning goals
37.  In Values Education, faith, hope, and love are believed to be permanent values whether they be valued by people or not. Upon what
philosophy is this anchored?
a.    Realism c. Idealism
b.    Existentialism d. Pragmatism
38.  In the schools, we teach realities that cannot be verified by the senses like an Invisible God or Allah. Whose beliefs does this practice
negate?
a.    Stoicists’ c. Skeptics’
b.    Rationalists’ d. Empiricists’
39.  Which emphasized on non-violence as the path to true peace as discussed in peace education?
a.    Taoism c. Hinduism
b.    Buddhism d. Shintoism
40.  I make full use of the question-and-answer as a model for discussion. From whom is this question-and-answer method?
a.    Socrates c. Aristotle
b.    Kant d. Plato

41.  A wife who loves her husband dearly becomes so jealous that in a moment of savage rage, kills him. Is the wife morally responsible
and why?
a.    Not necessarily. Antecedent passion may completely destroy freedom and consequently moral responsibility
b.    Yes, she killed her husband simply because of jealousy
c.     It depends on the case of the wife’s jealousy
d.    It depends on the mental health of the wife

42.  Martin Luther King Jr. once said, “Man may understand all about the rotation of the earth but he may still miss the radiance of the
sunset.” Which type of education should be emphasized as implied by Martin Luther King Jr.?
a.    Science and education
b.    Vocational education
c.    Liberal education
d.    Technical education

43.  Computer-assisted instruction is an offshoot of the theory of _____.


a.    J. Piaget c. B.F. Skinner
b.    J. Brunner d. J. Watson
44.  The use of varied teaching and testing strategies on account of students’ multiple intelligences is in line with the thoughts of ______.
a.    Daniel Goleman
b.    Jean Piaget
c.    Howard Gardner
d.    Benjamin Bloom

45.  Applying Confucius’s teachings, how would hiring personnel select the most fit in government positions?
a.    By record evaluation done by an accrediting body
b.    By government examinations
c.     By accreditation
d.    By merit system
46.  “Moral example has a greater effect on pupils’ discipline than laws and codes of punishment” is an advice of teachers from
_________.
a.    Confucius c. Lao tzu
b.    Mohammed d. Mencius
47.  “The principle of spontaneity against artificiality will make you accomplish something. Leave nature to itself and you will have
harmony” is an active advice from the _____.
a.    Hindu c. Shintoist
b.    Taoist d. Buddhist
48.  The significance of liberal education in holistic development of students is supported by _____.
a.    Perennialism
b.    Pragmatism
c.     Confucian teaching
d.    Perennialism and Confucian philosophy

49.  Is a sick teacher, the only one assigned in a remote school, excused from her teaching duty?
a.    No, she is the only one assigned in that school
b.    Yes, teaching is a demanding job
c.    Yes, she is sick
d.    No, she must abide by her contract
50.      What is a demonstration of your authority as a professional teacher?
a.    You make your pupils run errands for you
b.    You decide on how to teach a particular lesson
c.     You absent from class to enjoy your leave even without prior notice
d.    You select to teach only those lessons which you have mastered

LET Reviewer : Professional Education Part 3

- November 02, 2018

1.        Which statement on true authority is wrong?


a.    It sets an example
b.    It seeks its own satisfaction and privilege
c.       It acts in the best interest of others
d.    Its goal is to help, form, and guide others
2.        When a teacher teaches the idea that it is wrong to think that Filipino lifestyle, products and ideas are inferior to those of other
nationalities, he fights against ______.
a.    Acculturation            c. Ethnocentrism
b.    Xenocentrism         d. Culture shock
3.        Teacher Lolita, a teacher for forty years, refuses to attend seminars. She claims that her forty years of teaching is more than all the
seminars she is asked to attend. Is her actuation and thinking in accordance with the Code of Ethics for Professional Teachers?
a.    No, a professional teacher, regardless of teaching experience, ought to go through continuing professional education
b.    No, non-attendance to seminars means no professional growth
c.       Yes, because she has taught for forty years and may have mastered the trade
d.    Yes, provided she has an excellent performance rating
4.        A principal asked her good teachers to write modular lessons in Science, and then she had them published with her name printed as
author. Which is unethical in this case?
a.    She was the exclusive beneficiary of the royalty from the modules
b.    She burdened her teachers with work not related to teaching
c.       She had the modular lessons published when they were worth publishing
d.    She got the merit which was due for her teacher-writers

5.        Is it ethical on the part of the teacher to proselyte in her classroom every Friday?
a.    Yes, that strengthens values education
b.    Yes, that is religious instruction which is allowed by the Constitution
c.      No, a teacher shall not engage in the promotion of his/her religious interest in the classroom
d.    No, proselyting is no longer necessary in this age

6.        Which can promote national pride among pupils/students?


I.               Studying the lives of outstanding Filipinos here and abroad
II.    Reading the lives of saints of the Church
III.   Studying Philippine history with emphasis on the victories and greatness of the Filipino people
a.    I, II, III                                   c. I & II
b.    III only                                  d. I only
7.        Why is the exodus of Filipino professionals described as “brain drain”?
I.               Those who go abroad are usually the better
II.    Filipino professionals serve other countries instead
III.   They contribute to nation building through their dollar remittances
a.    I, II, III     b. I only   c. II only    d. I &II
8.        You want to report on a colleague's act of immorality. You don't have the courage to confront her. To end her illicit affair with a
married man you write and secretly distribute copies of your anonymous letter against your fellow teacher. What should have been
done instead?
a.    If the charge is valid; present such charge under oath before your school head
b.    Ask a third party to write the anonymous letter to prevent yourself from being involved
c.       Talk to the married man with whom she is having illicit affair
d.    Secretly give the anonymous letter only to the two people concerned
9.        Teachers often complain of numerous non-teaching assignments that adversely affect their teaching. Does this mean that teachers
must be pre-occupied only with their teaching?
a.    Yes, because teaching is enough full time job
b.    Yes, if they are given other assignments, justice demands that they be properly compensated
c.       No, because every teacher is expected to provide leadership in activities for the betterment of the communities where they live and
work
d.    No, they are also baby sitters especially in the pre-school
10.      My right ends where the rights of others begin. What does this mean?
a.    Rights are not absolute    
b.    Rights are alienable            
c.       Rights are inalienable       
d.    Rights are absolute
11.      History books used in the schools are replete with events portraying defeats and weaknesses of the Filipino people. In the spirit of
nationalism, how should you tackle them in the classroom?
a.    Present them as they are and tell the class to accept reality
b.    Present the facts and use them as means to teach and inspire your class
c.       Present the facts and express your feelings of regret
d.    Present the facts including those people responsible for the failures or for those who contributed
12.      Should an association of teachers obey a Supreme Court’s decision even if it conflict with its interest and opinions?
a.    No
b.    Yes
c.       Yes, provided they can make a bargain
d.    No, as a minority group they have the right to express their rejection
13.      A teacher does not agree with the selective retention policy of the school and she openly talks against it in her classes. Is her
behavior ethical?
a.    Yes, provided she got the permission from her superior to talk against the policy
b.    No, it is her duty to faithfully carry it out even if she does not agree
c.       No, in fact she is quite confused and passes on her confusion to others
d.    Yes, she is entitled to her opinion just as everybody is
14.      If you have a problem with another teacher, the first step towards resolution should be for you to:
a.    Talk directly with the teacher involved
b.    Ask your fellow teachers to intercede on your behalf
c.       Ask your fellow teachers for their suggestions
d.    Discuss it with your principal
15.      What is ethical for you to do if deep within your heart you do not agree with the school policy on student absences?
a.    Be vocal about your feeling and opinion against the policy
b.    Understand the policy and support the school in its implementation
c.       Argue with your superior to convince him to change the policy
d.    Keep your feeling to yourself but make insinuations that you are against it while you teach

CASE #1 – Mrs. Domingo developed a lesson on the concept of fraction this way: First, she presented one pizza, and then asked a
pupil to cut it into two. She called one part of the pizza 1/2 and the two parts of 2/2. Then she wrote 1/2 and 2/2 on the board. She
proceeded to ask another pupil to divide the half parts of the cake into two again, and then wrote 1/2, 2/4 and 3/4. Then she used the
model of fractions (wooden circles) divided into 2, 3 and 4 show 1/2, 1/4, 1/3, 2/4. Then she went back to the fractions she wrote on the
board. She asked her pupils for the meaning of 1/2, 1/3, 1/4, 2/4.

16.      Did Mrs. Domingo follow Bruner's three stage development of knowledge?
a.    Yes
b.    No
c.       Only in the first stage
d.    Cannot be determined because the lesson was not developed fully
17.      In the context of Bruner's principle of knowledge representation, which is the enactive phase of the lesson on fractions?
a.    Presenting the pizza and cutting it into two and four
b.    Using the model of fractions
c.       Writing the fractions 1/2, 1/4 and 1/3 on the board
d.    Asking the meaning of 1/2, 1/4 and 1/3
18.      Would it be easier to understand and retain the concept of fractions if Mrs. Domingo began the lesson on fractions with the meaning
of 1/2, 1/3, 1/4, etc.?
a.    Yes, provided we proceed to the concrete
b.    No, for better learning we proceed from the concrete to the abstract
c.       It depends on the teaching skills of the teacher
d.    Yes, provided we include a concrete application of the abstract
19.      Which part of the lesson is the symbolic stage?
a.    Using the model of fractions
b.    Dividing the pizza further into four
c.       Dividing the pizza into two
d.    Writing the fractions 1/2, 1/4, 1/3, 2/4 on the board
20.      Does the development of the lesson on fraction conform to the bottom-up arrangement of the learning experiences in Edgar Dale's
Cone of Experience?
a.    No
b.    Cannot be determined
c.    Yes
d.    Up to the second phase of the lesson only

CASE #2 – In a faculty recollection, the teachers were asked to share their thoughts of the learner, their primary customer. What
follow is the gist of what were shared:

Teacher A - “The learner is a product of his environment. Sometimes he has no choice. He is determined by his environment.”

Teacher B - “The learner can choose what he can become despite his environment.”

Teacher C - “The learner is a social being who learns well through an active interplay with other.”

Teacher D - “The learner is a rational being. Schools should develop his rational and moral powers.”

21.      Whose philosophical concept is that of Teacher A?


a.    Rationalist                c. Existentialist
b.    Behaviorist            d. Progressivist
22.      Teacher B's response comes from the mouth of a/an:
a.    Behaviorist               c. Essentialist
b.    Existentialist                       d. Perennialist
23.      If you agree with Teacher C, you are more of a/an:
a.    Perennialist              c. Essentialist
b.    Rationalist                d. Progressivist
24.      If you identify yourself with Teacher D, you adhere to what philosophy?
a.    Progressivist             c. Existentialist
b.    Behaviorist               d. Perennialist
25.      Whose response denies man's free will?
a.    Teacher A               c. Teacher D
b.    Teacher C                d. Teacher B

CASE #3 – School Head Amilia wants her teachers to be constructivist in their teaching orientation.

26.      Which material will her teachers most likely use?


a.    Facts                  c. Time-tested principles
b.    Hypotheses      d. Laws
27.      Which material will her teachers most likely avoid?
a.    Unquestionable laws
b.    Open-ended topics             
c.       Problems or cases            
d.    Controversial issues
28.      On which assumption/s is/are the principal's action anchored?
I.               Students learn by personally constructing meaning of what is taught
II.    Students construct and reconstruct meanings based on experiences
III.   Students derive meaning from the meaning that teacher gives
a.    I only                        c. I and II
b.    I and III                     d. II only
29.      Which does School Head Amilia want her teachers to do?
a.    Require their students to come up with a construct of the lesson
b.    Make their students derive meaning from what is presented
c.      Let their students construct meaningful sentences based on the lesson
d.    Give the meaning of what they present in class
30.      Which one should her teachers then avoid? Students':
a.    Reflection    
b.    Self-directed learning
c.      Memorization of facts for testing
d.    Inquiry         
        

CASE #4 – On the first day of school, Teacher Yveta oriented her class on procedures to be observed in passing papers, getting
textbooks from and returning the same on the bookshelf, leaving the room for necessity, and conducting group work. She arranged
the chairs in such a way that students can interact and can move around without unnecessarily distracting those seated. She
involved the class in the formulation of rules to ensure punctuality, order and cleanliness in the classroom.

31.      On what belief is Teacher Yveta's management practice anchored?


a.    Classroom rules need to be imposed for order's sake
b.    The classroom environment affects learning                                       
c.       A teacher must lord her power over her students to be an effective classroom manager
d.    A reactive classroom management style is effective
32.      Teacher Yveta involved her students in the formulation of class rules. Which describes her classroom management style?
a.    Benevolent               c. Democratic
b.    Authoritarian d. Laissez-faire
33.      Which adjective appropriately describes Teacher Yveta as a classroom manager?
a.    Proactive                 c. Reactive
b.    Modern                     d. Traditional  
34.      Was it sound classroom management practice for Teacher Yveta to have involved the students in the formulation of class rules?
a.    No, it weakens the teacher's authority over her students
b.    Yes, it makes students feel a sense of ownership of the rules
c.       Yes, it lessens a teacher's work
d.    No, it is a students' act of usurpation of teacher’s power              
35.      Which assumption underlies Teacher Yveta's classroom management practice?
a.    Classroom routines are boring and will work when imposed
b.    Classroom routines leave more time for class instruction
c.       Students need to be treated like adults to learn responsibility
d.    Teacher's personality is a critical factor in classroom discipline                

           

CASE #5 – Mr. Santo's lesson was on water conservation. He presented a graph that compared water consumption of small and big
families. Before he asked any of the questions, he asked someone to stand up to give an answer. He called only on those who
raised their hands. The questions he asked were:

1.     What do you see in the graph?

2.     How do you compare the water consumption of small and big families?

3.     Why do most of the big families consume more water than the smaller families?

4.     Do all the small families consume less water than the big families? Explain your answer.

5.     In your opinion, why does one small family consume more water than one big family?

6.     In what ways is water wasted?

7.     What are ways of conserving water?

8.     Are the families presented well at water conservation? Why or why not?

9.     What generalization can you draw about water consumption and size of families?

36.      Is there any convergent question from #1 to #8?


a.    Yes, question #4                 
b.    Yes, question #7               
c.       Yes, question #8
d.    None
37.      Which question technique/s of Mr. Santo do(es) not enhance interaction?
I.               Asking high-level questions
II.    Calling only on those who raised their hands
III.   Calling on someone before asking the question
a.    II and III                    c. III only
b.    I and II                      d. I and III
38.      Which statement on “wait time” is correct?
a.    The higher the level of the question, the longer the wait time
b.    Wait time turns off slow thinking students
c.       For quality response, “what” and “why” questions require equal wait time
d.    Wait time discourages the brighter group of students
39.      To connect the lesson on water conservation to the life of the students, which question is most appropriate?
a.    How can you help conserve water?
b.    Based on you observations, in what ways for people contribute to water wastage?        
c.    What maybe the reason why even if Family B is not as big as Family C, it consumes much more water than Family C?
d.    Among the families, who contributes most to water conservation?
40.      Were all the questions of Mr. Santo divergent?
a.    Yes                           c. No
b.    No, except #4           d. Yes, except #1       

CASE #6 – With a topic on the human circulatory system, Teacher Jan formulated the following lesson objectives:

1.    Given a model of the human circulatory system, the student must be able to understand the route of blood circulation

2.    After discussing the process of blood circulation, the teacher must be able to lead the pupils in enumerating circulatory system-
related diseases and in citing the causes and prevention of such diseases.          

41.      Is objective #1 in accordance with the principles of lesson objective formulation?


a.    No, the word “understand” is not a behavioral term
b.    No, it is not attainable         
c.       Yes, “understand” is an action word
d.    Yes, it is very specific
42.      How can you improve objective #2?
a.    Remove the phrase “After discussing the process of blood circulation.”
b.    Formulate it from the learner's point of view
c.       Cut it short; the statement is quite long
d.    No need to improve on it                            
43.      Do both objectives include a criterion of success, an element of a lesson objective cited by Robert Mager?
a.    Only objective #1 has         
b.    Only objective #2 has                     
c.    No, they don't        
d.    Yes, they do
44.      Is objective #2 in accordance with the principles of lesson objective formulation?
a.    No, it does not describe pupils' learning behavior
b.    Yes, it is formulated from the point of view of the teacher
c.       No, it is very broad
d.    Yes, it describes teacher's teaching activity
45.      Which one is the measurable learning behavior in objective #1?
a.    Able to understand  
b.    Route of blood circulation   
c.       To understand                   
d.    None 

               

CASE #7 – Teacher Fantina has a hard time getting the attention of her class. When she checks for understanding of the lesson
after a usual forty-five minute lecture, she finds out that only one or two can answer her questions. This has become a pattern so
one time, when the class could not answer, she gave a test.            

46.      What does the inattentiveness of most of Teacher Fantina's students confirm?
a.    The “ripple effect” of behavior               
b.    The lack of academic preparation of some teachers
c.       The strange behaviors of today's students
d.    The stubbornness of student groups

47.      Which method in dealing with classroom management problem is better than that of Teacher Fantina?
a.    Low level force and private communication
b.    Low level force and public communication
c.       High level force and private communication
d.    High level force and public communication
48.      Can the inattentiveness of Teacher Fantina's class be attributed to her use of the lecture method?
a.    Yes, if the lecture was not interactive
b.    Yes, if Teacher Fantina is an experienced teacher
c.       No, if the students are intelligent
d.    Cannot be determined
49.      With the principles of learning in mind, which one can help Teacher Fantina solve her student disciplinary problem?
a.    Call on their first names      
b.    Do interactive teaching                
c.       Change seat plan of the class
d.    Assign monitors in class
50.      Which act of Teacher Fantina is contrary to the principles of teaching?
a.    Asking questions to check for understanding                                 
b.    Giving a lecture
c.       Checking for understanding of the lesson in the process of teaching

         d.   Giving a test to discipline the class

LET Reviewer : Professional Education Part 4

- November 02, 2018

1.    Teacher Leon gives his students opportunities to be creative because of his conviction that much learning results from
the need to express creativity. On which theory is Teacher Leon’s conviction anchored? ______ theory

a.    Behaviorist c. Cognitive


b.    Associationist d. Humanist
2.    Which philosophy approves of a teacher who lectures most of the time and requires his students to memorize the rules of
grammar?
a.    Existentialism c. Pragmatism
b.    Realism d. Idealism
3.    Teacher Nikka wants to check prior knowledge of her pupils about water pollution. She writes the main topic water pollution
in the center of the chalkboard and encircles it. Then, she ask the pupils to provide information that can be clustered
around the main topic. Which technique did the teacher employ?
a.    Vocabulary building
b.    Semantic mapping
c.     Demonstration
d.    Deductive teaching
4.    The current emphasis on the development of critical thinking by the use of philosophic methods that emphasize debate and
discussion began with:
a.    Aristotle c. Confucius
b.    Socrates d. Plato
5.    Which refers to a single word or phrase that tells the computer to do something with a program or file?
a.    Computer program c. Computer language
b.    Password d. Command
6.    In instructional planning, which among these three; unit plan, course plan, lesson plan is most specific? _________ plan.
a.    Course c. Resources
b.    Unit d. Lesson
7.    The first American teachers on the Philippines were:
a.    Missionaries
b.    Soldiers
c.     Graduates of the normal school
d.    Elementary graduates
8.    By which process do children become participating and functioning members of society by fitting into an organized way to
life?
a.    Socialization c. Accommodation
b.    Acculturation d. Assimilation
9.    What is the mean of this score distribution 4, 5, 6, 7, 8, 9, 10?
a.    7.5 b. 8.5 c. 6 d. 7
10.  Which is a teaching approach for kindergartens that makes real world experiences of the child the focal point of
educational stimulation?
a.    Situation approach
b.    Traditional approach
c.    Montessori approach
d.    Eclectic approach
11.  Which among the following graphic organizers used helps to show events in chronological order?
a.    Time line and story map
b.    Time line and cycle
c.     Series of events chart and story map
d.    Time line and series of events chart

12.  Which schools are subject to supervision, regulation and control by the state?
a.    Public, private sectarian and non-sectarian schools
b.    Public schools
c.     Sectarian and non-sectarian schools
d.    Private schools

13.  Which among following can help student development the habit of critical thinking?
a.    Blind obedience of authority
b.    A willingness to suspend judgment until sufficient evidence is presented
c.     Asking convergent questions
d.    Asking low level questions

14.  Teacher Nelda wants to develop in her pupils comprehension skills. What order of skills will she develop?
I.              Literal comprehension
II.            Interpretation
III.           Critical evaluation
IV.          Integration
a.    II-III-IV-I c. I-II-III-IV
b.    III-IV-I-II d. IV-III-II-I
15.  An integrative, conceptual approach introduced by Roldan that has as its highest levels in the development of _____
thinking skills.
a.    Interpretative c. Critical
b.    Creative d. Literal
16.  If a student thinks about thinking, he is involved in the process called ________.
a.    Higher order thinking
b.    Metacognition
c.     Critical thinking
d.    Creative thinking
17.  The use of drills in the classroom is rooted on Thorndike’s law of:
a.    Readiness c. Exercise
b.    Effect d. Belongingness
18.  The following are some drill techniques, except:
a.    Challenging students to be above the level of the class
b.    Asking pupils to repeat answers
c.     Giving short quiz and having students grade papers
d.    Assigning exercises from a workbook
19.  The process of task analysis ends up in the formulation of:
a.    Instructional objectives
b.    Enabling objectives
c.     Goals of learning
d.    Behavioral objectives

20.  For city-bred students to think that their culture is better than those from the province is a concrete example of ____.
a.    Ethical relativism  c. Cultural relativism

b.    Ethnocentrism  d. Xenocentrism

21.  Who were the Thomasites?


a.    The first American teachers that help in establishing the public educational system in the Philippines
b.    The soldiers who doubted the success of the public educational system to be set in the Philippines
c.     The first religious group who came to the Philippines on board the US transports Thomas
d.    The devotees to St. Thomas Aquinas who came to evangelize
22.  Which teaching activity is founded on Bandura's social learning theory?
a.    Questioning
b.    Inductive reasoning
c.    Modeling
d.    Interactive teaching

23.  Which program is DepEd’s vehicle in mobilizing support from the private and non-government sectors to support programs
based on DepEd’s menu of assistance packages?

a.    Chili-Friendly-School System
b.    Adopt-A-School-Program

c.     Every Child A Reader Program

d.    Brigada Eskwela

24.  Researchers gave rats a dose of 3-m butyl phthalide and measured changes in the rats blood pressure. This statement is
best classified as
a.    Experiment c. Hypothesis
b.    Prediction d. Finding

25.  The right hemisphere of the brain is involved with the following functions except:
a.    Visual functions
b.    Nonverbal functions
c.     Intuitive functions
d.    Detail-oriented functions
26.  To build a sense of pride among Filipino youth, which should be done?
a.    Re-study our history from the perspective of our colonizer
b.    Re-study our history and stress on our achievements as a people
c.     Replace the study of folklores and myths with technical subjects
d.    Set aside the study of local history

27.  Which is in accordance with the “with-it-ness” principle of classroom management of Kounin?
a.    Students agree to disagree in class discussions
b.    Teacher is fully aware of what is happening in his classroom
c.     Student is with his teacher in everything he teaches
d.    Both parents and teachers are involved in the education of children

28.  Below are questions that must be considered in developing appropriate learning activity experiences except one. Which is
it?
a.    Can experiences benefit the pupils?
b.    Do the experiences save the pupils from learning difficulties?
c.     Are the experiences in accordance with the patterns of pupils?
d.    Do the experiences encourage pupils to inquire further?
29.  Which is a characteristic of an imperfect type of matching test?
a.    An item may have no answer at all
b.    An answer may be repeated
c.     There are two or more distracters
d.    The items in the right and left columns are equal in number
30.  Which is a proactive management practice?
a.    Tell them that you enforce the rules on everyone, no exception
b.    Set and clarify your rules and expectation on Day 1
c.     Punish the misbehaving pupils in the presence of their classmates
d.    Stress on penalty for every violation
31.  “Specialization is knowing more and more about less and less. Then it is better to be a generalist,” claims Teacher Patty.
On which philosophy does Teacher Patty learn?
a.    Essentialism c. Perennialism
b.    Progressivism d. Existentialism
32.  I like to develop the synthesizing skills of my students. Which one should I do?
a.    Ask my students to formulate a generalization from the data shown in the graphs
b.    Direct my students to point out which part of the graph are right and which part is wrong
c.     Ask my students to answer the questions beginning with “what if”
d.    Tell my students to state data presented in the graph
33.  To reach out to clientele who cannot be in the classroom for one reason or another, which of the following was
established?
a.    Informal
b.    Special education (SPED)
c.     Pre-school education
d.    Alternative learning delivery system
34.  Tira enjoys games like scrabble, anagrams and password. Which type of intelligence is strong in Tira?
a.    Interpersonal intelligence
b.    Linguistic intelligence
c.     Logical and mathematical intelligence
d.    Spatial intelligence
35.  Teacher Milla observes cleanliness and order in the classroom to create a conducive atmosphere for learning. On which
theory is her practice based?
a.    Psychoanalysis
b.    Gestalt psychology
c.     Behaviorism
d.    Humanistic psychology
36.  Which learning principles is the essence of Gardner’s theory of multiple intelligences?
a.    Almost all learners are linguistically intelligent
b.    Intelligence is not measured on one form
c.     Learners have different IQ level
d.    Learners have static IQ
37.  Who asserted that children must be given the opportunity to explore and work on different materials so that they will
develop the sense of initiative instead of guilt?
a.    Kohlberg c. Maslow
b.    Erickson d. Gardner
38.  Teacher Ada uses direct instruction strategy. Which will she first do?
a.    Presenting and structuring
b.    Independent practice
c.     Guided student practice
d.    Reviewing the previous day’s work
39.  The free public elementary and secondary educations in the country are in the line with the government effort to address
educational problems of _________.
a.    Productivity
b.    Relevance and quality
c.    Access and quality
d.    Effectiveness and efficiency
40.  Here are raw scores in a quiz 97, 95, 85, 83, 77, 75, 50, 10, 5, 2, 1. To get a picture of the group’s performance which
measure of central tendency is most reliable?
a.    Median
b.    Mode
c.     Mean
d.    None, it is best to look at the individual scores

41.  Which objective in the affective domain is the lowest level?


a.    To accumulate examples of authenticity
b.    To support viewpoints against abortion
c.     To respond positively to a comment
d.    To formulate criteria for honestly
42.  “Using the six descriptions of elements of good short story, IDENTIFY IN WRITING THE SHORT STORY BY O. HENRY, with
complete accuracy.” The words in capital letters are referred to as the ____________.
a.    Criterion of success
b.    Condition
c.    Performance statement
d.    Minimum acceptable performance

43.  You have presented a lesson on animal protective coloration. At the end, you ask if there are any questions. There are
none. You can take this to mean that ______.
a.    The students are not interested in the lesson
b.    You need to ask specific questions to elicit responses
c.     The students did not understand what you were talking about
d.    The students understood everything you presented

44.  Teacher Vina feels offended by her supervisor’s unfavorable comments after a classroom supervision. She concludes that
her supervisor does not like her. Which Filipino trait is demonstrated by Teacher Vina?

a.    Extreme family-centeredness
b.    Extreme personalism

c.     “Kanya-kanya” syndrome

d.    Superficial religiously

45.  Teachers are encouraged to make use of authentic assessments such as:
a.    De-contextualized drills
b.    Unrealistic performances
c.     Answering multiple choices test items
d.    Real world application of lessons learned
46.  What does the principle of individual difference require teachers to do?
a.    Give less attention to gifted learners
b.    Provide for a variety of learning activities
c.     Treat all learners alike while teaching
d.    Prepare modules for slow learners in class

47.  The source of energy responsible for life on earth is the ______.
a.    Moon c. Star
b.    Wind d. Sun
48.  Which quotation goes with a proactive approach to discipline?
a.    “An ounce of prevention is better than a pound of cure.”
b.    “Do not make a mountain out of a molehill.”
c.     “Walk your talk.”
d.    “Do not smile until Christmas.”

49.  I’d like to test whether a student knows what a particular word means. What should I ask the student to do?
a.    Give the word a tune then sing it
b.    Define the word
c.     Spell the word and identify its part of speech
d.    Give the etymology of the word

50.  Study this matching type of test. Then answer the question below.

    Measure of relationship a.  Mean


    Measure of central tendency b.  Standard Deviation
    Binet-Simon c.  Rho
    Statistical test of mean d.  T-ratio
difference e.  Intelligence testing movement
    Measure of variability

Which among the following is a way to improve the above matching test?

a.    Add five items in both columns


b.    Add one or two items in the right column
c.     Add ten items in both columns to make the test more comprehensive
d.    Add one or two items in the left column

LET Reviewer : Professional Education Part 5

- November 02, 2018

1.    A teacher’s quarrel with a parent makes her develop a feeling of prejudice against the parent’s child. The teacher’s
unfavorable treatment of the child is an influence of what Filipino trait?
    
a.    Lack of self-reflection

b.    Extreme personalism

c.     Extreme family-centeredness

d.    “Sakop-mentality

2.    In the context of grading, what is referred to as teacher’s generosity error? A teacher _______.
a.    Rewards students who perform well
b.    In overgenerous with praise
c.    Has a tendency to give high grades as compare to the rest
d.    Gives way to students’ bargain for no more quiz
3.    For mastery learning and in line with outcome-based evaluation model which element should be present?
a.    Inclusion of non-performance objectives
b.    Construction of criterion-referenced tests
c.     Construction of norm-referenced tests
d.    Non-provision of independent learning
4.    Which statement about standard deviation is correct?
a.    The higher the standard deviation the more spread the scores are
b.    The lower the standard deviation the more spread the scores are
c.     The higher the standard deviation the less spread the scores are
d.    It is a measure of central tendency
5.    What is the Teacher’s Professionalism Act?
a.    RA 7836 c. RA 7722
b.    RA 4670 d. RA 9263
6.    The State shall protect and promote the right of citizens to quality education at all levels. Which government program is in
support of this?
a.    Exclusion of children with special needs from the formal system
b.    Free elementary and secondary education
c.     Deregulated tuition fee hike
d.    Re-introduction of the NEAT and NSAT
7.    The American Teachers who were recruited to help set the public educational system in the Philippines during the
American regime were called Thomasite’s because:
a.    They were devotees of St. Thomas Aquinas
b.    They disembarked from the CIS Transport called Thomas
c.     They first taught at the University of Sto. Tomas
d.    They arrived in the Philippines on the feast of St. Tomas
8.    Zazha exhibits fear response to freely roaming dogs but does not show fear when a dog is on a leash or confined to a pen.
Which conditioning process is illustrated?
a.    Generalization c. Discrimination
b.    Acquisition d. Extinction
9.    The Filipino learner envisioned by the Department of Education is one who is imbued with the desirable values of person
who is:
a.    Makabayan, makatao, makakalikasan at maka-Diyos
b.    Makabayan, makatao, makahalaman, at maka-Diyos
c.     Makabayan, makasarili, makakalikasan, at maka-Diyos
d.    Makabayan, makakaragatan, makatao, at maka-Diyos
10.  The right hemisphere of the brain is involved with the following function except:
a.    Intuitive functions
b.    Nonverbal functions
c.     Visual functions
d.    Detail-oriented functions
11.  What function is associated with the right brain?
a.    Visual, non-verbal, logical
b.    Visual, intuitive, non-verbal
c.    Visual, intuitive, logical
d.    Visual, logical, detail-oriented
12.  Principal Connie tells her teachers that training in the humanities is most important. To which education philosophy does
he adhere?
a.    Existentialism c. Essentialism
b.    Progressivism d. Perennialism
13.  Behavior followed by pleasant consequences will be strengthened and will be more likely to occur in the future. Behavior
followed by unpleasant consequences will be weakened and will be less likely to be repeated in the future. Which one is
explained?
a.    Freud’s psychoanalytic theory
b.    Thorndike’s law of effect
c.     B.F. Skinner’s Operant conditioning theory
d.    Bandura’s social learning theory
14.  Theft of school equipment like TV, computer, etc. by people on the community itself is becoming a common phenomenon.
What does this signify?
a.    Prevalence of poverty in the community
b.    Inability of school to hire security guards
c.     Deprivation of Filipino schools
d.    Community’s lack of sense of co-ownership
15.  The main purpose of compulsory study of the Constitution is to _____.
a.    Develop students into responsible, thinking citizen
b.    Acquaint students with the historical development of the Philippine Constitution
c.     Make constitutional experts of the students
d.    Prepare students for law-making
16.  Which goals of educational institution, as provided for by the Constitution, is the development of work skills aligned?
a.    To develop moral character
b.    To develop vocational efficiency
c.     To teach the duties of citizenship
d.    To inculcate love of country

17.  Complete this analogy:

Spanish period: moral and religious person.


American period: _______

a.    Productive citizen c. Patriotic citizen

b.    Self-reliant citizen  d. Caring citizen

18.  Who is remembered for his famous quotation? “My loyalty to may party ends where my loyalty to my country begins.”
a.    Carlos P. Garcia c. Manuel L. Quezon
b.    Ferdinand Marcos d. Manuel Roxas
19.  Here is a question: “Is the paragraph a good one? Evaluate.” If broken down to simplify, which is the best simplification?
a.    Is the paragraph a good one? Why or why not?
b.    Why is the paragraph a good one? Prove
c.     If you asked to evaluate something, what do you do? Evaluate the paragraph
d.    What are the qualities of a good paragraph? Does the paragraph have these qualities?
20.  Manunulat ang tatay mo kaya sa personal mong kagustuhan makasunod sa kanyang mga yapak, magsusulat ka. Ano ang
kahulugan ng pagsusulat?
a.    Proseso na nagsisimula sa idea o karanasan
b.    Makalikha ng mga salita, pangungusap at talata
c.     Kakayahan ng tao na makapagtala o makapagimprenta ng malinaw
d.    Pagpili ng paksa at pagsasaliksik
21.  Anong bahagi ng pananalita ang nasa malaking titik? Malungkot ANG MGA nagtapos na wala pang trabaho.
a.    Pananda c. Pariralang pantukoy
b.    Pang-ukol d. Pangatnig

22.  Which is NOT a characteristic of education during the pre-Spanish era?

a.    Vocational training-oriented
b.    Structured

c.     Unstructured

d.    Informal

23.  Which is closest to the real human digestive system for study in the classroom?
a.    Model of the human digestive system
b.    Drawing of the human digestive system on the board
c.     The human digestive system projected on an OHP
d.    Drawing of human digestive system on a page of a textbook
24.  Which one is in support of greater interaction?
a.    Repeating the question
b.    Not allowing a student to complete a response
c.     Probing
d.    Selecting the same student respondents
25.  Which is/are effective methods/s in teaching students critical reading skills?
a.    Interpret editorials about a particular subject from three different newspapers
b.    Read and interpret three different movie reviews
c.     Distinguish fiction from non-fiction materials
d.    Interpret editorials and read and interpret three different movie reviews
26.  Which is true of a bimodal score distribution?
a.    The group tested has two different groups
b.    The scores are neither high nor low
c.     The scores are high
d.    The scores are low
27.  What is the mastery level of school in a 100 item test with a mean of 55?
a.    42% b. 50% c. 45% d. 55%

28.  Who stressed the idea that students cannot learn if their basic needs are not first met?
a.    Thorndike
b.    Maslow
c.     Wertheimer
d.    Operant conditioning

29.  A person, who has had painful experience as the dentist’s office, may become fearful at the mere sight of the dentist’s
office building. What theory can explain this?
a.    Attribution theory
b.    Classical conditioning
c.     Generalization
d.    Operant conditioning

30.  I want my students to have mastery learning of a basic topic. Which of the following can help?
a.    Socratic method and drill
b.    Drill
c.     Socratic method
d.    None of the above

31.  As a teacher you are a skeptic. Which among these will be your guiding principle?

a.    I must teach the child to the fullest


b.    I must teach the child every knowledge, skill, and value that he needs for a better future

c.    I must teach the child that we can never have real knowledge of anything

d.    I must teach the child so he is assured

32.  With forms of prompting in mind, which group is arranged from least to most instructive prompting?
a.    Verbal, physical, gestural
b.    Verbal, gestural, physical
c.     Gestural, physical, verbal
d.    Physical, gestural, verbal
33.  In which way does heredity affect the development of the learner?
a.    By providing equal potential to all
b.    By making acquired traits hereditary
c.     By compensating for what environment fails to develop
d.    By placing limits beyond which the learner cannot develop

34.  In writing performance objectives, which word is not acceptable?

a.    Integrate  c. Manipulate

b.    Delineate  d. Comprehend

35.  For which lesson objective will a teacher use the direct instruction method?
a.    Distinguish war from aggression
b.    Appreciate Milton’s Paradise Lost
c.    Use a microscope properly
d.    Become aware of the pollutants in the environment
36.  To encourage introspection, which teaching method is MOST appropriate?
a.    Cognitive c. Process
b.    Reflective d. Cooperative learning
37.  With indirect instruction in mind, which does not belong to the group?
a.    Lecture-recitation
b.    Experiential method
c.     Inductive method
d.    Discovery method
38.  A master teacher is the resource speaker in an in-service training. He presented the situated learning theory and
encouraged his colleagues to apply the same in class. Which of the following did he not encourage his colleagues to do?
a.    Apprenticeship
b.    Decontextualized teaching
c.     Learning as it normally occurs
d.    Authentic problem solving
39.  I want to engage my students in small group discussions. Which topic lends itself to a lively discussion?
a.    The exclusion of Pluto as a planet
b.    The meaning of the law of supply and demand
c.     The law of inertia
d.    Rules on subject-verb agreement
40.  Teacher Joshua discovered that his pupils are weak in comprehension. To further in which particular skill(s) his pupils are
weak, which test should Teacher Joshua give?
a.    Standardized test c. Placement
b.    Aptitude test d. Diagnostic
41.  The students of Teacher Kath scan an electronic encyclopedia, view a film on the subject, or look at related topics at the
touch of a button right there in the classroom. Which devices does teacher Kath’s class have?
a.    Videotape lesson c. Video disc
b.    Teaching machine d. CD
42.  If my approach to my lesson is behaviorist, what features will dominate my lesson?
I.              Lecturing III. Reasoning
II.            Copying notes IV. Demonstration
a.    III, IV c. I, II, III, IV
b.    I, III, IV d. II, III, IV
43.  You practice inclusive education. Which of these applies to you?

I.      You accept every student as full and valued member of the class and school community
II.    Your special attention is on learners with specific learning or social needs

III.   You address the needs of the class as a whole within the context of the learners with specific learning or social needs

a.    II only c. I only


b.    I and II d. I and III
44.  Which will be the most authentic assessment tool for an instructional objective on working with and relating to people?
a.    Organizing a community project
b.    Writing articles on working and relating to people
c.     Home visitation
d.    Conducting mock election
45.  Here is a test item:
“From the data presented in the table, form generalizations that are supported by the data.” Under what type of question
does this item fall?
a.    Convergent c. Application
b.    Evaluative d. Divergent
46.  I want to teach concepts, patterns, and abstractions. Which method will be most appropriate?
a.    Discovery c. Direct instruction
b.    Indirect instruction d. Problem solving
47.  We are very much interested in a quality professional development program for teachers. What characteristic should we
look for?
a.    Required for renewal of professional license
b.    Prescribed by top educational leader
c.    Responsive to identified teacher’s needs
d.    Dependent on the availability of funds
48.  What principle is violated by overusing the chalkboards, as though it is the only education technology available?
a.    Isolated use c. Variety
b.    Flexibility d. Uniformity
49.  Which statement applies correctly to Edgar Dale’s “CONE of experience”?
a.    The farther you are from the base, the more direct the learning experience becomes
b.    The farther you are from the bottom, the more direct the learning experience becomes
c.     The close you are to the base, the more indirect the learning experience become
d.    The closer you are to the base, the more direct the learning experience becomes
50.      To teach the democratic process to the pupils. Santo Domingo Elementary School decided that the election of class officers
shall be patterned after local elections. There are qualities set for candidates, limited period for campaign, rules for
posting campaign materials, etc. Which of the following did the school use?
a.    Symposium c. Role playing
b.    Simulation d. Philips 66

LET Reviewer : Professional Education Part 6

- November 02, 2018

1.        Which of the following are effective methods in teaching student critical reading skills?
I.      Interpret editorials
II.    Read and interpret three different movie reviews
III.   Read a position paper and deduce underlying assumptions of the position papers
a.    II and III c. I and II
b.    I and III d. I, II and III
2.        Here is a test item:
“The improvement of basic education should be the top priority of the Philippine government. Defend and refute the
position.”
Under what type of question does this test item fall?
a.    Low-level c. Analysis
b.    Evaluative d. Convergent
3.        Teacher Jenny teaches a lesson in which students must recognize that ¼ is the same as 0.25. They use this relationship to
determine that 0.15 and 0.20 are slightly less than ¼. Which of the following concept/s is/are being taught?
a.    Numeration skills
b.    Place value of decimals
c.    Numeration skills for decimal and relationship between fractions and decimals
d.    Relationship between fractions and decimals
4.        To nurture student’s creativity, which activity should the teacher avoid?
a.    Ask “what if” questions
b.    Ask divergent thinking questions
c.    Emphasize the need to give right answer
d.    Be open to “out-of-this world” ideas

5.        After reading an essay, Teacher Bebe wants to help sharpen her students’ ability to interpret. Which of these activities will
be most appropriate?
a.    Drawing conclusions
b.    Making inferences
c.     Getting the main idea
d.    Listing facts separately from opinion

6.        What is the best way to develop math concepts?


a.    Solving problems using multiple approaches
b.    Solving problems by looking for correct answer
c.    Learning math as applied to situations such as being a tool of science
d.    Solving problems by applying learned formulas
7.        Teacher Cita, an experienced teacher, does daily review of past lessons in order to ________.
a.    Provide her pupils with a sense of continuity
b.    Introduce a new lesson
c.     Reflect on how she presented the previous lesson
d.    Determine who among her pupils are studying

8.        Research says that mastery experiences increase confidence and willingness to try similar or more challenging tasks such
as reading. What does this imply for children reading performance?
a.    Children who have mastered basic skills are more likely to be less motivated to read because they get fed up with too much reading
b.    Children who have not mastered the basic skills are more likely to be motivated to read in order to gain mastery over basic skills
c.     Children who have a high sense of self-confidence are not necessarily those who can read
d.    Children who have gained mastery over basic skills are more motivate to read

9.        Bruner’s theory on intellectual development moves from enactive to iconic and symbolic stages. Applying Bruner’s theory,
how would you teach?
a.    Begin with the abstract
b.    Be interactive in approach
c.    Begin with the concrete
d.    Do direct instruction

10.      Teacher Nene asked this question: “What conclusion can you draw based on your observation?” Nobody raised a hand so
she asked another question: “Based on what you observe, what can you now say about the reaction of plants to light.”
What did Teacher Nene do?

a.    Redirecting c. Repeating


b.    Probing d. Rephrasing

11.      Under no circumstance shall a teacher be prejudiced nor discriminatory against any learner according to the Code of
Ethics. When is a teacher prejudice against any learner?
a.    When he makes a nearsighted pupil sit at the front
b.    When he considers multiple intelligences in the choice of his teaching strategies
c.     When he makes a farsighted pupil sit at the back
d.    When he refuses a pupil with a slight physical disability in class

12.      Which learning activity is most appropriate if teacher’s focus in attitudinal change?
a.    Role play c. Exhibit
b.    Field trip d. Game
13.      The mode of a score distribution is 25. This means that:
a.    There is no score of 25
b.    Twenty five (25) is the score that occurs most
c.     Twenty five is the average of the score distribution
d.    Twenty five is the score that occurs least
14.      The following characterize a child-centered kindergarten except:
a.    Focus on the education of the whole child
b.    Importance of play in development
c.    Extreme orientation on academic
d.    Emphasis on individual uniqueness

15.      As a classroom manager, how can you exhibit expert power on the first day of school?

a.    By citing to my students the important of good grades


b.    By making my students feel my authority over them

c.     By making them feel a sense of belongingness and acceptance

d.    By making my students feel I know what I am talking about

16.      Which may help an adolescent discover his identity?


a.    Parents pushing in to follow a specific path
b.    Relating to people
c.     Decision to follow one path only
d.    Exploring many different roles in a healthy manner
17.      Which terms refers to a teacher helping a colleague grow professionally?
a.    Technology transfer
b.    Peer mentoring
c.     Facilitating
d.    Independent study
18.      What primary criterion should guide a teacher in the choice of instructional devices?
a.    Novelty c. Appropriateness
b.    Cost d. Attractiveness
19.      Some of your students don’t seem to like you as their teacher. If you will regard the situation, on the level of the ego, what
will you most likely think about?
a.    Why should I care if they like me or not
b.    To hell with them
c.     What’s wrong have I done to deserve this?
d.    What is it about me that they do not like?
20.      Which of the following is considered a peripheral device?
a.    Printer c. CPU
b.    Keyboard d. Monitor
21.      Which questioning practice will promote more class interaction?
a.    Asking rhetorical question
b.    Rejecting wrong answer
c.     Focusing on convergent question
d.    Asking divergent question
22.      For grades to be (made) valid indicators of students’ achievements, which process should be observed?
a.    Adopting letter grades such as A, B, C, D
b.    Explaining the meaning of grades
c.    Defining the course objectives as intended learning outcomes
d.    Giving objective type of test
23.      The claim of a benefactor to the gratitude of his protégé is an example of a (an):
a.    Acquired right c. Imperfect right
b.    Perfect right               d. Alienable right
24.      Which appropriate teaching practice flows from this research finding on the brain: “The brain’s emotional center is tied its
ability to learn.”
a.    Create a learning environment that encourages students to explore their feelings and ideas freely
b.    Come up with highly competitive games where winners will feel happily
c.     Establish this discipline of being judgmental in attitude
d.    Tell the students to participate in class activities or else they won’t receive plus points in class recitation
25.      Research on Piagetian tasks indicates that thinking becomes more logical and abstract as children reach the formal
operations stage. What is an educational implication of this finding?
a.    Expect hypothetical reasoning for learners between 12 to 15 years of age
b.    Learners who are not capable of logical reasoning from ages 8 to 11 behind in their cognitive development
c.     Engage children in analogical reasoning as early as preschool to train them for higher order thinking skills (HOTS)
d.    Let children be children
26.      A mathematics test was given to all Grade V pupils to determine the contestants for the math quiz bee. Which statistical
measure should be used to identify the top 15?
a.    Percentage score
b.    Mean percentage score
c.     Quartile score
d.    Percentile score
27.      Which is the true foundation of the social order?
a.    Strong, political leadership
b.    The reciprocation of rights and duties
c.     Equitable distribution of wealth
d.    Obedient citizenry
28.      With which is true authority equated?
a.    Service c. Suppression
b.    Power d. Coercion
29.      The following are sound specific purposes of questions except:
a.    To stimulate learners to ask questions
b.    To call the attention of an inattentive student
c.     To arouse interest and curiosity
d.    To teach via student answers
30.      What is not a sound purpose in making questions?
a.    To remind students of a procedure
b.    To probe deeper after an answer is given
c.     To encourage self-reflection
d.    To discipline a bully in class
31.      A student passes a book report written but ornately presented in a folder to make up for the poor quality of the book
content. Which Filipino trait does this practice prove?
a.    Art over science
b.    Art over academic
c.     Substance over “porma”
d.    “Porma” over substance
32.      Which one should a teacher avoid to produce an environment conducive for learning?
a.    Games c. Tests
b.    Seat plan d. Individual competition

33.      Between pursuing a college course where there is no demand and a vocational course which is highly in demand, the
Filipino usually opts for the college course. Which Filipino trait is shown?

a.    Interest to obtain a skill


b.    Penchant for a college diploma

c.     Desire for entrepreneurship

d.    Appreciation of manual labor

34.      Which of the following does extreme authoritarianism in the home reinforce in pupils?

a.    Sense of initiative


b.    Dependence on other for direction
c.     Ability for self-direction

d.    Creativity in work

35.      Teacher Bart wants his students to master the concept of social justice. Which series of activities will be most effective?
a.    Pre-teaching > posttest > re-teaching of unlearned concept > posttest
b.    Pre-test > teaching > posttest
c.    Review > pretest > teaching > posttest
d.    Teaching > posttest
36.      To provide for individual differences how is curriculum designed?
a.    Minimum learning competencies are included
b.    Realistic and meaningful experiences are provided
c.     Some degree of flexibility is provided
d.    Social skills are emphasized
37.      Which types of play is most characteristic of a four- to six-year-old child?
a.    Solitary and onlooker plays
b.    Associative and cooperative plays
c.     Associative and onlookers plays
d.    Cooperative and solitary plays
38.      How can you exhibit referent power on the first day of school?
a.    By making the students feel you know what you are talking about
b.    By telling them the importance of good grades
c.     By reminding your students your authority over them again and again
d.    By giving your students a sense of belonging and acceptance

39.      To ensure that all Filipino children are functionally literate, which mechanism is meant to reach out to children who are far
from a school?

a.    A school in every barangay b.    Multi-grade classroom c.     Mobile teacher
d.    Sine’s skwela

40.      Referring to the characteristics of the latest Basic Educational Curriculum which does not belong to the group?

a.    More flexible


b.    Less prescriptive

c.    More compartmentalized

d.    More integrated

41.      If student’s inappropriate behavior is low level or mild and that it appears that the misbehavior will not spread to others, it is
sometimes best for the teacher not to take notice of it. What influence technique is this?

a.    Planned ignoring


b.    Antiseptic bouncing

c.     Proximity control

d.    Signal interference

42.      Which among the following objectives in the psychomotor domain is highest in level?
a.    To distinguish distant and close sounds
b.    To contract a muscle
c.     To run a 100-meter dash
d.    To dance the basic steps of the waltz
43.      Which material consists of instructional units that cater to varying mental level pupils?
a.    Plantilia
b.    Multi-level materials
c.     Multi-grade materials
d.    Minimum learning competencies
44.      Which statement on counseling is false?
a.    For counseling to be successful, the counselee is willing to participate in the process
b.    The ultimate goal of counseling is greater happiness on the part of the counselee
c.     Counseling is the program that includes guidance
d.    The school counselor is primarily responsible of counseling

45.      Arianna describes Teacher Monica as “fair, caring and approachable.” Which power does Teacher Monica possess?

a.    Legitimate power


b.    Expert power

c.    Referent power

d.    Reward power

46.      Kounin claims “with-it-ness” is one of the characteristics of an effective classroom manager. Which among the following is
a sign of “with-it-ness”?

a.    Giving attention to students having difficulty with school work


b.    Seeing only a portion of the class but intensively

c.     Knowing where instructional materials are kept

d.    Aware of what’s happening in all part of the classroom

47.      In the K-W-L technique, K stands for what the pupils already knows, W for what he wants to know and L for what he:
a.    Learned c. Failed to learn
b.    He like to learn d. Needs to learn
48.      When a significantly greater number from the lower group gets a test item correctly, this implies that the test item:
a.    Is not highly reliable
b.    Is not very valid
c.     Is highly reliable
d.    Is very valid
49.      Which statement about guidance is false?
a.    The classroom teacher is not part of the school guidance program since she is not trained to be a guidance counselor
b.    Guidance embraces curriculum, teaching, supervision and all other activities in school
c.     Guidance is a function of the entire school
d.    A guidance program is inherent in every school
50.      A teacher should not be a slave of his lesson plan. This means that:
a.    A teacher must be willing to depart from her lesson plan if students are interested in something other than her intended
lesson
b.    A lesson plan must be followed by a teacher no matter what
c.     A teacher must be ready to depart from her lesson plan if she remembers something more interesting than what she earlier planned
d.    Teacher is the best lesson plan designer

You might also like